Sie sind auf Seite 1von 267

РАДЕ КРЕНКОВ СЕЉАМИ ТАХИР

ЗБИРКА ЗАДАЧИ ПО МАТЕМАТИКА


ЗА НАТПРЕВАРИ

СОУ „ГОЦЕ ДЕЛЧЕВ“


ВАЛАНДОВО
2011
ZBIRKA ZADA^I PO MATEMATIKA
ZA NATPREVARI

Rade Krenkov Seqami Tahir

Recenzenti:

D-r Valentina Miov


Miovska Prirodno-matemati~ki fakultet, Skopje
M-r Aneta Gacovska Prirodno-matemati~ki fakultet, Skopje
M-r Del~o Le{kovski Me|unaroden Balkanski Univerzitet, Skopje

Izdava~ na knigata: SOU „Goce Del~ev“, Valandovo

Za izdava~ot: Du{ko Kortov, direktor

Lektura: Branko \or|iev, profesor po makedonski jazik i literatura

Dizajn na koricata: Radomir Kne`evi}, profesor po likovna umetnost

Kompjuterska obrabotka: Seqami Tahir, Rade Krenkov

Pe~ati: „Unikat-Komerc“, Valandovo

Tira`: 300

CIP - Каталогизација во публикација


Национална и универзитетска библиотека "Св. Климент Охридски", Скопје

51(079.1)

КРЕНКОВ, Раде
Збирка задачи по математика за натпревари / Раде Кренков, Сељами
Тахир. - Валандово : #СОУ #Гоце Делчев, 2011. - 265 стр. : граф.
прикази ; 23 см

Библиографија: стр. 263-264. - Содржи и: Прилог

ISBN 978-9989-57-736-9
1. Тахир, Сељами [автор]
а) Математика - Задачи од натпревари
COBISS.MK-ID 86965514
Predgovor

Knigata e nameneta za site u~enici i nastavnici od srednite


u~ili{ta koi sakaat da gi pro{irat i prodlabo~at svoite znaewa od
oblastite po matematika predvideni za matemati~kite natprevari.
Odredeni temi opfateni vo ovaa zbirka zada~i po matematika mo`e da
se koristat i od strana na u~enici od osnovnite u~ili{ta, a istata
mo`e da ja koristat i studentite na prirodno-matemati~kite i
tehni~kite fakulteti. Vo samata zbirka se opfateni golem broj temi,
koi formiraat {iroka lepeza na zada~i, razli~ni po priroda i te`ina.
Vo nea vnimatelno e odbran redosledot na zada~ite po te`ina. Na
„polesnite“ zada~i se dadeni samo odgovori, na nekoi i upatstva, dodeka
na dobar del od zada~ite se dadeni i re{enija. Sekako deka ponudenoto
re{enie ne sekoga{ e mo`ebi najdobroto i najednostavnoto, taka {to na
~itatelite im prepora~uvame da se obidat da pronajdat i drugi metodi i
na~ini za re{avawe na zada~ite. Vo re{avaweto na odredeni
matemati~ki problemi, nie kako avtori imame koristeno i sovremeni
informacisko-komunikaciski tehnologii t.e. matemati~kite programi:
Geogebra, Dr.geo, Kig, Kmplot.
Zada~ite nameneti za natprevari ~esto pati po svojata
priroda se „nestandardni“, baraat golemo poznavawe na matemati~kite
ve{tini, upornost i istrajnost. No, zadovolstvoto od uspe{no re{ena
zada~a e golemo.
Sodr`ina
Zada~i Odgovori

Matemati~ka logika i mno`estva....................5 104

Osnovni brojni mno`estva................................10 110

Algebarski racionalni izrazi.......................13 116

Algebarski neravenstva....................................18 120

Geometriski neravenstva.................................23 130

Stepeni i koreni..................................................27 135


.
Geometriski figuri vo ramnina......................30 140

Linearni ravenki i neravenki.......................42 170

Teorija na broevi................................................45 172

Kompleksni broevi..............................................49 177

Kvadratni ravenki i kvadratni funkcii.....51 181

Plo{tina na ramninski figuri.......................54 183

Elementi od stereometrija...............................62 196

Eksponencijalni i logaritamski ravenki....67 209

Sistemi ravenki i neravenki........................69 210

Elementi od trigonometrija.............................75 215

Analiti~ka geometrija........................................81 228

Nizi i progresii..................................................86 236


..
Elementi od kombinatorika i verojatnost...93 244

Funkcii. Funkcionalni ravenki.....................101 254


Matemati~ka logika i mno`estva

Matemati~ka logika i mno`estva

1. Branko, Goran i Nikola se o`eneti so Daniela, Milena i Marija


(ne se znae koj so kogo). Sekoj par ima sin. Imiwata na sinovite se:
Daniel, Milan i Jovan. Opredeli gi imiwata na sekoe semejstvo,
ako se znae deka: а) sinovite na Milena i Nikola se zaqubeni vo
ista devojka; б) sinot na Branko ne e Milan; v) Marija ne e `ena na
Goran.
2. Neka K, M i P se `iteli na eden ostrov od la`govci i lu|e koi ja
zboruvat vistinata. Liceto K izjavilo: ,,nie site la`eme”, a liceto
M izjavilo: ,,eden od nas ja zboruva vistinata”. Opredeli koj od
`itelite K, M i P ja zboruva vistinata, a koj la`e?
3. Se sobrale trite papagali - Goki, Koki i Roki. Eden od niv
sekoga{ ja zboruval vistinata, drug sekoga{ la`el, a tretiot ili
la`el ili ja zboruval vistinata (mudrec). Na pra{aweto - koj e
Koki? - papagalite odgovorile:
Goki: Koki e la`go.
Koki: Jas sum mudrec.
Roki: Toj e apsolutno praveden papagal.
Opredeli koj od papagalite e ~esen, koj e la`go, a koj e mudrec?
4. Petar i Nikola razgovarale za godinite na sinovite na Petar.
Petar rekol deka ima tri sina, deka proizvodot na nivnite godini
e 36, a zbirot na godinite e ednakov na brojot na avtobusot koj
pominuva pokraj niv. Nikola seu{te ne mo`el da ja odredi nivnata
vozrast. Samo najstariot e crn kako mene – dodal Petar. So ova
informacija Nikola doznal po kolku godini imaat decata.
Opredeli po kolku godini imaat decata i koj e brojot na avtobusot?
5. Trojca u~enici, Ana, Marija i Olga, izbrale po eden od trite
predmeti: topka, {estar i lenir. Odredi koja u~eni~ka kakov
predmet izbrala, ako od trite tvrdewa:
a) Ana izbrala topka;
b) Marija ne izbrala topka;
v) Olga ne izbrala lenir;
edno e vistinito, a drugite dve se nevistiniti.
6. Vo edno u~ili{te ima 50 u~enici koi igraat ko{arka, fudbal i
tenis. Nekoi u~enici igraat pove}e od eden sport. Ako 15 u~enici
igraat tenis, 25 igraat ko{arka, 30 fudbal, 8 igraat tenis i
ko{arka, 5 igraat tenis i fudbal i 10 igraat ko{arka i fudbal,
toga{ odredi kolku u~enici gi igraat site tri sporta.
7. Vo eden klas, 25 u~enici u~at geografija, 17 u~at istorija i 21
u~enik u~at angliski jazik. Od tie 25 u~enici {to u~at geografija,
10 u~at i angliski jazik, a 7 u~at i istorija. Postojat 8 u~enici koi
u~at istorija i angliski jazik. To~no 3 u~enici gi slu{aat site tri

5
Matemati~ka logika i mno`estva

predmeti. Znaej}i deka sekoj u~enik od klasot posetuva barem eden


od ovie tri predmeti, opredeli go vkupniot broj na u~enici vo
klasot.
8. Angova, Ba~eva, Vasileva i Go{eva se ~etiri talentirani `eni.
Ednata od niv e balerina, drugata slikarka, tretata violinistka i
~etvrtata pisatelka. Za niv znaeme deka:
(1) Angova i Vasileva ne bile na koncertot na violinistkata;
(2) Ba~eva i pisatelkata zaedno i pozirale na slikarkata;
(3) Pisatelkata ja napi{ala biografijata na Go{eva i se
podgotvuva da napi{e biografija za Angova;
(4) Angova nikoga{ ne se videla so Vasileva.
Opredeli gi profesiite na sekoja od ~etirite `eni.
9. Dadeno e mno`estvoto A = {1,2,3...7,8} . Kolku petelementni
podmno`estva od mno`estvoto A imaat zbir na elementi pogolem
od 19?
10. Opredeli kolku (neprazni) podmno`estva od mno`estvoto
{1,2,3,...,12} go imaat svojstvoto zbirot na najmaliot i najgolemiot
element da e 13?
11. Kolku sedumelementni podmno`estva ima mno`estvoto
S={1,2,3, …,14}, takvi {to zbirot na nivnite elementi e deliv so
14? (Zbirot na elementite vo komplementot na takvoto podmno`estvo ne
e deliv so 14)
12. Neka A e mno`estvo od m posledovatelni celi broevo ~ij zbir e
2m, a B e mno`estvo od 2m posledovatelni celi broevi ~ij zbir e
m. Razlikata mu|u najgolemiot element na B i A e 99. Najdi go
brojot m.
13. Opredeli go brojot na trielementnite podmno`estva na
mno`estvoto {1,2,3,…,19,20}, takvi {to vo sekoe od niv proizvodot
na elementite e broj deliv so 4.
14. Neka S e podmno`estvo od mno`estvoto {1,2,…,100}. Opredeli go
maksimalniot broj elementi na podmno`estvoto S, za koi
proizvodot na koi bilo dva elementi ne e to~en kvadrat.
15. Kolku podmno`estva na mno`estvoto S={1,2,…,17} postojat koi ne
sodr`at dva posledovatelni broja?
16. Broevite 1 do 2018 se zapi{ani eden do drug. Kolku pati e
zapi{ana cifrata 9?
17. Prirodnite broevi od 1 do n se zapi{ani vo eden red, a pod niv se
zapi{ani istite broevi, no vo drug redosled. Dali mo`e zbirot na
sekoj broj i brojot zapi{an pod nego da e to~en kvadrat za
a) n=11; b) n=2010?
18. Dadena e nizata 1,10,11,100,101,110, … . Koj e 99 -tiot ~len vo
nizata?
19. Na tabla se napi{ani broevite 1, 1 , 1 , ..., 1 . Izbirame dva
2 3 2021
proizvolni broja i na nivnite mesta go zapi{uvame brojot a+b+aÿb.
Ovaa postapka se povtoruva 2020 pati, se dodeka na tablata ne
ostane napi{an samo eden ist broj. Koj e toj broj?

6
Matemati~ka logika i mno`estva

20. Prvite 2011 prirodni broevi se rasporedeni vo krug, pri {to se


bri{e sekoj vtor broj (2,4,...,2010,...). Toj proces prodol`uva se
dodeka ne se dobie samo eden neizbri{an broj. Koj e toj broj?
21. Site broevi od 1 do 1000 se zapi{ani eden do drug
(posledovatelno) vo krug. Zapo~nuvaj}i od 1, se precrtuva sekoj
15-ti broj (1,16,31,46,...), taka {to pri sekoe povtoruvawe se
precrtuvat i prethodno precrtanite broevi. Toj proces prodol`uva
se dodeka povtorno ne se precrta brojot 1. Opredeli kolku broja
ostanale neprecrtani.
22. Site prirodni broevi od 1 do 2n se zapi{ani vo niza a 1 , a 2 , ..., a 2n
taka {to a1 - a 2 + a 2 - a 3 + ... + a 2n-1 - a 2n + a 2n - a1 = 2n 2 . Doka`i deka
a 1 - a 2 + a 3 - a 4 + ... + a 2n -1 - a 2n = n 2 .
23. Neka S e mno`estvoto od site trojki (i,j,k) od pozitivni celi
broevi, za koi i+j+k=10. Presmetaj go zbirot od proizvodite na
sekoja takva trojka.
24. Vo temiwata na triagolnikot ABC se zapi{ani broevite 1, 2 i 3.
Potoa sekoj broj se zamenuva so zbirot na dva sosedni broja.
Operacijata se povtoruva kone~en broj pati. Dali mo`e zbirot na
trite dobieni broja, pri nekoja operacija da bide 3000?
25. 11-ti Juni 2009 godina e ~etvrtok. Koj den od nedelata e 11-ti
Avgust 2012 godina?
26. Doka`i deka od n+1 razli~ni pozitivni broevi pomali od 2n, mo`e
da se izberat tri broja, za koi zbirot na dva od niv e ednakov so
tretiot broj.
27. Na kolku nuli zavr{uva brojot
6 1 2 3 4 5
5 6 1 2 3 4
34 45 56 61 12 23
N = 12 ⋅ 23 ⋅ 34 ? ⋅ 45 ⋅ 56 ⋅ 61
28. Na tabla bile napi{ani dva broja. Petar sekoj den gi bri{el dvata
broja i na nivno mesto gi pi{uval broevite koi se aritmeti~ka i
harmoniska sredina na izbri{anite broevi. Ako dvata napi{ani
broja, prviot den, bile 1 i 2, toga{ presmetaj go proizvodot na
broevite zapi{ani vo 2023-tiot den.
29. Odredi go zbirot na prvite 500 prirodni broevi koi ne se delivi
so 4 ili 5.
30. Odredi go brojot na pojavuvawa na brojot 2 kako mno`itel vo 100!.
31. Dadeni se 555 tegovi so masa: 1g, 2g, …, 555g. Razdeli gi na tri
dela so ednakva masa.
32. Vo krug se postaveni 10 `elezni tegovi. Me|u bilo koj dva tega se
postaveni bronzeni medali ~ija masa e ednakva so razlikata od
masata na sosednite tegovi. Doka`i deka medalite mo`e da se
razdelat na dva dela so ista masa.
33. Srebreni moneti od 1, 2, 3 i 5 denari te`at soodvetno 1, 2, 3 i 5g.
Pome|u ~etirite moneti edna e neispravna. Kako mo`e so dve
merewa, na vaga bez tegovi da se opredeli neispravnata moneta?

7
Matemati~ka logika i mno`estva

34. Pet ma~ki, jadat 5 gluvci, za 5 minuti. Za kolku minuti, 3 ma~ki, }e


izedat 3 gluvci?
35. Petranka napravila vebsajt na koj taa prodavala univerzitetski
uniformi. Vo ponedelnik, taa prodala 13 trenerki od skopskiot i
9 od bitolskiot univerzitet, za vrednost od 370 denari. Vo
vtornik, prodala 9 trenerki od skopskiot i 2 od bitolskiot
univerzitet za vrednost od 180 denari. Vo sredata prodala 12
trenerki od skopskiot i 6 od bitolskiot. Ako Petranka ne ja
promenila cenata na proizvodite vo tekot na nedelata, toga{
kolku pari dobila vo sreda?
36. Dejan, rabotej}i na nekoi podatoci za gradskata bolnica, posakal
da se napie voda. No, koga se vratil, negoviot ekran od kompjuterot
bil blokiran. Toj zapomnil deka bil blizu do presmetuvawe na
rezultatot pred da gi memorira podatocite. Go presmetal
proizvodot na tri prirodni broevi a, b i c koj iznesuval 24, no ne
gi zapomnil nivnite vrednosti. Ona {to Dejan trebal da go
presmeta e zbirot na broevite a, b i c. Dejan znae deka zbirot e
paren dvocifren broj pomal od 25 i so pomalku od 6 deliteli.
Pomogni mu na Dejan da go opredeli zbirot a+b+c.
37. Ivan ima 2n listovi hartija numerirani od 1 do 2n. Toj izvadil n
liv~iwa numerirani so posledovatelni broevi. Zbirot na broevite
napi{ani na liv~iwata {to ostanale bil 1615. Opredeli gi site
mo`ni vrednosti na n.
38. Na eden ostrov 2/3 od site ma`i bile `eneti i 3/5 od site `eni
bile ma`eni. Ako vkupniot broj `iteli na ostrovot bil 1900, toga{
kolku od niv ne bile vo brak?
39. Eden ~ovek imal tri sina. Toj mu rekol na popi{uva~ot, koj do{ol
na negovata vrata deka proizvodot od godinite na negovite sinovi
e 72, a zbirot na nivnite godini e ednakov so brojot na nivnata
ku}a. Popi{uva~ot, poglednuvaj}i vo brojot na ku}ata rekol, „mene
mi se potrebni u{te informacii“. ^ovekot odgovoril -„se
razbira“-, jas zaboraviv da ti ka`am deka moeto najstaro dete
mnogu saka pita od jagodi. Toga{ popi{uva~ot gi napi{al godinite
na sinovite. Koj e brojot na ku}ata?
40. Tatko vo negoviot testament, gi ostavil site svoi pari na negovite
deca po sledniot redosled:
- $1000 na prvorodeniot i 1/10 od toa {to ostanalo;
- $2000 na vtororodeniot i 1/10 od toa {to ostanalo;
- $3000 na tretorodeniot i 1/10 od toa {to ostanalo i na istiot
na~in za ostanatite deca. Koga ova bilo napraveno, sekoe dete
imalo ist iznos. Kolku deca bile?
41. U~enicite Aleksandar, Boris i Vlatko re{avat matemati~ki test
od 100 zada~i. Sekoj od niv re{il po 60 zada~i. Ako zada~ite {to
gi re{il samo eden u~enik gi smetame za „te{ki“, a zada~ite {to gi
re{ile site trojca u~enici za „lesni“, toga{ za kolku brojot na
te{ki zada~i e pogolem od brojot na lesni zada~i.

8
Matemati~ka logika i mno`estva

42. 5 ku}i se oboeni vo 5 razli~ni boi. Vo sekoja od ku}ite `ivee


li~nost od razli~na nacionalnost. Sopstvenici pijat razli~en
pijalok, svirat na razli~en instrument i odgleduvat razli~ni
doma{ni `ivotni. Vrz osnova na dolu dadenite podatoci odgovori
na pra{aweto: Koj ~uva ribi?
Poznato e deka: angli~anecot `ivee vo crvenata ku}a; {ve|anecot
odgleduva ku~iwa; danecot pie ~aj; zelenata ku}a se nao|a levo od
belata ku}a; sopstvenikot na zelenata ku}a pie kafe; li~nosta koja
svira violina odgleduva ptici; sopstvenikot na `oltata ku}a
sviri klavir; sopstvenikot na srednata ku}a pie mleko;
norve`aninot `ivee vo prvata ku}a; ~ovekot koj sviri truba `ivee
do onoj koj odgleduva ma~iwa; ~ovekot koj odgleduva kowi e sosed
so ~ovekot koj sviri klavir; sopstvenikot koj sviri harmonika pie
pivo; germanecot sviri na gitara; norve`aninot `ivee pokraj
sinata ku}a; ~ovekot koj sviri na truba ima sosed koj pie voda.
Postoi edinstveno re{enie koe gi zadovoluva gorenavedenite
uslovi. (Zada~a na Ajn{tajn))
43. Dadeni se 101 prirodni broevi ~ij zbir e 200. Doka`i deka od tie
broevi sekoga{ mo`e da se izberat nekolku ~ii zbir }e bide 100.
44. Dadeni se 10 otse~ki, pri {to e poznato deka sekoja od niv e so
dol`ina cel broj. Ako dve od niv se so najmala dol`ina 1, a
najdolgata e so dol`ina 50, doka`i deka me|u otse~kite postojat
tri od koi mo`e da se konstruira triagolnik.
45. Sumata na pet broevi e 10. Doka`i deka me|u niv postojat dva
broja ~ij zbir e najmalku 4.
46. Dadeni se dvaeset prirodni broevi takvi {to 1§a1§a2§…§a20§70.
Doka`i deka pome|u razlikite ai+1-ai (i=1,2,…,19) postojat barem
~etiri ednakvi me|usebe.
47. Eden u~enik zamislil 5 pozitivni broja a, b, c, d, e i gi soop{til na
drug u~enik site mo`ni zbirovi a+b, a+c, ..., d+e. Kako od tie 10
broevi vtoriot u~enik mo`e da gi odredi zamislenite broevi?

9
Osnovni brojni mno`estva

Osnovni brojni mno`estva

48. Presmetaj ja vrednosta na izrazot


a) 1+2-3-4+5+6-7-8+...+2006-2007-2008+2009+2010-2011;
b) (1002-992)+(982-972)+…+(22-12).
49. Proizvodot od cifrite na tricifren broj e 6 pati pogolem od
nivniot zbir. Cifrata na edinicite e zbir od drugite dve cifri.
Presmetaj go zbirot na site takvi tricifreni broevi.
50. Presmetaj 1 + 1 + 1
+ ... +
1
+
1 .
1+ 2 1+ 2 + 3 1 + 2 + ... + 2018 1 + 2 + ... + 2018 + 2019
1
 1⋅ 2 ⋅ 4 + 2 ⋅ 4 ⋅ 8 + ... + n ⋅ 2n ⋅ 4n  3
51. Presmetaj ja vrednosta na izrazot   .
 1⋅ 3 ⋅ 9 + 2 ⋅ 6 ⋅ 18 + ... + n ⋅ 3n ⋅ 9n 
52. Kolku nuli sodr`i brojot 1000!?
53. Doka`i deka 21n + 4 e neskratliva dropka za sekoj priroden broj n .
14n + 3
54. Najdi go najmaliot priroden broj n za koj 2n − 685 e skratliva
3n + 643
dropka.
55. Odredi gi site celi broevi n za koi brojot 19n + 7 e cel broj.
7n + 11
56. Neka S=1+2+3+...+2017. Odredi go ostatokot {to se dobiva koga S
se deli so 1000.
57. Re{i ja vo mno`estvoto prirodni broevi ravenkata a+b+ab=2012.
1 1 1 2
58. Vo mno`estvoto prirodni broevi re{i ja ravenkata − + = .
x y xy 5
59. Vo mno`estvoto prirodni broevi re{i ja ravenkata
(x+y+z)2=x3+y3+z3 , ako x<y<z.
60. Za trojkata prirodni broevi (x,y,z) e poznato deka zbirot na koi
bilo dva se deli so tretiot. Odredi gi site takvi trojki.
61. Ako a, b i c se neparni broevi toga{ barem eden od broevite ab-1,
bc-1, ca-1 e deliv so 4. Doka`i.
62. Vo mno`estvoto prirodnite broevi re{i ja ravenkata 1 + 2 − 3 = 1 .
x y z
63. Odredi go najgolemiot priroden broj koj e ednakov so kubot od
zbirot na negovite cifri.
64. Neka B e zbirot od cifrite na brojot A=44444444 , a C e zbirot od
cifrite na brojot B. Odredi go zbirot od cifrite na brojot C.

10
Osnovni brojni mno`estva

65. Odredi go najgolemiot priroden broj so svojstvo na negovite cifri


da se pomali od aritmeti~kata sredina na nivnite sosedni cifri
(osven za prvata i poslednata cifra).
66. Odredi gi site prirodni broevi n za koi va`i ravenstvoto
1
= 0, abcabcabc. .. kade a, b i c se tri razli~ni cifri.
n
67. Vo mno`estvoto na prirodnite broevi re{i ja ravenkata
2x2y2+2y2z2+2z2x2-x4-y4-z4=576.
68. Odredi gi site celi broevi a i b, koi {to se zaemno prosti i za koi
a+b 3
va`i 2 = .
а − аb + b 2
13
69. Doka`i deka pome|u proizvolni deset posledovatelni prirodni
broevi postoi broj, zaemno prost so ostanatite.
70. Neka а, b i c se me|usebno zaemno prosti prirodni broevi. Odredi
(a + b )(b + c )(c + a )
gi site mo`ni vrednosti na izrazot , ako e poznato
abc
deka toj e cel broj.
71. Ako a i b se zaemno prosti toga{ postoi priroden broj k, takov {to
ak-1 e deliv so b. Doka`i.
72. Najdi gi site trojki posledovatelni neparni prirodni broevi ~ij
zbir od kvadrati e ednakov so ~etiricifren broj so ednakvi
cifri.
73. Odredi gi site prirodni broevi a, b i c (a§b§c) za koi brojot
1 1 1
+ + e priroden broj.
a b c
74. Poznato e deka zbirot od cifrite na prirodniot broj N e 100, a
zbirot od cifrite na brojot 5N e 50. Doka`i deka N e paren broj.
75. Presmetaj NZD (22022 -1; 22020-1).
1 1 1
76. Neka a,b,cœN za koi va`i + = . Ako NZD(a,b,c)=d, toga{
a b c
proizvodot abcd e poln kvadrat.
77. Presmetaj NZD( 2020+2,20202+2,20203+2,...);
78. Za prirodnite broevi m i n va`i НЗС(m,n)+НЗД(m,n)=m+n. Doka`i
deka edniot od broevite e delitel na drugiot.
79. Odredi gi site celi broevi x i y koi se re{enie na ravenkata
x 2 − y 2 = 133 .
80. Re{i ja vo mno`estvoto celi broevi ravenkata 2x-1=5y.
81. Poznato e deka a-b+2010, b-c+2010 i c-a+2010 se tri
posledovatelni celi broevi. Najdi gi tie broevi.
82. Ako za celite broevi x, y i z va`i x 3 + y 3 = z 3 , doka`i deka barem
eden od niv e deliv so 3.
83. Doka`i deka ne postojat neparni celi broevi x, y, z za koi va`i
ravenstvoto (x+y)2+(x+z)2=(y+z)2.

11
Osnovni brojni mno`estva

84. Ako dol`inite na katetite na eden pravoagolen triagolnik se


kvadrati na prirodni broevi, doka`i deka dol`inata na
hipotenuzata ne e priroden broj.
85. Odredi go brojot na podredeni dvojki od prirodni broevi (a,b) za
1 1 1
koi va`i a<b i + = .
a b 2014
86. Najdi gi site prirodni broevi n za koi proizvodot n(n + 16) e to~en
kvadrat.
87. Neka a, b i c se proizvolni realni broevi. Doka`i deka barem eden
od broevite (a+b+c)2-9ab, (a+b+c)2-9bc, (a+b+c)2-9ca e nenegativen.
88. Re{i ja vo mno`estvoto celi broevi ravenkata 1!+2!+3!+...+x!=y2.
89. Doka`i deka racionalnite koreni na ravenkata so celi
koeficienti x n + a 1x n-1 + ...a n-1x + a n = 0 , se celi broevi.
90. Presmetaj ja vrednosta na izrazot
1 1
a) A = 1 + b) B = 1 +
1 1
1+ 2+
1 1
1+ 2+
1 + ... 2 + ...
1
91. Neka a i b se realni broevi za koi va`i a = b + i
1
a+
1
b+
a + ...
1 . Odredi ja vrednosta na izrazot a2+b2.
b=a-
1
b+
1
a-
b + ...
92. Odredi gi site parovi realni broevi (x,y) za koi
1
(2x+1)²+y²+(y–2x)²=
3.
93. Re{i ja vo R ravenkata (x2+3x-4)3+(2x2-5x+3)3=(3x2-2x-1)3.
94. Neka a1, a2, ..., a2019 ; b1, b2, ..., b2019 ; p i q se realni broevi razli~ni od 0
za koi va`i:
1) a 12 + a22 + ... + a20192 = p2
2) b12 + b22 + ... + b20192 = q2
3) a1b1 + a2b2 + ... + a2019b2019 = p ÿ q
Doka`i deka a1 = a 2 = ... = a 2019 .
b1 b 2 b 2019

95. Neka a, b, c, d se realni broevi od intervalot [0,1]. Doka`i deka


postoi broj xœ[0,1] za koj va`i neravenstvoto
1 1 1 1
+ + + < 40.
x-a x -b x-c x-d

12
Algebarski racionalni izrazi

Algebarski racionalni izrazi

96. Razlo`i go na mno`iteli polinomot 2x4+x3+4x2+x+2.


97. Razlo`i go na mno`iteli polinomot a5+a4+a3+a2+a.
98. Razlo`i go na mno`iteli polinomot a3+b3+c3-3abc
99. Razlo`i go na mno`iteli polinomot (x+1)(x+3)(x+5)(x+7)+15.
100. Izrazot (ab+ac+bc)·(a+b+c)-abc razlo`i go na mno`iteli.
101. Doka`i gi ravenstvata:
a) (a-b)3+(b-c)3+(c-a)3=3(a-b)(b-c)(c-a);
b) a(b2+c2)+b(c2+a2)+c(a2+b2)+2abc=(a+b)(b+c)(c+a).
102. Doka`i gi ravenstvata:
1
a) a3+b3+c3-3abc= (a+b+c)·[(a-b)2+(b-c)2+(c-a)2];
2
b) (a+b+c) -a -b -c3=3(a+b)(b+c)(c+a).
3 3 3

103. Doka`i deka polinomot x44+x33+x22+x11+1 e deliv so polinomot


x4+x3+x2+x+1.
104. Za polinomot Р(x) va`i P(1)=3 i Р(2)=5. Odredi go polinomot od prv
stepen {to se dobiva kako ostatok pri deleweto na Р(x) so
(x-1)(x-2).
1
105. Daden e polinomot Р(х) od {estti stepen, za koj Р(n) = za
n
n =1, 2, 3, ..., 7. Presmetaj Р(8).
106. Odredi go koeficentot pred x² vo polinomot
P(x)=(1-x)·(1+2x)·(1-3x)·...·(1+14x)·(1-15x).
107. Ako xy=3 i x2y+xy2+2x+2y=25 toga{ odredi go x2+y2.
108. Doka`i deka brojot od vidot 8n+7 ne mo`e da se pretstavi kako
zbir na tri polni kvadrati.
109. Ako x+y=5 i xy=1, toga{ opredeli ja vrednosta na izrazot x3+y3.
110. Neka x i y se broevi za koj x3+y3+(x+y)3+30xy=2000. Odredi ja
vrednosta na zbirot x+y.
1 1 1
111. Neka xyz=1 i a = x + , b = y + , c = z + . Odredi ja vrednosta na
x y z
izrazot a2+b2+c2-abc.
1 3 1
112. Ako a + = 6 , toga{ presmetaj ja vrednosta na izrazot a + 3 .
a a
113. Ako x4+y4+z4+x2y2+x2z2+y2z2=2xyz(x+y+z), toga{ x=y=z. Doka`i.

13
Algebarski racionalni izrazi

a3 b3 c 3
114. Presmetaj ja vrednosta na izrazot + + , ako abc∫0 i
b3 c 3 a3
a b c a c b
+ + =4, + + = 5.
b c a c b a
115. Ako a+b+c=3, ab+ac+bc=4 i abc=5, toga{ odredi ja vrednosta na
1 1 1
izrazot 2 + 2 + 2 .
a b c
116. Ako x+y+z=3, x2+y2+z2=1 i x3+y3+z3=32 toga{ presmetaj ja vrednosta
na izrazot xyz.
117. Ako x+y+z=0 i x2+y2+z2=1, presmetaj kolku e x4+y4+z4.
118. Neka a, b, c se pozitivni realni broevi za koi a+b+c=1 i
1
ab+bc+ca= . Najdi ja vrednosta na izrazot:
3
a b c a b c
a) + + ; b) + + .
b c a b +1 c +1 a +1
119. Ako za broevite x i y e poznato deka x + y + x − y = 2 , toga{
x−y x+y
4 4 4 4
x +y x −y
presemetaj ja vrednosta na izrazot 4 4
+ 4 4
.
x −y x +y
120. Odredi ja najmalata vrednost na polinomot
P(x)=(x-1)(x-2)(x-3)(x-4)+10 , kade xœR .
121. Daden e polinomot P(x)=(x-2)·(2x-3)2·(3x-4)3·…·(2013x-2014)2013.
Odredi go zbirot na koeficientite na polinomot P(x).
122. Neka P(x)=1+x+x2+...+xn-1+xn i neka x1, x2, ..., xn se koreni na
polinomot P(x). Opredeli ja vrednosta na izrazot
(1-x1)(1-x2)·...·(1-xn).
123. Neka P(x ) = a n x n + a n-1x n-1 + ... + a 1x + a 0 e polinom so celi koeficenti.
Neka a, b, c, d se razli~ni celi broevi takvi {to
P(a) = P(b) = P(c ) = P(d) = 4 . Doka`i deka ne postoi cel broj m za koj
P(m ) = 7 .
124. Odredi go ostatokot {to se dobiva koga polinomot
P(x)=1-2x+3x2-4x3+...-100x99 se deli so polinomot Q(x)=x2-1.
125. Neka P(x) e polinom so celobrojni koeficienti, za koj rastojanieto
me|u dve to~ki od negoviot grafik koi imaat celobrojni
koordinati e cel broj. Doka`i deka otse~kata koja gi povrzuva tie
dve to~ki e paralelna so x-oskata.
126. Odredi gi site polinomi P(x) so realni koeficienti takvi {to za
sekoj realen broj x va`i ravenstvoto (1+2x)·P(2x)=(1+22019)·P(x).

14
Algebarski racionalni izrazi

127. Za broevite x, y, z i k se ispolneti ravenstvata 7 k 11 .


= =
x+y x+z z−y
Opredeli ja vrednost na brojot k.
(x + y + z)(xy + yz + zx)
128. Odredi ja najmalata mo`na vrednost na izrazot
xyz
ako x, y, z se pozitivni realni broevi.
129. Neka x, y, z se tri razli~ni realni broevi za koi va`at
ravenstvata y = x + y = x . Odredi ja vrednosta na x .
x−z z y y
a b c a 2
b 2
c 2
130. Ako + + = 1, toga{ + + = 0 . Doka`i.
b+c c +a a+b b+c c +a a+b
ay − bx cx − az bz − cy x y z
131. Ako = = , toga{ = = . Doka`i.
c b a a b c
2 2 2
132. Za x, y, z œ R i x +y +z =xy+xz+yz doka`i deka x=y=z.
133. Ako x3+y3+z3=3xyz , toga{ x+y+z=0 ili x=y=z. Doka`i.
x y z 2 2 2
134. Ako a = b = c , a + b + c = 1 i a +b +c =1 toga{ xy+xz+yz=0. Doka`i.
135. Ako za razli~nite realni broevi a, b, c va`i a3+pa+q=0, b3+pb+q=0,
c3+pc+q=0 toga{ a+b+c=0. Doka`i.
a b c 1 1 1 3
136. Ako + + = 0 toga{ + + = . Doka`i.
b+c c +a a+b b+c c +a a+b a+b+c
137. Ako a + bx = b + cx = c + ax toga{ a3+b3+c3=3abc. Doka`i.
b + cy c + ay a + by
2 2 2
138. Neka a, b, c, x, y, z se broevi za koi va`i y + yz + z = a ,
z 2 + zx + x 2 = b 2 , x 2 + xy + y 2 = c 2 i xy + yz + zx = 0 . Doka`i deka
(a + b + c) ⋅ (a + b - c) ⋅ (a - b + c) ⋅ (a - b - c) = 0 .
139. Broevite a, b, c se razli~ni od nula i za niv se to~ni ravenstvata:
b c a
a+ =b+ =c+ =1. Doka`i deka ab+bc+ca=0.
c a b
a b c
140. Ако + + = 1 toga{ a3 + b3 + c 3 + abc = 0 . Doka`i.
b +c c +a a +b
141. Broevite a, b, c i d se takvi {to a+b=c+d i a2+b2=c2+d2. Doka`i
deka a3+b3=c3+d3.
a-b b-c c -a
142. Dokaæi deka zbirot na dropkite ; ; e ednakov so
1+ ab 1 + bc 1+ ac
nivniot proizvod.
x y z a b c 2 2 2
143. Ako + + = 0 i + + = 1 , toga{ a + b + c = 1 . Doka`i.
a b c x y z x2 y2 z2

15
Algebarski racionalni izrazi

a2 + b2 + c 2 1
144. Ako a+b+c=0 toga{ 2 2 2
= . Doka`i.
(a - b) + (b - c) + (c - a) 3
2 2 2 2 n n n n
145. Ako x+y=a+b i x + y = a + b , toga{ x + y = a + b , "nœN. Doka`i.
x 2 y 2 z 2 x 3 z3 y 3
146. Za pozitivnite broevi x, y, z va`i + + = + + .
y z x z y x
Doka`i deka x = y ili x = z ili x = z .
147. Ako n e neparen priroden broj i ako vaæi ravenstvoto
1 1 1 1 1 1 1 1
+ + = , toga{ n + n + n = n n n .
x y z x+y+z x y z x +y +z
1 1 1
148. Ako xyz = 1 , toga{ va`i + + = 1.
1 + x + xy 1 + y + yz 1 + z + zx
149. Broevite a, b, c se realni i so isti znaci. Doka`i deka ako
a4+b4+c4+d4 =4abcd toga{ a=b=c=d.
150. Ako (a+b+c)2=3(ab+ac+bc-x2-y2-z2) toga{ a=b=c i x=y=z=0. Doka`i.
151. Za prirodnite broevi a, b, c i d e ispolneto ravenstvoto
a b ab + 1
= = . Doka`i deka a = c i b = d.
c d cd + 1
1 1 1 ax3 + by3 + cz3 3
152. Ako ax3=by3=cz3 i + + = 1, toga{ 3 = a +3 b +3 c .
x y z 3
Doka`i.
153. Ako m, n, p, a, b, c se pozitivni realni broevi za koi va`i
m:a=n:b=p:c, toga{ (ma)1/2+(nb)1/2+(pc)1/2=[(m+n+p)(a+b+c)]1/2. Doka`i.
154. Ako za realnite broevi x, y i z e to~no ravenstvo
(x-y)2+(y-z)2+(z-x)2=(x+y-2z)2+(y+z-2x)2+(z+x-2y)2, toga{ x=y=z.
Doka`i.
a b c a b c
155. Ako + + = 0 , toga{ + + = 0.
b −c c −a a −b (b − c ) (c − a ) (a − b )2
2 2

Doka`i.
a3 b3 c3
156. Uprosti go izrazot + + .
(a - b )(a - c ) (b - a)(b - c ) (c - a )(c - b )
157. Ako dva broja mo`e da se zapi{at kako zbir od kvadratite na dva
broja toga{ i nivniot proizvod mo`e da se zapi{e kako zbir od
kvadrati na dva broja. Doka`i!
158. Ako za broevite a, b i c se ispolneti neravenstvata |a-b|¥|c|,
|b-c|¥|a|, |c-a|¥|b| toga{ edniot od broevite e ednakov so zbirot na
drugite dva broja.
159. Neka a1, a2, …, an se razli~ni prirodni broevi. Doka`i deka
proizvodot (a12+1)·(a22+1)·…·(an2+1) mo`e da se pretstavi kako zbir
na kvadrati na dva prirodni broja.

16
Algebarski racionalni izrazi

160. Za prirodnite broevi a, b i c va`i ravenstvoto


a2+b2+c2=(a-b)2+(b-c)2+(c-a)2. Doka`i deka broevite ab, bc, ca i
ab+bc+ca se polni kvadrati.
161. Prirodnite broevi x i y se takvi {to zbirot xy+x+y e poln kvadrat
od cel broj. Doka`i deka postoi priroden broj z, taka {to sekoj od
zbirovite xy+z, xz+y i yz+x se polni kvadrati na celi broevi.
 x2 y2  x y
162. Doka`i deka izrazot A =  2 + 2  − 24 +  + 34 e poln kvadrat.
y x  y x
163. Za pozitivnite realni broevi a, b i c va`i ravenstvoto a+b+c=6.
Presmetaj ja maksimalnata mo`na vrednost na izrazot
a bc + b ac + c ab .
164. Neka а, b, с se pozitivni realni broevi i а+b+с=1. Odredi ja
1  1  1 
najmalata vrednost na izrazot  - 1 - 1 - 1 .
a  b  c 
2 2 2
165. Ako x -yz=a, y -zx=b, z -xy=c, toga{ ax+by+cz=(a+b+c)(x+y+z).
Doka`i.
166. Opredeli gi site realni vrednosti na promenlivite x i y za koi
x2020+y2020=x2021+y2021=x2022+y2022.

17
Geometriski neravenstva

Algebarski neravenstva

167. Dokaæi deka va`i neravenstvoto


1 2 2018
+ + ... + < 1.
2! 3! 2019!
1 3 5 99 1
168. Doka`i deka va`i neravenstvoto ⋅ ⋅ ⋅ ... ⋅ < .
2 4 6 100 10
169. Doka`i deka za sekoj priroden broj n va`at neravenstvata
1 1 1 2
a) + + ..... + 2 < ;
22 3 2 n 3
1 1 1 1 1
b) 3 + 3 + 3 + ... + 3 < .
2 3 4 n 4
1  1 1 1  1  1 1 1 
170. Ako a = 1 + + + ... +  i b= 1 + + + ... +  toga{
2019  2 3 2019  2018  2 3 2018 
a<b. Doka`i.
1  1
171. Ako x>0, y>0 i x+y=1 toga{  1 +  1 +  ≥ 9 . Doka`i.
 x  y
172. Za pozitivnite realni broevi x, y i z va`i x+y+z=1. Doka`i deka
 1  1  1 
1+ 1+ 1+  ≥ 64 .
 x  y  z 
173. Doka`i deka a1 + a2 + …+ an ≥ n ⋅ n (a1a2 ….an ) ( ai > 0).
(Neravenstvo na Ko{i)
174. Ako a>0, b>0, c>0 toga{ (a + b - c )(b + c - a )(c + a - b ) ≤ abc . Doka`i.
175. Doka`i go neravenstvoto
(a1b1+a2b2+…anbn)2≤(a12+a22+a32+…+an2)(b12+b22+…+bn2), kade {to ai i bi
se proizvolni realni broevi. Koga va`i znakot za ravenstvo?
(Neravenstvo na Ko{i- -Bunjakovski--[varc)
176. Zbirot na najgolemiot i najmaliot ~len (pozitivni ~lenovi) na
edna geometriska progresija e pogolem od zbirot na drugite dva
~lena. Doka`i. (Neravenstvo na Evklid)
177. Ako a>0, b>0, c>0 toga{ (a+b)(b+c)(c+a)>8abc. Doka`i.
178. Neka a, b i c se pozitivni realni broevi. Doka`i deka va`i
a2 b2 c 2
+ + ≥ a+b+c.
b c a
179. Neka a, b i c se pozitivni realni broevi. Doka`i deka
 a 1  b 1  c 1
 +  +  +  ≥ 1.
 b + c 2  a + c 2  a + b 2 

18
Geometriski neravenstva

180. Doka`i deka za pozitivni realni broevi a, b i c va`i


a2 b2 c2 3
neravenstvoto + + ≥ .
(a + b)(a + c) (b + c)(b + a) (c + a)(c + b) 4
181. Doka`i deka za proizvolni pozitivni realni broevi a, b i c va`at
slednite neravenstva:
a) (a+b+c)2¥3(ab+ac+bc);
b) a2(1+b2)+b2(1+c2)+c2(1+a2)¥6abc
v) a3+b3+c3¥a2b+b2c+c2a.
1 1 1 9
182. Ako a>0, b>0 , c>0 toga{ + + ≥ . Doka`i.
a b c a+b+c
3
( )
183. Ako a>0, b>0 ,c>0 toga{ 27abc § (a + b + c) § 9 a + b + c . Doka`i.
3 3 3

184. Neka a, b, c se pozitivni realni broevi za koi va`i a+b+c=1.


a3 + b3 + c 3 1
Doka`i deka va`i neravenstvoto 2 2 2
≥ .
a +b +c 3
185. Za pozitivnite broevi a, b i c va`i abc=1. Doka`i deka
1 1 1
+ + ≤ 1.
1+ a + b 1+ b + c 1+ c + a
186. Ako a, b i c se pozitivni realni broevi toga{
1 1 1 1 1 1
+ + ≥ + + . Doka`i.
a b c ab bc ca
187. Neka a, b i с se pozitivni realni broevi za koi va`i abc=1. Doka`i
ab bc ca
deka 5 5 + 5 5 + 5 5 ≤ 1.
a + b + ab b + c + bc c + a + ca
188. Doka`i deka za proizvolni pozitivni realni broevi a, b i c va`i
a−b b−c c −a
neravenstvoto + + < 1.
a+b b+c c+a
189. Ako a i b se pozitivni realni broevi, doka`i go neravenstvoto
a3+b3+2¥2ab+a+b.
190. Neka za pozitivnite realni broevi a, b i c va`i a+b+c=1. Doka`i gi
slednite neravenstva:
ab bc ca 1 a b c
a) + + ≤ ; b) + + ≥ 9.
a+b b+c c +a 2 bc ca ab
191. Ako a, b i c se pozitivni realni broevi za koi va`i a+b+c=1 toga{
a2 b2 c2 1
+ + ≥ . Doka`i.
a+c b+a c +b 2
192. Neka a, b, c se pozitivni realni broevi za koi va`i a+b+c=1.
3 3 3
Doka`i deka va`i neravenstvoto 2 a 2 + 2 b 2 + 2 c 2 ≥ 1 .
a +b b +c c +a 2

19
Geometriski neravenstva

193. Neka a, b i c se pozitivni realni broevi za koi a+b+c=1. Doka`i go


1 1 1 2 2 2
neravenstvoto + + ≥ + + .
1− a 1− b 1− c 1+ a 1+ b 1+ c
1 1 1
194. Ako x, y, z se pozitivni realni broevi i + + = 1 toga{
x y z
(x-1)(y-1)(z-1) ≥ 8. Doka`i.
195. Neka x, y i z se pozitivni realni broevi za koi va`i xyz=1. Doka`i
x −1 y −1 z −1
deka + + ≥0.
y +1 z +1 x +1
196. Neka x, y i z se pozitivni realni broevi. Doka`i deka va`i
 1 1 1
( )
neravenstvoto x 2 + y 2 + z 2 ⋅  + +  ≥ 3 ⋅ (x + y + z ) .
x y z
197. Neka x, y i z se realni broevi za koi va`i 0§x§1, 0§y§1, 0§z§1 i
x+y+z=2. Doka`i deka 1§ x + y + z §2.
x +1 y +1 z +1
198. Neka x, y i z se pozitivni realni broevi za koi va`i x+y+z=3.
Doka`i go neravenstvoto x + y + z ≥ xy + xz + yz.
199. Neka x, y i z se pozitivni realni broevi takvi {to xyz=1. Ako
1 1 1 1 1 1
+ + ≥ x + y + z toga{ 3 + 3 + 3 ≥ x3 + y3 + z3 . Doka`i.
x y z x y z
200. Ako x, y se realni broevi toga{ va`i (x2 +y2)2>xy(x+y)2. Doka`i.
201. Ako x>0 toga{ x12-x9+x4-x+1>0 . Doka`i.
202. Doka`i deka za site realni broevi x i y va`i neravenstvoto
x6 y6
+ ≥ x4 + y4 .
y2 x2
203. Ako x, y i z se razli~ni realni broevi toga{
6 6 6 2 2 2 3 3 3 3 3 3
x +y +z +3x y z ¥2(x y +x z +y z ). Doka`i.
204. Za pozitivnite realni broevi a, b i c va`i a2 + b2 - ab = c2 . Doka`i
deka (a-c)ÿ(b-c)<0.
205. Neka a, b i c se pozitivni realni broevi takvi {to a 2 + b 2 + c 2 = 1 .
Doka`i deka a(b + c ) § 2 .
2
206. Dokaæi deka za proizvolni realni broevi a, b, c vaæi
neravenstvoto a2 + b2 + c 2 + 32 ≥ 2(a + b + c + 3) .
207. Neka a, b i c se realni broevi takvi {to 0<a§b§c. Doka`i deka
va`i neravenstvoto (a + 3b )(b + 4c )(c + 2a ) ¥ 60abc . Koga va`i znakot
za ravenstvo?

20
Geometriski neravenstva

208. Neka a, b i c se pozitivni realni broevi za koi va`i a2+b2+c2=3.


1 1 1
Doka`i deka + + ≥ 1.
1 + 2ab 1 + 2bc 1 + 2ca
209. Doka`i deka za proizvolni realni broevi a, b i c va`i
neravenstvoto a 4 + b 4 + c 4 ¥ abc (a + b + c ) .
210. Neka a, b i c se pozitivni realni broevi za koi va`i
1 1 1
a + b + c ≥ + + . Doka`i deka va`i a3+b3+c3¥a+b+c .
a b c
211. Ako za pozitivnite realni broevi a, b i c va`i aÿbÿc=1 toga{
 1  1  1
doka`i go neravenstvoto  a - 1 +  b - 1 +  c − 1 +  ≤ 1 .
 b  c  c
212. Ako a, b, c i d se pozitivni realni broevi toga{ va`i
1 1 1
neravenstvoto + ≤ . Doka`i.
1 1 1 1 1 1
+ + +
a b c d a+c b+d
213. Realnite broevi x i y pripa|aat na segmentot [0,1]. Doka`i deka
va`i x5 +y5 + (x-y)5 § 2.
214. Neka x, y i z se pozitivni realni broevi za koi va`i xyz=1. Doka`i
x9 + y9 y9 + z9 z9 + x9
deka S = 6 + + ≥ 2.
x + x3 y3 + y 6 y 6 + x3 y3 + z6 z6 + z3 x3 + x 6
215. Ako x, y i z se razli~ni realni broevi, doka`i go neravenstvoto
IxI+IyI+IzI§Ix+y-zI+Ix-y+zI+I-x+y+zI.
n
 n + 1
216. Za sekoj priroden broj n>1 va`i n! <   . Doka`i.
 2 
217. Neka a, b i c se pozitivni realni broevi i n e nenegativen cel broj.
Dokaæi deka vaæi neravenstvoto an+2 + bn+2 + c n+2 ≥ anbc + bnca + c nab.
218. Doka`i deka ako 0<a≤b≤c toga{ ac-a·ba-b·cb-c≤1.
219. Neka x i y se nenegativni realni broevi. Doka`i deka
4(x9+y9)¥(x2+y2)·(x3+y3)·(x4+y4).
220. Neka x1, x2, …, xn se pozitivni realni broevi. Doka`i deka
2 2 2
 x   x   x 
 1 + 1  +  1 + 2  + ... +  1 + n  ≥ 4n .
 x2   x3   xn 
221. Doka`i deka za sekoi x,y,z œ R va`i
 3(x − y )2 3(y − z)2 3(z − x )2 
x 2 + y 2 + z2 − xy − yz − zx ≥ max , , .
 4 4 4 
222. Neka a,b,cœR. Doka`i deka va`i neravenstvoto
1
[ 2 2 2
] (
min (a − b ) , (b − c ) , (c − a ) ≤ a 2 + b 2 + c 2 .
2
)

21
Geometriski neravenstva

223. Doka`i deka za p¥0 va`i


p
1−20132p 1−20132013p
(2013 ) p 1−2013
+ 20132p ( )
+ ... + 20132013p ( ) ≤ 2013 .
224. Ako 1<a<b<c toga{ loga(logab)+logb(logbc)+logc(logca)>0. Doka`i.
225. Neka a, b i c se realni broevi pogolemi od 1. Doka`i go
b2 c2 a2
neravenstvoto loga( − b+ac)ÿlogb( − c+ab)ÿlogc( − a+bc)≥1.
a⋅c a⋅b b⋅c
226. Neka a, b i c se realni broevi pogolemi od 1.
Doka`i deka za sekoj realen broj r va`i neravenstvoto
(log a bc )r + (log b ca )r + (log c ab )r ≥ 3 ⋅ 2 r .
227. Doka`i deka ako a 1 ⋅ a 2 ⋅ ... ⋅ a n = 1, a i > 0, i = 1, 2, ..., n toga{
a1 + a2 + ... + an ≤ a1 + a 2 + ... + an .
228. Neka a1, a2 ,..., an i b1, b2, ..., bn se nenegativni realni broevi.
Doka`i deka va`i n a 1a 2 ...a n + n b 1b 2 ...b n ≤ n (a 1 + b 1 )(a 2 + b 2 )...(a n + b n ) .
229. Neka a1, a2, …, an (n¥2) se pozitivni realni broevi i s=a1+a2+…+an.
a1 a2 an n
Doka`i go neravenstvoto + + ... + ≥ .
s − a1 s − a 2 s − an n − 1
230. Ako a>0, b>0, c>0 toga{
a + b + c ≥ k + n + m ak ⋅ bn ⋅ c m + k + n + m an ⋅ bm ⋅ c k + k + n + m am ⋅ bk ⋅ c n . Doka`i.
231.
Ako a, b i c se pozitivni realni broevi toga{ va`i neravenstvoto
a b c
1
a a +b+c
⋅b a+b +c
⋅c a +b+c
≥ ⋅ (a + b + c) . Doka`i.
3

22
Geometriski neravenstva

Geometriski neravenstva

232. Neka ha , hb и hc se visini vo triagolnikot ABC, a r radiusot na


vpi{anata kru`nica. Doka`i deka ha + hb + hc ¥9r.
233. Ako ta, tb, tc se te`i{nite linii a s poluperimetarot vo
triagolnikot ABC toga{ va`i neravenstvoto ta2+tb2+tc2¥s2. Doka`i.
234. Za proizvolen triagolnik ABC doka`i gi neravenstvata:
27
a) 1 < 1 + 1 < 1 ; b) hahbhc ≥ 27r2; v) P 2 ≥ R ⋅r3 .
2r h a h b r 2
235. Za pravoagolniot triagolnik ABC (®C=90±), doka`i gi
neravenstvata: a) a+b<c+h; b) a3+b3<c3.
236. Daden e triagolnik ABC so tap agol kaj temeto A. Doka`i deka
a+ha>b+hb.
237. Ako ta, tb i tc se te`i{nite linii vo triagolnikot АВС a s
3
poluperimetarot toga{ va`i s < t a + t b + t c < 3 ⋅ s . Doka`i.
2
238. Vo vnatre{nosta na triagolnikot ABC so perimeter L e zemena
L
to~ka O. Doka`i deka < AO + BO + CO <L .
2
239. Ako ta, tb i tc se te`i{nite linii vo triagolnikot ABC toga{ doka`i
deka tatb+tbtc+tcta< 5 ·(ab+bc+ca).
4
240. Neka ta i tb se te`i{ni linii vo triagolnikot ABC, a P e
plo{tinata na triagolnikot ABC. Doka`i deka taÿtb¥ 3 P . Koga va`i
2
znakot za ravenstvo?
241. Niz vnatre{na to~ka na daden triagolnik se povle~eni tri pravi
koi triagolnikot go delat na 6 dela. Neka S1, S2 i S3 se plo{tinita
na tri taka dobieni delovi na triagolnikot. Doka`i deka
1 1 1 9
+ + > , kade S e plo{tinata na triagolnikot ABC.
S1 S 2 S 3 S
242. Doka`i go neravenstvoto  t a 2 + t b 2 + t c 2  ⋅  ha 2 + hb 2 + hc 2  ≥ 27P2 kade
   
ha , hb , hc se visinite, a t a , t b , t c se te`i{nite linii na
triagolnikot АВС.

23
Geometriski neravenstva

243. Doka`i deka vo sekoj triagolnik ABC so strani a, b, c i


1 1 1  1 1 1
poluperimetar s va`i neravenstvoto + + ≥ 2 + + 
s−a s −b s−c a b c .
244. Ako a, b, c se strani na triagolnikot
ABC a P negovata plo{tina toga{
doka`i go neravenstvoto
2 2 2
a + b + c ≥ 4 3P .
245. Neka r e radiusot na vpi{anata
kru`nica vo triagolnikot ABC, a
ra, rb, rc se radiusite na kru`nicite koi
ja dopirat soodvetnata strana i
prodol`enijata na drugite dve strani
(crt. 1). Ako P e plo{tinata na
triagolnikot ABC toga{
r2+ra2+rb2+rc2¥2P . Doka`i.
Crte` 1
246. Daden e triagolnik ABC so strani a, b, c i vnatre{na to~ka О.
Neka x, y, z se rastojanijata od О do temiwata A, B, C, soodvetno.
Ako p, q, r se rastojanijata do stranite BC, AC и AB, soodvetno,
toga{ va`i neravenstvoto ax¥qb+rc. Doka`i.
247. Dve strani na triagolnikot АВС se 10 i 15. Doka`i deka
simetralata na agolot me|u niv e pomala od 12.
248. Za proizvolen ostroagolen triagolnik ABC so ortocentar H doka`i
AH BH CH
go neravenstvoto + + ≥ 3.
a b c
249. Neka a, b i c se strani na triagolnik. Doka`i go neravenstvoto
a 2 + 2bc b 2 + 2ac c 2 + 2ab
+ 2 + 2 > 3.
b2 + c 2 c + a2 a + b2
250. Niz to~kata O koja le`i vo vnatre{nosta na triagolnikot ABC se
povle~eni otse~ki paralelni so stranite
koi go delat toj triagolnik na tri
triagolnici i tri paralelogrami. Neka
plo{tinite na taka dobienite delovi
bidat ozna~eni kako na crte` 2. Doka`i
a b c 3
deka + + ≥ .
x y z 2
Crte` 2
251. Ako a,b,c i d se stranite na tetiven ~etiriagolnik ABCD toga{
(ac+bd)2¥4abcd. Doka`i.

24
Geometriski neravenstva

a b c
252. Neka a, b i c se strani na triagolnik za koi p= + + i
b c a
a c b
q= + + . Doka`i deka p - q < 1 .
c b a

253. Za proizvilen triagolnik ABC doka`i gi neravenstvata:


α β γ 1 3 3
a) sin sin sin ≤ ; b) sinα ⋅ sinβ ⋅ sinγ ≤ .
2 2 2 8 8
254. Ako a, b, c se stranite a α, β i γ soodvetnite vnatre{ni agli vo
triagolnikot АВС toga{ va`i neravenstvoto:
3
1 < cosα + cosβ + cosγ≤ . Doka`i.
2
255. Doka`i deka vo sekoj triagolnik ABC so strani a, b, c i agli α, β, γ
cosα cosβ cosγ 3
va`i neravenstvoto + 3 + 3 ≥ ⋅ abc.
a3 b c 2
256. Neka a,b i c se stranite a α, β, γ soodvetnite agli vo triagolnikot
ABC. Ako R e radiusot na opi{anata a r radiusot na vpi{anata
kru`nica vo triagolnikot ABC toga{ doka`i go neravenstvoto
cosα cosβ cosγ R
+ + ≥ .
sin 2 α sin 2β sin 2 γ r
257. Ako a, b, c se stranite, a α, β i γ soodvetnite agli vo triagolnikot
a+b+c
ABC, toga{ va`i neravenstvoto a·cos α +b·cos β +c·cosγ§ .
2
Doka`i.
258. Neka a, b, c se stranite a α, β, γ soodvetnite agli vo ostroagolniot
triagolnik ABC. Ako R i r se radiusite na opi{anata i vpi{anata
kru`nica vo triagolnikot ABC toga{ va`i neravenstvoto
1 1 1 9R
+ + ≥ . Doka`i.
cosα cosβ cos γ R + r
259. Ostroagolniot triagolnik ABC so sredini A1, B1 i C1 na stranite
BC, CA i AB e vpi{an vo kru`nica so radius 1. Doka`i deka
1 1 1
+ + ≥ 6 , kade {to O e centarot na kru`nicata.
OA 1 OB1 OC1
260. Neka a, b i c se stranite, a V centarot na vpi{anata kru`nica vo
abc
triagolnikot ABC. Doka`i go neravenstvoto VA ⋅ VB ⋅ VC ≤ .
3 3
261. Neka a, b, c se strani, a R radiusot na opi{anata kru`nica okolu
triagolnikot ABC. Doka`i go neravenstvoto a2+b2+c2≤9R2.

25
Geometriski neravenstva

262. Vo triagolnikot ABC to~kite M, L, K le`at na stranite AB, BC, CA,


soodvetno. Ako AK = k, BL = l, CM = m toga{ va`at neravenstvata
1 k +l+m 3
< < . Doka`i.
2 a+b+c 2
263. Neka sa, sb i sc se dol`inite na simetralite na aglite α, β i γ,
soodvetno, vo triagolnikot АВС, a ta, tb i tc se te`i{nite linii.
s s s
Doka`i deka a + b + c > 1.
ta tb tc
264. Ako H e ortocentarot a to~kite D, E i F se sredinite na stranite
AB, BC i CD soodvetno na triagolnikot ABC toga{ va`i
3R
neravenstvoto HD + HE + HF ≥ . Doka`i.
2

26
Stepeni i koreni

Stepeni i koreni

265. Vo mno`estvoto na realnite broevi re{i ja ravenkata


x3-(a+b+c)x2+(ab+ac+bc)x-abc=0 (a,b,cœR). (Zada~a na Viet)
266. Doka`i go ravenstvoto (a12 + a22 + ... + an2 ) ⋅ (b12 + b22 + ... + bn2 )
= (a1b1 + a 2 b 2 + ... + a n b n ) 2 + (a1 b 2 − a 2b1 ) 2 + (a1b 3 − a 3 b1 ) 2 + ... + (a n −1b n − a nb n −1 ) 2 .
(Ravenstvo na Lagran`)

267. Uprosti go izrazot


(3 ) − (3 )
2020 2 2018 2
.
(3 ) − (3 )
2019 2 2017 2

268. Doka`i deka brojot 77777-777 e deliv so 10.


269. Doka`i deka zbirot 7 + 7 2 + 7 3 + ... + 712n e deliv so 100 za
proizvolen priroden broj n.
270. Doka`i deka izrazot 11n+2 + 12 2n+1 e deliv so 133 za sekoj nœN .
271. Ako brojot 3 n + m e deliv so brojot 10 toga{ brojot 3 n+4 + m isto
taka se deli so 10, za m,nœN. Doka`i.
272. Ako p i q se posledovatelni neparni broevi, kade (р>q), toga{
pp+qq se deli so p+q. Doka`i.
273. Doka`i deka zbirot od kubovite na tri posledovatelni prirodni
broevi e deliv so 9.
274. Odredi gi site dvocifreni broevi, koi stepenuvani na proizvolen
pokazatel, zavr{uvaat na istite dve cifri od koi se obrazuvani
tie broevi.
275. Odredi gi site vrednosti na n za koi barem eden od izrazite
2n-1 i 2n+1 e deliv so 7.
276. Dadeni se n1>n2>…>nr nenegativni celi broevi i broevi ak
(1§k§r) koi primat vrednost samo 1 ili -1. Ako
a1 ⋅ 3n + a 2 ⋅ 3n + ... + ar ⋅ 3n = 2019 toga{ presmetaj ja vrednosta na
1 2 r

zbirot n1+n2+…+nr .
9
277. Na koja cifra zavr{uva brojot 9 9 ?
278. Na koja cifra zavr{uva brojot 777 777 ?
279. Koi se poslednite dve cifri vo brojot ((20032005)2017)2019?
2
.3
..
12 11
280. Odredi gi poslednite dve cifri na prirodniot broj 13 .
2019
281. Najdi gi poslednite tri cifri na brojot 2019 .
282. Odredi go ostatokot pri delewe na 22012 so 2012.
283. Odredi gi site celobrojni re{enija na ravenkata 2 x + 1 = y 2 .
284. Najdi go najgolemiot priroden broj n za koj brojot 2n e delitel na
brojot 32013+1.

27
Stepeni i koreni

285. Odredi go najgolemiot priroden broj n za koj izrazot 2 n + 2 11 + 2 8 e


to~en kvadrat.
286. Neka a, b i c se pozitivni realni broevi takvi {to am+bm=cm i
an+bn=cn, kade m i n se isto taka pozitivni realni broevi. Doka`i
deka m=n.
287. Doka`i deka brojot 400...0 23
1233 e deliv so brojot 600...0
1233 + 200...0
43
1236 .
n n n
2 2 2
     
288. Neka n ∈ N. Doka`i deka  33...3   123 44...4
123  =  55...5 1235  .
 123  +  55...5 123 44...4
 n   n−1 n−1   n−1 n −1 
100


( 100
) (
289. Dokaæi deka izrazot 10  1+ 10 - 1- 10  e cel broj. ) 
290. Najdi ja vrednosta na izrazot:
a) 3 20 + 392 + 3 20 − 392 ; b) 3 5 2 + 7 − 3 5 2 − 7
1
291. Presmetaj ja vrednosta na izrazot B = 3 10 + 6 3 − 2 .
3
10 + 6 3
2+ 3 2− 3
292. Uprosti go izrazot + .
2+ 2+ 3 2− 2− 3

293. Presmetaj: a) 3 9 + 4 5 + 3 9 − 4 5 ; b) 3 20 +14 2 + 3 20 −14 2 .


n 4 4 48
6 4 4 47
2- 2+ 2 + ...... + 2 1
294. Doka`i go neravenstvoto > .
4
2- 2 + 2 + ..... + 2
1 4 442 4 4 43
n -1

295. Doka`i deka ravenkata x + x + x + ... + x = y (so 2019 vlo`eni


koreni) nema re{enie vo N.
296. Re{i ja ravenkata x + x = y - 2012 vo mno`estvoto celi broevi.
297. Vo mno`estvoto realni breovi re{i ja ravenkata
12 + 2 12 =x2 (x ≠ 0 ).
12 − x − 2
x2 x
298. Re{i ja ravenkata 3
x2 − 3 x − 1 = 1.
a−x n b+x
299. Re{i ja ravenkata n + = 2.
b+x a−x
300. Re{i ja ravenkata ( 2 − 3 ) x + ( 2 + 3 ) x = 4 .
301. Re{i ja ravenkata x − 2 x − 1 - x + 3 − 4 x − 1 =1.
302. Re{i ja ravenkata 3 4(3x+ 4) - 3 3(4x- 7) =1 vo mno`estvoto na realni
broevi.

28
Stepeni i koreni

303. Najdi ja najgolemata


i najmalata vrednost na izrazot
4a + 1 + 4b + 1 + 4c + 1 ako a + b + c = 1.
304. Najdi gi realnite koreni na ravenkata

x + 2 x + 2 x + ... + 2 x + 2 3x = x .
144444424444443
n-koreni

305. Najdi tricifren broj xyz koj ja zadovoluva ravenkata


xy - z = xy + z .
306. Neka a i b se zaemno prosti celi broevi takvi {to koli~nikot a/b
e ednakov na zbirot na razli~nite realni re{enija na ravenkata
3
3x − 4 + 3 5x − 6 = 3 x − 2 + 3 7x − 8 . Odredi go zbirot a+b.
307. Ako x e najmaloto realno re{enie na ravenkata
x2 + 25x+ 25 =10x⋅ x +1 , toga{ najdi ja vrednosta na [ x ].
308. Neka a i b se to~ni kvadrati za koi razlikata na nivniot proizvod
so nivniot zbir e 4844. Presmetaj ja vrednosta na izrazot
( a +1)( b +1)( a - 1)( b - 1)- ( 68 +1)( 63 +1)( 63- 1)( 68 +1).
309. Odredi gi site cifri x, y, z ako e poznato deka ravenstvoto
xx...x
123 - yy...y = zz...z va`i barem za dve razli~ni vrednosti na n.
123 123
2n n n

29
Geometriski figuri vo ramnina

Geometriski figuri vo ramnina

310. Neka А i В se dve proizvolni to~ki na pravata p. Vo ista


poluramnina opredelena so pravata p, se povle~eni normalite
AA 1 = a и BB 1 = b . Doka`i deka presekot na pravite АВ1 i А1В se
nao|a na isto rastojanie od pravata р nezavisno od izborot na
to~kite А i В.
311. Doka`i deka te`i{nite linii povle~eni od temiwata A i B na
triagolnikot ABC se me|usebno normalni ako i samo ako za
2 2 2
dol`inite na stranite va`i ravenstvoto BC + AC = 5 AB .
312. Doka`i deka te`i{nata linija ta od temeto A e ednakva na
1
⋅ 2b 2 + 2c 2 − a 2 .
2
313. Ako za poluperimetarot s na triagolnikot ABC so strani a, b i c
ab bc ca
va`i ravenstvoto s = + + toga{ triagolnikot ABC e
a +b b+c c +a
ramnostran. Doka`i.
314. Vo triagolnikot ABC to~kata D le`i na stranata BC, a B1, C1, D1 se
prese~nite to~ki na proizvolna prava p so AB , AD , AC ,
AC AB AD
soodvetno. Doka`i deka BD ⋅ + CD ⋅ = BC ⋅ .
AC1 AB1 AD1
315. Neka M e to~ka od vnatre{nosta na triagolnikot ABC od koja se
spu{teni normalite MA1, MB1 i MC1 kon pravite BC, CA i AB,
a b c
soodvetno. Za koja to~ka M izrazot + + prima najmala
MA 1 MB1 MC1
vrednost?
316. Na stranata AC vo triagolnikot ABC zemena e to~ka K koja ja deli
stranata AC vo odnos 1:3, a na stranata BC to~ka L takva {to
BL : LC = 1: 4. Doka`i deka pravata AL ja prepolovuva otse~kata BK.
317. Ако a + ha = b + hb = c + hc toga{ triagolnikot ABC e ramnostran.
Doka`i.
318. Vo vnatre{nosta na triagolnikot ABC zemena e to~ka O, pri {to
pravite AO, BO и CO gi se~at stranite vo to~kite A1, B1 и C1 ,
soodvetno. Doka`i deka:

30
Geometriski figuri vo ramnina

OA 1 OB1 OC 1 AC1 BA 1 CB1


a) + + = 1; b) ⋅ ⋅ =1
AA 1 BB 1 CC1 C1B A 1C B1A
319. Vo triagolnikot ABC e vpi{ana
kru`nica k so radius r. Tangentite na
kru`nicata k paralelni so stranite
na triagolnikot ABC, otsekuvaat od
nego tri mali triagolnici (crt. 3). Ako
r1, r2, r3 se radiusite na vpi{anite
kru`nici vo tie triagolnici doka`i
deka r1+r2+r3=r.

Crte` 3
320. Ako to~kite P. Q i R se na stranite ili na prodol`enijata na
stranite BC, CA i AB na triagolnikot ABC, soodvetno, toga{ P, Q i
→ → →
R se kolinearni ako i samo ako AR ⋅ BP ⋅ CQ = −1 . Doka`i.
→ → →
RB PC QA
(Teorema na Menelaj)
321. Na otse~kata AB e zemena proizvolna to~ka C i nad otse~kite AC,
BC i AB kako nad dijametar se konstruirani polukru`nici koi
le`at na ista strana vo odnos na
pravata AB. Niz to~kata C e
povle~ena prava p normalna na
prava AB koja go deli najgolemiot
polukrug na dva dela ACD i CBD, vo
koi se vpi{ani dve kru`nici k1 i k2
kako na crte` 4. Doka`i deka dvete
kru`nici imaat ednakov radius.

Цrtеж 4
322. Na lakot BC od kru`nicata k opi{ana okolu ramnostraniot
triagolnik ABC, {to ne go sodr`i temeto A, e zemena proizvolna
to~ka Р. Doka`i deka AP = BP + CP .
323. Vo triagolnikot ABC povle~eni se visinata AH, simetralata BL i
te`i{nata linija CM.. Ako
triagolnikot HML e ramnostran
toga{ triagolnikot ABC e istotaka
ramnostran. Doka`i.
324. Vo ~etiriagolnikot ABCD (vo koj na
mo`e da se vpi{e ili opi{e
kru`nica) to~kite E, F, G, H se
centri na opi{anite kru`nici
okolu triagolnicite ABC, BCD, CDA
i DAB, soodvetno. Crte` 5

31
Geometriski figuri vo ramnina

Presekot na dijagonalite na ~etiriagolnikot EFGH neka e


to~kata P. Doka`i deka AP = CP i BP = DP .
325. Na edniot krak od daden agol so teme О zemena e to~ka A, a na
drugiot krak to~kite B i C, pri {to to~kata B le`i me|u O i C.
Konstruirana e kru`nica k1 so centar О1, vpi{ana vo triagolnikot
ОАB i kru`nica k2 so centar O2, koja ja dopira stranata AC i
prodol`enijata na stranite ОА i ОС na triagolnikot АОС (crt. 5).
Doka`i deka ako O1A = O 2 A toga{ triagolnikot АBС e ramnokrak.
326. Neka k1, k2 i k3 se pripi{ani kru`nici na triagolnikot ABC
(crt. 6). Doka`i deka pravite AA1, BB1 i CC1 se se~at vo edna to~ka.
(To~ka na Nag
Nagel)
327. Neka A1, B1 i C1 se dopirnite to~ki na
vpi{anata kru`nica vo triagolnikot
ABC. Doka`i deka pravite AA1, BB1 i CC1
se se~at vo edna to~ka.
To~ka na Geron)
(To~ka
328. Za proizvolen (ne ramnokrak) trapez
ABCD (AB II CD) va`i ravenstvoto
2 2
AC - BD AB + CD . Doka`i.
2 2
=
AD - BC AB - CD

Crte` 6
329. Vo triagolnikot ABC so agol kaj temeto B ednakov na 60±,
simetralite AD i CE na aglite kaj temiwata A i C, soodvetno, se
se~at vo to~ka O. Doka`i deka OD = OE .
330. Neka P, A, B i C se kolinearni to~ki, a takvi se i to~kite P, X, Y i
Z. Ako pravite AX i BZ se paralelni i ako BX i CY se paralelni
toga{ BY i CZ se isto taka paralelni pravi. Doka`i.
331. Na stranite BC, CA, AB od triagolnikot АВС se izbrani to~kite A1,
B1, C1 za koi pravite АА1, ВВ1 i СС1 se se~at vo to~kata М. Ако
AM : MA 1 = BM : MB 1 = CM : MC 1 toga{ AA1, BB1 i CC1 se visini vo
triagolnikot АВС. Doka`i.
332. Ako stranite na triagolnikot se tri posledovatelni prirodni
broja toga{ R = 2r + 1 (R-radius na opi{ana kru`nica; r-radius na
2r
vpi{ana kru`nica). Doka`i.
333. Vo triagolnikot ABC so strani a, b, c se povle~eni tangenti na
vpi{anata kru`nica k, paralelni so stranite. Neka otse~kite od
tangentite obrazuvani me|u stranite se a1, b1, c1, soodvetno. Doka`i
a b c
deka 1 + 1 + 1 = 1.
a b c
334. Vo triagolnikot ABC niz to~kite A, B, C se povle~eni normali na
stranite AB, BC, CA koi normali se se~at vo to~kite A1, B1, C1

32
Geometriski figuri vo ramnina

(crt. 7). Ako R, R1, R2, R3 se radiusite na


kru`nicite opi{ani okolu triagolnicite
ABC, BCA1, CAB1, ABC1 soodvetno, toga{
R1·R2·R3=R3. Doka`i.
335. Ako O e prese~na to~ka na dijagonalite na
vpi{an ~etiriagolnik ABCD vo kru`nica k
2 2 2 2 2 2
toga{ AC +OB +OD = BD +OA +OC . Doka`i.

Crte` 7

336. Na crte` 8 se dadeni dve kru`nici k1 i k2


so ednakov radius koi vnatre{no dopiraat
kru`nica so pogolem radius vo to~ki A i B,
soodvetno, a i me|usebno se dopirat. Neka C
e proizvolna to~ka od „pogolemata”
kru`nica i neka CA ja se~e kru`nicata k1 vo
M, a CB ja se~e k2 vo N. Doka`i deka MN i
AB se paralelni.
Crte` 8
337. Nad katetite na pravoagolniot triagolnik ABC se pripi{ani dve
kru`nici so radius r i R. Odredi ja plo{tinata na triagolnikot
ABC (crt. 9).
338. Neka vo triagolnikot ABC na stranite AB, BC, CA le`at to~kite M,
N, P, soodvetno i neka kru`nicite opi{ani
okolu triagolnicite AMP, BMN i CNP gi
se~at visinite od temiwata A, B i C vo
to~kite X, Y, Z, soodvetno. Doka`i deka
triagolnikot ABC e sli~en so
triagolnikot XYZ. Crte` 9
339. Odredi go odnosot od zbirot na kvadratite na te`i{nite linii vo
ABC i zbirot na kvadratite na stranite na toj triagolnik.
340. Doka`i deka vo pravoagolniot triagolnik simetralata na praviot
agol se javuva istovremeno i kako simetrala na agolot me|u
visinata i te`i{nata linija povle~eni od temeto na praviot agol.
341. Od site triagolnici vpi{ani vo daden triagolnik, odredi go onoj
{to ima najmal perimeter.
342. Katetite na pravoagolniot triagolnik se 4cm i 3cm. Opredeli go
rastojanieto me|u centrite na vpi{anata i opi{anata kru`nica vo
toj triagolnik.
343. Doka`i deka zbirot od kvadratite na rastojanijata od proizvolna
to~ka M do temiwata na triagolnikot ABC e minimalna ako to~kata
M e te`i{teto na toj triagolnik.
344. To~kite A1, B1, C1 se to~ki od stranite BC, CA, AB na triagolnikot
ABC, pri {to otse~kite AA 1, BB 1, CC1 se se~at vo edna to~ka M.

33
Geometriski figuri vo ramnina

MA 1 MB1 MC1
Pri kakva polo`ba na to~kata M izrazot ⋅ ⋅ prima
AA 1 BB1 CC1
maksimalna vrednost?
345. Nad dijametarot AV na polukrugot ANB e konstruiran pravoagolnik
ACDB, taka {to polukrugot i pravoagolnikot se na razli~ni strani
od AB. Drugata strana na pravoagolnikot e ednakva so stranata na
vpi{aniot kvadrat vo krugot so dijametar AB. Ako N e proizvolna
to~ka od polukrugot i ako CN i DN go se~at dijametarot AB vo
to~kite E i L, soodvetno, toga{ AL ² + BE² = AB² . Doka`i.
(Zada~a na Ferma)
346. Neka triagolnikot ABC e tapoagolen (crt. 10). To~kite P i Q le`at
na stranata BC taka {to P e pome|u
B i Q i BP =21, PQ =35,
QC =100. Ako AP i AQ go delat
agolot BAC na tri ednakvi agli,
toga{ odredi go odnosot AB .
AC
Crte` 10
347. Od vnatre{na to~ka O na triagolnikot ABC se spu{teni normali
na, nb, i nc kon stranite a, b i c, soodvetno. Doka`i deka
na nb n c
+ + = 1.
ha hb h c
348. Neka ha, hb, hc se soodvetnite visini povle~eni od temiwata A, B, C
na triagolnikot ABC, a ra, rb, rc se radiusite na kru`nicite
pripi{ani na toj triagolnik ABC, a r e radiusot na vpi{anata
1 1 1 1 1 1 1
kru`nica. Doka`i deka a) + + = + + = ;
ha hb hc ra rb rc r
2 1 ra1 r r
b) = + + b + c ≥3.
; v)
ha rb rc
ha hb hc
349. Neka O e centarot na kru`nicata k
opi{ana okolu ostroagolniot
triagolnik ABC, AC > AB , a D e
simetri~na na A vo odnos na O. Ako E e
vnatre{na to~ka na otse~kata BC za
koja AB = AE toga{ va`i ravenstvoto
AC ⋅ BD = AB ⋅ CD + AD ⋅ CE . Doka`i.
Crte` 11

350. Neka VK e simetrala na agolot β vo triagolnikot ABC. Poznato e


deka ®AKB:®CKB=4:5. Najdi ja razlikata na aglite α i γ.

34
Geometriski figuri vo ramnina

351. Na stranata AB od triagolnikot ABC e izbrana to~ka M (crt. 11).


Neka pravata niz A paralelna so pravata CM ja se~e BC vo to~ka P,
a pravata niz B paralelna so pravata CM ja se~e AC vo to~ka Q.
1 1 1
Doka`i deka + = .
AP BQ CM
352. Na crte` 12 e daden ramnostran triagolnik
ABC i pravoagolnik BCDE za koj BC = 2 ⋅ CD .
Ako M e sredina na stranata AC toga{
presmetaj go agolot CMD .
353. 353. Ako A1, B1 i C1 se podno`jata na visinite
povle~eni od temiwata A, B i C na
triagolnikot ABC, a A2, B2 i C2 se sredinite
na otse~kite AA1, BB1 i CC1 toga{ presmetaj
go zbirot na aglite C2A1B2 , A2B1C2 i B2C1A2.
Crte` 12
354. Ako T e te`i{teto na triagolnikot ABC i R to~ka {to le`i na
ramninata opredelena so triagolnikot ABC toga{ va`i
ravenstvoto PA2 + PB2 + PC2 = 3PT2 + 1 ( AB2 + AC2 + BC2 ) . Doka`i.
3
(Teorema na Lajbnic)
355. Doka`i deka sredinite na dijagonalite na ~etiriagolnik i
sredinata na otse~kata koja gi soedinuva prese~nite to~ki na
sprotivnite strani, le`at na edna prava. (Prava na Gaus)
356. Od to~kite A1, B1 i C1 koi le`at na stranite BC, CA i AB na
triagolnikot ABC, se povle~eni normali kon stranite. Doka`i
deka potreben i dovolen uslov trite normali da se se~at vo edna
to~ka e da bide ispolneto ravenstvoto
2 2 2 2 2 2
AC1 + BA 1 + CB 1 = B1A + C1B + A 1C . (Teorema na Karnot)
357. Vo ostroagolen triagolnik AVS rastojanijata od centarot na
opi{anata kru`nica do stranite se OM1 =d1, OM 2 =d2 i
OM 3 =d3. Doka`i deka d1+ d2 +d3=R + r. (Zada~a na Karnot)
358. Okolu triagolnikot ABC e opi{ana kru`nica. Doka`i deka
podno`nite to~ki na normalite spu{teni od proizvolna to~ka na
kru`nicata kon stranite na triagolnikot ABC le`at na edna
prava. (Prava na Simpson)
359. Niz temiwata A, B, C na ostroagolen triagolnik i centarot na
opi{anata kru`nica se povle~eni pravi koi gi se~aat sprotivnite
strani vo to~ki M, N, P. Doka`i deka 1 + 1 + 1 = 2 .
ΑΜ ΒΝ CP R
360. Neka N e sredina na stranata BC vo triagolnikot ABC. Nad
stranite AB i AC, nadvor od triagolnikot ABC, konstruirani se
ramnokraki pravoagolni triagolnici ABM i ACP, soodvetno, taka

35
Geometriski figuri vo ramnina

{to AB i AC se nivni hipotenuzi. Doka`i deka triagolnikot MNP e,


isto taka, ramnokrak pravoagolen triagolnik.
361. Neka V e centarot na vpi{anata kru`nica na triagolnikot ABC, a
O1, O2 i O3 se centrite na opi{anite kru`nici okolu triagolnicite
BCV, CAV i ABV, soodvetno. Doka`i deka kru`nicite opi{ani
okolu triagolnicite O1O2O3 i ABC se koncentri~ni.
362. Neka a, b, c i a1, b1, c1 se strani na triagolnicite ABC i A1B1C1, a
α, β, γ i α1, β1, γ1, nivnite agli. Ako α+α1=π i
β=β1 toga{ a·a1=b·b1+c·c1. Doka`i.
363. Doka`i deka rastojanieto od centarot na
opi{anata kru`nica okolu triagolnikiot ABC,
do edna od stranite, e polovina od rastojanieto
me|u ortocentarot i sprotivnoto teme na
stranata (crt. 13).
Crte` 13
364. Ako H e ortocentarot a r i R radiusite na vpi{anata i opi{anata
kru`nica okolu triagolnikot ABC toga{
AH + BH + CH = 2( r + R ) . Doka`i.
365. Vo triagolnikot ABC to~kite X, Y ja delat
stranata AB na tri ednakvi dela, to~kite Z,
W ja delat stranata BC, a to~kite U, V
stranata CA, isto taka na tri ednakvi dela.
Neka P, Q i R se prese~nite to~ki na pravite
povle~eni niz temiwata i delbenite to~ki
od sprotivnata strana na triagolnikot ABC
(crt. 14). Doka`i deka triagolnikot ABC e
sli~en so triagolnikot PQR.
(Teorema na Morli) Crte` 14
366. Neka AD, BE i CF se visinite vo triagolnikot ABC. Ako R, R1, R2 i
R3 se soodvetnite radiusi na kruænicite opi{ani okolu
triagolnicite ABC, AEF, BFD, CED i r e radiusot na kruænicata
vpi{ana vo triagolnikot DEF, toga{ R12 + R2 2 + R3 2 = R(R- r) . Doka`i.
367. Vo ostroagolniot triagolnik ABC so visina CD, od to~kata D e
povle~ena normala DE kon stranata BC (E œ BC). Na otse~kata DE e
zemena to~ka H, za koja EH: HD = AD: DB . Doka`i deka CH i AE se
zaemno normalni pravi.
368. Vpi{anata kru`nica k vo triagolnikot ABC gi dopira stranite AB
i BC vo to~kite M i N, soodvetno. Ako sa e simetralata na agolot
kaj temeto A koja ja se~e MN vo to~ka P, toga{ CP i sa se zaemno
normalni. Doka`i.
369. Neka H e ortocentarot, a R e radiusot na opisanata kru`nica okolu
triagolnikot ABC. Doka`i deka radiusite na opi{anite kru`nici
okolu triagolnicite ABH, BCH i CHA se ednakvi so R.
(Teorema na Hamilton)

36
Geometriski figuri vo ramnina

370. Na stranata AD vo paralelogramot ABCD kako dijametar e


konstruirana kru`nica k, koja minuva niz prese~nata to~ka O na
dijagonalite i niz sredi{nata to~ka E na stranata AB. Najdi gi
aglite na paralelogramot.
371. Neka O e centarot na opi{anata kru`nica okolu triagolnikot ABC.
Na stranite AC i BC izbrani se to~ki M i N, soodvetno, pri {to
2®MON=®AOB. Doka`i deka perimetarot na triagolnikot MCN ne e
pomal od stranata AB.
372. Otse~kata EF go deli triagolnikot ABC na dva dela so ednakvi
perimetri i plo{tini. Doka`i deka centarot na vpi{anata
kru`nica vo triagolnikot ABC le`i na taa otse~ka .
373. Ako R i r se radiusite na opi{anata i vpi{anata kru`nica vo
proizvolen triagolnik ABC so centri vo to~kite O i V, soodvetno,
toga{ OV = R ⋅ (R - 2r ) . (Ojlerovo ravenstvo)
374. Te`i{nite linii AM, BN i CP vo triagolnikot ABC ja se~at
opi{anata kru`nica soodvetno vo to~kite M1, N1 i P1. Doka`i deka
AM ⋅ AM1 + BN ⋅ BN1 + CP ⋅ CP1 = a 2 + b 2 + c 2 .
375. Trite visini vo ostroagolniot triagolnik ABC se se~at vo to~kata
H, koja ednata od niv ja deli na polovina, a drugata vo odnos 2:1. Vo
koj odnos ortocentarot H ja deli tretata negova visina?
376. Vo ostroagolniot triagolnik АВС povle~eni se visinite AHA, BHB i
CHC. Doka`i deka triagolnikot ~ii temiwa se ortocentrite na
triagolnicite АНВНС, ВНАНС i СНАНВ e skladen so triagolnikot
НАНВНС .
377. Daden e ostroagolen triagolnik ABC so ortocentar H. Doka`i deka
1 2 2 2
AHÿha + BH ÿhb+ CH ÿhc= (a +b +c ) .
2

378. Neka O e centarot na vpi{anata kru`nica vo


triagolnikot ABC i neka triagolnikot XYZ e
dobien kako presek na pravite povle~eni niz
temiwata A, B i C normalno na pravite OA, OB
i OC, soodvetno (crt. 15). Doka`i deka to~kata
O e ortocentar na triagolnikot XYZ.

Crte` 15
379. Vo ramnostran triagolnik so strana a, vpi{ani se tri kru`nici,
so ednakvi radiusi takvi {to poparno se dopiraat i sekoja od niv
dopira dve strani od triagolnikot. Najdi ja dol`inata na
radiusite na kru`nicite.
380. Nad stranite na paralelogramot ABCD konstruirani se
ramnostranite triagolnici BQC i CPD. Doka`i deka triagolnikot
AQP e isto taka ramnostran.

37
Geometriski figuri vo ramnina

381. Vo paralelogramot ABCD od temeto C e povle~ena normala CE na


prodol`enieto na stranata AB. To~kata E ja povrzuvame so
sredinata M na stranata AD. Doka`i deka 2·®BEM=®EMC.
382. Vo paralelogramot ABCD temeto C se povrzuva so sredinite na
stranite AB i AD. Doka`i deka na toj na~in dijagonalata BD se
deli na tri ednakvi dela.
383. Na stranite AB i AD na paralelogramot ABCD se izbrani to~kite M
AM AN
i N, za koi =m i = n . Neka prese~nata to~ka na NM so AC e
AB AD
AP
to~kata P. Opredeli go odnosot .
AC
384. Daden e ramnokrakiot triagolnik ABC so kraci CA i CB i agol me|u
niv 20 o . Neka D e to~ka od krakot CA taka {to ®ABD= 60 o , a E e
to~ka od krakot CB taka {to ®BAE= 50 o . Presmetaj go agolot BDE .
385. a) Proizvodot od dvete dijagonali kaj tetiven ~etiriagolnik e
ednakov so zbirot od proizvodot na dvete sprotivni strani
AC·BD = AB·CD + BC·AD . Doka`i. (Prva teorema na Ptolomej)
b) Ako ABCD e tetiven ~etiriagolnik, toga{
AC AB ⋅ AD + BC ⋅ CD
= . Doka`i. (Vtora teorema na Ptolomej)
BD AB ⋅ BC + AD ⋅ CD
386. Tri skladni kvadrati ABB1M, BCC1B1 i
CDNC1 formirat pravoagolnik ADNM.
Doka`i deka ®MBA+®MCA+®MDA=90±.

Crte` 16
387. Vo konveksniot ~etriagolnik ABCD va`i AB = CD i
®ADB+®CBD=180± (crt. 16). Dokaæi deka ®BAD=®BCD
.
388. Vo konveksniot ~etiriagolnik АВСD, AB = CD ,
agolot АВС e 77± i agolot ВСD e 150±. Neka Р e
prese~nata to~ka na simetralite na stranite ВС и
АD. Najdi go agolot ВРС.
Crte` 17

389. Najdi go zbirot na aglite МАN, МВN, МСN, МDN i МЕN ozna~eni na
kvadratnata {ema prika`ana na crte` 17.

390. Neka p e prava koja e tangenta na tri kru`nici koi nadvore{no se


dopiraat (crt. 18). Ako R, R1 i R2 se nivnite radiusi toga{ va`i
1 1 1
= + . Doka`i.
R R1 R2

38
Geometriski figuri vo ramnina

391. Vo ~etiriagolnikot АВСD


dijagonalite se se~at vo to~kata
О. Doka`i deka pravata koja
minuva niz ortocentrite na
triagolnicite АОD i BОC e
normalna so pravata koja minuva
niz te`i{tata na triagolnicite
ABO i CDО.
Crte` 18
392. Okolu kru`nicata so radius R e opi{an trapez АВСD. Ako M, N, P,
Q se posledovatelni dopirni to~ki na stranite na trapezot so
kru`nicata, toga{ AM ⋅ BN ⋅ CP ⋅ DQ = R 4 . Докажи.
393. Ako dijagonalite na eden tetiven ~etiriagolnik ABCD se se~at vo
2 2 2 2 2 2
to~kata О, toga{ AC + OB + OD = BD + OA + OC . Doka`i.
394. To~kata K e sredina na stranata AB na kvadratot ABCD, a to~kata L
ja deli dijagonalata AC vo odnos AL : LC =3:1. Doka`i deka KLD e
prav agol.
395. Trapezot ABCD e prese~en so proizvolna otse~ka MN paralelna so
osnovata , pri {to M le`i na AD, a N na BC. Otse~kata MN ja se~e
pravata AC vo to~ka P, a pravata BD vo to~ka Q. Doka`i deka
MP = QN .
396. Daden e pravilen 2n-agolnik i to~ka M od negovata vnatre{nost.
2 2 2 2 2 2
Doka`i deka MA1 + MAn+1 = MA2 + MAn+2 = ... = MAn + MA2n .
397. Ako ~etiriagolnikot ABCD e paralelogram, toga{
2 2 2 2
AC + BD = 2 ⋅ AB + 2 ⋅ BC . Doka`i.
398. Nad stranite na ostroagolen triagolnik se konstruirani
ramnostrani triagolnici. Ako I, J i Q se te`i{ta na tie
triagolnici toga{ triagolnikot IJQ e ramnostran. Doka`i.
(Teorema na Napoleon)

399. Za proizvolen konveksen ~etiriagolnik ABCD doka`i go


2 2 2 2 2 2 2
ravenstvoto AB + BC + CD + DA = AC + BD + 4MN kade M i N se
sredini na dijagonalite AC i BD. (Ojlerovo ravenstvo)

400. Doka`i deka pravata, koja minuva niz prese~enata to~ka na kracite
i prese~enata to~ka na dijagonalite na trapez, gi prepolovuva
osnovite. (Teorema na [tajner)

401. Neka M e proizvolna to~ka vo vnatre{nosta na kvadratot ABCD. Da


gi ozna~ime so A1, B1, C1 i D1 to~kite {to se presek na pravite AM,
BM, CM i DM so kru`nicata opi{ana okolu kvadratot ABCD,
soodvetno. Doka`i go ravenstvoto A 1B 1 ÿ C1D1 = A 1D 1 ÿ B1C1 .

39
Geometriski figuri vo ramnina

402. Vo ~etiriagolnikot ABCD e vpi{ana, a isto taka i


opi{ana kru`nica, pri {to dopirnite to~ki na
tangentite na vpi{anata kru`nica se X, Y, Z, W
(crt. 19). Doka`i deka pravite XZ i YW se zaemno
normalni.

Crte`.19
403. Doka`i deka vo sekoj konveksen ~etiriagolnik АВСD presekot na
simetralite na vnatre{nite agli pri temiwata А i С so
simetralite na vnatre{nite agli pri temiwata B i D le`at na
edna kru`nica .
404. To~kite K, L, M, N se sredini na stranite AB, BC, CD i DA,
soodvetno, na ~etiriagolnikot ABCD koj e vpi{an vo kru`nicata k.
Doka`i deka ortocentrite na triagolnicite AKN, BKL, CLM i DMN
se temiwa na paralelogram.
405. Niz to~kata A na paralelogramot ABCD e konstruirana proizvolna
kru`nica k koja gi se~e AB, AC i AD vo to~kite M, N, P, soodvetno.
Doka`i deka AB ⋅ AM + AD ⋅ AP = AC ⋅ AN .
406. Daden e kvadratot ABCD so centar O. Neka s e simetrala na agolot
ACD, b e normala niz B na s koja ja se~e dijagonalata AC vo to~ka P,
a stranata CD vo to~ka Q. Doka`i deka DQ = 2PO .
407. Tetiven ~etiriagolnik ABCD ima strani so dol`ini AB =9cm, BC
=2cm, CD =3cm i DA =10cm. Neka M i N se sredinite na stranite AD
i BC, soodvetno i neka AC i BD ja se~at MN vo to~kite P i Q,
PQ
soodvetno. Opredeli go odnosot .
MN
408. Neka to~kata C e sredina na tetivata AB od kru`nicata k i neka
tetivite DE i FG minuvat niz to~kata C. Ako to~kite M i N se
prese~nite to~ki na tetivite DF i EG so
tetivata AB toga{ to~kata C e sredina na
otse~kata MN. Doka`i.
409. Pravoagolnik e razdelen na {est kvadrati, kako
{to e prika`ano na crte` 20. Odredi gi
stranite na pravoagolnikot, ako stranata na
najmaliot kvadrat e 1. Crte` 20
410. Stranite na pravilen petagolnik, {estagolnik i desetagolnik
vpi{ani vo krug so radius R , soodvetno se R 10 − 2 5 , R i
2
R
( 5 − 1) . Doka`i deka kvadratot od stranata na petagolnikot e
2
ednakva na zbirot od kvadratite na {estagolnikot i
desetagolnikot. (zada~a na Evklid)

40
Geometriski figuri vo ramnina

411. Niz ortocentarot H na ostroagolniot triagolnik ABC, minuvaat tri


kru`nici taka {to sekoja od kru`incite dopira edna od stranite
na triagolnikot vo podno`jata na visinite. Doka`i deka vtorite
prese~ni to~ki na kru`nicite se temiwa na triagolnik, sli~en so
triagolnikot ABC.
412. Kru`nicata k vpi{ana vo triagolnikot ABC, gi dopira stranite AB,
BC i AC vo to~kite C1, A1 i B1, soodvetno. Poznato e deka
AA 1 = BB 1 = CC 1 . Doka`i deka triagolnikot ABC e ramnostran
triagolnik.
413. Neka pravata p ja dopira kru`nicata k vo to~kata M. Od to~kite A
i B od kru`nicata k se povle~eni normali kon p ~ii dol`ini
soodvetno se a i b. Najdi go rastojanieto od to~kata M do pravata
AB.
414. Vo vnatre{nosta na agolot XOY
e konstruirana kru`nica k1 so
radius 2 koja gi dopira kracite
na dadeniot agol. Potoa, e
konstruirana kru`nica k2 so
radius 3 koja ja dopira
odnadvor k1 i kracite na agolot
XOY. Ako k3 e kru`nica koja ja
dopira odnadvor k2 i kracite na
agolot XOY toga{ opredeli go
radiusot na kru`nicata k3.

Crte` 21
415. Neka pravite p i q se se~at i se tangenti na kru`nicite k1 i k2 ~ii
radiusi se 1 (crt. 21). Neka k3 e kru`nica koja ja dopira k1
(nadvore{no) pri {to p i q se nejzini tangenti, a k4 kru`nica koja ja
dopira k2 (nadvore{no) pri {to p i q se isto taka nejzini tangenti.
Ako radiusot na k3 e 4 toga{ najdi go radiusot na kru`nicata k4.
416. Vo kru`nica k e vpi{an ramnostran triagolnik ABC so strana a.
Neka rastojanijata od prozvolna to~ka od kru`nicata k do
temiwata na triagolnikot se x, y, z. Doka`i go ravenstvoto
x2+ y2+z2=2a2.
417. Neka O e centarot na opi{anata kru`nica okolu triagolnikot ABC
i p prava koja minuva niz O i gi se~e stranite AC i BC vo to~kite M
i N. Ako P i Q se sredini na otse~kite AN i BM, soodvetno toga{
doka`i deka agolot POQ e ednakov so agolot ACB.

41
Linearni ravenki i neravenki

Linearni ravenki i neravenki

418. Re{i ja ravenkata 1-(2-(3-(...(2018-(2019-(2020-x)))...))=1010.


419. Za koj realen broj a, ravenkata │x-2│+│3-x│=a ima to~no dve
re{enija?
420. Najdi gi site realni re{enija na ravenkata
│x│-│x-1│+2│x-2│=3.
421. Najdi gi site realni broevi x koi ja zadovoluvaat ravenkata
x = x − x − x − ... x − x − 1 ... , ako vo ravenkata se vlo`eni 100
apsolutni vrednosti.
422. Najdi gi site vrednosti na realniot parametar a za koi ravenkata
x + x + a + x − x − a = 2 ima to~no tri re{enija.
423. Kolku re{enija ima ravenkata x + 2 ⋅ y = 100 vo mno`estvoto celi
broevi?
x - ab x - bc x - ca
424. Re{i ja ravenkata + + = a +b +c .
a+b b +c c +a
x-a x-b b a
425. Re{i ja ravenkata + = + .
b a x -a x -b
a+b - x b+c - x c +a - x 4x
426. Re{i ja ravenkata + + + = 1.
c a b a+b+c
427. Odredi go najmaliot priroden broj koj pri delewe so tri dava
ostatok 1, pri delewe so pet ostatok 2, a pri delewe so sedum dava
ostatok 3.
428. Najdi gi site tricifreni broevi koi pri delewe so 7 davaat
ostatok 2, pri delewe so 9 davaat ostatok 4 i pri delewe so 12
davaat ostatok 7.
429. 109 jabolki se rasporedeni vo nekolku paketi. Vo nekoi paketi se
javuvat po x jabolki, a vo ostanatite po 3 jabolki. Odredi gi site
vrednosti za x, ako imalo vkupno 20 paketi.
430. Od desnata strana na prirodniot broj A, dopi{ani se tri cifri.
Dobieniot broj e ednakov so zbirot na prirodnite broevi od 1 do A.
Najdi go brojot A.
431. Eden ~etiricifren broj e devet pati pomal od brojot zapi{an so
istite cifri, no vo obraten red. Odredi go zbirot od cifrite na
toj broj.
432. Ako na ~etiricifreniot broj mu se dopi{e cifrata 2 od levo ili
cifrata 4 od desno, toga{ vo dvata slu~ai se dobiva poln
kvadrat. Najdi go ~etiricifreniot broj.

42
Linearni ravenki i neravenki

433. Prirodniot broj n e ednakov so zbirot na tri razli~ni deliteli na


brojot n-1. Najdi gi site takvi broevi.
434. U~enik pro~ital kniga od 480 stranici, sekojdnevno ~itaj}i ist
broj stranici. Koga bi ja ~ital knigata taka {to sekoj sleden den
~ita po 16 stranici pove}e, toga{ bi ja pro~ital za 5 dena porano.
Kolku dena u~enikot ja ~ital knigata?
435. Od nekoja to~ka A na kruæna pateka, trgnale dvajca velosipedisti
vo sprotivna nasoka. Posle nekoe vreme tie se sretnale i se
poka`alo deka prviot velosipedist pominal 7m pove}e od vtoriot.
Prodolæuvaj}i ponatamu (vo isti nasoki), prviot velosipedist
pristignal vo to~kata A za 6 sekundi, a vtoriot velosipedist za 9
sekundi. Kolku e dolga patekata?
436. Sekoj ~len na edno semejstvo ispil po edna polna ~a{a kafe so
mleko, pri {to Katerina ispila ~etvrtinka od celoto mleko i
{estinka od celoto kafe. Kolku ~lena ima vo semejstvoto?
437. Za edna godina, brojot na `iteli na edno selo se zgolemil za n
lu|e, a vo vtorata godina za 300 lu|e. Pritoa, vo prvata godina
brojot na `iteli se zgolemil za 300%, a vo vtorata godina za n%.
Posle tie dve godini, kolku `iteli imalo seloto?
438. Godinite na nekoj gra|anin na Makedonija roden vo XX vek, vo 2017
godina, bile ednakvi so zbirot na cifrite na godinata vo koja bil
roden. Kolku godini imal toj gra|anin?
439. Patnik, ~ekaj}i na aerodromot, re{il da napravi eksperiment.
Dvi`ej}i se po elevator (podvi`na lenta), patot go pominal za
edna minuta. Dvi`ej}i se vo sprotiven pravec istiot pat go
pominal za tri minuti. Za kolku vreme patnikot bi go izminal
istiot pat po elevatorot, ako toj e vo miruvawe?
440. Dvajca lu|e stoele eden do drug, svrteni so grb, na prugite na edna
mala `elezni~ka stanica. Koga glavata na ekspresniot voz (koj ne
zapira na ova stanica) stignuva do niv, tie po~nuvaat da odat vo
sprotivni nasoki, paralelno so prugite. Koga krajniot del na vozot
go stignuva sekoj od niv, tie zastanuvaat, imaj}i pritoa pominato
edniot trieset metri a drugiot ~etirieset metri. Ako i dvajcata se
dvi`ele so ista (konstantna) brzina , a i vozot se dvi`el so druga
(konstantna) brzina toga{ kolku bil dolg vozot?
441. Za numerirawe na stranicite na edna kniga se iskoristeni n pati
pove}e cifri, otkolku brojot na stranici. Kolku stranici ima
knigata?
442. Most e napraven od legura na dva metali koi se vo odnos 1:2, a drug
most gi sodr`i istite metali, no vo odnos 2:3. Kolkav del mo`e da
se zeme od sekoj most, za da se napravi nov most, koj }e gi sodr`i
metalite vo odnos 17:27?
443. Atanas, Blagoja i Vasko trebalo da pominat so avtobus odreden pat
vozej}i na sledniot na~in:
Atanas vozel 90 km/h-2 min., potoa 100 km/h-2 min. i se taka
naizmeni~no. Blagoja vozel 2km so 90 km/h, potoa 2km so 100 km/h

43
Linearni ravenki i neravenki

i se taka naizmeni~no. Vasko vozel 2km so 90 km/h, potoa 100 km/h-


2 min. i se taka naizmeni~no. Podredi gi imiwata na voza~ite,
po~nuvaj}i od najkratkoto vreme {to mu e potrebno za da go pomine
odredeniot pat.
444. Na kolku razli~ni na~ini mo`eme da dobieme 2000 denari od to~no
1000 moneti so vrednost od 1, 2 ili 5 denari?
445. Ako m i n se celi broevi za koi 49<n<101 i 19<m<51, toga{ koja e
najgolemata vrednost na izrazot (n+m)/n?

44
Teorija na broevi

Teorija na broevi

446. Doka`i deka p(n)=n(n+1)(4n+5) e deliv so 6 za sekoj priroden broj n.


447. Doka`i deka p(n)=n(n2-1)(n2-5n+26) e deliv so 120 za sekoj
priroden broj n.
448. Doka`i deka p(n)=n47-n11 e deliv so 10 za sekoj priroden broj n.
449. Doka`i deka 7|(22225555+55552222).
450. Doka`i deka a) 17|(3ÿ52n+1)+(33n+1); b) 84|(42n-32n-7) za sekoj priroden
broj n.
451. Doka`i deka za sekoj nenegativen cel broj n, brojot an=n5-5n3+4n e
deliv so 120.
452. Neka n e neparen cel broj. Doka`i deka n3 + 3n2 –n – 3 e deliv so 48.
453. Ako n e neparen broj toga{ 7 e delitel na izrazot 6n+8n.Doka`i!
454. Najdi gi site prirodni broevi n za koi brojot n10+1 e deliv so 10.
455. Prirodnite broevi a, b, c i d se takvi {to ab=cd. Doka`i deka
brojot a+b+c+d ne mo`e da bide prost broj.
456. Dali postojat 2011 razli~ni prirodni broevi za koi zbirot na koi
bilo 2010 broja e deliv so preostanatiot broj?
457. Kaj kolku ~etiricifreni broevi zbirot na prvite dve cifri e
ednakov so zbirot na poslednite dve cifri.
458. Ako 5 ne e delitel na nitu eden od broevite a, b, c, d i e, doka`i
deka 5 e delitel na zbirot a4+b4+c4+d4+e4.
459. Kolku deliteli na 30101 ne se deliteli na 20100?
460. Za prirodnite broevi x i y brojot x2+xy+y2 e deliv so 10 . Doka`i
deka brojot x2+xy+y2 e deliv i so 100.
461. Neka a, b, c se celi broevi za koi izrazite ab+9b+81 i bc+9c+81 se
delivi so 2015. Doka`i deka i ca+9a+81 isto taka e deliv so 2015.
462. Ako a+b+c e deliv so 30 toga{ a5+b5+c5 isto taka e deliv so 30.
Doka`i.
463. Ako razlikata abc - def e deliva so 7 toga{ i brojot abcdef e deliv
so 7. Doka`i.
464. Ako za celite broevi x, y, z va`i (x-y)(y-z)(z-x)=x+y+z toga{ x+y+z e
deliv so 27. Doka`i.
465. Najdi gi site vrednosti na x za koi brojot 88...8 123 e deliv so
123 x 99...9
50 pati 50 pati
7.
466. Za koi prirodni broevi n, brojot 1313...13
14243 e deliv so 63?
2n cifri
467. Najdi gi site nenegativni celi broevi n za koi 2010+n e deliv so
n+1.

45
Teorija na broevi

468. Neka S e mno`estvo od 400 posledovatelni celi broevi. Kolku


razli~ni parovi (x,y) postojat (x,yœS), za koi se ispolneti uslovite:
x∫y i x+y e deliv so 80 ili x=y i x+y e deliv so 160.
469. Odredi go zbirot na site dvocifreni broevi koi se delivi so
svoite cifri.
470. Odredi go ~etiricifreniot broj xyzt , koj ja zadovoluva ravenkata
4xyzt = tzyx .
471. Neka a, b, c se neparni prirodni broevi. Doka`i deka barem eden
od broevite ab-1, bc-1, ac-1 e deliv so 4.
472. Odredi gi site tricifreni broevi xyz koi se ednakvi so
vrednosta na izrazot x+y+z+xy+xz+yz+xyz .
473. Odredi gi site tricifreni broevi abc koi {to se poln kvadrat,
taka {to i brojot (a − 1)(b − 1)(c − 1) e poln kvadrat.
474. Odredi gi site tricifreni broevi abc za koi va`i ravenstvoto
abc = 11⋅ (a2 + b2 + c 2 ) .
475. Kolku petcifreni broevi abcde postojat za koi cifrite b i d se
zbir od nivnite sosedni cifri (b=a+c; d=c+e)?
476. Opredeli {estcifren broj koj pomno`en so 2, 3, 4, 5 i 6 gi dava
broevite (vo proizvolen red) koi {to se dobivaat so cikli~no
premestuvawe na negovite cifri. (Pr: So cikli~no premestuvawe
na cifrite na 395017 se dobivaat 950173, 501739…).
ab ab a
477. Najdi gi site dropki od vidot < 1 , ako = .
bc bc c
478. Odredi go brojot na neparni deliteli na brojot 7!.
479. Neka a, b, c i d se celi broevi taka {to izrazot a4+b4+c4+d4 e deliv
so 5. Doka`i deka izrazot abcd e deliv so 625.
480. Ako p i q se dva posledovatelni neparni broevi (p>q) toga{ pp+qq e
deliv so p+q .Doka`i.
481. Neka a i b se celi broevi za koi broevite a+2b i b+2a se to~ni
kvadrati. Doka`i deka sekoj od broevite a i b e deliv so 3.
482. Doka`i deka za sekoj priroden broj n, izrazot
xn(y-z)+yn(z-x)+zn(x-y) e deliv so (x-y)(y-z)(z-x).
483. Presmetaj go zbirot na site ~etiricifreni broevi {to se
sostaveni od neparni cifri.
484. Neka a1, a2, …, ap se site deliteli na m i b1, b2, …, bq se site
deliteli na n. Ako a1+a2+…+ap=b1+b2+…+bq i
1 1 1 1 1 1
+ + ... + = + + ... + toga{ m=n. Doka`i.
a1 a2 ap b1 b2 bq
485. Odredi go najmaliot priroden broj n za koj 29| aa...a
123 .
n cifri

46
Teorija na broevi

486. Vo edna nagradna igra u~estvuvaat taloni na koi se napi{ani


{estcifreni broevi (cifrata 0 mo`e da stoi napred. Pr: 015420).
Talonot go narekuvame sme{en ako zbirot na prvite tri cifri e
ednakov na zbirot na poslednite tri cifri. Doka`i deka zbirot na
broevite na site sme{ni taloni e deliv so 13.
487. Proizvodot na tri posledovatelni prirodni broevi e 33 pati
pogolem od nivniot zbir. Odredi go zbirot na cifrite na ovoj
proizvod.
1 1 1
488. Doka`i deka ravenkata 2
+ + 2 = 1 nema re{enie vo
x xy y
mno`estvoto prirodni broevi.
489. Vo mno`estvoto na celi broevi re{i ja ravenkata
x2(y-1)+y2(x-1)=1;
490. Vo mno`estvoto na celi broevi re{i ja ravenkata
x3+2021y=y3+2021x.
491. Doka`i deka broevite 11...1
123 i 11...1
1 23 se zaemno prosti ako i samo ako
m n
m i n se zaemno prosti.
492. Brojot A se sostoi od 2n edinici, brojot B se sostoi od n+1 edinici,
a brojot C se sostoi od n edinici. Doka`i deka brojot N=A+B+6C+8
e to~en kvadrat.
493. Odredi go brojot na prirodni broevi pomali od 2015 takvi {to
( n ) (
2015 e deliv so 2014 .
n )
494. Sekoj prost broj р pogolem od 3 e od oblik 6k-1 ili 6k+1. Doka`i.
495. Doka`i deka broevite 2 n − 1 i 2 n + 1 (n > 2) , nemo`at istovremeno da
bidat prosti.
496. Odredi gi site prosti broevi p, q i r i site prirodni broevi n za koi
1 1 1 1
va`i ravenstvoto + + = .
p q r n
497. Ako stranite na triagolnikot ABC se prosti broevi toga{ negovata
plo{tina ne e cel broj. Doka`i.
498. Neka a=32012+2. Dali a2+2 e prost broj?
499. Najdi gi site prirodni broevi n za koi 6n2+5, 2n2+3 i n2+1 se prosti
broevi.
500. Najdi gi site prosti broevi r za koi 2r+r2 e isto taka prost broj.
501. Odredi gi site prosti broevi p takvi {to 4 p + p 4 e isto taka prost
broj.
502. Odredi gi site prosti broevi p za koi p| (7p+13).
503. Odredi gi site prosti broevi p za koi broevite p+10 i p+20 se isto
taka prosti broevi .
504. Ako broevite p i 8p-1 se prosti toga{ brojot 8p+1 e slo`en.
Doka`i.
505. Ako a>1 i ap+1 e prost broj toga{ p e stepen na 2. Doka`i.

47
Teorija na broevi

506. Najdi gi site prosti broevi p i q takvi {to p+q=(p-q)3.


507. Poznato e deka broevite 2n+1 i 3n+1, za n priroden broj, se to~ni
kvadrati. Dali brojot 5n+3 e prost broj?
508. Ako p e prost broj, najdi gi site broevi od oblik 2p+1 koi
pretstavuvaat tret stepen na nekoj priroden broj.
509. Neka n e priroden broj pogolem od 1. Doka`i deka ako 8n + 1 i
24n+1 se istovremeno polni kvadrati toga{ 8n+3 e slo`en broj.
510. Odredi gi site prosti broevi koi mo`at da se zapi{at i kako zbir
i kako razlika na dva prosti broja.
511. Ako p e prost broj pogolem od 3, doka`i deka p2-1 e deliv so 24.
512. Odredi gi site prosti broevi p za koi p2-1 e kub na nekoj priroden
broj.
513. Odredi gi site prosti broevi p za koi postojat prirodni broevi a i
1 1 1
b takvi {to = 2 + 2 .
p a b
514. Doka`i deka brojot 101010...101, napi{an so 100 nuli i 101
edinici, ne e prost broj.
515. Doka`i deka za proizvolen prost broj p pogolem od 5, brojot
N = 11...1
123 22...2
1 3 33...3
424 1
424 1
424 3 − 123...9 e deliv so p.
3...99...9
p p p p
516. Neka а, b i c se prirodni broevi i p= bc+а, q=ab+c, r=ca+b. Doka`i
deka ako p, q i r se istovremeno prosti toga{ barem dva od niv se
ednakvi.
517. Ako p e prost broj pogolem od 3 toga{ p2+11 ima pove}e od {est
prirodni deliteli. Doka`i.
518. Vo mno`estvoto na prosti broevi re{i ja ravenkata xy+1=z.

48
Kompleksni broevi

Kompleksni broevi

519. Presmetaj ja vrednosta na izrazot


(1 + i)2014 .
(1 − i)2010 − i ⋅ (1 + i)2012
520. Presmetaj ja vrednosta na izrazot A = 1 + i 3 + 1 − i 3 .
(Zada~a na Lajbnic)
2 3 2018
1+ i  1+ i   1+ i   1+ i 
521. Presmetaj ja vrednosta na zbirot +   +   + ... +   .
2  2  2  2
2015
 1+ i 
522. Ako z =   toga{ presmetaj 6z6.
 2
523. Za koi vrednosti na n va`i ravenstvoto i+2·i2+3·i3+…+n·in=48+49i.
524. Za kompleksnite broevi z i w va`i z = w = z − w .
2019
z
Presmetaj   .
w
525. Ako 
cosx + cosy + cosz = 0
toga{ sin3x=sin3y=sin3z. Doka`i.
sinx + siny + sinz = 0
526. Neka z=a·i e re{enie na ravenkata z4-2z3+7z2-4z+10=0. Odredi ja
vrednosta na a i re{enijata na dadenata ravenka.
527. Neka x i y se kompleksni broevi razli~ni od nula koi ja
zadovoluvaat ravenkata x2+xy+y2=0. Presmetaj ja vrednosta na
2013 2013

izrazot  x   y 
+   .
x+y
  x+y
528. Ako za kompleksnite broevi a, b, c i d va`at ravenstvata
1 1 1 1
a+b+c+d=-2 i + + + = −2 (a, b, c, d se razli~ni od nula i
a b c d
1 1 1 1
eden) toga{ + + + = −2 . Doka`i.
a -1 b -1 c -1 d -1
529. Nizata od kompleksni broevi z0, z1, z2, ... e definirana so zë=1-i i
z +i
zn+1 = n . Najdi z2021.
zn − i
530. Ako z1 i z2 se kompleksni broevi toga{ va`at ravenstvata:
a) |z1+z2|2+ |z1−z2|2=2(|z1|2+|z2|2 );
b) |1+z1· z 2 |2 + |z1−z2 |2=(1+|z1|2)·(1+|z2|2). .

49
Kompleksni broevi

531. Ako za kompleksnite broevi z1, z2 va`at ravenstvata |z1|=|z2|=1 i


z1 + z 2
z1·z2∫−1 toga{ e realen broj. Doka`i.
1 + z1 ⋅ z 2
532. Neka za kompleksniot broj z va`i |z|=1 i z2n∫-1. Doka`i deka
zn e realen broj.
1 + z 2n
533. Neka z1, z2, ..., zn se n kompleksni broevi za koi |z1|=|z2|=...=|zn|=r>0.
Doka`i deka
(z 1
+z
2
)⋅ (z 2
+z
3
)⋅ ... ⋅ (z n −1
+z
n
)⋅ (z n
+z
1
) e realen broj.
z ⋅ z ⋅ ... ⋅ z
1 2 n
2017
534. Ako S=Re(1+i) toga{ presmetaj log2S.
535. Odredi go vkupniot broj parovi (a,b) od realni broevi za koi va`i
ravenstvoto (a-bi)2020=a+bi.
536. Neka z1, z2 i z3 se kompleksni broevi za koi |z1|=|z2|=|z3|=1 i
z1+z2+z3=0. Doka`i deka z12+z22+z32=0.
537. Ako za kompleksnite broevi a, b, c va`i a = b = c = 1 toga{
a + b + c = ab + bc + ca . Doka`i.
538. Ako z1, z2, z3 se kompleksni breovi takvi {to z1 = z 2 = z 3 i
2 2 2
z1ÿz2ÿz3=0 тогаш изразот z1 + z 2 + z 2 + z 3 + z 3 + z1 ima konstantna
vrednost. Doka`i.
539. Neka z1, z2 i z3 se kompleksni broevi takvi {to z1+z2+z3=0 i
z1z2+z2z3+z3z1=0. Doka`i deka va`i z1 = z2 = z3 .
540. Neka z1, z2 i z3 se kompleksni broevi takvi {to |z1|=|z2|=|z3|=r>0
z z +z z +z z
i z1+z2+z3∫0. Doka`i deka 1 2 2 3 3 1
=r.
z +z +z
1 2 3
2
1+ z + z
541. Neka zœC\R i œR. Doka`i deka |z|=1.
1− z + z 2
sinx + siny + sinz = 0
542. Neka x, y, z se realni broevi za koi  . Doka`i
cosx + cosy + cosz = 0
sin2x + sin2y + sin2z = 0
deka  .
cos2x + cos2y + cos2z = 0
543. Neka z1, z2 i z3 se kompleksni broevi za koi va`at ravenstvata
z1+z2+z3=1, z12+z22+z32=3 i z13+z23+z33=7. Presmetaj ja vrednosta na
izrazot z110+z210+z310.

50
Kvadratni ravenki i kvadratni funkcii

Kvadratni ravenki i kvadratni funkcii

544. Re{i ja ravenkata (x+1)2+(2x+1)2+…+(99x+1)2=99.


545. Re{i ja ravenkata (x-4)(x-5)(x-6)(x-7)=1680 vo mno`estvoto R.
546. Vo mno`estvoto prirodni broevi, re{i ja ravenkata x2-y2=xy+61.
547. Doka`i deka ravenkata x2+x=y2 nema re{enie vo mno`estvoto
prirodni broevi.
548. Ravenkata 2m2+n2=2mn+3n, re{i ja vo mno`estvoto celi broevi.
549. Re{i ja ravenkata 1 2 2015
+ + ... + = 2015 .
x - x +1 x - x + 2
2 2
x - x + 2015
2

550. Re{i gi ravenkite:


a) 2 1 + 2
2
= 2
6
;
x - 2x + 2 x - 2x + 3 x - 2x + 4
b) 1 1 1 1
+ + + = 2.
x(x + 1) (x + 1)(x + 2) (x + 2)(x + 3) (x + 3)(x + 4)
551. Najdi gi site prirodni broevi n za koi n2+25n+19 e to~en kvadrat.
552. Za koi vrednosti na celiot broj n, kvadratniot trinom n2-20n+244
e to~en kvadrat?
553. Korenite na ravenkata x2+px+q=0 se celi broevi, pri {to p+q=198.
Odredi gi korenite na ravenkata, p i q.
554. Za koi vrednosti na р i q, ravenkata х2+рх+q=0 ima dva razli~ni
koreni 2р i p+q?
555. Za koi vrednosti na parametrot m, razlikata od korenite na
kvadratnata ravenka x2-6x+m2-4m+12=0 ima najgolema vrednost?
556. Za koi vrednosti na parametrot q, zbirot od ~etvrtite stepeni na
korenite na ravenkata x2-x+q=0, ima najmala vrednost?
557. Presmetaj ja minimalnata vrednost na izrazot x12+x22, ako x1, x2 se
realni re{enija na ravenkata x2-2mx-(2m+2)=0, za nekoj realen
parametar m.
558. Neka a, b se razli~ni celi broevi za koi ravenkata (x-a)(x-b)=-1 go
ima kako cel koren brojot r. Doka`i deka a+b=2r.
559. Ravenkite ax²+bx+c=0 i cx²+bx+a=0 imaat zaedni~ki koren. Doka`i
deka (a+c)²=b².
560. Za kvadratniot trinom x2+bx+c e poznato deka i dvata koreni na
soodvetnata kvadratna ravenka le`at vo intervalot (2,3). Doka`i
deka 5b+2c+12<0.
561. Koeficientite na ravenkata ax2+bx+c=0 go zadovoluvaat uslovot
2a+3b+6c=0. Doka`i deka ravenkata ima koren na intervalot (0,1).

51
Kvadratni ravenki i kvadratni funkcii

562. Za pozitivnite broevi a, b, c va`i a+b+c=1. Doka`i deka ravenkata


a+bx+cx2=x ima koren x0œ(0,1) ako i samo ako b+2c>1.
563. Pozitivnite broevi a, b i c go zadovoluvaat ravenstvoto a+b+c=1.
Doka`i deka ravenkata ax²+ bx+c=x ima koren x1œ(0,1) ako i samo
ako a+b>1.
564. Dadeni se celite broevi a, b i c (c≠b). Poznato e deka kvadratnite
trinomi ax2+bx+c i (c-b)x2+(c-a)x+(a+b) imaat zaedni~ki koreni (ne
mora da se celi broevi). Doka`i deka zbirot a+b+2c e deliv so 3.
565. Poznato e deka x1>0 i x2=4x1, kade x1 i x2 se nuli na funkcijata
f(x)=ax2+bx+c. Za realnite koeficienti a, b i c va`i ravenstvoto
3a=2(c-b). Odredi ja vrednosta na x1.
566. Neka x1, x2 se re{enija na ravenkata x2-1154x+1=0. Presmetaj ja
vrednosta na izrazot 4 x 1 + 4 x 2 .
567. Opredeli gi site parovi celi broevi (a,b), taka {to a+b e koren na
ravenkata x2+ax+b=0.
568. Za koi prirodni broevi a, b korenite na ravenkata x2-abx+a+b=0 se
celi broevi?
569. Opredeli go zbirot od kubovite na korenite na ravenkata
x2+rx+q=0.
570. Odredi gi site vrednosti na k (k∫0), za koi korenite x1 i x2 na
ravenkata kx²+kx-2=0 gi zadovoluvaat neravenstvata |x1|<1 i |x2|>1.
571. Voz koj se dvi`i so konstantna brzina treba da pomine pat dolg
360km. Ako vozot se dvi`i so brzina pogolema za 5 km/~ toga{ }e
pristigne 1~ porano. Odredi ja brzinata na vozot.
572. Razli~nite broevi a, b, c se takvi {to x2+ax+1=0 i x2+bx+c=0 imaat
zaedni~ki realen koren. Osven toa zaedni~ki realen koren imaat i
ravenkite x2+x+a=0 i x2+cx+b=0. Najdi go zbirot a+b+c.
573. Razli~nite realni broevi а, b, c i d se takvi {to a i b se koreni
na ravenkata x 2 - 3cx - 8d = 0, a c i d се корени на равенката
x 2 - 3ax - 8b = 0 . Presmetaj go zbirot а+b+c+d.
574. Ako a i b se koreni na ravenkata x2+cx+d=0, a c i d koreni na
ravenkata x2+ax+b=0, toga{ odredi gi a, b, c i d.
575. Korenite na ravenkata x2+mx+1=n se celi broevi, razli~ni od
nula. Doka`i deka m2+n2 e slo`en broj.
576. Doka`i deka kvadratnite ravenki ax 2 + bx + c = 0 i bx 2 + cx + a = 0
kade а,b,сœR a∫0 , b∫0, imaat zaedni~ko re{enie ako i samo ako
a 3 + b 3 + c 3 = 3abc .
577. Neka a i b se realni broevi. Doka`i deka barem edna od ravenkite
x2+2ax+b=0, ax 2 +2bx+1=0, bx2+2x+a=0 ima realni koreni.
578. Ako za kvadratnata funkcija f(x)=ax2+bx+c e poznato deka ima dva
razli~ni realni koreni i f(5-x)=f(x-5) "xœR, toga{ presmetaj go
zbirot na nejzinite koreni.

52
Kvadratni ravenki i kvadratni funkcii

579. Neka f(x)=ax2+bx+c e kvadratna funkcija so realni koeficienti.


Ako f(2i)=0 toga{ opredeli ja vrednosta na zbirot a+b+c.
580. Najdi gi site realni broevi a i b za koi funkcijata f(x)=x2+ax+b
prima realni vrednosti od intervalot [-2,2], za site xœ[0.4]
581. Poznato e deka (a+b+c)c<0. Doka`i deka b2>4ac.
582. Doka`i deka ako ravenkata ax2+bx+c=0 (a≠0) so celi koeficienti
ima racionalen koren toga{ barem eden od koeficientite a, b, c
e paren broj.
583. Najdi gi site prosti broevi p i q za koi ravenkata px2-qx+p=0 ima
racionalni re{enija.
584. Poznato e deka ravenkata х2+5х+с=0 ima koreni х1 i х2 (х1∫х2), a
ravenkite у2+2х1·у+2х2=0 i z2+2х2·z+2x1=0 imat zaedni~ki koren.
Kolku e c ?
585. Ako r e re{enie na ravenkata x2+x-1=0, toga{ presmetaj ja
1 3 5 23
 1  2 1   4 1   2048 1 
vrednosta na izrazot r +  + r + 2  + r + 4  + ...+ r + 2048  .
 r  r   r   r 
586. Odredi gi site parovi (a,b) od realni broevi, takvi {to koga x1 e
re{enie na ravenkata x2+ax+b=0, toga{ i x12-2 da e istotaka
re{enie na dadenata ravenka.
587. Najdi go zbirot na site prirodni broevi B takvi {to
(111)B=(aabbcc)6 kade a, b i c se cifri vo broen sistem so osnova 6.

53
Plo[tina na ramninski figure

Plo{tina na ramninski figuri

588. a) Presmetaj ja plo{tinata na kvadratot vpi{an vo triagolnikot


ABC (crt. 22);
b) Presmetaj ja plo{tinata na kvadratot vpi{an vo triagolnikot
ABC (crt. 23).

Crte` 22 Crte` 23
589. Doka`i deka zbirot od rastojanijata od
proizvolna vnatre{na to~ka na ramnostran
triagolnik do stranite, e ednakov so visinata na
triagolnikot.
590. Nad stranite na triagolnikot ABC, vpi{an vo
kru`nica so dijametar AB, se konstruirani
polukrugovi (kako na crt. 24). Doka`i deka
plo{tinata na {trafiraniot del e ednakva so
plo{tinata na triagolnikot ABC.
Crte` 24
591. Tri kruga me|usebno nadvore{no se dopiraat pri
{to rastojanijata me|u nivnite centri iznesuvat
3cm, 4cm i 5cm. Presmetaj go zbirot od plo{tinite
na tie tri kruga.
592. Dva skladni ramnostrani triagolnici formiraat
yvezda kako na crte` 25. Ako pravilniot
{estagolnik ima plo{tina 60 toga{ kolkava e
plo{tinata na eden ramnostran triagolnik?
Crte` 25
593. a) Doka`i deka od te`i{nite linii na proizvolen triagolnik ABC
mo`e da se konstruira drug triagolnik.
b) Odredi go odnosot od plo{tinite na tie dva triagolnika.
v) Odredi go zbirot od plo{tinite na triagolnicite dobieni od
te`i{nite linii na prethodniot triagolnik ako po~etniot
triagolnik e so strani 5, 12, 13.
594. Presmetaj ja plo{tinata na ramnokrak triagolnik, ako dol`inata
na otse~kata {to gi soedinuva sredinite na osnovata i krakot e

54
Plo[tina na ramninski figure

ednakva na polovina od radiusot na opi{anata kru`nica okolu toj


triagolnik.
595. Daden e triagolnik ABC i to~ka O. Ako T1, T2 i T3 se te`i{tata vo
triagolnicite AOB, BOC i COA, soodvetno, toga{ doka`i deka
1
P∆ T1T2 T3 = P∆ABC .
9
596. Daden e triagolnik ABC. Na polupravite AB, CB i AC opredeleni
se to~ki A1, B1 i C1, soodvetno, takvi {to
BA 1 = 2BA, CB 1 = 2CB, AC 1 = 2 AC. Odredi go odnosot od plo{tinite
na triagolnicite ABC i A1B1C1 .
1
597. Ako T e te`i{teto na triagolnikot ABC toga{ PóATB= PóABC.
3
Doka`i.
598. Vo ramnokrak triagolnik ABC (so AB = a za osnova) e daden
odnosot pome|u osnovata i krakot (a:b). Odredi go odnosot na
plo{tinata na triagolnikot ABC i triagolnikot ~ii temiwa se
prese~nite to~ki na simetralite na aglite i stranite na dadeniot
triagolnik.
599. To~kite M i N le`at na stranite AB i BC, soodvetno, na
triagolnikot ABC. Otse~kite AN i CM se se~at vo to~ka О. Neka
plo{tinite na triagolnicite AOM, AOC i CON se S1, S2, i S3,
soodvetno. Najdi ja plo{tinata na triagolnikot MBN.
600. Neka to~kite D, E, F gi delat stranite na triagolnikot ABC vo
2
odnos 2:1. Doka`i deka PóADF=PóDBE=PóECF= PóABC.
9
601. To~kata O le`i vo vnatre{nosta na
triagolnikot ABC. Pravite AX, BY i CZ
minuvat niz to~kata O i go delat
triagolnikot ABC na {est pomali
triagolnici. Ako {rafiranite
triagolnici (crt. 26) imaat ednakvi
plo{tini toga{ site {est triagolnici
imaat ednakvi plo{tini. Doka`i. Crte` 26
602. Niz proizvolna to~ka K od
vnatre{nosta na triagolnikot
ABC se povle~eni tri pravi
paralelni so stranite na
triagolnikot ABC. Tie pravi go
delat triagolnikot ABC na tri
triagolnici so plo{tini P1, P2,
P3. Presmetaj ja plo{tinata na
triagolnikot ABC. Crte` 27

55
Plo[tina na ramninski figure

603. Doka`i deka va`i ravenstvoto P = P1 + P2 + P3 + 2 P4 kade P1,


P2, P3 i P4 se plo{tinite na triagolnicite ozna~eni na crte` 27, a
P e plo{tinata na triagolnikot ABC.
604. Niz to~kata O, koja le`i vo vnatre{nosta na triagolnikot ABC se
povle~eni otse~ki paralelni so stranite.
Otse~kite AA1, BB1 i CC1 go delat
triagolnikot ABC na ~etiri triagolnici i
tri ~etiriagolnici (crt. 28). Doka`i deka
zbirot od plo{tinite na trite triagolnici
{to gi sodr`at temiwata A, B i C, e ednakov
so plo{tinata na ~etvrtiot triagolnik .
Crte` 28
605. Presmetaj ja plo{tinata na pravoagolen triagolnik so hipotenuza c
i zbir na kateti k.
606. To~kite M, N, Q le`at na stranite AB, BC i CA, soodvetno pri {to
AM : AB = BN: BC = CQ : CA =1:3. Ako plo{tinata na triagolnikot MNQ
e S toga{ najdi ja plo{tinata na triagolnikot ABC.
607. Neka a e dol`inata na stranata na triagolnik so soodvetna visina
h. Ako a+h=16 toga{ opredeli ja maksimalnata mo`na vrednost na
plo{tinata na toj triagolnik.
608. Neka to~kata S e od vnatre{nosta na triagolnikot ABC. Doka`i
deka proizvodot na rastojanijata od to~kata S do stranite na
triagolnikot ABC e najgolem koga to~kata S e te`i{te na toj
triagolnik .
609. Doka`i deka od site triagolnici so ednakvi plo{tini,
ramnostraniot triagolnik ima najmal perimeter.
610. Doka`i deka od site triagolnici so ednakvi perimetri
ramnostraniot triagolnik ima najgolema plo{tina.
611. Neka М e sredina na hipotenuzata АВ vo pravoagolniot triagolnik
АВС. Doka`i deka pravata {to minuva niz centrite na vpi{anata
kru`nica vo triagolnicite AMC i BMC go deli triagolnikot ABC
na dve dela so ednakvi plo{tini.
612. Neka a, b, c se strani, s poluperimetar, a P plo{tina na
triagolnikot ABC pri {to a≤c i b≤c. Doka`i deka triagolnikot
ABC e pravoagolen ako i samo ako (s-a)(s-b)=P.
613. Neka M i N se to~ki na stranite AB i BC na
triagolnikot ABC,takvi {to
AM : MB = BN : NC = m : n i neka AN i CM se
se~at vo to~kata D. Doka`i deka
PóADC=PóMBND.
614. Visinata CD i bisektrisata AE na
pravoagolniot triagolnik АВС (®C=90o) se
se~at vo to~ka F. Neka G e prese~nata to~ka
na pravite ED i BF. Doka`i deka PCEGF=PDBDG. Crte` 29

56
Plo[tina na ramninski figure

615. Daden e ramnostran triagolnik ABC so strana 1. To~kite D, E i F se


taka izbrani {to AD = BE = CF = 1 . Pravite AE, BF i CD se se~at vo
4
to~kite G, H i I (crt. 29). Odredi go odnosot od plo{tinite na
triagolnicite GHI i ABC.
616. Vo vnatre{nosta na ramnostran triagolnik ABC e izbrana to~kata
P, taka {to PA = 3 , PB = 4 , PC = 5 . Presmetaj ja plo{tinata na
triagolnikot ABC.
617. Neka V e centarot na vpi{anata
kru`nica, a V1 centarot na pripi{anata
kru`nica na triagolnikot ABC i neka E i
D se dopirnite to~ki so prodol`enijata
na stranite AC i AB, soodvetno (crt. 30).
Ako G i F se prese~nite to~ki na
otse~kite VD i VE so stranata BC,
soodvetno, doka`i deka
P∆VGF = P∆BDG + P∆FEC . Crte` 30
618. Neka O e centarot na opi{anata kru`nica, a V centarot na
vpi{anata kru`nica vo
triagolnikot ABC (crt. 31). Neka
I, J i Q se centrite na
pripi{anite kru`nici vo
triagolnikot ABC, a K, M i T se
dopirnite to~ki na tie kru`nici
so triagolnikot ABC. Ako E, F i D
se dopirnite to~ki na vpi{anata
kru`nica vo triagolnikot ABC
toga{ va`i ravenstvoto
P∆EFD = P∆KMT .
619. Doka`i deka plo{tinata na
paralelogramot, ~ii temiwa se
sredini na konveksen
~etiriagolnik, e polovina od
plo{tinata na ~etiriagolnikot. Crte` 31
620. Neka O e vnatre{na to~ka na paralelogramot ABCD. Doka`i deka
zbirot od plo{tinite na
triagolnicite AOB i COD e ednakov
so zbirot od plo{tinite na
triagolnicite BOC i AOD.
621. Neka P1 i P3 se plo{tinite na
triagolnicite ozna~eni na crte` 32, a
P2 i P se plo{tinite na ozna~enite
~etiriagolnici. Doka`i deka
P=P1+P2+P3. Crte` 32

57
Plo[tina na ramninski figure

622. Neka P1, P2 i P se plo{tinite na


~etiriagolnicite ozna~eni na crte` 33
kade M i N se sredini na osnovite, a K e
sredina na krakot AD na trapezot ABCD.
Doka`i deka P=P1+P2.

Crte` 33
623. Plo{tinata na paralelogramot ABCD
(crt. 34) e 120. Presmetaj ja plo{tinata
na paralelogramot MNPQ ako to~kite E,
F, G i H se sredini na stranite na
paralelogramot.

Crte` 34
624. Na stranite BC, CA i AB na triagolnikot АВС se zemeni to~kite М,
N i P, soodvetno, pri {to ~etiriagolnikot СNPM e paralelogram.
Neka {O} = AM ∩ BN,{A 1} = AM ∩ PN i {B1} = BN ∩ PM . Doka`i deka
P∆MNO = P∆B1OA! ⋅ P∆B1CA1 .
625. Neka pravata p koja e paralelna so stranata AV
na paralelogramot ABCD, ja se~e BC vo to~ka K,
AC vo to~ka L i AD vo to~ka M. Doka`i deka
P∆ABK = P∆ALD .
626. Vo kru`nica so r = 2 e vpi{an kvadrat a nad
kvadratot e vpi{an pomal kvadrat (crt. 35).
Odredi go odnosot od plo{tinite na pogolemiot
sprema pomaliot kvadrat.
Crte` 35
627. Na stranite na edini~niot kvadrat se zemeni to~kite K, L, M i N
taka {to KM e paralelna so dve strani na kvadratot a LN e
paralelna so drugite dve strani. Ako otse~kata KL otsekuva od
kvadratot triagolnik so perimeter eden, toga{ presmetaj ja
plo{tinata na triagolnikot {to go otsekuva od kvadratot
otse~kata MN.
628. Vo pravoagolnikot ABCD dadeni se to~kite E i F na stranite BC i
CD soodvetno. Ako triagolnikot AEF e ramnostran toga{ doka`i
deka P∆ECF=P∆ABE+P∆AFD.
629. Presmetaj ja plo{tinata na rombot ABCD ~ii radiusi na
kru`nicite opi{ani okolu triagolnicite ABC i ABD se r i R,
soodvetno.
630. Vo vnatre{nosta na kvadratot ABCD izbrani se to~ki P i Q takvi
{to ®PAQ=®PCQ=45°. Doka`i deka:
2 2 2
a) PQ = BP + QD ; b) PóPAQ+PóPCB+PóQCD=PóQCP+PóQAD+PóPAB.

58
Plo[tina na ramninski figure

631. Vo kvadratot ABCD to~kite D1, A1, B1 i C1 gi delat stranite


AB, BC, CD, DA vo odnos 1:2 (crt. 36). Neka otse~kite
AA1, BB1, CC1, DD1 se se~at vo to~kite M, N, P, Q.
Ako plo{tinata na kvadratot ABCD e P toga{
odredi ja plo{tinata na ~etiriagolnikot MNPQ.
632. Ako sekoja od dijagonalite AC i BD go deli
~etiriagolnikot ABCD na dva triagolnika so
ednakva plo{tina toga{ ABCD e paralelogram.
Doka`i.

Crte` 36
633. Daden e romb ABCD kaj koj radiusite na kru`nicite opi{ani okolu
triagolnicite ABD i ABC se R i R, soodvetno. Presmetaj ja
2
plo{tinata na rombot.
634. Dijagonalite na rombot se odnesuvaat kako 3:4. Odredi go odnosot
od plo{tinata na rombot i plo{tinata na vpi{aniot krug vo
rombot.
635. Dijagonalite na ~etiriagolnikot ABCD se se~at vo to~kata O pri
{to P2óABO+P2óCDO=P2óBCO+P2óADO. Doka`i deka to~kata O e sredina
na edna od dijagonalite.
636. Trapez so osnovi a i b se deli so otse~ka MN, paralelna so
osnovite, na dva dela so ednakvi plo{tini. Odredi ja dol`inata
na otse~kata MN.
637. Vo trapez ABCD dijagonalite AC i BD se se~at vo to~ka O. Ako
P∆AOD=m2 i P∆BOC=n2 toga{ PABCD=(m+n)2. Doka`i.
638. Na prodol`enieto na stranite AB, BC, CD, DA na konveksen
~etiriagolnik ABCD se naneseni vo edna nasoka soodvetno
otse~kite DM = AD, CQ = DC, BP = BC, AN = AB . Doka`i deka
PMNPQ=5PABCD .
639. Stranite BC i AD na konveksen ~etiriagolnik ABCD se razdeleni
na tri ednakvi delovi BM = MN = NC i AQ = QP = PD . Doka`i deka
1
PMNPQ= PABCD .
3
640. Vo konveksen ~etiragolnik ABCD sredinite na stranite AB i CD se
E i F. Neka otse~kite AF i DE se se~at vo to~ka K, a otse~kite BF i
CE se se~at vo to~ka L. Doka`i deka P∆AKD+P∆BLC=PELFK.
641. Vo ~etiriagolnikot ABCD to~kite K i L gi delat stranite AB i CD
vo odnos m:n. Otse~kite BL i CK se se~at vo to~ka P, otse~kite DK i
AL vo to~ka Q. Doka`i deka PKPLQ=PDBPC+ PDAQD.
642. Doka`i deka ako vo ~etiriagolnik so strani a, b ,c i d mo`e da se
vpi{e i opi{e kru`nica, toga{ va`i ravenstvoto P = abcd .
643. Neka M, N i P se dopirnite to~ki na vpi{anata kru`nica vo
triagolnikot ABC. Doka`i deka PóMNP:PóABC=r:2R.

59
Plo[tina na ramninski figure

644. Neka perimetarot na konveksniot ~etiriagolnik ABCD e 4. Doka`i


deka najgolemata mo`na vrednost na negovata plo{tina e 1.
645. Niz sredinata na sekoja dijagonala kaj
konveksniot ~etiriagolnik ABCD se
povle~eni pravi paralelni so drugata
dijagonala (crt. 37). Tie pravi se se~at
vo to~ka O. Doka`i deka otse~kite ~ii
kraevi se to~kata O i sredinite na
stranite na ~etiriagolnikot, go delat
toj ~etiriagolnik na ~etiri
~etiriagolnici so ednakvi plo{tini.

Crte` 37

646. Na crte` 38 e dadena kru`nica k so dijametar AB. Kru`nicata k1 ja


dopira kru`nicata k, a go dopira i
dijametarot AB vo centarot na k. Kru`nicata
k2 gi dopira kru`nicite k i k1 i dijametarot
AB. Najdi go odnosot na plo{tinata na
kru`nicata k i plo{tinata na kru`nicata k2.

Crte` 38

647. Neka L i K se sredini na stranite AB i CD vo konveksniot


~etiriagolonik ABCD i neka otse~kite AK i
LD se se~at vo to~kata P, a LC i BK vo to~kata
Q. Doka`i deka PLQKP = PóAPD + PóBCQ.

648. Plo{tinite na odredeni delovi na pravilen


petagolnik ABCDE se ozna~eni so x, y, z (crt.
39). Ako e poznata plo{tinata x toga{ odredi
gi plo{tinite y i z, kako i plo{tinata na
celiot petagolnik. Crte` 39

649. Vo konveksen {estagolnik AC1BA1CB1 e


poznato deka AB1 = AC 1 , BC 1 = BA 1 ,
CA 1 = CB 1 i ®A+®B+®C=®A1+®B1+®C1
1
(crt. 40). Doka`i deka P∆ABC = PAC 1BA 1CB 1 .
2

Crte` 40

60
Plo[tina na ramninski figure

650. Sekoja od stranite na konveksen ~etiriagolnik e razdelen na tri


ednakvi dela i soodvetni to~ki me|usebno se
povrzani formiraj}i pritoa devet
~etiriagolnici. Doka`i deka plo{tinata na
centralniot ~etiriagolnik e devet pati
pomala od plo{tinata na celiot
~etiriagolnik (crt. 41).

Crte` 41

651. Doka`i deka plo{tinata na kvadratot


ABCD (crt. 42) e ednakva so
plo{tinata na triagolnikot DEF .

Crte` 42

61
Elementi od stereometrija

Elementi od stereometrija

652. Paralelopiped so plo{tina na osnovata 70cm2 ima bo~ni yidovi so


plo{tini 28cm2 i 10cm2. Kolku e volumenot na paralelopipedot?
653. Vo topka so radius R e vpi{an kocka. Presmetaj ja plo{tinata na
kockata.
654. Odredi gi odnosite od plo{tinite na tri topki od koi ednata e
vpi{ana, drugata gi dopira rabovite a tretata e opi{ana okolu
kocka so rab a.
655. Presmetaj go volumenot na pravilna
~etiristrana prizma so osnoven rab a, ako
agolot me|u nejzinata dijagonala i eden bo~en
yid e 30о.
656. Odredi go odnosot od plo{tinite na kocka i
sfera vpi{ana vo taa kocka.
657. Presmetaj go volumenot na pravilna
triagolna prizma ABCA1B1C1 ako radiusot na
sferata vpi{ana vo piramida BCC1B1A e r. (crt. 43)
Crte` 43
658. To~kite M i N se sredini na rabovite AA1 i CC1 na
paralelopipedot ABCDA1B1C1D1. Neka pravite A1C , B1M i BN se
me|usebno zaemno normalni. Presmetaj go volumenot na
paralelopipedot, ako e poznato deka A 1C =a, B 1M =b i BN =c.
659. Kocka so rab a e prese~ena so ramnina koja sodr`i edna od
nejzinite dijagonali. Odredi ja najmalata mo`na vrednosta na
plo{tinata na presekot.
660. Vo kockata ABCDA1B1C1D1 to~kata P e na sredina na stranata BC, a
to~kata Q e presekot na dijagonalite na kvadratot CC1D1D.
Ramninata koja minuva niz to~kite A, P i Q ja deli kockata na dva
dela. Odredi go odnosot od volumenite na tie delovi.
661. Neka a, b и c se strani na paralelopiped, a d negova prostorna
1
dijagonala. Doka`i deka a 2 + b 2 + c 2 ¥ d 2 .
3
662. Doka`i deka zbirot na rastojanija od proizvolna to~ka do
temiwata na kocka so rab 1 e pogolem ili ednakov na 4 3 .
663. Tri ramnini razdeluvat kocka so rab 1 na osum paralelopipedi.
Doka`i deka me|u niv postojat ~etiri paralelopipedi ~ij volumen
na sekoj od niv ne e pogolem od 1/8.
664. Vo pravilna ~etiriagolna piramida so osnovni rabovi a ednakvi so
generatrisata e vpi{ana kocka, pri {to ~etiri temiwa le`at na

62
Elementi od stereometrija

apotemite, a ~etiri vo osnovata na piramidata. Presmetaj ja


plo{tinata i volumenot na kockata.
665. Najdi go agolot me|u osnovata i bo~niot zid na pravilna
~etiristrana piramida, ako ramninata koja go deli toj agol na
polovina, ja deli bo~nata povr{ina na piramidata na dva dela so
ednakvi plo{tini.
666. Dadena e piramida ABCD. Poznato e deka ®ADB=®DBC, ®ABD=®BDC
i ®BAD=®ABC. Presmetaj ja plo{tinata na piramidata ako
plo{tinata na osnovata e 10.
667. Neka volumenot na piramidata ABCD (D-vrv) e 5. Niz sredinata na
rabovite AD i BC e povle~ena ramnina koja go se~e rabot CD vo
to~ka M, pri {to DM : MC = 2 : 3 . Najdi ja plo{tinata na presekot,
ako rastojanieto od temeto A do prese~nata ramnina e 1.
668. Najdi go odnosot od volumenite na paralelopipedot ABCDA1B1C1D1 и
тетраедарот ACB 1D1 .
669. Ako bo~nite rabovi na triagolna piramida se me|usebe normalni i
ednakvi na a, b i c toga{ doka`i deka visinata spu{tena kon
osnovata e ednakva na 1 .
1 1 1
+ +
a2 b2 c2
670. Osnovata na ~etiriagolna piramida e paralelogram. Niz eden od
osnovnite rabovi i srednata otse~ka (linija) na sprotivniot yid, e
povle~ena ramnina. Opredeli vo kakov odnos taa ramnina gi deli
volumenite na dvata delovi od piramidata.
671. Niz eden rab na osnovata na pravilna triagolna piramida
povle~ena e ramnina koja e normalna na sprotivniot bo~en rab na
piramidata. Odredi go volumenot na piramidata, ako povle~enata
ramnina go deli bo~niot rab vo odnos 5:1 smetano od vrvot na
piramidata (stranata na osnovata e a).
672. Dadena e triagolna piramida SA1A2A3 so volumen V. Na stranite na
osnovata A1A2, A2A3, A3A1 izbrani se to~ki K1, K2, K3, soodvetno, taka
{to A1K1 : K1A 2 = A 2K 2 : K 2A 3 = A 3K3 : K 3A1 = 2 . Niz sredinata na rabot
SA1, paralelno so osnovata povle~ena e ramnina π koja gi se~e
otse~kite SK1, SK2, SK3 vo to~ki L1, L2, L3, soodvetno. Presmetaj go
volumenot na pravata prizma ~ija gorna osnova e triagolnikot
L1L2L3, a dolnata osnova le`i vo ramninata na osnovata na
piramidata.
673. Vo triagolna piramida ABCD e poznato deka CD =9, BC = AC , a
rabot AD e normalen na ramninata na osnovata ABC. Sfera so
radius 2 go dopira yidot ABD, rabot CD i osnovata ABC vo
prese~nata to~ka na te`i{nite linii. Presmetaj go volumenot na
piramidata.

63
Elementi od stereometrija

674. Ako aglite pri vrvot na piramidata ABCV ( V – vrv) se pravi


agli toga{ va`i ravenstvoto Ρ2 ∆ABV + Ρ2 ∆ ACV + Ρ2 ∆BCV = Ρ2 ∆ ABC .
Doka`i.
675. Sredinite na sekoja strana od osnovata na ~etiriagolna piramida,
se povrzani so te`i{tata na sprotivnite bo~ni strani. Doka`i
deka:
a) Tie otse~ki se se~at vo edna to~ka koja gi deli vo odnos 3:2
smetano od stranite na osnovata.
b) Sredinite na tie otse~ki obrazuvaat paralelogram.
v) Najdi go odnosot od plo{tinata na paralelogramot i plo{tinata
na osnovata.
676. Vo pravilna ~etiristrana potse~ena piramida, presekot na
ramninata koja minuva niz dijagonalite na dvete osnovi ima ista
plo{tina so presekot na ramninata koja gi sodr`i rabot od
dolnata osnova i negoviot sprotiven rab od gornata osnova. Odredi
go agolot me|u prese~enite ramnini.
677. Doka`i deka okolu piramida mo`e da se opi{e sfera ako i samo
ako okolu osnovata na taa piramida mo`e da se opi{e kru`nica.
678. Doka`i deka te`i{nite linii vo tetraedarot ABCV se se~at vo
to~ka koja gi deli vo odnos 3:1, smetano od vrvovite na
tetaraedarot.
679. Neka T e pravilen tetraedar, a T1 tetraedarot ~ii temiwa se
te`i{tata na yidovite na tetraedarot T. Presmetaj go odnosot
VT1 : VT .
680. Robert trebalo da pomogne za izgradba na scena za {kolska
predstava. Za edna scena, toj treba da izgradi koloriten tetraedar
od platno i bambus. Toj zapo~nuva so montirawe na tri bambusi,taka
{to tie se sre}avaat vo ista to~ka, i sekoj par od bambusovite
stapovi se sre}avaat pod prav agol. Tri drugi bambusovi stapovi se
povrzuvaat so kraevite na prvite tri stapovi. Robert, potoa
otsekol triagolnici od tkaenina so: sino, crveno, zeleno i `olto
par~e. Ovie triagolni delovi od tkaenina gi postavil vo
triagolniot prostor pome|u bambusite pravej}i ~etiri yida na
tetraedarot. Plo{tinite na crveniot, `oltiot i zeleniot del se
so 60, 20 i 15, soodvetno. Ako siniot del e najgolemiot od ~etirite
yida toga{ opredeli ja plo{tinata na toj del.
681. Vo osnovata na tetraedarot ABCD e izbrana to~ka M niz koja se
povle~eni pravi, paralelni so rabovite AD, BD i CD. Neka tie
pravi gi proboduvaat bo~nite yidovi BCD, ACD i ABD vo to~kite P,
MP MQ MR
Q i R, soodvetno. Doka`i go ravenstvoto + + = 1.
AD BD CD
682. Tetraedarot ABCP ima bo~ni rabovi PA, PB i PC koi se zaemno
normalni pri {to PA =a, PB =b , PC =c. Odredi ja dol`inata na
PT, kade T e te`i{te na osnovata ABC.

64
Elementi od stereometrija

683. Presmetaj go volumenot na pravilen tetraedar, ako radiusot na


opi{anata kru`nica okolu eden sid e R.
(crt. 44)
684. Rabot AB na tetraedarot ABCD e
dijagonala na osnovata na ~etiriagolna
piramida, pri {to sprotivniot rab CD e
paralelen so drugata dijagonala od taa
osnova i temiwata mu le`at na bo~nite
rabovi od piramidata. Odredi go
najmaliot mo`en volumen na piramidata,
ako volumenot na tetraedarot e V.
Crte` 44
685. Osnovata na pravilna ~etiriagolna piramida ABCDP, e kvadratoto
ABCD. Niz sredinata na osnovniot rab AB, e povle~ena ramnina
paralelna so ramninata opredelena so to~kite A, D, P. Presmetaj ja
plo{tinata na presekot, ako plo{tinata na bo~niot zid e 16.
686. Niz osnovniot rab na pravilna ~etiriagolna piramida e povle~na
ramnina, koja gi se~e sprotivnite bo~ni rabovi vo odnos 1:2
smetano od vrvot na piramidata. Presmetaj go odnosot od
volumenite na dvete tela dobieni so prese~nata ramnina.
687. Vo vnatre{nosta na pravilen tetraedar e izbrana to~ka P. Pravata
PO, kade O e centarot na opi{anata sfera, gi proboduva ramninite
na stranite vo to~kite A ′, B′, C′, D′ . Doka`i deka
PA ′ PB ′ PC ′ PD ′
+ + + =4.
OA ′ O B ′ OC ′ O D ′
688. Doka`i deka sprotivnite rabovi kaj tetraedarot ABCD se zaemno
normalni ako i samo ako AB 2 + CD 2 = AC 2 + BD 2 = AD 2 + BC 2.
689. Neka HA, HB, HC, HD se visinite vo tetraedarot ABCD, a R e
radiusot na vpi{anata sfera. Doka`i deka
a) 1 + 1 + 1 + 1 = 1 ; b) HA+HB+HC+HD¥16R.
HA HB HC HD R
690. Vo tetraedarot ABCD site agli pri temeto A se po 60±. Doka`i deka
AB + AC + AD ≤ BC + CD + DB .
691. Bo~nite rabovi na tetraedar me|u sebe se normalni. Doka`i deka
9 2
P1+P2+P3¥ H kade P1, P2 i P3 se plo{tinite na bo~nite yidovi na
2
tetraedarot, a H negovata visina.
692. Vo tetraedarot PABC se povle~eni PA1, PB1 i PC1, simetrali na
aglite kaj temeto P na triagolnicite PBC, PAC i PAB, soodvetno.
Doka`i deka pravite AA1, BB1 i CC1 se se~at vo edna to~ka.
693. Odredi go odnosot od volumenite na ramnostran cilindar i
ramnostran konus koj imaat ednakvi plo{tini.
694. Prav potse~en konus ima radius na osnovite R i r, pri {to
dijagonalite na oskiniot presek se se~at pod prav agol. Presmetaj

65
Elementi od stereometrija

ja plo{tinata na paralelniot presek koj minuva niz prese~nata


to~ka na dijagonalite na oskiniot presek.
695. Odredi go odnosot od volumenite na telata dobieni so rotacija na
paralelogramot ABCD okolu svoite dve sosedni strani.
696. Odredi go odnosot od volumenite na telata dobieni so rotacijata
na triagolnikot ABC okolu sekoja strana a, b i c.
697. Opredeli go radiusot na sferata, koja gi dopira rabovite na
pravilen tetraedar so rabovi a.
698. Okolu topka so radius R opi{i konus so najmal volumen.
699. Konus e vpi{an vo topka, a topkata e vpi{ana vo cilindar.
Presmetaj go odnosot pome|u ovie geometriski tela.

66
Eksponencijalni i logaritamski ravenki

Eksponencijalni i logaritamski ravenki

700. Presmetaj ja vrednosta na izrazot


lgtg1 ° + lgtg2 ° + lgtg3 ° + ... + lgtg88 ° + lgtg89 ° .

701. Doka`i deka a) logax·logby=logbx·logay; b) logab  b  = log ac − logbc .


 a  log ac + logbc
702. Za koi racionalni vrednosti na x izrazot log2 (x2 − 4x − 1) e cel
broj ?
703. Ako 9 x + 9 − x = 14 toga{ kolku e 81 x + 81 − x ?
704. Krug so radius lg(a2) ima perimetar lg(b4). Kolku e logab?
705. a) Znaej}i deka log 6 9 = a i log6 5 = b opredeli log 3 5 .
b) Poznato e deka log14 7 = a, log14 5 = b. Presmetaj log35 28.
706. Doka`i deka
1 1 1 1
a) + + ... + = (2016! = 1 ⋅ 2 ⋅ ... ⋅ 2016);
log 2 x log 3 x log 2016 x log 2016! x
b) logax·logbx+logbx·logcx+logcx·logax= log a x ⋅ logb x ⋅ log c x .
log abc x
2
707. 21 x + x − 88
=10 lg441.
1
log 2 3 124 +
708. Presmetaj 8 3
.

Re{i gi ravenkite:

709. 4 log 10 x − 32 + x log 10 4 = 0.


710. log x 3x • log 3 x = -1.
711. 5
lgx
- 3lgx-1 = 3lgx+1 - 5lgx-1.
x-2 x-2
712. lg2 + lg(4 + 9) = 1 + lg(2 + 1).
x−2 x−2
713. 4 + 16 = 10 ⋅ 2
.
714. lg x − 5 + lg 2x − 3 + 1 = lg30 .
715. 3 1 .
=
log3 x 64
2
716. log 4 (x + 12) ⋅ log x 2 = 1 .
717. 22xÿ9x-2ÿ63x-1+42x-1ÿ34x-2=0.
718. log x 2 ⋅ log 2x 2 = log 4x 2 ;
lgx
719. x = 100x.

67
Eksponencijalni i logaritamski ravenki

720. 42x+1+22x+6=4·8x+1.
721. 312x−1 − 96x−1 − 27 4x−1 + 813x+1 = 2192 .
722. Presmetaj go zbirot na site re{enija na ravenkata
2
(x2 + 3x + 1)x +2x−8
=1 .
723. Re{i ja ravenkata 5lgx=50-xlgx.
8x − 8
724. Re{i ja ravenkata x = 7.
4 − 2 x +1
725. Ako 4a=5, 5b=6, 6c=7 i 7d=8, presmetaj ja vrednosta na izrazot abcd.
726. Opredeli gi site vrednosti na parametarot k za koi ravenkata
log(kx)=2 log(x+1) ima to~no eden pozitiven koren.
lgx 2 lgx3 lgx 4 lgxn
727. Re{i ja ravenkata + + + ... = 8.
lg2 x lg3 x lg4 x lgn x
728. Ako n e priroden broj, re{i ja eksponencijalnata ravenka
x n + 5n 1
n
n n
= .
80 + x 4
729. Neka a, b, c se prirodni broevi za koi a·log1443+b·log1442=c. Kolku e
a+b
?
c

730. Re{i ja ravenkata lg(lgx ) =


(m + n )lg(m −n ) .
(m − n )lg(m +n )
731. Odredi ja minimalnata vrednost na izrazot 3x+y·(3x-1+3y-1-1) za site
realni broevi x i y .
732. Ako a, b i c se strani na pravoagolen triagolnik (s-hipotenuza,
s-b ≠ 1, s+b ≠ 1) toga{ logc +ba + logc −ba = 2logc +ba ⋅ logc −ba .
733. Neka a, b i c se pozitivni realni broevi za koi va`i
logab+logbc+logca=0. Presmetaj ja vrednosta na izrazot
(logab)3+(logbc)3+(logca)3.
734. Neka a, b i c se pozitivni realni broevi takvi {to
log 3 7 log 7 11 log 11 25
a = 27 , b = 49 i c = 11 . Presmetaj go zbirot
2 2
   
S = a
 log 
37
+b
(log 7 11 )2 + c
 log 
11 25 
.
4 2
735. Presmetaj + .
log 3 2025 6 log 5 2025 6
736. Ravenkata 2333x-2+2111x+2=2222x+1+1 ima tri realni koreni. Poznato e
m
deka nivniot zbir e racionalniot broj , kaj koj m i n se zaemno
n
prosti prirodni broevi. Odredi go zbirot na broevite m i n.
737. Re{i ja neravenkata log2(x-1)+log2(x+2)-log2(3x-1)<1.

68
Sistemi ravenki i neravenki

Sistemi ravenki i neravenki

x 2 − y − z = 100
738. Re{i go vo mno`estvoto celi broevi sistemot   .
x − y2 − z = 124
739. Vo mno`estvoto na celite broevi re{i ja ravenkata
x2y2-x2-8y2=2xy.
740. Odredi gi x, y, z ako (x + y) : (5 − z) : (y + z) : (9 + y) = 3 : 1: 2 : 5 .

Re{i gi sistemite ravenki:

741.  x + y = 4 .
3 3

 xy = 27

742. 
x!+ y! = z!
.
x + y = z
 2 2
743. (x + y )(x + y) = 65 .
 xy(x + y) = 30
x 2 + y 2 = 8
744.  1 1 1 .
 x2 + y2 = 2

(x + y)4 + 4(x + y)2 = 117
745.  .
x − y = 25
746. 
lgx + lgy = lg2
.
2 2
x + y = 5
log x + log x y = 2
747.  y (x>0, y>0).
 x 2 + y = 12
x lgy = 100
748.  .
xy = 1000
3 x − 2 y = 77
2

749.  x y2
.
3 2 − 2 2 = 7
 8
750. logy x − logx y = 3 , (x>0,y>0) .
x ⋅ y = 16

69
Sistemi ravenki i neravenki

x + y + z = 9

751.  1 + 1 + 1 = 1 , (x ≠ 0, y ≠ 0 , z ≠ 0 ).

x y z
 xy + xz + yz = 27

 xy
x + y = 1

752.  xz .
 =2
x + z
 yz
y + z = 3

x + y + z = a

753. x2 + y 2 + z2 = a2 .
 3 3 3 3
x + y + z = a

log 2 x + log 4 y + log 4 z = 2


754. log 3 y + log 9 z + log 9 x = 2 .
log z + log x + log y = 2
 4 16 16

(a + b)(x + y) − cz = a − b
755. Re{i go sistemot ravenki 
(b + c)(y + z) − ax = b − c , ako a+b+c≠0.
(c + a)(z + x) − by = c − a

756. Odredi gi site realni re{enija na sistemot ravenki
x 2 − y 2 = 2(xz + yz + x + y )
 2
y − z = 2(yx + zx + y + z ) .
2

z 2 − x 2 = 2(zy + xy + z + x )

757. Vo mno`estvoto pozitivni realni broevi re{i go sistemot
 4
x + xy = 3

 4 .
y + =3
 yz
 4
z + =3
 zx

70
Sistemi ravenki i neravenki

x(1 + y) = 1 + z
+ 
758. Vo R , re{i go sistemot ravenki y(1 + z) = 1 + x .
z(1 + x) = 1 + y

 x
x + y + y = 19
759. Broevite x i y se re{enija na sistemot 
 . Presmetaj go
 x(x + y) = 60
 y
zbirot x+y.
760. Ako 4x+3y+7z=12, 9x-2y-3z=8 i 6x+4y-z=9 toga{ kolku e x-7y+6z ?

761. Odredi ja vrednosta na izrazot xy+yz+zx ako x, y, z se re{enija na


x 2 + xy + y 2 = 2

sistemot ravenki  y 2 + yz + z 2 = 3 .
z 2 + zx + x 2 = 5

762. Ako a, b, c se razli~ni broevi, re{i go sistemot ravenki
x + ay + a 2 z + a 3 = 0
 2 3
x + by + b z + b = 0 .
 2 3
x + cy + c z + c = 0

763. Re{i go sistemot ravenki


x + y + z = 2

(x + z) ⋅ ( y + z) + ( y + z) ⋅ (z + x) + (z + x) ⋅ ( x + y) = 1 .
x 2 ⋅ ( y + z) + y 2 ⋅ (z + x) + z 2 ⋅ ( x + y) = -6

x + yzt = 2
y + xzt = 2
764. Re{i go sistemot ravenki 
 .
 z + xyt = 2
t + xyz = 2
a + b = c 2

b + c = d2
765. Odredi gi pozitivnite re{enija na sistemot 
 .
2
c + d = a
d + a = b 2

71
Sistemi ravenki i neravenki

 2x 1 = x 5 ² - 23

 4x 2 = x 1 ² + 7
766. Re{i go sistemot ravenki 
 6x 3 = x 2 ² + 14 .
 8x 4 = x 3 ² + 23

 10x 5 = x 4 ² + 34
x1 + x 2 = 1

x 2 + x 3 = 2
x 3 + x 4 = 3

...
767. Daden e sistemot ravenki  . Odredi ja vrednosta
...
...

x 2018 + x 2019 = 2018
x
 2019 + x 2020 = 2019
na zbirot x1+x2020.

768. Vo mno`estvoto na pozitivnite realni broevi re{i go sistemot


 1
x1 + x = 4
 2

 1
x2 + x = 1
 3

 1
ravenki  x 3 + = 4 .
x4

 ...
 ...

 ...
 1
 x 99 + = 4
 x 100

 x 100 + 1 = 1
 x1
2x1 + x 2 + x 3 + ... + x n -1 + x n = 1

x1 + 2x 2 + x 3 + … + x n -1 + x n = 1
x1 + x 2 + 2x 3 + … + x n-1 + x n = 1

769. Re{i go sistemot ravenki … .

…
…

x1 + x 2 + x 3 + … + 2x n-1 + x n = 1
x + x + x + … + x + 2x = 1
 1 2 3 n-1 n

72
Sistemi ravenki i neravenki

770. Za koi vrednosti na parametarot m sistemot ravenki


 x 2 + y 2 = 25
 ima ednakvi koreni?
mx - y + 4 - 3m = 0
771. Odredi gi site vrednosti na parametarot m taka {to sistemot
ravenki 
(m + 1)x − my = 4 , da ima re{enie (x,y) {to go zadovoluva
3x − 5y = m
uslovot x-y<2.
a + b + c = 10
772. Neka a, b i c se realni broevi i neka  . Najdi ja
2 2 2
a + b + c = 50
maksimalnata vrednost na a.
773. Neka a1, a2, ..., an, A se dadeni realni broevi, takvi {to
a1+a2+…+ an ≠ 0. Re{i go sistemot ravenki
 x1 x 2 x 2 x 3 x x
 = , = , ..., n −1 = n
 a1 a 2 a 2 a3 an −1 an .
x + x + ... + x = A
 1 2 n
774. Tri petli i edna guska ~inat isto kolku i dva fazani. Eden petel,
dve guski i tri fazani ~inat 25 denari. Kolku ~ini eden petel,
edna guska i eden fazan?
775. Moite godini sega, se dvapati pove}e od godinite {to si gi imal ti
koga jas sum imal tolku godini, kolku {to ima{ sega ti. Koga ti }e
ima{ tolku godini kolku {to imam sega jas, toga{ zbirot na na{ite
godini }e bide 63. Kolku godini imam jas a kolku ti?
776. Po kru`na pateka dolga 2000m se dvi`at dvajca motorcikli so
postojana brzina. Ako motorciklistite se dvi`at vo sprotivna
nasoka, }e se sre}avaat sekoja minuta, a ako se dvi`at vo ista
nasoka toga{ motorciklistot koj se dvi`i so pogolema brzina }e go
prestignuva drugiot na sekoi 12 minuti. Opredeli gi brzinite na
motorciklistite.
777. Dva broda zapo~nuvaat da se dvi`at istovremeno od dve mesta A i
B na ezerskoto pristani{te eden nasproti drug, pravoliniski. Po
izvesno vreme se sre}avaat na rastojanie 500 metri oddale~eni vo
odnos na mestoto A. Razminuvaj}i se, prodol`uvaat da se dvi`at
odej}i do mestata A i B, pristignuvaat i vedna{ se vra}aat nazad
pri {to se sre}avaat na rastojanie 150 metri oddale~eni od
drugoto mesto B. Brzinata na brodovite e konstantna vo tekot na
dvi`eweto. Kolku iznesuva rastojanieto pome|u mestata A i B?
xy(10x + 10y + 7z ) ≥ 27
778. Daden e sistemot neravenki  yz(10y + 10z + 7x ) ≥ 27 kade x¥0, y¥0 i
zx(10z + 10x + 7y ) ≥ 27

z¥0. Odredi ja minimalnata vrednost na zbirot x+y+z.

73
Sistemi ravenki i neravenki

779. Dadeni se 100 broja a 1, a 2 ,..., a 100 koi gi zadovoluvaat neravenstvata


a1 - 3a2 + 2a3 ≥ 0
a2 - 3a 3 + 2a 4 ≥ 0
...
...
...
a99 - 3a100 + 2a1 ≥ 0
a100 - 3a1 + 2a 2 ≥ 0 . Doka`i deka a1=a2=a3=...=a99=a100.

74
Elementi od trigonometrija

Elementi od trigonometrija

8
780. Ako za ostriot agol α va`i 8cos2 α + = 65 toga{ opredeli ja
cos2 α
vrednosta na cosα.
π 1
781. Ako x i y se ostri agli takvi {to x+y = i tgy = . Toga{ opredeli
4 6
ja vrednosta na tgx.
782. Opredeli ja vrednosta na sin36± (bez kalkulator).
783. Najdi cos36 ° - sin18 ° .
784. Presmetaj: cos21±+cos22±+cos23±+…+cos289±+cos290±.
π 4π 7π
785. Presmetaj a) cos cos cos ; b) cos π + cos 3 π + cos 5 π .
9 9 9 7 7 7
786. Ako α + β + γ = 180 ° toga{ sin2α + sin2β + sin2 γ = 4sinα ⋅ sinβ ⋅ sinγ
787. Doka`e deka ako α + β + γ = 180o toga{
sin2α + sin2β + sin2 γ − 2cosα ⋅ cosβ ⋅ cosγ = 2
.
788. Doka`i deka vo bilo koj triagolnik ABC e to~no ravenstvoto
tgα a 2 + c 2 − b 2 .
=
tgβ b 2 + c 2 − a 2
789. Aglite α, β, γ go zadovoluvat ravenstvoto
2tg2α ⋅ tg2β ⋅ tg2 γ + tg2α ⋅ tg2β + tg2β ⋅ tg2 γ + tg2 γ ⋅ tg2α = 1 . Presmetaj go
zbirot sin2α + sin2β + sin2 γ .
790. Stranite na triagolnikot ABC se a=10, b=8 i c=3. Doka`i deka
α=3ÿβ.
791. Za aglite na triagolnikot ABC va`i α : β : γ =1 : 2 : 4. Doka`i deka
1 1 1
= + .
a b c
792. Opredeli go odnosot na stranite vo triagolnikot ABC ako
tgα:tgβ:tgγ=1:2:3.
793. Ako sin2α+sin2β+sin2γ=2 toga{ triagolnikot ABC so agli α, β, γ e
pravoagolen. Doka`i.
794. Doka`i deka vo pravoagolniot triagolnik ABC ( γ = 90 ° ) va`at
slednite ravenstva:
2b2 2ab b2 − a2
a) sin2α ⋅ tgβ = 2 ; b) sin2α = 2 ; v) cos2α = .
c c c2
795. Doka`i deka vo sekoj triagolnik ABC va`at ravenstvata

75
Elementi od trigonometrija

α β γ
a) P = 2R 2 sinα ⋅ sinβ ⋅ sinγ ; b) s = 4R cos ⋅ cos ⋅ cos .
2 2 2

796. Doka`i deka 1 − 2sin α = 1 − tgα , α ≠
2
+ kπ .
1 + sin2α 1 + tgα 4
α
797. Doka`i deka sinα ⋅ (1+ tgα ⋅ tg ) = tgα .
2
π
798. Ako α+β+γ= toga{ tgα·tgβ+tgβ·tgγ+tgγ·tgα=1. Doka`i.
2
799. Ako α, β i γ se agli vo triagolnikot AVS toga{ va`i ravenstvoto
(sinα + sinβ + sinγ)2 − (1+ cosα + cosβ + cosγ)2 = 4cosα ⋅ cosβ ⋅ cosγ .
Doka`i.
800. Aglite α, β i γ se takvi {to β=α+60± i γ=β+60±. Doka`i deka
izrazot tgα·tgβ+tgβ·tgγ+tgγ·tgα ne zavisi od aglite α, β i γ.
801. Najdi ja vrednosta na izrazot
a) (1+tg1º)·(1+tg2º)·...·(1+tg44º); b)(1-ctg1º)·(1-ctg2º)·...·(1-ctg44º).
802. Presmetaj go zbirot
sin1o sin1o sin1o sin1o .
+ + + ... +
cos0 o ⋅ cos1o cos1o ⋅ cos2o cos2 o ⋅ cos3 o cos2024 o ⋅ cos2025 o
2 2
803. Najdi ja najgolemata i najmalata vrednost na izrazot (a - c) + (b - d)
ako a2 + b2 = 1 i c2 + d2 = 4 .
1+ cosα + cos2α + cos3α
804. = 2cosα . Doka`i.
2cos2α + cosα − 1
805. 1 − cos2α + sin2α = tgα . Doka`i.
1 + cos2α + sin2α
806. sin2 α ⋅ tgα + cos2 α ⋅ ctgα + 2sinαsinα = tgα + ctgα . Doka`i.
2
807. 1 + cos 2α + cos4α + cos6α = 4cosα ⋅ cos2 ⋅ cos3α
808. Ako α + β + γ = 0 toga{ tgα + tgβ + tg γ = tgα ⋅ tgβ ⋅ tg γ .

809. Uprosti go izrazot sin2 α(1 + ctgα) + cos2 α(1 + tgα) , π < α < .
2
1 1 1 1
810. Presmetaj sin 2 2α ako + + + =7.
tg α ctg α cos α sin 2 α
2 2 2

Re{i gi ravenkite:

811. a) sin6x = sin4x ; b) tg2x − ctg3x = 0 .


812. sin2 x + sin2 2x = 1 .
813. 3sinx + cosx = 1 .
814. 16sinx = cosx 4 .
815. cos 2 x + cos 2 2x + cos 2 3x = 1 .

76
Elementi od trigonometrija

816. 12sin 2 x + 3sin2x − 2cos 2 x = 2 .


817. Presmetaj go zbirot od re{enijata na ravenkata sin5x+sinx=sin3x za
koi 0 ≤ x < 2π .
818. Odredi ja vrednosta na n ako lg(sinx)+lg(cosx)=-1
i 2ÿlg(sinx+cosx)=lgn-1.
819. Doka`i deka vo ostroagolen triagolnik ABC va`at ravenstvata
a2 + b2 + c 2 2 2 2
a) ctg α + ctg β + ctg γ = ; b) a ctgα + b ctgβ + c ctg γ = 4P .
4P
α β γ
820. Ako α + β + γ = 180° toga{ sin α + sin β + sin γ = 4cos cos cos .
2 2 2
821. Doka`i deka plo{tinata na konveksen ~etiriagolnik ABCD se
presmetuva po formulata P2=(s-a)(s-b)(s-c)(s-d)-abcd·cos2 β + γ .
2
822. Ako vo ~etiriagolnikot so agli α, β, γ i δ va`i
cosα + cosβ + cosγ + cosδ = 0 toga{ toj e paralelogram, trapez ili
tetiven ~etiriagolnik. Doka`i.
823. Doka`i deka vo proizvolen triagolnik ABC va`at slednite
α+β α
sin sin
ravenstva: a) sin(α − β) = a −2 b ; b)
2 2
2 = 1 ; v) 2 a
=
sin(α + β) c γ β γ s
cos cos ⋅ cos
2 2 2
2 2 2
α β γ r asinα + bsinβ + csin γ a +b +c
g) tg tg tg = ; d) = ;
2 2 2 s α β γ a+b+c
4cos cos cos
2 2 2
|) asin(β − γ) + bsin(γ − α) + csin(α − β) = 0 ;
a2sin(β − γ) b2sin(γ − α) c 2sin(α − β)
e) + + =0;
sinα sinβ sinγ
β− γ
cos
α β γ r 2 b+c
`) sin sin sin = ; z) = .
2 2 2 4R α a
sin
2
824. Ako triagolnikot ABC e ostroagolen toga{
cos2α+cos2β+cos2γ+2cosα cosβ cosγ=1. Doka`i.
825. Doka`i deka vo sekoj triagolnik ABC so plo{tina P strani a, b, c i
soodvetni agli α, β i γ va`i ravenstvoto
a²(sin2β+sin2γ)+b²(sin2α+sin2γ)+c²(sin2α+sin2β)=12P.
826. Vo triagolnikot ABC va`i ravenstvoto
α β γ
cosα + cosβ + cosγ = 1+ 4sin ⋅ sin ⋅ sin . Doka`i.
2 2 2

77
Elementi od trigonometrija

827. Vo triagolnikot ABC so strani 7cm, 8cm i 5cm odredi ja vrednosta


na izrazot ( sin α + sin β + sin γ )·( ctg α + ctg β + ctg ).
γ
2 2 2
828. Vo ramnokrak triagolnik agolot pri osnovata e α . Presmetaj ja
osnovata na triagolnikot ako rastojanieto od vrvot do centarot na
vpi{anata kru`nica e m.
829. Ako plo{tinata na triagolnikot ABC e P, a vnatre{nite agli α, β i
γ toga{ opredeli ja plo{tinata na triagolnikot ~ii temiwa se
podno`nite to~ki na visinite.
830. Presmetaj ja plo{tinata na paralelogram so dijagonali m i n
(m > n) i ostar agol α.
831. Neka O e prese~nata to~ka na dijagonalite vo konveksniot
~etiriagolnik ABCD i neka triagolnicite AOB, BOC, COD i DOA
imaat ednakvi perimetri. Ako radiusite na vpi{anite kru`nici vo
triagolnicite AOB, BOC i COD se soodvetno 3, 4 i 6 toga{ najdi go
radiusot na vpi{anata kru`nica vo triagolnikot DOA.
832. Stranite na ostroagolniot triagolnik ABC se a, b i c a
rastojanieto od centarot na opi{anata kru`nica do stranite a, b i
1
c se x, y i z, soodvetno. Doka`i deka ayz + bxz + cxy = abc .
4
833. Neka P = sin α + sin β + sin γ i Q = cos α + cos β + cos γ + 1 kade α, β, γ
se agli vo triagolnikot ABC. Doka`i deka triagolnikot ABC e
pravoagolen ako i samo ako P=Q.
834. Vo krug so radius r e vpi{an periferen agol α. Kolkavi treba da
bidat dol`inite na tetivite a i b koi go obrazuvat toj agol za da
zbirot nivni bide maksimalen?
835. Ako x, y i z se rastojanijata od centarot na vpi{anata kru`nica O
do temiwata A, B i C na triagolnikot ABC, soodvetno toga{
x ⋅ y ⋅ z = 4r 2R kade r e radius na vpi{anata a R radius na opi{anata
kru`nica. Doka`i.
836. Neka a i b se zaemno prosti prirodni broevi takvi {to а/b e
maksimalnata mo`na vrednost na izrazot
sin2 x1 + sin2 x2 + sin2 x3 + ... + sin2 x2019 , kade za 1 ≤ i ≤ 2019, xi e nenegativen
realen broj i x 1 + x 2 + x 3 + ... + x 2019 = π . Odredi ja vrednosta na
zbirot а+b.
837. Neka T e te`i{teto, a O centarot na opi{anata kru`nica okolu
triagolnikot ABC. Doka`i deka pravata OT e normalna na
te`i{nata linija CC1 ako i samo ako a2+b2=2c2 .
838. Ako cosx+cosy+cosz=sinx+siny+sinz=0 toga{ odredi ja vrednosta na
izrazot cos(x−y)+cos(y−z)+cos(z−x).

78
Elementi od trigonometrija

839. Vo vnatre{nosta na triagolnikot ABC se nao|a to~ka M za koja


®MAB=®MBC=®MCA=j. Doka`i deka 12 = 12 + 12 + 12 kade
sin ϕ sin α sin β sin γ
α, β i γ se aglite vo triagolnikot ABC.
840. Dol`inite na stranite na triagolnikot ABC se a, b, c i agol
3 1 1
α = 60 ° . Doka`i deka = + .
a+b+c a+b a+c
841. Neka Е e to~ka od vnatre{nosta na kvadratot АВСD za koja
®EAB=®EBA=15±. Doka`i deka triagolnikot СDЕ e ramnostran.
842. Opredeli go perimetarot na ramnokrak trapez so golema osnova a i
ostar agol α , opi{an okolu kru`nica.
843. Osnovniot rab na pravilna triagolna piramida e a, a bo~niot rab
so ramninata na osnovata zafa}a agol α. Opredeli go radiusot na
opi{anata sfera okolu piramidata.
844. Ramninata gi se~e bo~nite rabovi na pravilna ~etiriagolna
piramida vo to~ki, ~ii rastojanija do vrvot na piramidata se a, b,
c i d. Doka`i deka 1 + 1 = 1 + 1 .
a c b d
845. Vo triagolna piramida SABC, site ramninski agli pri vrvot S se
pravi, a SO e visinata na piramidata. Odnosot na plo{tinata na
triagolnikot AOB kon plo{tinata na triagolnikot BOC e ednakva
na k.Opredeli go odnosot od plo{tinata na triagolnikot ASB kon
plo{tinata na triagolnikot BSC.
846. Vo konus e vpi{ana sfera. Potoa, e vpi{ana druga sfera koja ja
dopira prvata sfera i bo~nata povr{ina na konusot, na sli~en
na~in e vpi{ana treta sfera i t.n. Presmetaj go zbirot od
plo{tinite na sferite ako visinata na konusot e H i agolot pri
vrvot e α.
847. Doka`i deka vo sekoja pravilna ~etiriagolna piramida va`at
ravenstvata
a) sinα = ctg (φ/2); b) sin(φ/2)cos( β /2) = 2 ; v) cosφ = −cos2 γ ; kade {to:
2
α e agolot {to go zafa}a sekoj bo~en rab so ramninata na
osnovata,
β e agolot me|u koj bilo dva sosedni bo~ni raba,
γ e prostorniot agol me|u proizvolen bo~en yid i osnovata
na piramidata,
φ e agolot me|u bilo koj dva sosedni bo~ni yida.
848. Odredi ja vrednosta na izrazot N=32sin α sin 5α ako cosα = .
3
2 2 4
849. Odredi go bez tablica ostriot agol α ako tgα = 6 + 3 − 2 − 2 .

79
Elementi od trigonometrija

kπ 1 1
850. Ako α ≠ toga{ doka`i deka + ≥ 8.
2 sin α cos4 α
4

851. Ako x i y se realni broevi toga{ va`i neravenstvoto


|cosx|+|cosy|+|cos(x+y)|¥1. Doka`i.
852. Neka a, b i c se strani na triagolnik, a α, β i γ soodvetnite agli.
π a α + b β + cγ π
Doka`i deka ≤ < .
3 a+b+c 2
853. Vo pravoagolniot triagolnik ABC (®ACB=90±), so φ e ozna~en
agolot me|u te`i{nata linija i simetralata na agolot od temeto A,
a so ψ agolot me|u medijanata i simetralata na agolot od temeto B.
( )( )
Doka`i deka 1 + 3 tgϕ ⋅ 1 + 3 tgψ = 2 .
854. Neka a, b i c se strani na triagolnikot ABC. Doka`i deka
a +b-c α β
= tg ⋅ tg
a+b+c 2 2.
855. Vo konus e vpi{ana topka, pri {to plo{tinata na topkata i
plo{tinata na osnovata na konusot se ednakvi. Presmetaj go
kosinusot od agolot na oskiniot presek na konusot vo temeto {to e
i vrv na konusot.
856. Vo ostroagolniot triagolnik ABC, presekot na visinite e to~kata
N. Doka`i deka radiusite na opi{anite kru`nici okolu
triagolnicite ABH, BCH i ACH se ednakvi me|u sebe.
857. Najdi ja zavisnosta me|u stranite na triagolnikot ABC ako
α β γ β−γ α−γ α−β
sin cos cos = cos sin sin .
2 2 2 2 2 2

80
Analiti~ka geometrija

Analiti~ka geometrija

858. Kolku to~ki so celobrojni koordinati le`at vnatre ili na


granicata na kru`nica, definirana vo ramninata XOY, so ravenka
x2+y2=100?
859. Presmetaj ja plo{tinata na mnoguagolnikot koj se dobiva od site
to~ki na kru`nicata x2+y2=100 so celobrojni koordinati.
860. Ako to~kite A(-2,10), B(1,-8), C(4,10) le`at na grafikot na
kvadratnata funkcija y=ax2+bx+c, toga{ kolku e abc?
861. Za koe k pravite 2x+7y=14 i kx-y=k+1 se se~at vo prviot kvadrant.
862. Vo ramninata se dadeni ~etiri to~ki A(-1,2), B(3,-4), C(5,-6) i
D(-2,8). Opredeli to~ka P od ramninata za koja izrazot
PA + PB + PC + PD ima minimalna vrednost.
863. Nad stranite na pravoagolen triagolnik ABC (so prav agol vo
temeto C), so kateti a i b, konstruirani se kvadrati. Odredi ja
plo{tinata na triagolnikot ~ii temiwa se centrite na tie
kvadrati (centar na kvadratot se smeta presekot na negovite
dijagonali).
864. Vo igrata hokej, Atanas se nao|a vo to~kata A(2,17) i saka da go
isprati “pakot”(top~eto) do Bor~e, koj se nao|a vo to~kata B(13,3),
kako {to e poka`ano na crt. 45. Me|u
niv se nao|aat nekolku odbranbeni
igra~i. Granici na hokearskoto
igrali{te pretstavuvaat oskite h i u.
Atanas planira da go {utne pakot taka
{to udiraj}i vo u-oskata, a potoa vo h-
oskata da stigne do Bor~e. Kolkav pat
bi pominal pakot?

Crte` 45

865. Mravka so zanemarliva golemina


stoi vo koordinatniot po~etok na
dvodimenzionalen koordinaten
sistem. Mravkata zapo~nuva da se
dvi`i na sledniot na~in: pominuva
edna merna edinica nadesno, potoa
1/2 merna edinica nagore, potoa 1/4
merna edinica levo, 1/8 merna
edinica nadole i taka natamu
Crte` 46

81
Analiti~ka geometrija

(crt. 46). Vo sekoe dvi`ewe mravkata se zavrtuva za agol od 90 ° i


pominuva polovina od patot izminat vo
prethodnoto dvi`ewe. Vo koja to~ka (x,y)
od ramninata }e zastane mravkata?

866. Mravka (so zanemarliva golemina) se


dvi`i vo XOY ramninata na sledniot
na~in: Startuva od to~kata (0,0), prviot
~ekor e vo to~kata (1,0) potoa (1,1); (0,1);
(-1,1); (-1,0) i taka natamu vo vid na
spirala (crt. 47). Do koja to~ka mravkata
}e stigne pri nejziniot 2016-ti ~ekor?
Crte` 47
867. Odredi gi site to~ki vo koordinantnata ramnina XOY, ~ii
koordinati ja zadovoluvaat ravenkata (x2+2x+4)(y2-6y+11)=6.
868. Odredi gi ravenkite na pravite koi istovremeno so kru`nicata
x2+y2=1 i so parabolata y=x2+1, imaat samo po edna zaedni~ka
to~ka.
869. Vo vnatre{nosta na ostroagolniot triagolnik ABC, najdi to~ka M
za koja, zbirot na rastojanijata do stranite i temiwata na
triagolnikot ABC e najmal.
870. Dadeni se dve koncentri~ni kru`nici k1 i k2, so radiusi r i 2r,
soodvetno. Ako to~kata P le`i na kru`nicata k2, odredi go agolot
{to go zafa}aat tangentite na kru`nicata k1, povle~eni od
to~kata P.
871. Odredi gi parovite od prirodni broevi (x,y) koi gi zadovoluvat
ravenkite x+y=2n i x2+y2=2m, kade m,n se prirodni broevi.
872. Oblasta Q e zadadena so neravenkata 3x + 2y ≤ 12 . Odredi ja
plo{tinata na taka definiranata oblast.
873. Vo XOY ramninata, odredi ja plo{tinata na oblasta ograni~ena so
neravenkata x + y + x + y ≤ 2 .
874. Neka R e oblasta sostavena od to~ki (x,y) od xOy -ramninata
 x − y ≤ 16
koi gi zadovoluvaat neravenkite  . Presmetaj ja
 y ≤ 16
plo{tinata na oblasta R.
875. Neka M e to~ka od vnatre{nosta na kvadratot ABCD. Ako
rastojanijata od to~kata M do temiwata B, C i D se 14, 6 i 10
soodvetno, presmetaj ja plo{tinata na kvadratot ABCD.
876. Neka ABCD e kvadrat so temiwa A(0,0), B(5,0), C(5,5) i D(0,5). Neka
P e to~ka koja le`i na pravata y=2x. Odredi gi koordinatite na
to~kata P taka {to zbirot od kvadratite na rastojanijata od P do
temiwata na kvadratot e minimalen.

82
Analiti~ka geometrija

877. Neka to~kata A(20,20) e teme na ramnokrak triagolnik ~ii temiwa


imaat celobrojni koordinati. Odredi go brojot na site takvi
triagolnici, so edno teme vo to~kata A, ~ija plo{tina e 9.

878. Parabola deli pravoagolnik na dva dela ~ii


povr{ini se odnesuvaat 2:1 (crt. 48). Kolku
prirodni broevi k postojat, 0<k§2018, takvi
{to plo{tinata ograni~ena me|u parabolata
y=k-x2 i x-oskata, e priroden broj?

Crte` 48
879. Neka ABCD e kvadrat ~ie teme A le`i vo koordinatniot po~etok i
neka BEFG e isto taka kvadrat pri {to
temeto F zaedno so temeto C le`i na
parabolata y=k· x (crt. 49). Odredi ja
EF
vrednosta na koli~nikot .
BC

Crte` 49
880. Vo vnatre{nosta na kvadratot ABCD e vlo`en
kvadrat MNPQ ~ii strani se paralelni so
stranite na kvadratot ABCD (crt. 50). Ako S e
prese~nata to~ka na otse~kite BQ i DN doka`i
deka i otse~kite AP i CM se se~at vo to~kata S.

Crte` 50
881. Dadeni se pet edine~ni kvadrati postaveni
vo koordinatnata XOY ramnina (crt. 51). Ako
otse~kata AB ja deli dobienata figura na dva
dela so ednakva plo{tina toga{ odredi ja
apcisata a na to~kata A.

Crte` 51
882. Doka`i deka geometrisko mesto na to~ki M, za koi rastojanijata do
m
dve dadeni to~ki A i B, se vo daden odnos ≠ 1 , e kru`nica.
n
(Apolonieva kru`nica)
883. Odredi go geometriskoto mesto na to~ki M za koi, tangentite
x2 y2
povle~eni vo to~kata M kon elipsata + = 1 , se vzaemno
16 9
normalni .

83
Analiti~ka geometrija

884. Neka C1(-r,0) i C2(r,0) se centrite na kru`nicite k1 i k2 so radiusi r


i 5r, taka {to k1 celosno se sodr`i vo k2. Odredi go geometriskoto
mesto na to~ki M za koi, rastojanijata od M do kru`incite k1 i k2,
se ednakvi.
(Rastojanie od to~ka M do kru`nica k1(C1,r) e rastojanieto od to~kata M, do
to~kata C1M∩k1={P}. Soodvetno, rastojanie od M do k2(C2,5r) e rastojanieto od M,
do to~kata C2M∩k2={Q}.)
885. Neka p e prava koja minuva niz temeto B na triagolnikot ABC i e
paralelna so stranata AC. Ako to~kata B se dvi`i po pravata p,
toga{ opredeli go geometriskoto mesto na to~ki {to go opi{uva
ortocentarot H na triagolnikot ABC.
886. Neka A i B se dve fiksni to~ki na rastojanie AB = 2 . Odredi go
geometriskoto mesto na to~ki M za koi, agolot AMB e 45±.
887. Neka AB [A(0,0); B(4n,0)] e dijametar na kru`nica k, pri {to
to~kata A e fiksna to~ka. Odredi go geometriskoto mesto na to~ki
M koi se sredini na otse~kite AB , koga B se dvi`i po kru`nicata
opredelena so dijametarot AB .
888. Odredi go geometriskoto mesto na to~ki M, koe go opi{uva
ortocentarot H na triagolnikot ABC, koga to~kata C se dvi`i po
kru`nicata k opi{ana okolu triagolnikot ABC.
889. Dadena e kru`nica k i dve nejzini tetivi AB i CD. Ako A i B se
fiksni to~ki, a to~kite C i D se dvi`at po kru`nicata k, toga{
odredi go geometriskoto mesto na prese~nite to~ki na pravite AD
i BC.
890. Dadeni se dve fiksni to~ki A, B i to~ka C, koja ne le`i na pravata
AB, pritoa formiraj}i triagolnik ABC so plo{tina P. Odredi go
geometriskoto mesto na to~ki, koe go opi{uva to~kata C, taka {to
plo{tinata na triagolnikot ABC e konstantna.
891. Dadena e kru`nica k so centar O(5,3), radius r=1 i prava p
zadadena so ravenkata y=-2, koja nema zaedni~ka to~ka so
kru`nicata k. Odredi go geometriskoto mesto na to~ki C, centri na
kru`nicata k1, koja gi dopira pravata p i kru`nicata k.
892. Neka A i B se dve fiksni to~ki od ramninata XOY. Najdi go
geometriskoto mesto na to~ki M od ramninata XOY, takvi da
to~kite A, B i M formiraat ramnokrak triagolnik.
893. Odredi go geometriskoto mesto na to~ki, koi se nao|aat vo
vnatre{nosta na ramnostran triagolnik ABC, so ~ii rastojanija do
stranite na triagolnikot ABC mo`e da se konstruira triagolnik.
894. Odredi go geometriskoto mesto na to~ki M, koi le`at vo
vnatre{nosta na rombot ABCD i go zadovoluvat ravenstvoto
®AMD+®BMC=180±.
895. Daden e ~etiriagolnikot ABCD so plo{tina P i neparalelni
sprotivni strani AB i CD. Odredi go geometriskoto mesto na to~ki

84
Analiti~ka geometrija

X, koi le`at na ~etiriagolnikot ABCD (na stranite ili vo


P
vnatre{nosta), a za koi va`i ravenstvoto P∆ABX + P∆CDX = .
2

896. Dadena e otse~ka BC. Odredi go


geometriskoto mesto na to~ki A, so
slednoto svojstvo: postoi to~no edna to~ka
D, takva {to ~etiriagolnikot ABDC e
tetiven i otse~kata BC ja prepolovuva
otse~kata AD (crt. 52).

Crte` 52
897. Niz to~ka koja le`i na strana od triagolnikot ABC, povle~i prava
koja }e go deli triagolnikot ABC na dva dela so ednakva
plo{tina.
898. Neka y=x2+2ax+a e mno`estvo od paraboli so realen parametar a.
a)Doka`i deka site paraboli minuvaat niz ista to~ka i odredete
gi koordinatite na taa to~ka.
b)Temiwata na site paraboli le`at na edna kriva. Odredi ja
ravenkata na taa kriva.
899. Vo ramninata Σ se dadeni dve otse~ki AB i CD. Odredi go
geometriskoto mesto na to~ki M, za koi zbirot od plo{tinite na
triagolnicite AMB i CMD e a2.

85
Nizi i progresii

Nizi i progresii

900. Dadena e nizata 1, 2, 4, 7, 11, 16, 22, … . Odredi go 64-tiot ~len od


nizata.
901. Odredi go 100-tiot ~len i zbirot na prvite 100 ~lena od nizata 1,
3, 7, 13, 21, 31, ...
(razlikata na sosednite ~lenovi obrazuvat aritmeti~ka progresija).
902. Opredeli go zbirot od kvadratite na prvite n prirodni broevi.
(Arhimedova zada~a)
3 3 3 3 4 4 4 4
903. Presmetaj a) S3 = 1 + 2 + 3 + ... + n ; b) S 4 = 1 + 2 + 3 + ... + n .
(Zada~a na Ferma)
 1  1  1  1  1 
904. Presmetaj S =  1 +  ⋅  1 + 2 1 + 4 1 + 8  ⋅ ... ⋅ 1 + 2n .
 2   2  2  2   2 
905. Presmetaj go zbirot:
1 1 1 1
S= + + + ... + + ... .
1+ 3 3+ 5 5+ 7 2n − 1 + 2n + 1
906. Odredi go 2018-tiot ~len vo nizata
1,2,2,3,3,3,4,4,4,4,5,5,5,5,5,6,6,6,6,6,6...
907. Kolku ednakvi ~lenovi imaat dvete aritmeti~ki progresii
2,7,12,17...
2,5,8,11...
ako sekoja od niv e sostavena od 2021 ~lena?
908. Dadena e nizata 2,3,5,6,7,8,10,11,12,13,14,15,17,18,… od prirodni
broevi vo koja se otstraneti site to~ni kvadrati. Koja e vrednosta
na 2010-ot ~len vo nizata?
909. Dadena e niza (an) so prv ~len a1=100 definirana so an-1+an=2n za
site n¥2. Najdi ja vrednosta na ~lenot a1000.
910. Odredi gi poslednite dve cifri na zbirot 1!+2!+3!+…+2016!.
911. Najdi gi site vrednosti na n za koj N=1!+2!+3!+…+n! e to~en
kvadrat.
912. Odredi ja vrednosta na izrazot
S=1! ⋅ 3-2! ⋅ 4+3! ⋅ 5-4! ⋅ 6+...-2012! ⋅ 2014+2013!.
913. Najdi gi site prirodni broevi n za koi zbirot
n1 n 2 nn-1 nn
S= + + ... + + e cel broj .
1! 2! (n - 1)! n!
914. Odredi ja sumata Sn= 1 + 22 + 33 + ... + nn .
2 2 2 2

86
Nizi i progresii

915. Nizata od broevi a0, a1, a2, …, an e zadadena so a0=0, a1=1 i


an=an-1-an-2 kade n¥2. Presmetaj ja vrednosta na ~lenot a500.
an −1
916. Definirana e nizata a1, a2, ..., an so a1=1, a2=2 i an = , n¥3.
an − 2
Kolku e a2009?
917. Nizata od realni broevi (an) za m¥n¥0 e zadadena so ravenstvoto
1
am+n+am-n= ·(a2m+a2n). Odredi ja vrednosta na ~lenot a2013 ako a1=1.
2
918. Doka`i deka site broevi od vidot 1331, 1030301, 1003003001,
1000300030001, ... se javuvaaat kako to~en kub.
919. Doka`i deka broevite
а) 16, 1156, 111556, ... ;
b) 49, 4489, 444889, ... ;
v) 361, 39601, 3996001, ... se polni kvadrati.
( ) ( ) (
920. Opredeli go zbirot 1! 12 + 1 + 2! 2 2 + 1 + ... + n! n 2 + 1 . )
921. Neka n e neparen priroden broj. Presmetaj go zbirot
1 1 1 1
S= + + + ... + .
1!⋅(n − 1)! 3!⋅(n − 3 )! 5!⋅(n − 5 )! n!⋅(n − n)!

x 4 + x3 + x 2 − x + 1
922. Presmetaj go zbirot na beskrajniot red ∑
x =2 x6 − 1
.

923. Presmetaj ja sumata S2015=[ 1 ]+[ 2 ]+[ 3 ]+…+[ 2015 ].


 3 44 2013 
924. Presmetaj ja vrednosta na izrazot:  2 + 3 + + ... + 2013 .
 2 3 2012 
925. Neka S e zbir od site x, 1 ≤ x ≤ 99 i {x²}={x}². Presmetaj [S].
926. Neka za prirodnite broevi k, n ≥ 2, definirame
 2n+1 + 1  3n+1 + 1  kn+1 + 1
S(k,n) =  n−1  +  n−1  + ... +  n−1  . Presmetaj ja vrednosta na
 2 + 1  3 + 1  k + 1
izrazot S(10,112)-S(10,55)+S(10,2).
927. Neka a e fiksen priroden broj. Najdi gi site prirodni broevi n za
1 1 1 1 a-n +a
koi + + + ... +
( )
= .
1• 2 2 • 3 3 • 4 n n +1 a - n + a +1
928. Doka`i deka za bilo koj broj nœN, va`i:
(n+1)·(n+2)·…·(n+(n-1))·(n+n)=2n·1·3·5·…·(2n-1).
n
929. Presmetaj go zbirot ∑ k ⋅ k! .
k =1
n
k −1 1
930. Doka`i deka ∑ = 1− .
k =1 k! n!
931. Dadena e nizata od realni broevi x1, x2, …, x2013, x2014

87
Nizi i progresii

x n+1 + 2x n-1
takva {to x1=x2014 i xn = za n=2,3,…1013. Doka`i deka
3
site ~lenovi od nizata se me|usebno ednakvi .
932. Dadena e niza od mno`estva {1}, {3,5}, {7,9,11}, ... obrazuvani od
neparnite prirodni broevi. Doka`i deka zbirot na elementite od
sekoe mno`estvo e ednakov so kubot od brojot na elementite vo
samoto mno`estvo.
933. Dadena e nizata a1,a2,…,a16 od 16 razli~ni prirodni broevi takvi
{to k§ai§k+29, "kœN. Doka`i deka pome|u dadenite broevi
postojat dva broja zaemno prosti.
934. Od ~etvorkata broevi (a,b,c,d) se dobiva vtora ~etvorka
(a+b,b+c,c+d,d+a) od nea po isto pravilo se dobiva treta ~etvorka i
t.n. Doka`i deka vo dobienata niza
nema dve ednakvi ~etvorki (osven koga
a=b=c=d=0).
935. Formirani se nizite kako na crte` 53.
Doka`i deka zbirot na ~lenovite vo
sekoj horizontalen red e to~en kvadrat
na neparen broj.
Crte` 53

936. Dadena e slednata triagolna tablica od broevi:

0 1 2 ... 2018 2019 2020


1 3 ... 4037 4039
4 ... 8076
… ... ...
...
Sekoj broj, (po~nuvaj}i od vtoriot red) e ednakov so zbirot na dvata
sosedni broja od prethodniot red. Doka`i deka brojot {to se nao|a
vo posledniot red e deliv so 2020.
937. Dadeni se broevite a0,a1,a2,...,an-1,an za koi a0=an=0 i ak-1-2ak+ak+1>0
(k=1,2,...,n-1). Doka`i deka site broevi a1,a2,...,an-1 se negativni.
938. Dadena e nizata an od prirodni broevi za koja a1=1, ak=ak-1+k.
Doka`i deka an+an+1=(n+1)2.
939. Dadena e nizata a0, a1, a2, …, a100 vo koja e poznato deka
а1 > a0,
a2 = 3a1 – 2a0,
a3 = 3a2 – 2a1,…,a100 = 3a99 – 2a98.
Doka`i deka а100 > 299.
 20  21   2n  2n+1
940. Doka`i go ravenstvoto  2 + 1 2 + 1. ⋅ ... ⋅  2 + 1 = 2 - 1.
    
1 1 1 n -1 1
941. Doka`i go ravenstvoto: + + ... + n-1 n
= ⋅ .
lg2lgx 4 lgx 4lgx 8 lgx 2 lgx 2 n lg2x 2

88
Nizi i progresii

942. Neka e dadena nizata a1=2, a2=3, a3=7, a4=43 i op{to


1 1 1
an+1=1+a1ÿa2ÿ...ÿan za site n≥1. Ako bn = + + ... + i
a1 a 2 an
1
cn = toga{ presmetaj ja vrednosta na izrazot b100+c100.
a 1 ⋅ a 2 ⋅ ... ⋅ a n
943. Neka F1=1, F2=1, F3=2, F4=3, F5=5 i op{to Fn=Fn-1+Fn-2 za site n≥3.
17
a)Doka`i deka Fn/Fn-1< za site n¥4.
10
b)Doka`i deka F2n/Fn e sekoga{ cel broj.
(Zabele{ka: Ovaa niza e poznata kako Fibona~ieva niza).
944. Doka`i deka brojot 120 33...3
123 08 se deli so 19.
k - trojki

945. Kolku ~lenovi od nizata 1, 11, 111, ..., 111


142 11 se delivi so 7?
...4
3
2010
946. Ako broevite a2, b2, c2 (a>b>c) obrazuvaat aritmeti~ka progresija
toga{ a2-c2 se deli so 48. Doka`i.
947. ^etirite realni re{enija na ravenkata (x2-2x+m)ÿ(x2-2x+n)=0
1
formirat aritmeti~ka progresija so prv ~len . Presmetaj 6m-n6.
4
948. Ako a2, b2 i c2 se posledovatelni ~lenovi na aritmeti~ka progresija
1 1 1
toga{ , , se isto taka posledovatelni ~lenovi na
b+c c +a a+b
aritmeti~ka progresija. Doka`i.
949. Vo aritmeti~ka progresija zbirot na prvite n ~lenovi e ednakov so
zbirot na prvite m ~lenovi (m ∫ n). Dokaæi deka zbirot na prvite
m+n ~lenovi e nula.
950. Aritmeti~kata progresija a1, a2, …, an se sostoi od n celi broevi,
pri {to zbirot na prvite n ~lena e stepen so osnova 2. Doka`i deka
i n e stepen so osnova dva .
2
951. Ako za aritmeti~kata progresija va`i ravenstvoto Sm = m2 toga{
Sn n
a m 2m − 1
doka`i deka va`i = .
an 2n − 1
952. Ako a1, a2, …, an obrazuvaat aritmeti~ka progresija toga{
1 1 1 n -1
+ + ... + = . Dokaæi.
a1a 2 a 2 a 3 a n-1a n a1a n
953. Vo sekoja aritmeti~ka progresija a1, a2, …, an so pozitivni ~lenovi
vaæi 1 1 1 n - 1 . Dokaæi.
+ + ... +
a1 + a2 a2 + a3 an - 1 + an a1 + an

89
Nizi i progresii

954. Ako logk x , logm x i logn x se posledovatelni ~lenovi na aritmeti~ka


progresija toga{ va`i n2 = (kn )log k m . Dokaæi.
955. Za koja vrednost na x trite izrazi lg2, lg(2x-1) i lg(2x+3) se
posledovatelni ~lenovi na aritmeti~ka progresija?
956. Ako vo ostroagolniot triagolnik ABC broevite tgα, tgβ, tgγ se
posledovatelni ~lenovi na aritmeti~ka progresija toga{ i
sin2α, sin2β, sin2γ isto taka se posledovatelni ~lenovi na
aritmeti~ka progresija. Doka`i.
957. Ako vo ostroagolniot triagolnik ABC va`i ravenstvoto
sin α +sin β +sinγ= 3 ⋅ (cosα + cosβ + cosγ ) toga{ aglite vo toj
triagolnik se posledovatelni ~lenovi na aritmeti~ka progresija.
Doka`i.
958. Ako stranite a, b, c vo triagolnikot
ABC (crt. 54) obrazuvaat aritmeti~ka
progresija (vo dadeniot redosled)
toga{ otse~kata koja gi soedinuva
te`i{teto i centarot na vpi{anata
kru`nica vo triagolnikot ABC e
paralelna so stranata b. Doka`i.
Crte` 54
959. Proizvodot na soodvetnite ~lenovi na dve aritmeti~ki progresii
ja davat nizata so ~lenovi 1440, 1716, 1848, … . Odredi go osmiot
~len vo novata niza .
960. Ako lgm, lgn i lgp se tri posledovatelni ~lenovi na geometriska
progresija toga{ logmx, lognx i logpx se isto taka tri posledovatelni
~lenovi na geometriska progresija. Doka`i.
961. Pozitivnite broevi a, b, c se ~lenovi na geometriska progresija
log a N log a N - logb N
ako i samo ako = , N>0. Dokaæi.
log c N log bN - log c N
962. Ako (an) e geometriska progresija so pozitivni ~lenovi za koi
a1+a2+…+a2017=2 i 1 + 1 + ... + 1 = 1 toga{ odredi ja vrednosta na
a1 a 2 a 2017
proizvodot a1·a2·…·a2017 .
963. Dadena e nizata a1, a2, ..., a2019 za koja a1+a2+...+a2019=0 i
|a1-2a2|=|a2-2a3|=...=|a2018-2a2019|=|a2019-2a1|. Doka`i deka
a1=a2=...=a2019=0.
964. Neka pozitivnite broevi a, b, c se posledovatelni ~lenovi na
geometriska progresija za koi b-a e poln kvadrat i
log6a+log6b+log6c =6. Presmetaj go zbirot a+b+c.
965. Neka a1, a2, ..., an e niza od pozitivni realni broevi za koi va`i
ravenstvoto (a1+a2+...+an-1)(a2+a3+...+an)=(a1a2+a2a3+a3a4+...+an-1an)2
Doka`i deka nizata a1, a2, ..., an obrazuva geometriska progresija.
966. Dadeni se sistemite:

90
Nizi i progresii

a1 + a 2 + a3 = 21 a1 + a 2 + a 3 = 195


a)  1 1 1 7 i b)  .
a a+ + = a 3 − a 1 = 120
 1 2 a 3 12
Opredeli gi a1, a2 i a3 ako se posledovatelni ~lenovi na
geometriska progresija .
967. Neka Sn e zbirot od prvite n ~lenovite na geometriska progresija,
Sn S − Sn
Sn∫0, q ∫0. Doka`i deka = 2n .
S 2n − S n S 3n − S 2n
968. Neka O e centarot na vpi{anata kru`nica vo triagolnikot ABC.
Ako eden od triagolnicite AOB, BOC ili COA e sli~en so
triagolnikot ABC toga{ aglite vo triagolnikot ABC obrazuvaat
geometriska progresija. Doka`i.
969. Ako prvite dva ~lena na aritmeti~ka progresija se pozitivni
(razli~ni me|usebe) i se sovpa|aat so prvite dva ~lena na
geometriska progresija, toga{ site ~lenovi na aritmeti~kata
progresija po~nuvaj}i od tretiot ~len se pomali od soodvetnite
~lenovi na geometriskata progresija. (Teorema na Bernuli)
970. Ako pozitivnite broevi a, b, c obrazuvaat geometriska progresija
1 1 1
toga{ , , obrazuvaat aritmeti~ka progresija.
log a x log b x logc x
Doka`i.
971. Tri broja obrazuvaat geometriska progresija. Ako tretiot ~len se
namali za 64 toga{ dobienite tri broja obrazuvaat aritmeti~ka
progresija. Ako vtoriot ~len na aritmeti~kata progresija se namali
za 8 toga{ povtorno se dobiva geometriska progresija. Koi se tie
broevi?
972. Ako a, b, c obrazuvat aritmeti~ka progresija a x, y, z geometriska
progresija toga{ va`i ravenstvoto xbycza=xcyazb . Doka`i.
973. Ako broevite a, b i c se 5-ti, 17-ti i 37-mi ~len na edna
aritmeti~ka i na edna geometriska progresija toga{ opredeli ja
vrednosta na izrazot ab-c·bc-a·ca-b.
974. Niza od tri realni broja formira aritmeti~ka progresija so prv
~len 9. Ako na vtoriot ~len se dodade brojot 2 a na tretiot ~len se
dodade brojot 20 toga{ taka dobienata niza e geometriska
progresija. Koja e najmalata vrednost na tretiot
~len na geometriskata progresija?
975. Neka a, b i c se strani, a α, β i γ se agli vo
triagolnikot AVS. Ako tgα, tgβ, tgγ obrazuvat
harmoniska progresija toga{ a2, b2, c2 obrazuvat
aritmeti~ka progresija. Doka`i.
976. Beskone~na niza od sferi e vpi{ana vo prav
kru`en konus so radius na osnovata 2 (crt. 55).
Crte` 55

91
Nizi i progresii

Radiusot na najgolemata sfera e 1. Najdi go volumenot na delot od


konusot nadvor od site sferi.
n
2
977. Neka kn se koncentri~ni kru`nici so radiusi   za n=0,1,2,... i
3
neka Pn e plo{tinata na povr{inata ograni~ena
megu vnatre{niot del na kru`nicite k2n i
nadvore{niot del na kru`nicite k2n+1 za
n=0,1,2,... (prvite tri povr{ini se pretstaveni na

crte` 56). Najdi ja sumata ∑P
n=0
n .

Crte` 56
978. Dol`inite na dve strani na ramnostran triagolnik se udvojuvaat
pri {to se dobiva ramnokrak triagolnik. Ovie dve podolgi strani
se udvojuvaat povtorno pri {to se dobiva tret (ramnokrak)
triagolnik. Ovoj proces se povtoruva. Ako aglite pri vrvovite gi
ozna~ime so α1, α2, α3, ... presmetaj ja vrednosta na zbirot
S = (1 - cosα1 ) + (1 - cosα 2 ) + (1 - cosα 3 ) + ... .

92
Elementi od kombinatorika i verojatnost

Elementi od kombinatorika i verojatnost

979. Na eden plo{tad mo`e da se vleze od {est uli~ki od koi dve se


ednonaso~ni.Na kolku na~ini mo`e da se pomine preku plo{tadot.
980. Na kolku razli~ni na~ini mo`e da se rasporedat 12 knigi vo tri
kutii, taka {to vo prvata kutija ima dve knigi, vo vtorata kutija
tri knigi, a ostanatite knigi se vo tretata kutija?
981. Pri sleguvawe od skala so tri nivoa,mo`ni se ~etiri na~ini na
sleguvawe do poslednoto nivo (crt. 57). Ako skalata e so sedum
nivoa, na kolku razli~ni na~ini e mo`no sleguvawe do poslednoto
nivo?

Crte` 57
982. Na kolku na~ini m+n+p predmeti mo`e da se rasporedat vo tri
grupi taka {to vo ednata ima m, vo drugata n a
vo tretata p predmeti?
983. Na kolku razli~ni na~ini, dvi`ej}i se od
bukva do (sosedna) bukva vo slednata {ema
(crt. 58), mo`e da se pro~ita zborot
,,kvadrat”?.
Crte` 58

984. Izgradena e piramida od kocki na 100 nivoa, kako


na crte` 59:
a)Kolku kocki ima vo poslednoto nivo,
smetano od vrvot na piramidata?
b) Kolku vkupno kocki ima vo piramidata?

Crte` 59

985. Na kolku delovi se deli prostorot so n ramnini koi {to minuvaat


niz edna to~ka, taka {to koi bilo tri od niv ne minuvaat niz ista
prava.
986. Povr{inata na sfera e podelena so n ramnini koi {to minuvaat
niz nejziniot centar i koi bilo tri od niv ne minuvaat niz ist
dijametar. Odredi go brojot na delovi pri podelbata.

93
Elementi od kombinatorika i verojatnost

987. Pravoagolnik ABCD so dimenzii 7x4 e


podelen na 28 edine~ni kvadrati. Na kolku
razli~ni na~ini mo`e da se stigne od
temeto A do temeto C, dvi`ej}i se po
stranite na kvadrat~iwata samo desno i
nagore (crt. 60).
Crte` 60

988. Dali mo`e proizvolen triagolnik da se podeli na 2014 sli~ni


triagolnici?
989. Sekoja strana vo triagolnik ABC e podelena na 8 ednakvi delovi
(otse~ki). Odredi go brojot na triagolnici so temiwa vo delbenite
to~ki (A, B i C ne se smetat za delbeni to~ki) koi {to nemaat
strana paralelna so nekoja od stranite na triagolnikot ABC.
990. Neka A i B se sprotivni temiwa na edna kocka. Mravka trgnuva od
temeto A i saka da stigne vo temeto B, dvi`ej}i se samo po rabovite
na kockata. Na kolku na~ini mravkata mo`e da go pomine patot,
pravej}i najmnogu pet ,,~ekori''. Za ,,~ekor'' se smeta patot pominat
me|u dve sosedni temiwa na kockata.
991. Na matemati~ki kongres u~estvuvale 26 matemati~ari: Aleksandar,
Bojan, Viktor, ..., Marko i Nikola. Aleksandar se rakuval samo so
edna li~nost, Bojan se rakuval samo so dvajca, Viktor samo so
trojca lu|e i taka natamu, Marko se rakuval so 25 lu|e. So kolku
lu|e se rakuval Nikola?
992. Grupa od 10 u~enici re{avaat matemati~ki test. Sekoj problem bil
re{en od to~no 7 u~enici. Ako sekoj od prvite 9 u~enici re{il
to~no 4 problemi, kolku problemi re{il desetiot u~enik?
993. Vo ramnina se dadeni 5 to~ki. Me|u pravite koi tie gi formiraat
nema nitu paralelni, nitu normalni. Od sekoja od pette to~ki se
povle~eni normali na pravite opredeleni so ostanatite to~ki.
Odredi go maksimalniot broj na prese~ni to~ki na tie normali,
nesmetaj}i gi pette dadeni to~ki.
994. Od grupa od 9 lu|e treba da se sostavi komisija od 5 ~lena. Na
kolku na~ini e mo`en izborot ako Zoran i Ilija mora da bidat
izbrani zaedno ili voop{to da ne bidat izbrani, dodeka Ana i
Ivana odbivaat da ~lenuvaat zaedno vo komisijata.
1 () () ()
2 3 ()
995. Doka`i deka 1⋅ n + 2 ⋅ n + 3 ⋅ n + ... + n ⋅ n
n = n⋅2 .
n -1

1() () ()
2 3 ()
996. Doka`i deka 12 ⋅ n + 22 ⋅ n + 32 ⋅ n + ... + n2 ⋅ n
n = n ⋅ (n + 1)⋅ 2
n-2.

1 1 1 2 n-1
997. Doka`i go ravenstvoto + + + ... = .
1! (n - 1)! 3! (n - 3)! 5! (n - 5)! n!
998. Presmetaj: a) C 100 - C 100 + C 100 - ... + C 100 ;
0 2 4 100

b) C99 - C99 + C99 - ... - C99 .


1 3 5 99

94
Elementi od kombinatorika i verojatnost
2 2 2
n n n
999. Presmetaj go zbirot S =   +   + ... +   , nœN.
0  1 n
1000. Odredi go brojot na {estcifreni broevi kaj koi tri cifri se
parni, a tri neparni.
1001. Odredi go brojot na ~etiricifreni neparni broevi ~ii cifri
(gledano od levo na desno) se vo strogo opa|a~ki redosled.
1002. Vo vnatre{nosta na stranite AB, BC i CA na triagolnikot ABC,
soodvetno se obele`ani 2, 3 i 7 to~ki. Kolku triagolnici mo`e da
se formiraat od temiwata na triagolnikot ABC i obele`anite
to~ki?
1003. Odredi go zbirot na site ~etiricifreni broevi zapi{ani so
cifrite 1, 2, 3 i 4 (broevite se zapi{ani so razli~ni cifri).
1004. Na kolku delovi se razdeluva eden sedumagolnik so svoite
dijagonali, ako e poznato deka ne postojat tri dijagonali {to
minuvaat niz ista to~ka?
1005. Odredi go brojot na site trojki (a,b,c) taka {to a, b, c se prirodni
broevi pomali od 10 i proizvodot abc e deliv so 20.
1006. Odredi ja najmalata vrednost za n taka {to vo razvojot na binomot
(1+x)n postojat sosedni ~lenovi ~ii {to binomni koeficienti se
odnesuvaat kako 7:5.
( )
1007. Odredi gi x i y taka {to va`i Cyx−1 : Cyx-2 + Cyx−−22 + 2 ⋅ Cxy--12 : Cyx+1 = 3 : 5 : 5 .
1008. Doka`i deka me|u 100 proizvolni prirodni broevi mo`e da se
izberat nekolku ~ija suma e deliva so 100.
1009. Vo vnatre{nosta na kvadrat so strana 1 se nao|aat 51 to~ka.
Doka`i deka postojat tri to~ki koi mo`at da se pokrijat so krug so
radius 1/7.
1010. Vo ramnina se dadeni 25 to~ki taka {to od
proizvolni tri to~ki, dve se na rastojanie
pomalo od 1. Doka`i deka postoi krug so
radius 1 vo koj {to se nao|aat najmalku 13 od
dadenite to~ki.
1011. Doka`i deka figurata sostavena od 54
edine~ni kvadrati (crt. 61 1) nemo`e da se
pokrie so pravoagolnici 3x1. Crte` 61

1012. Dadeni se 18 otse~ki so dol`ini xi i=1,2,...,18, pri {to 1 ≤ xi ≤ 2009


za sekoj i=1,2,...,18. Doka`i deka me|u ovie otse~ki postojat tri so
koi {to mo`e da se konstruira triagolnik.
1013. Doka`i deka me|u 20 posledovatelni prirodni broevi, postoi broj
~ij zbir na cifri e deliv so 11.
1014. Sekoja od stranite na eden kvadrat e podelen
podelena so n to~ki. Odredi
go vkupniot broj na triagolnici so temiwa vo tie to~ki (temiwata
na kvadratot ne se teme na nekoi od triagolnicite)
triagolnicite).

95
Elementi od kombinatorika i verojatnost

1015. Doka`i deka pome|u 99 posledovatelni prirodni broevi postoi


broj ~ij zbir na cifri e deliv so 14.
1016. Doka`i deka vo sekoe mno`estvo od 11 prirodni broevi, postojat 6
~ii zbir e deliv so 6.
1017. Dadeni se osum razli~ni prirodni broevi, ne pogolemi od 15.
Doka`i deka me|u nivnite pozitivni razliki postojat tri koi se
ednakvi me|usebe.
1018. Vo kvadrat so strana 1 se smesteni 100 figuri, ~ii zbir na
plo{tini e pogolem od 99. Doka`i deka vo kvadratot postoi to~ka
koja pripa|a na site 100 figuri.
1019. Ramnina e podelena so n pravi. Odredi go maksimalniot broj na
oblasti pri taa podelba.
1020. Na prava le`at n otse~ki. Dokaæi deka ako koi bilo dve otse~ki
imaat zaedni~ka to~ka, toga{ postoi barem edna to~ka koja pripa|a
na site otse~ki.
1021. Na masa so plo{tina 6m2 postaveni se tri prekriva~i, sekoj so
povr{ina od 3m2. Doka`i deka postojat dva od niv koi se
prepokrivaat so povr{ina ~ija plo{tina ne e pomala od 1m2.
1022. Vo pravoagolnik so plo{tina 5 se smesteni devet pravoagolnici,
sekoj so plo{tina 1. Doka`i deka postojat dva pravoagolnici ~ij
zaedni~ki del ima plo{tina ne pomala od 1/9.
1023. Broevite 50, 100, 150, ..., 1500 (vkupno 30) se napi{ani vo niza po
nekoj redosled. Doka`i deka postojat 3 posledovatelni broevi vo
nizata ~ij zbir e pogolem ili ednakov na 2350.
1024. Dali kvadrat mo`e da se podeli na 2014 pomali kvadrati (ne mora
da bidat so ednakvi strani)?
1025. Dali kocka mo`e da se podeli na 2012 pomali kocki (ne mora da
bidat so ednakvi rabovi)?
1026. Odredi go ostatokot pri delewe na brojot
 2017   2017   2017 
S = 1  + 2
  2 
 + ... + 2017 
 2017  so 25.
 1     
1027. 17 nau~nici korespondiraat me|usebno (sekoj nau~nik
korespondira so ostanatite 16). Vo nivnite korespondencii tie
pi{uvat samo na tri temi. Sekoj par nau~nici korespondira samo
na edna tema. Doka`i deka postojat trojca nau~nici koi
korespondiraat me|u sebe na edna ista tema.
1028. Doka`i deka pome|u sekoi {est lica postojat tri lica koi
me|usebno se poznavaat ili tri lica koi me|usebno ne se poznavaat
(dve lica se poznavaat ako sekoj od niv go poznava drugiot).
1029. Vo grupa od 2n lica sekoj se poznava so ne pomalku od n lica.
Doka`i deka od grupata mo`e da se izberat ~etiri lica i da se
rasporedat okolu kru`na masa , taka {to sekoj od niv sedi do svoi
poznanici.

96
Elementi od kombinatorika i verojatnost

1030. Vo edna kompanija rabotat 2n+1 lu|e (n¥1). Za proizvolni n lu|e,


postoi ~ovek od ostanatite n+1 koj gi poznava site niv. Doka`i
deka vo kompanijata postoi ~ovek koj se poznava so site.
1031. Na eden natprevar po matematika u~estvuvaat 35 u~enici. Doka`i
deka me|u niv postojat barem dvajca koi imaat ist broj poznanici.
1032. Stranite i dijagonalite na konveksen {estagolnik se oboeni so
sina ili crvena boja. Doka`i deka postoi barem eden triagolnik
~ii strani se oboeni so ista boja .
1033. Neka ramninata e oboena so tri boi. Doka`i deka postojat dve
to~ki so ista boja ~ie {to rastojanie e 1.
1034. Sekoja to~ka vo ramninata e oboena so crvena, zelena ili sina
boja. Doka`i deka vo ramninata postoi pravoagolnik ~ii temiwa
se oboeni so ista boja.
1035. Temiwata na site ramnostrani triagolnici vo ramninata se bojat
so edna od dve dadeni boi. Doka`i deka postoi ramnostran
triagolnik so temiwa so ista boja.
1036. Sekoja to~ka so celobrojni koordinati od XOY ramninata e oboena
so edna od tri razli~ni boi. Doka`i deka postoi pravoagolen
triagolnik ~ii temiwa se so razli~ni boi (site tri boi se
zastapeni).
1037. Site 6 strani na kocka so dimenzii nxnxn se oboeni, a potoa
kockata e podelena na edine~ni kocki (so dimenzii 1x1x1). Odredi
go brojot na edine~ni kocki kaj koi najmalku dve strani se oboeni.
1038. Sakame broevite 1, 2, ..., 10 da gi oboime so crvena, zelena i sina
boja, taka {to ako razlikata na dva broja e neparna, tie mora da
bidat oboeni so razli~na boja. Na kolku na~ini e mo`no boeweto,
ako pritoa ne mora da se upotrebat site tri boi?
1039. Na dvata polulaci od kru`nicata k (crt. 62)
izbrani se po pet razli~ni to~ki. So
povrzuvawe na to~kite od edniot lak so
to~kite od drugiot lak, se dobivaat 25
razli~ni tetivi. Odredi go brojot na
triagolnicite formirani od tie tetivi takvi
{to najmalku edno nivno teme le`i na
kru`nicata (koi bilo tri tetivi ne se se~at vo
ista to~ka). Crte` 62

1040. Odredi go vkupniot broj kvadrati formirani od edine~nite


kvadrati na standardna {ahovska tabla.
1041. Na kolku na~ini mo`at da se postavat 2 topa na standardna
{ahovska tabla, taka {to tie se nao|aat na poliwa so razli~na
boja i ne se napa|aat.
1042. Na kolku razli~ni na~ini mo`e da se rasporedat osum {ahovski
figuri na crnite poliwa od {ahovskata tabla, taka {to koi bilo
dve figuri ne se nao|aat vo ist red ili vo ista kolona.

97
Elementi od kombinatorika i verojatnost

1043. Doka`i deka {ahovska tabla so dimenzii 8x8 ne mo`e da se pokrie


so 15 figuri od oblik i edna figura od oblik .
1044. Na tabla so dimenzii 2nx2n se nao|aat crni i beli figuri. Vo
sekoja vertikala kade {to ima bela figura, crnite figuri se
zamenuvaat so beli, a potoa, vo sekoja horizontala kade {to ima
crna figura, belite figuri se zamenuvaat crni. Doka`i deka na
krajot
jot brojot na crnite ili brojot na belite fiuri na tablata ne e
pogolem od n2.
1045. Vo tabla so dimenzii nxn (n-paren paren broj) po~nuvaj}i od gorniot lev
agol do dolniot desen agol, posledovatelno gi zapi{uvame
prirodnite broevi od 1 do n2. Na toj na~in vo prvata redica se
zapi{ani broevite 1, 2, ..., n, vo vtorata broevite n+1, n+2, ..., 2n i
se taka do poslednata redica redica, vo koja se zapi{ani broevite
(n-1)n+1, (n-1)n+2, ..., n2. Doka`i deka при sekoja promena na znacite
n n
na broevi во sekoja redica, no taka {to broevi istovremeno
2 2
go menuvaat znakot i vo sekoja kolona, zbir
zbirot na site broevi od
tablata е 0.
1046. Na kolku na~ini mo`at da se izberat edine~ni kvadrati na tabla
so dimenzii 8x8,, taka {to ne postojat dva koi se nao|a
nao|aat vo ista
redica ili ista kolona.
1047. Vo poliwata na {ahovska tabla, proizvolno se zapi{ani broevite
od 1 do 64. Doka`i deka postojat dva sosedni broevi ~ija razlika
ne e pomala od 5 (sosedni broevi se zapi{ani vo kvadratni
poliwa koi imaat zaedni~ka strana).
1048. Na eden {ahovski turnir u~estvuvale u~enici od treta i ~etvrta
godina od srednite u~ili{ta. Site u~enici igrale me|usebno po
edna partija. U~enicite od ~etvrta godina bile ~etiri pati pove}e
od u~enicite od treta godina i osvoile {est pati pove}e poeni od
niv. Kolku u~enici i od treta godina u~estvuvale na turnirot i
kolku vkupno poeni osvoile?
1049. Eden {ahist, podgotvuvaj}i se, igral najmalku edna partija dnevno.
Za da ne se izmori, toj igral najmnogu 12 partii nedelno. Doka`i
deka postojat nekolku posledovatelni denovi vo koi { {ahistot
odigral vkupno 20 partii.
1050. Na standardna {ahovska tabla 8x8 se obele`uvaat 8 edine~ni
kvadrati. Pri obele`uvaweto se vodi smetka edine~nite kvadrati
vo }o{iwata
o{iwata da ostanat neobele`ani i da ne postojat dva
obele`ani kvadrati vo ista redica ili vo istista kolona. Na kolku
na~ini e mo`no obele`uvaweto?
1051. Na kolku na~ini mo`at da se rasporedat 4 sivi i 4 zeleni top~iwa
vo kutija od 4x4 dela (pregradi)
(pregradi), taka {to sekoja redica i sekoja
kolona sodr`i samo edno sivo i edno zeleno top~e (vo sekoj del
mo`e da se postavi
stavi samo edno top~e
top~e)?

98
Elementi od kombinatorika i verojatnost

1052. Avtomobilska registarska tabli~ka se sostoi od tri kirili~ni


bukvi i tri cifri. Ako site oznaki se ednakvo verojatni, odredi ja
verojatnosta bukvite ili cifrite da formiraat palindrom.
1053. Standardna kocka za igrawe e frlena 3 pati. Koja e verojatnosta
na nastanot ,,Proizvodot na trite dobieni broja e deliv so 8''.
1054. Od kutija so {est crveni, ~etiri sini i dve zeleni top~iwa
slu~ajno se izbiraat tri top~iwa. Koja e verojatnosta deka trite
izbrani top~iwa ke bidat so ista boja?
1055. Paskal ima {est `olti i ~etiri sini top~iwa, sekoe razli~no od
drugite. Toj gi postavuva top~iwata vo redica. Koja e verojatnosta
deka nitu edno sino top~e ne se nao|a do drugo sino top~e?
1056. Dva tima igraat serija od 9 igri. Pobednik e timot koj }e pobedi vo
5 igri. Prviot tim pobeduva vo sekoja igra so verojatnost 2/3,
dodeka vtoriot tim pobeduva so verojatnost 1/3. Koja e
verojatnosta deka vo to~no 7 odigrani igri }e bide dobien
pobednikot?
1057. Daniel i Ivan igraat edna igra pove}e pati. Pri pobeda vo sekoja
igra se dobiva 1 poen. Kone~en pobednik e onoj natprevaruva~ koj
}e ima dva poeni pove}e od svojot protivnik. Ako e poznato deka
Daniel ima 60% {ansi da pobedi vo sekoja igra, koja e
verojatnosta toj da bide kone~en pobednik?
1058. Neka a/b e verojatnosta deka proizvolno izbran pozitiven delitel
na 122017 e delitel i na 122010, kade {to a i b se zaemno prosti
pozitivni celi broevi. Odredi go ostatokot koga a+b se deli so
2017.
1059. Neka a/b e verojatnosta deka slu~ajno izbran delitel na 2025 e
deliv so 5. Ako a i b se zaemno prosti prirodni broevi, odredi go
zbirot “a+b”.
1060. Sekoe edine~no kvadrat~e od kvadrat so dimenzii 3x3 e oboeno so
crvena ili sina boja (so ednakva verojatnost). Odredi ja
verojatnosta na nastanot ,,Ne postoi kvadrat so dimenzii 2x2
oboen so crvena boja”.
1061. Tri kocki, numerirani od 1 do 6, istovremeno se frlaat. Koja e
verojatnosta deka trite kocki na gornata povr{ina }e poka`at
broevi koi mo`at da pretstavuvaat dol`ini na strani na
triagolnik?
1062. Kaj drvena kocka so dimenzii 4x4x4 se oboeni pet strani, a potoa
kockata e podelena na 64 edine~ni kocki. Koja e
verojatnosta kaj slu~ajno izbrana edine~na kocka,
to~no dve od pette vidlivi strani da se oboeni?
1063. So to~kite M i N, sredini na otse~kite AB i CD,
kvadratot ABCD so strana 2x e podelen na
petagolnik i triagolnik (crt. 63). Odredi ja
verojatnosta slu~ajno izbrana to~ka od kvadratot
da le`i vo triagolnikot MBN . Crte` 63

99
Elementi od kombinatorika i verojatnost

1064. Eden u~enik fiksira edno teme od konveksen


2018-agolnik. Koja e verojatnosta dijagonala povle~ena od drug
u~enik da minuva niz toa teme?
1065. Od kutija so crveni i sini top~iwa izvlekuvame naedna{ dve
top~iwa. Poznato e deka verojatnosta da se izvle~at dve crveni
top~iwa e 5 pati pogolema od verojatnosta da se izvle~at dve sini
top~iwa. Osven toa, poznato e deka verojatnosta da se izvle~at
dve top~iwa so razli~na boja e 6 pati pogolema od verojatnosta da
se izvle~at dve sini top~iwa. Kolku crveni, a kolku sini top~iwa
se nao|aat vo kutijata?

100
Funkcii. Funkcionalni ravenki

Funkcii. Funkcionalni ravenki

1066. Za realnata funkcija f(x) va`i f(x)=1-f(x-1) i f(2)=12. Presmetaj


f(2019).
1067. Realnata funkcija f ja zadovoluva funkcionalnata ravenka
1
f( )-3f(x)=x, x∫0. Presmetaj f(2).
x
1068. Neka f(x) e definirana so f(x+y)=f(x)+f(y)+6xy+1 ("x,yœZ) i
f(-x)=f(x). Presmetaj f(3).
1069. Neka f(x)=x3+px2+qx+r=0 e polinom od tret stepen so eden dvoen
koren a i drug koren b, kade {to a i b se realni broevi.
Ako p=–6 i q=9, presmetaj ja vrednosta na r?
1070. Funkcijata f: N→N ja zadovoluva ravenkata f(f(x)+f(y))=x+y
(x,yœN). Presmetaj f(2011).
1071. Za funkcijata f: N→N va`i f(1)+f(2)+...+f(n)=n2f(n) i f(1)=1010.
Presmetaj f(2020).
x −1
1072. Dadena e funkcijata f(x) = , xœR. Presmetaj f(2010)(x)
x +1
(f(2)(x)=f(f(x)); f(n+1)(x)=f(f(n)(x)).
x
1073. Odredi funkcija f: R+→R+ za koja va`i f(f(…f(x))) = .
14243 x +1
2018
1
1074. Neka za funkcijata f: R→R va`i f(x)+(x+ )ÿf(1-x)=1. Presmetaj
2
f(0) i f(1).
1075. Odredi gi site funkcii f: R\{0}→R za koi va`i

f(x)+8f   =-63x, x∫0 .


1
x
1
1076. Neka m e maksimalnata celobrojna vrednost na x16 + , kade
x16
{to x e re{enie na ravenkata x6-4x4-6x3-4x2+1=0. Odredi go
ostatokot pri deleweto na m so 2017.
1077. Odredi gi site funkcii f: Q→Q za koi va`i f(1)=2 i
f(xy)=f(x)f(y)-f(x+y)+1.
1078. Dadena e funkcijata f: R→R definirana so f(x)=x2+2x. Odredi gi
site vrednosti na x za koi f(f(f(x)))=255.
1079. Odredi gi site funkcii f: R/{0.1}→R za koi va`i f(x)+f 
1 
 =x.
 1- x 

101
Funkcii. Funkcionalni ravenki

 
1080. Odredi gi site funkcii f: R→R za koi f(x) + f  1  = 1
 1 - x  x ⋅ (
1 − x)
(x∫0, x∫1).
1081. Odredi gi site funkcii f: R→R za koi va`i f(-x)=-f(x) i
f(x+1)=f(x)+1.
1082. Odredi gi site realni funkcii f koi gi ispolnuvaat ravenstvata
f(xy)=f(x)ÿf(y) i f(x+y)=f(x)+f(y)+2xy.
1083. Odredi gi site funkcii f: R→R koi ja zadovoluvaat ravenkata
f((x − y)2 ) = (f(x))2 − 2xf(y) + y 2
1084. Odredi gi site funkcii f: R→R koi {to ja zadovoluvat ravenkata
f(xf(y)+x)=xy+f(x), "x,yœR.
1085. Odredi gi site funkcii f(x) za koi e ispolneto f(xf(y))+f(yf(x))=2xy
(x,yœR).
1086. Odredi gi site funkcii f: R→R za koi e ispolneto
2 4
x f(x) + f(1-x) = 2x-x , "x,yœR.
1087. Odredi gi site realni funkcii f: R→R za koi va`i ravenstvoto
f(xf(y) + yf(x))=y f(x) (x,yœR).
1088. Odredi gi site neprekinati funkcii f: R→R za koi va`i
f(x + f(y))= f(x) + y.
1089. Odredi gi site funkcii f: R→R koi gi zadovoluvaat uslovite:
a) f(x+f(y))= f(x+y)+1, "x,yœR;
b) e strogo monotono raste~ka funkcija.
1090. Odredi gi site funkcii f: R→R za koi e ispolneto ravenstvoto
f(x2-y2)=(x-y)[f(x)+f(y)], "x,yœR.
1091. Odredi gi site funkcii f: R→R za koi va`i f(1)=1 i
y x
f(x+y)=3 f(x)+2 f(y), "x,yœR.
1092. Odredi gi site realni funkcii f: R→R za koi
f(x)f(y)f(z)-f(xyz)=xy+yz+zx+x+y+z "x,y,zœR.
1093. Opredeli gi site funkcii y=f(x) koi ja zadovoluvaat ravenkata
(x − 1) ⋅ f  x + 1 − f (x ) = x , (x∫1).
 x − 1
1094. Odredi gi site funkcii f: Z→Z za koi e ispolneto
f(f(x)+y+1)=x+f(y)+1 "x,yœZ.
1095. Odredi gi site funkcii f: R→R za koi {to va`i
f(4xy)=2y(f(x+y)+f(x-y)), "x,yœR i f(2020)=2021.

102
Odgovori, upatstva i re[enija

103
Odgovori, upatstva i re[enija

Tema 1: Matemati~ka logika i mno`estva


1. Branko, Milena i Jovan; Goran, Daniela i Daniel; Nikola, Marija i Milan.
2. K i P la`at a M ja zboruva vistinata. 3. Goki-~esen, Koki-la`go, Roki-
mudrec. Roki ne mo`e da bide praveden bidej}i toga{ i Koki }e bide
praveden, {to e nevozmo`no. Zna~i Roki ili la`e ili e praveden. Da
pretpostavime deka Roki la`e. Toga{ Koki e itrec (ne mo`e da bide la`go i
ne mo`e da bide praveden, bidej}i negoviot iskaz bi bil nevistinit). No,
toga{ Koki mora da bide praveden {to protivre~i na negoviot iskaz.
Sleduva deka Roki e itrec. Spored iskazot na Koki toj e la`go, a ostanuva
Roma da bide praveden, ~ij iskaz e i vistinit. 4. 2, 2, i 9, brojot na avtobusot
e 13. 5. Ana izbrala lenir, Marija izbrala topka, a Olga {estar. 6. 3 u~enici.
Neka TKF e vkupniot broj na u~enici koi gi igrat site tri sportovi, K brojot
na u~enici koi igraat ko{arka, T brojot na u~enici koi igraat tenis, F brojot
na u~enici koi igraat samo fudbal. Neka TK, KF, TF gi pretstavuvaat
broevite na u~enici koi igraat po dva od soodvetnite sportovi. Toga{,
vkupniot broj na u~enici e ednakov na T+K+F-TK-TF-F+TKF. Zna~i,
50=15+25+30-8-5-10+TKF od kade se dobiva deka TKF=3. 7. 41.
8. Spored uslovite na zada~ata, Angova ne e nitu violinistka, nitu
pisatelka. Ba~eva ne e nitu slikarka, nitu pisatelka. Vasileva ne e
violinistka, a Go{eva ne e pisatelka. Vo kolonata „pisatelka” edinstvenoto
prazno mesto e za Vasileva. Spored uslovot (2), Vasileva i pozirala na
slikarkata, a spored (4) Vasileva ne se videla so Angova, zna~i Angova ne e
Balerina slikarka violinistka pisatelka
Angova + - - -
Ba~eva - - + -
Vasileva - - - +
Go{eva - + - -
slikarka.
Vo redot na Angova edinstvenoto nepotpolneto pole e ona za balerina. Toga{
Ba~eva ne e balerina i edinstvenoto prazno pole vo nejziniot red e
violinistka. Zna~i, Go{eva ne e violinistka, a bidej}i ne e nitu balerina,
ostanuva da bide slikarka. Kone~no, Angova e balerina, Ba~eva e
violinistka, Vasileva e pisatelka, a Go{eva slikarka.
10. 1365. Ako a e najmaliot, toga{ 13-a e najgolemiot element od takvoto
podmno`estvo i ostanatite elementi mo`e da se izberat od 12-2a elementi
12-2a
a+1, a+2, ..., (13-a)-1. Brojot na site vakvi podmno`estva e 2 so najmal
element a. Od toa {to 13-a ≥ a sleduva deka a<7. Sobiraj}i go brojot na takvi
podmno`estva za a=1, 2, ..., 6 dobivame deka vkupniot broj na vakvi
10 8 6 0 5 4 0 6
podmno`estva e 2 +2 +2 +...+2 =4 +4 +...+4 =(4 -1)/(4-1)=1365.
11. Da gi formirame za k=1, 2, … sedumelementnite podmno`estva od
posledovatelni broevi V1 = {7k − 6,7k − 5,7k − 4,7k − 3,7k − 2,7k − 1,7k} ,
V2 = S \ V1 . Za k=1 gi dobivame V1 = {1,2,3,4,5,6,7} i V2 = {8,9,10,11,12,13,14}. Od
ostanatite sedumelementni podmno`estva na mno`estvoto S, to~no 1/14 od

104
Odgovori, upatstva i re[enija
niv imaat zbir na elementi deliv so 14. Sleduva deka baraniot odgovor e
7
C14 −2 3430
x = 1+ = 1+ , odnosno x=1+245. Baraniot broj e x=246.
14 14
12. 201. Za elementite vo mno`estvoto A va`i (a + 1) + (a + 2) + ...+ (a + m) = 2m .
m ⋅ (m + 1) m(2a + m + 1)
Toga{ m ⋅ a + = 2m; = 2m; m ≠ 0. Dobivame 2a+ m+1= 4 od
2 2
3 −m
kade 2a + m = 3 , pa sleduva deka a = ...(1) . Za elementite od mno`estvoto
2
2m⋅ (2m+ 1)
B imame (b + 1) + (b + 2) + ... + (b + 2m) = m . Toga{ 2mb+ = m , od kade
2
m(2b+ 2m + 1) = m ⇒ b = −m …(2). Od b + 2m − a − m = 99 sleduva b + m − a = 99…(3).
3-m
Zamenuvaj}i gi ravenkite (1) i (2) vo ravenkata (3) dobivame - m + m − = 99
2
od kade pak m=201.
13. 795. Proizvodot na tri neparni broja ne e deliv so 4, a neparni broevi od
dadenoto mno`estvo ima 10. Brojot na takvi trielementni podmno`estva e
( )
10
3 . Od podmno`estvoto parni broevi {2,4,6,…,20}, broevite od mno`estvoto
{2,,6,10,14,18,} ne se delivi so 4. Kombiniraj}i eden takov paren broj so dva
neparni broja dobivame istotaka trielementno podmno`estvo so elementi ~ij
proizvod ne e deliv so 4. Brojot na takvi podmno`estva e 1 2 . Toga{ ( )( )
5 10

baraniot broj se dobiva koga od vkupniot broj podmno`estva }e se odzeme


brojot na podmno`estva so proizvod na elementi koj ne e deliv so 4, odnosno
( )( ) ( )
20 10 10
x = 3 − 3 − 5 2 = 795 .
14. b(S)=|S|=61. Ako ab=c za nekoe c>1, toga{ ili a ili b e deliv so d za
2 2

nekoe d>1. Za da go poka`eme toa, prvo da zabele`ime deka mora da postoi


najmalku eden prost mno`itel na c pogolem od 1. Neka p e takviot mno`itel.
2 2
Toga{, p e delitel na a ili p e delitel na b ili p e delitel na a,b i c,
2
pritoa p ne deli nitu eden od niv. Ako posledniot slu~aj e to~en, pritoa a ne
e ednakov so b, toga{ mora da postoi drug prost mno`itel na c i taa postapka
mo`e da se povtoruva pove}e pati. Postapkata nekade zavr{uva, pri{to
dobivame deka ili a ili b e deliv so broj koj e poln kvadrat. Taka, mo`eme da
gi isklu~ime site broevi od dadenoto mno`estvo koi se delivi so broj koj e
poln kvadrat. Me|u 1 i 100 ima 25 broevi delivi so 4. Postojat 11 broevi
delivi so 9, no 2 od niv se delivi i so 36. Postojat 4 broevi delivi so 25, no
eden od tie broevi e deliv i so 100. Na kraj, postojat 2 broja delivi so 49.
Baraniot broj e x=100-25-11+2-4+1-2=61.
15. 4181. Neka S1={1}, S2={1,2}, S3={1,2,3}, S4={1,2,3,4}, ..., S17={1,2,3,...,17}. Da go
razgledame brojot na podmno`estva na dadenite mno`estva koi ne sodr`at
dva posledovatelni broja. Jasno e deka S1 ima 2 takvi podmno`estva, S2 ima
3, S3 ima 5 takvi podmno`estva, a S4 ima 8. Zabele`uvame deka brojot na
podmno`estva se dobiva kako zbir od brojot na podmno`estva na prethodnite
dve mno`estva odnosno xn+2=xn+1+xn , nœ{1,2, ..., 15}. Zna~i, x5=13, x6=21, x7=34,
x8=55, x9=89, x10=144, x11=233, x12=377, x13=610, x14=987, x15=1597, x16=2584,

105
Odgovori, upatstva i re[enija

x17=4181. 16. 601. 17. a) ne; b) da. 18. 1100011. 19. 2021. Upatstvo:
 1  1   1   1 
 1 +  ⋅  1 +  ⋅  1 +  ⋅ ... ⋅  1 +  − 1 = 2022 − 1 = 2021 .
 1  2   3   2021 
20. 1975. Neka 2011 broja se posledovatelno rasporedeni vo krug i neka na
sekoj broj mu go pridru`ime brojot f(n). Na toj na~in ja dobivame rekurentnata
vrska f(2n)=2f(n)-1 i f(2n+1)=2f(n)+1, pri {to jasno e deka f(1)=1. Primenuvaj}i
ja ova vrska dobivame f(2011)=2f(1005)+1; f(1005)=2f(502)+1; f(502)=2f(251)-1;
f(251)=2f(125)+1; f(125)=2f(62)+1; f(62)=2f(31)-1; f(31)=2f(15)+1; f(15)=2f(7)+1;
f(7)=2f(3)+1; f(3)=2f(1)+1; f(1)=1. Vra}aj}i se nazad dobivame f(3)=3; f(7)=7;
f(15)=15; f(31)=31; f(62)=61; f(125)=123; f(251)=247; f(502)=493; f(1005)=987;
f(2011)=1975. 21. 800 броја.
22. Neka S = a 1 - a 2 + a 2 - a 3 + ... + a 2n -1 - a 2n + a 2n - a 1 . Pri osloboduvawe od
sekoja apsolutnata vrednost se dobivaat dva celi broja od mno`estvoto
{- 2n,-2n + 1, ..., 2n} . Za zbirot Ѕ da ja dostigne najgolemata vrednost potreben i
dovolen uslov e broevite od 1 do n da se javat kako negativni, a broevite od
(n+1) do 2n da se pozitivni. Toga{
S = 2[(2n + (2n - 1) + ... + (n + 2) + (n + 1)) - (n + (n - 1) + ... + 2 + 1)]
= 2[(1 + 2 + ... + (2n - 1) + 2n) - 2(1 + 2 + ... + n)] = 2n 2 .
Zna~i, vo apsolutnata vrednost edniot broj e pogolem od n, drugiot e pomal
ili ednakov na n. Toga{
a1 - a2 + a3 - a4 + ... + a2n-1 - a2n = (2n + (2n - 1) + ... + (n + 1)) - (n + (n - 1) + ... + 2 + 1) = n2 .
23. 792. Upatstvo: S n = ( ) odnosno S
n+ 2
5 10 = ( ). 24. Da. Mo`eme od trojkata
12
5
(1,2,3) posle kone~en broj ~ekori da ja dobieme trojkata (1,1500,1499), ~ij zbir
e 3000.
25. Sabota.
-Od 11-ti Juni 2009 g. do 11-ti Juni 2010 g. ima 365 dena [365ª1(mod7)];
-Od 11-ti Juni 2010 g. do 11-ti Juni 2011 g. ima 365 dena [365ª1(mod7)];
-Od 11-ti Juni 2011 g. do 11-ti Juni 2012 g. ima 366 dena [366ª2(mod7)];
-Od 11-ti Juni 2012 g. do 11-ti Juli 2012 g. ima 30 dena [ 30ª2(mod7)];
-Od 11-ti Juli 2012 g. do 11-ti Avgust 2010 g. ima 31 den [31ª3(mod7)].
Zna~i 1+1+2+2+3=9ª2(mod7).
26. Neka 0 < a1 < a 2 < ... < an < an+1 se tie broevi. Gi razgleduvame razlikite
a 2 - a 1, a 3 - a 1, ..., a n+1 - a 1 i broevite a2 , a3 , ..., an+1 . Vo dvete grupi, broevite
vo grupata se razli~ni. Razgleduvame vkupno 2n broevi, site pomali od 2n.
Toga{, spored principot na Dirihle, postojat dva broja koi se ednakvi
me|usebe i toa od dvete razli~ni grupi. Zna~i za nekoi j i k ak - a1 = a j , od
kade se dobiva a k = a1 + a j. .
27. 15625 nuli. Upatstvo: brojot na nuli na koi zavr{uva brojot, zavisi od
brojot na proizvodi 2·5.
28. 2. Proizvodot na broevite od tablata ne se menuva navistina,
a+b 2 a + b 2ab
⋅ = ⋅ = a ⋅b.
2 1 1 2 a+b
+
a b

106
Odgovori, upatstva i re[enija

29. 75000. Broevi od nizata 1, 2, 3, …, 499, 500 …(1) delivi so 4 se:


4=4·1, 8=4·2, 12=4·3, …, 500=4·125 ~ij zbir e S1 = 4 ⋅ (1+ 2 + ... + 125)
125 ⋅ 126
= 4⋅ = 31500 . Broevi od nizata (1) delivi so 5 se: 5=5·1, 10=5·2, 15=5·3,
2
100 ⋅ 101
…, 500=5·100 ~ij zbir e S 2 = 5 ⋅ (1 + 2 + ... + 100) = 5 ⋅ = 25250 . Broevite
2
koi se povtoruvat se vsu{nost broevite delivi so 4 i 5 odnosno delivi so 20.
Takvi broevi od nizata (1) se: 20=20·1, 40=20·2, 60=20·3, …, 500=20·25 ~ii zbir
25 ⋅ 26
e S 3 = 20 ⋅ (1 + 2 + ... + 25) = 20 ⋅ = 6500 .
2
500 ⋅ 501
Ako S = 1 + 2 + ... + 500 = = 125250 toga{ baraniot zbir e
2
x=S-S1-S2+S3=125250-31500-25250+6500 od kade se dobiva deka x=75000.
30. 97 pati
31. Devet tegovi so masa m, m+1, …, m+8 mo`e da se razlo`at na tri dela so
ista masa na sledniot na~in:
I. del gi sodr`i tegovite so masa m, m+4, m+8
II. del gi sodr`i tegovite so masa m+1, m+5, m+6
III. del gi sodr`i tegovite so masa m+2, m+3, m+7.
Toj na~in ovozmo`uva razlo`uvawe na tegovite so masa od 1,2,...,549=61ÿ9.
Ostanuvaat u{te {est tega so masa 550g,551g,…555g ~ija podelba e mo`na na
toj na~in {to prviot del da gi sodr`i tegovite so masa 550g i 555g. Vtoriot
del gi sodr`i tegovite so masa 551g i 554g i tretiot del gi sodr`i tegovite
so masa 552g i 553g.
32. Neka mi e masata na i-tiot teg, a xi masata na i-tiot medal.
Od (m1-m2)+(m2-m3)+(m3-m4)+...+(m9-m10)+(m10-m1)=0 dobivame deka i
≤x1≤x2≤x3≤...≤x9≤x10=0 pri {to pred nekoi xi stoi znakot +, a pred nekoi stoi
znakot-.Jasno, zaradi zbirot ednakov na nula, zbirot na masite koi imaat
negativen predznak e ednakov so zbirot na masite so pozitiven predznak.
Stavaj}i gi medalite xi pred koi stoi znakot „+“ na edna strana a ostanatite
na druga strana, dobivame dve grupi na medali koi imaat ista masa.
33. Prvo merewe: na edniot tas gi stavame monetite od 2 i 3 denari, a na
drugiot tas monetata od 5 denari. Gi razgleduvame slednite slu~ai:
I. Ako tasovite se vo ramnote`a toga{ neispravna e monetata od 1 denar.
II. Ako monetata od 5 denari e pote{ka, toga{ pravime vtoro merewe: na
edniot tas gi stavame monetite od 1 i 2 denari, a na drugiot od 3 denari
toga{:
a) pri ramnote`a-neispravna e monetata od 5 denari;
b) ako monetata od 3 denari e pote{ka toga{ neispravna e
polesnata moneta, a toa e monetata od 2 denari.
v) ako monetata od 3 denari e polesna, toga{ taa e neispravna
moneta.
III. Ako monetata od 5 denari e polesna (monetata od 1 denar e ispravna),
toga{ pravime vtoro merewe: na edniot tas gi stavame monetite od 1 i 2
denari, a na drugiot od 3 denari toga{:
a) pri ramnote`a-neispravna e monetata od 5 denari;

107
Odgovori, upatstva i re[enija
b) ako monetata od 3 denari e pote{ka, toga{ taa e neispravna moneta.
v) ako monetata od 3 denari e polesna, toga{ neispravna e monetata od 2
denari.
34. 5 minuti. Ja sostavuvame {emata
od kade sleduva deka x:5=3:5
=5:3
odnosno ja dobivame proporcijata x:5=(3Α5):(5Α3), ~ie re{enie e x=5.
35. Petranka dobila 300 den.
36. aÿbÿc=24=2ÿ2ÿ2ÿ3. Postojat 6 mo`nosti (i nivnite permutacii) za
razlo`uvawe na mno`iteli 1.) a=1, b=1, c=24; 2.) a=1, b=2, c=12; 3.) a=1, b=3,
c=8; 4.) a=1, b=4, c=6; 5.) a=2, b=2, c=6; 6.) a=2, b=3, c=4. Od site 6 slu~ai
uslovite od zada~ata gi ispolnuva samo pettiot slu~aj. Zna~i zbirot na
broevite a, b i c e 10. 37. 34 i 38.
38. 700 `iteli. Neka n ma`i bile vo brak, toga{ i n `eni bile vo brak. Od
 2  3n  
uslovot na zada~ata imame deka 3n/2 bile ma`i    = n  , 5n/3 bile `eni.
3 2  
3n 5n
Zna~i vkupniot broj `iteli na ostrovot e + =1900, od kade se dobiva
2 3
deka n=600. Bidej}i vo brak bile vkupno 2n=1200 `iteli, baraniot odgovor e
700 `iteli. 39. Brojot na ku}ata e 14.
40. I re{enie: Neka x e vkupnata suma koja treba da se podeli. Toga{
x − 1000 x 9x
prvorodenoto dete dobilo 1000 + = + 900 a ostanuvaat − 900 .
10 10 10
9x
− 900 + 2000
9x
Vtororodenoto dete dobilo 2000 + 10 = + 1710 . Taka imame
10 100
x 9x
deka + 900 = + 1710 , od kade se dobiva deka x=81000$. Toga{,
10 100
81000 − 1000
prvorodenoto dete dobilo 1000 + = 9000 $ kako i ostanatite
10
deca. Zna~i dobivame deka bile vkupno 81000:9000=9 deca.
II re{enie: Neka S=nk, kade n e brojot na deca, a K iznosot pari. Toga{,
S − 1000
prvorodeniot }e dobie iznos 1000 + =K ...(1).
10
S − K − 2 ⋅ 1000
Vtororodeniot }e dobie 2·1000 + =K …(2).
10
S − 2K − 3 ⋅ 1000
Tretorodeniot }e dobie 3·1000 + =K …(3).
10
S − (n − 1)K − n ⋅ 1000
Sli~no, n-rodeniot }e dobie n·1000 + =K.
10
Koga }e gi sobereme site ovie ravenstva dobivame
nS − K ⋅ (1 + 2 + ...(n − 1)) − 1000 ⋅ (1 + 2 + ... + n)
1000 ⋅ (1 + 2 + ...n) + = n ⋅K .
10

108
Odgovori, upatstva i re[enija

n(n + 1) (n − 1)n n(n + 1) n(n + 1)


Ottuka, 10000 ⋅ + nS − K − 1000 = 10nK . (1+2+...+n= )
2 2 2 2
n(n + 1) (n − 1)n
Sleduva deka 9000 ⋅ + nS − K = 10nK , od kade delej}i so n
2 2
K(n − 1)
dobivame deka 4500 ⋅ (n + 1) + S − = 10K .
2
Poslednoto ravenstvo go mno`ime so 2 i dobivame 9000(n+1)+2Kn-Kn+K=20K,
od kade 9000(n+1)+Kn+K=20K (S=Kn), odnosno S=19K-9000·(n+1)...(4).
Od ravenstvoto (1) imame 10000+S-1000=10K, od ravenstvoto (2) imame
20000+S-K-2000=10K, od kade gi dobivame ravenstvata
S=10k-9000·1
S=11k-9000·2
...


S=19k-9000·10...(5). Od ravenstvata (4) i (5) zaklu~uvame deka n+1=10. Zna~i,
n = 9. Odgovorot e 9 deca.
41. Za 20 zada~i. Neka A1, A2, A3 e brojot na zada~i {to gi re{il samo eden
u~enik, A12, A13, A23 se broevite na zada~i {to gi re{ile samo dva u~enika i
A123 e brojot na zada~i {to gi re{ile site trojca
u~enici (crt. 64). Od uslovot na zada~ata gi imame
slednite tri ravenstva:
A1+A12+A13+A123=60 ...(1), A2+A12+A23+A123=60 ...(2);
A3+A13+A23+A123=60 ...(3) . Od toa {to vkupniot broj
zada~i bil 100, imame ravenstvo
A1+A2+A3+A12+A13+A23+A123=100 ...(4). Sobiraj}i gi
ravenstvata (1), (2) i (3) dobivame
A1+A2+A3+2A12+2A13+2A23+3A123=180 ...(5). Crte` 64
Od ravenstvata (4) i (5) dobivame deka A12+A13+A23+2A123=80 ...(6). Sega, od
ravenstvata (4) i (6) se dobiva razlikata (A1+A2+A3)-A123=20. Zna~i, brojot na
te{ki zada~i e za 20 pogolem od brojot na lesni zada~i.
42. Ribite gi ~uva germanecot..
43. Neka a1, a2 , a3 , ...,a101 se dadenite broevi. Izbirame kru`nica so perimeter
200, koja ja delime so 200 to~ki na 200 ednakvi laci. Po~nuvaj}i od edna
to~ka, vo nasoka na strelkite na ~asovnikot, gi bele`ime kraevite na lacite
so dol`ini a1, a2 , a3 , ..., a101 . Od toa {to postojat 100 para dijametralno
sprotivni to~ki, a obele`ani se 101 to~ka, postoi par obele`ani
dijametralno sprotivni to~ki. Takviot dijametar gi deli lacite so dol`ini
a1, a2 , a3 , ..., a101 na dve grupi (vo sekoja grupa vleguva po eden od broevite {to
stojat na dijametarot), vo koi zbirot na dol`inite na lacite e 100, {to
treba{e i da se doka`e.
44. Da go pretpostavime sprotivnoto, odnosno ne postojat tri otse~ki od koi
mo`e da se konstruira triagolnik. Da gi razgledame otse~kite m1=1, m2=1,
m1=m2§m3§m4§…§m10=50. Toga{ m3¥m1+m2=2. Analogno m4¥m2+m3¥1+2=3.
Ponatamu m5¥m3+m4¥2+3=5; m6¥m4+m5¥3+5=8; m7¥5+8=13; m8¥8+13;
m9¥13+21=34 i na krajot dobivame deka m10¥21+34=55 koe neravenstvo
protivre~i na uslovot m10=50.

109
Odgovori, upatstva i re[enija
45. Upatstvo: Doka`i go tvrdeweto so koristewe na metodot na kontradikcija.
46. Da pretpostavime sprotivno, odnosno ne postojat 4 ednakvi razliki. Jasno
e deka a20-a1§70-1=69. Od druga strana, poradi pretpostavkata, imame
a20-a1=(a20-a19)+(a19-a18)+...+(a2-a1)¥3·1+3·2+...+3·6+7=70.
Dobivame kontradikcija, od kade {to sleduva deka postojat 4 ednakvi
razliki.
47. Da pretpostavime deka pozitivnite broevi se a§b§c§d§e. Od desette
zbira najmaliot zbir e a+b, vtoriot najmal zbir e a+c. Analogno, najgolemiot
zbir e d+e, a vtoriot po golemina zbir e c+e. Sobiraj}i gi site sumi i delej}i
go dobienoto ravenstvo so 4 go dobivame zbirot na site pet broja. Neka
a+b+c+d+e=S...(1). Zamenuvaj}i ja vrednosta na a+b i d+e vo (1) ja nao|ame
vrednosta na brojot c. Zamenuvaj}i go c vo a+c ja dobivame vrednosta na a.
Zamenuvaj}i go e vo c+e ja dobivame vrednosta na e. Zamenuvaj}i go a vo a+b ja
dobivame vrednosta na b. Zamenuvaj}i go e vo d+e ja dobivame vrednosta na d.

Tema 2: Osnovni brojni mno`estva


48. a) 0 b) 5050. 49. 1680. Neka prvite dve cifri se a i b toga{ tretata cifra
e a+b. Od uslovot na zada~ata imame deka a ⋅ b ⋅ (a + b ) = 6(a + b + a + b ) ,
a ⋅ b(a + b) = 12(a + b) , ab = 12 . Od poslednoto ravenstvo sleduvat ~etiri
slu~ai i toa:
I. a = 3; b = 4; II. a = 4; b = 3; III. a = 2; b = 6; IV. a = 6; b = 2 . Baranite broevi se
2
347, 437, 268, 628. 50. 2019 . 51. . 52. 249 nuli.
1010 3
53. Ako broevite od vidot 21n+4 i 14n+3 imaat zaedni~ki delitel d toga{ d e
delitel i na nivnata razlika (21n+4)-(14n+3)=7n+1. Broevite 2ÿ(7n+1)=14n+2
i 14n+3 isto taka imaat zaedni~ki delitel d pa i nivnata razlika
(14n+3)-(14n+2)=1 se deli so d. No, toa e mo`no samo ako d=1. Sleduva deka
dadenata dropka e neskratliva.
54. 2013. Neka d>1 e zaedni~ki delitel na 2n–685 i 3n+643. Toga{
d|2·(3n+643)-3(2n-685)=3341 koj e prost broj. Zna~i d=3341. Ako dropkata e
skratliva toga{ 2n–685=3341k, kœZ. Za k=1 imame 2n=4026 odnosno n=2013.
19n + 7
55. nœ{3, -1, -3, -13}. Ako brojot e cel broj toga{ i brojot 7 ⋅ 19n + 7 - 9
7n + 11 7n + 11
-160
e cel, odnosno e cel broj. Zna~i, 7n+11|160 od kade {to so proverka se
7n + 11
utvrduva deka nœ{3, -1, -3, -13}. 56. Ostatokot e 153.
57.(2,670), (10,182), (32,60), (60,32), (182,10), (670,2). Ravenstvoto a+b+ab=2012 e
ekvivalentno so ravenstvoto (a+1)(b+1)=2013. Od 2013=3·11·61 gi imame
slednive mo`nosti:
a+1=1, b+1=2013 (nevozmo`no vo mno`estvoto na prirodni broevi);
a+1=3, b+1=671 implicira a=2, b=670;
a+1=11, b+1=183 implicira a=10, b=182;
a+1=33, b+1=61 implicira a=32, b=60;
a+1=61, b+1=33 implicira a=60, b=32;

110
Odgovori, upatstva i re[enija
a+1=183, b+1=11 implicira a=182, b=10;
a+1=671, b+1=3 implicira a=670, b=2.
58. x=2, y=5. Upatstvo: Dadenata ravenka se transformira vo vidot
(2x-5)(2y+5)=-15. 59. x=1, y=2, z=3. Upatstvo: x+y+z<3z-3 odnosno
3 3 2 3 2 3 2
x +y §9(z-1) -z <9z -z =z (9-z). 60. Trojkite (k,k,k); (l,l,2l); (m,2m,3m) k,l,mœN i
nivnite permutacii.
61. Neparnite broevi a, b i c pri delewe so 4 davat ostatok 1 ili 3. Spored
Principot na Dirihle , dva od tie broevi davaat ist ostatok pri delewe so 4.
Neka se toa broevite a i b. Toga{ 4 e delitel na ab-1.
2 3
62. Za x=1 ravenkata se sveduva na = , odnosno 2z=3y. Re{enie na ovaa
y z
ravenka e z=3k, y=2k, kade {to k e priroden broj. Zna~i edno mno`estvo
re{enija e {(1,2k,3k)|kœN}. Neka x>1. Po~etnata ravenka se transformira vo
2x
oblikot z(2x+y)=xy(z+3). Od z<z+3 sleduva 2x+y>xy odnosno > y , odnosno
x -1
2
2+ > y . Od poslednovo zaklu~uvame deka yœ{1,2,3}.
x -1
I. Neka y=1 toga{ ravenkata (1) se transformira vo oblikot z(x+1)=3x...(3).
Bidej}i x ne e delitel na x+1 (za x>1) sleduva deka x e delitel na z odnosno
3
z=kx. Zamenuvaj}i za z vo (3) imame deka x + 1 = od kade dobivame deka k=1;
k
z=2; x=2. Zna~i re{enie na ravenkata e trojkata (2,1,2);
II. Neka y=2 toga{ ravenkata (1) se zapi{uva kako z=3x. Zna~i re{enie na
ravenkata vo ovoj slu~aj e sekoja trojka (t,2,3t) kade t e priroden broj;
III. Neka y=3 toga{ (1) se transformira vo vidot 3(z-3x)=xz...(4). Ako 3Ix
9k
toga{ x=3k pa ravenkata (4) se zapi{uva vo oblikot z = koja nema
1− k
re{enie. Ako 3Iz toga{ z=3k pa ravenkata (4) se transformira vo oblikot
9
x =3− ~iij re{enija se k=6; x=2; z=18.
k+3
Zna~i (x,y,z) œ{(2,1,2),(2,3,18),(1,2t,3t),(t,2,3t)} za sekoj tœN.
63. Neka N e baraniot broj , d brojot na negovite cifri i S(N) e zbirot
na cifrite na N. Od S(N)≤ 9d i N ≥ 10 d −1 sleduva deka
9 3 d3 ≥ [S(N)] = N ≥ 10d−1 . So principot na matemati~ka indukcija lesno se
3

poka`uva deka za d¥7 va`i 93 ⋅ d3 < 10 d-1 . Za d=6 va`i S(N) ≤ 9 ⋅ 6 = 54 i


[S(N)]3 ≤ 54 3 = 157464 odnosno N§157464. Zabele`uvame deka
S(N)<S(199999)=46 {to povlekuva [S(N) ] ≤ 45 3 = 91125
3
odnosno N ≤ 91125 .
Zna~i N ima najmnogu 5 cifri. Da pretpostavime deka 8000 < N < 125000 . Toga{
3
8000 < [S(N)] < 125000 odnosno 20<S(N)<50.

Od priznakot za delivost so 9 imame deka va`i N = [S(N) ]3 ≡ [S(N) ](mod9)


odnosno [S(N)]3 − S(N) ≡ 0(mod9), pa [S(N) − 1] ⋅ S(N) ⋅ [S(N) + 1] e deliv so 9. Taka
gi razgleduvame samo broevite

111
Odgovori, upatstva i re[enija
3
26 = 17576 → S(17576) = 26 36 3 = 46656 → S(46656) = 27
3
27 = 19683 → S(19683) = 27 37 3 = 50653 → S(50653) = 19
3
28 = 21952 → S(21952) = 19 44 3 = 85184 → S(85184) = 26
3 45 3 = 91125 → S(91125) = 18
35 = 42875 → S(42875) = 26
46 3 = 97336 → S(97336) = 28
Od prethodnoto sleduva deka edinstveno re{enie e brojot 19683.
64. 7. Od toa {to sekoj priroden broj e kongruenten so zbirot na negovite
4444 4444
cifri po modul 9, imame deka CªBªA (mod9). Pa od A=4444 ª7 (mod9) i
3 3·1481+1
7 ª1(mod9) imame deka A=7 ª7(mod9). Od toa {to A ima pomalku od 20000
cifri (sekoja od niv mo`e da ima najgolema vrednost 9), B }e ima vrednost
pomala od 9·20000=180000. Taka C }e ima vrednost pomala od 1+5·9=46.
Broevi pomali od 46 a kongruentni so 7 po modul 9 se 7,16,25,34 i 43. Da
zabele`ime deka zbirot na cifrite na site ovie broevi e 7. Zna~i, zbirot na
cifrite na brojot C e 7.
65. Neka baraniot broj vo dekaden zapis e a 1a 2 a 3 ....a n . Toga{
a i + a i+ 2
a i+1 < povlekuva a i − a i+1 > a i+1 − a i+2 . Cifrite na baraniot broj
2
mora prvo da se vo opa|a~ki redosled, a potoa vo raste~ki. Za da go
opredelime najgolemiot takov broj, po~nuvame so cifrata 9. Neka
9>a2>a3>...>aj<=aj+1<...<an. Sleduva deka j∫5, bidej}i vo sprotivno bi imale
9-a5=(9-a2)+(a2-a3)+(a3-a4)+(a4-a5)¥10, {to ne e mo`no. Zna~i j§4. Ova
povlekuva deka n=8 (Zo{to?). Ne e te{ko da se proveri deka najgolemiot broj
{to mo`e da se dobie pri ovie uslovi e 96433469. 66. 27 i 37. 67. Ravenkata
mo`e da se transformira vo oblikot (x+y+z)(x+y-z)(x-y+z)(y+z-x)=576 odnosno
6 2
(x+y+z)(x+y-z)(x-y+z)(y+z-x)=2 3 . Poradi simetri~nosta na ravenkata, bez
gubewe od op{tosta mo`eme da pretpostavime deka x¥y¥z.
Neka x+y+z=k1; x+y-z=k2; x-y+z=k3; y+z-x=k4. Od x¥y¥z sleduva deka
k1¥k2¥k3¥k4. Od k1=k2+2z=k3+2y=k4+2x sleduva deka k1, k2, k3 i k4 se so ista
parnost, dodeka od k1k2k3k4=576 sleduva deka site se parni. Isto taka, site
broevi k1, k2, k3 i k4 mora da se pozitivni (Slu~ajot k1¥k2>0>k3¥k4 povlekuva
deka z<0, {to e kontradikcija). Zna~i imame deka k1k2k3k4=576, k1¥k2¥k3¥k4>0 i
site se parni broevi. Od poslednoto sleduva deka (k1,k2,k3,k4)œ{(6,6,4,4),
(8,6,6,2), (12,12,2,2), (12,6,4,2), (18,8,2,2), (18,4,4,2), (24,6,2,2), (72,2,2,2)}. No, se
x + y + z = k 1

x + y - z = k 2
proveruva deka sistemot  ima re{enie samo za k1=12, k2=6,
x - y + z = k 3
- x + y + z = k 4

k2 + k3 k + k4 k + k4
k3=4 i k4=2. Ottuka x = = 5, y = 2 =4 i z= 3 = 3 . Zna~i,
2 2 2
re{enie na ravenkata e trojkata (5,4,3) i sekoja nejzina permutacija.
68. a=2, b=7 i a=7, b=2.
70. 8, 9, 10. Neka
(a + b)(b + c )(c + a) e cel broj. Od abc|ac2+a2c+bc2+b2c+ab2+a2b
abc
i NZD(a,b)=NZD(a,c)=NZD(b,c)=1, sleduva deka a|b+c, b|a+c i c|a+b, odnosno
postojat prirodni broevi k1, k2 i k3 takvi {to b+c=k1a, a+c=k2b i a+b=k3c.

112
Odgovori, upatstva i re[enija

- k1a + b + c = 0

Sistemot a − k 2b + c = 0 ima netrivijalno re{enie po a, b i c ako i samo ako
a + b − k c = 0
 3

− k1 1 1
1 − k2 1 = 0 odnosno k1k2k3=2+k1+k2+k3. Da ja re{ime vo mno`estvoto na
1 1 − k3
prirodni broevi dobienata ravenka. Poradi simetri~nosta mo`eme da
pretpostavime deka k1¥k2¥k3>0.
i) Ako k3¥3 toga{ k1k2k3¥9k1¥3k1+2>2+k1+k2+k3 od kade {to sleduva deka
ravenkata nema re{enie.
ii) Ako k3=1 toga{ ravenkata se reducira do 3+k1+k2=k1k2 odnosno
(k1-1)(k2-1)=4. Re{enie na ova ravenka vo N se k1=5,k2=2 i k1=3,k2=3
(pri uslov k1¥k2¥k3>0).
iii) Ako k3=2 toga{ ravenkata se reducira do 4+k1+k2=2k1k2 odnosno
(2k1-1)(2k2-1)=9. Re{enie na ova ravenka vo N, pri uslov k1¥k2¥k3>0, e
k1=2, k2=2.
Zna~i, re{enija na ravenkata k1k2k3=2+k1+k2+k3 se trojkite (5,2,1), (3,3,1),
(2,2,2) i site nivni permutacii. Poradi simetri~nosta na po~etniot izraz,
dovolno e da ja razgledame negovata vrednost za (k1,k2,k3)œ[(5,2,1), (3,3,1),
(2,2,2)].
1) Za k1=5, k2=2, k3=1 re{enija na sistemot se trojkite (a,2a,3a), aœN.
Me|utoa a, 2a i 3a se me|usebno zaemno prosti samo za a=1 odnosno b=2
i c=3. Vo toj slu~aj po~etniot izraz e cel broj i iznesuva 10.
2) Za k1=3, k2=3, k3=1 re{enija na sistemot se trojkite (a,a,2a), aœN.
Me|utoa a, a i 2a se me|usebno zaemno prosti samo za a=1 i toga{
vrednosta na izrazot e 9.
3) Za k1=k2=k3=2 se dobiva a=b=c. Vo toj slu~aj a, b i c se zaemno prosti
ako a=b=c=1. Vo toj slu~aj vrednosta na izrazot e 8.
2 3 b b+1
71. Da gi razgledame broevite a,a ,a ,...,a ,a . Nivniot broj e b+1 i od
principot na Dirihle sleduva deka najmalku dva od niv pri delewe so b
n m
davat isti ostatoci. Neka se toa broevite am i an, m<n. Toga{ b|a -a i
n m m n-m m
a -a =a (a -1). Od NZD(a,b)=1 sleduva deka NZD(a ,b)=1, odnosno dobivame
n m
deka b|a - -1. Zna~i, k=n-m e brojot koj {to treba{e da se odredi.
72. (41,43,45).
1 1 1 1 1 1
73. Jasno e deka + + < 1 za a¥4, b¥4, c¥4, odnosno izrazot + + ne bi
a b c a b c
bil priroden broj. Ostanuva da gi razgledame slu~aite za aœ{1,2,3}. Neka a=2.
1 1 1 1
Toga{ + = d + ≥ …(1), kade {to d e 0 ili nekoj priroden broj. Ako b>4 i
b c 2 2
1 1 1
c>4 toga{ + < , {to e vo kontradikcija so (1). Taka dobivame deka
b c 2
2=a§a§b§4. Ostanuva da gi razgledame slu~aite za bœ{2,3,4}. Za b=2 zada~ata
nema re{enie. Za b=3, c=6. Za b=4, c=4. Analogno ako a=1 toga{ b=1, c=1 ili
b=2, c=2. Ako a=3 toga{ b=3, c=3. Kone~no, re{enija na zada~ata se trojkite
(1,1,1), (1,2,2), (2,3,6), (2,4,4), (3,3,3).

113
Odgovori, upatstva i re[enija
74. Neka S(N) e zbir od cifrite na brojot N. Za dva broja A i B va`i
S(A+B)§S(A)+S(B), pri {to ravenstvoto va`i ako zbirot na cifrite od isti
red (razmer) e pomal od 10 .Od S(10N)=S(N)=100 i S(5N)=50 sleduva
S(5N)+S(5N)=S(10N) {to povlekuva deka zbirot na cifrite od ist red e pomal
od 10. Bidej}i brojot 5N zavr{uva na 5 ili 0 sleduva deka N mo`e da bide
neparen ili paren. Prviot slu~aj otpa|a bidej}i predizvikuva prenos vo
naredniot razmer. Zna~i N e paren broj.
75. NZD na dadenite broevi e delitel na nivnata razlika
2022 2020 2020
2 -2 =2 ·3. Bidej}i dadenite broevi se neparni sleduva deka NZD=1
2022 2020 2022 2020
ili NZD=3. Bidej}i 3|2 -1 i 3|2 -1, sleduva deka NZD(2 -1; 2 -1)=3.
76. Dadenoto ravenstvo e ekvivalentno so raenstvoto c·(a+b)=a·b. Neka
NZD(a,b)=e odnosno a=ep i b=eq kade {to NZD(p,q)=1. Dobivame deka
c(p+q)=epq. Od NZD(p,q)=1 sleduva deka NZD(p+q, p·q)=1 {to povlekuva deka
(p+q) e delitel na e. Zna~i e=t(p+q), za nekoj priroden broj t. Dobivame deka
a=tp(p+q), b=tq(p+q), c=tpq. Od NZD(pq, p(p+q), q (p+q))=1 следува дека
2 2
d=NZD(a,b,c)=t. Kone~no abcd=tp(p+q)·tq(p+q)·tpq·t=[t (p·q)·(p+q)] .
2
77. 2020 +2=2020(2018+2)+2=2018(2020+2)+6. Od Evklidoviot algoritam
2
imame NZD(2020+2,2020 +2)=NZD(2022,6)=6. Od druga strana, sekoj broj od
2 3
nizata 2020+2,2020 +2,2020 +2,... e deliv so 2. Od toa {to
k k
2020=2019+1=673·3+1 imame deka 2020 =3·a +1 za sekoj priroden broj k. Taka
k
sleduva deka 2020 +2 e deliv so 3 za sekoj kœN. Bidej}i 2 i 3 se vzaemno
prosti, sleduva deka sekoj broj od nizata e deliv so 6.
78. Neka НЗД(m,n)=d. Toga{ m=kd, n=ld i НЗС(m,n)=kld kade {to NZD(k,l)=1. Od
uslovot na zada~ata sleduva deka kld+d= kd+ld odnosno (k-1)(l-1)=0.Od
poslednoto ravenstvo sleduva deka k=1 i m|n ili l=1 i n|m. 79. (13,6), (-13,-6),
(67,66), (-67-66), (-13,6), (13,-6), (-67,66), i (67,-66). 80. x=1, y=0. 81. 2009,
2010, 2011. 82. Upatstvo: Barem eden od broeviete x,y ili z e 0.
85. 13. Jasno e deka a>2014. Neka a=2014+k za nekoj pozitiven cel broj k.
1 k 2014 2
Toga{ = odnosno b = 2014 + , od kade imame deka
b 2014 ⋅ (2014 + k) k
2 2014 2
k|2014 . Od a<b sleduva deka k < , odnosno k<2014. Zna~i k e delitel na
k
2
2014 , pomal od 2014. Takvi broevi se vkupno 13, {to povlekuva deka postojat
vkupno 13 parovi koi go zadovoluvaat uslovot na zada~ata.
2 2
86. n=2 i n=9.Upatstvo:n <n(n+16)<(n+8) .
87. Da go pretpostavime sprotivnoto, odnosno deka site tri broja se
2 2 2
negativni. Toga{ (a+b+c) -9ab<0, (a+b+c) -9bc<0, (a+b+c) -9ca<0. So sobirawe
na ovie neravenstva i so sreduvawe se dobiva neravenstvoto
2 2 2 1 2 2 2
a +b +c -ab-ac-bc<0 odnosno [(a-b) +(b-c) +(c-a) ]<0. Me|utoa zbirot na
2
kvadratite na tri realni broja ne mo`e da bide negativen, od kade {to
zaklu~uvame deka barem eden od dadenite tri broja e nenegativen.
88. x=1, y= ± 1 i x=3, y= ± 3 .
s
89. Neka x = pri {to NZD(s,t)=1. Zamenuvaj}i za x vo ravenstvoto dobivame
t
sn s n−1 s n −2 s
n
= −a 1 ⋅ n −1
− a2 ⋅ n −2
− ... − a n −1 − a n . Mno`ej}i go poslednoto ravenstvo
t t t t

114
Odgovori, upatstva i re[enija
n n n-1 n-2 2 n-1 n
so t imame s =-a1·s ·t-a2·s· ·t -…-an-1·s·t -an·t …(1). Dobivame deka desnata
n
strana vo (1) e deliva so t, od kade sleduva deka s e deliv so t. No, od druga
strana s i t se zaemno prosti, pa sleduva deka t=1 odnosno x=s.
1+ 5
90. a) A = b) B = 2 .
2
2 2 1
91. a +b = 5 . Dadenite ravenstva gi zapi{uvame vo oblikot a = b + i
1
a+
a
1
b = a- . Zamenuvaj}i ja vtorata vo prvata ravenka ja dobivame
1
b+
b
4 2 2 5 −1
bikvadratnata ravenka b +b -1=0, od kade se dobiva deka b = .
2
2 5 +1 2 2
Zamenuvaj}i vo prvata ravenka dobivame deka a = i kone~no a +b = 5 .
2
2 2 2
1
92. Po~etnata ravenka (2x+1) +y +(y-2x) = e ekvivalentna so ravenkata
3
2 2 1
(1+3x) +3(x-y) =0 koja e zadovolena za x=y= − .
3
2 2
93. Da ozna~ime a=x +3x-4 i b=2x -5x+3. Toga{ po~etnata ravenka se
3 3 3
transformira vo a +b =(a+b) , odnosno vo 3ab(a+b)=0. Re{enija na poslednata
2 2 2
ravenka se a=0 (x +3x-4=0), b=0 (2x -5x+3=0) i a+b=0 (3x -2x-1=0). Gi
razgleduvame slednite tri ravenki:
2
I. x +3x-4=0 ~ii re{enija se x1=1, x2=-4;
2 3
II. 2x -5x+3=0 ~ii re{enija se x3= , x4=x1=1;
2
2 1
III. 3x -2x-1=0 ~ii re{enija se x5=x1=x4=1, x6= − .
3
3 1
Zna~i re{enija na po~etnata ravenka se: x1=1, x2=-4, x3= i x4= − .
2 3
2 2
94. Ako prvata ravenka ja pomno`ime so q , vtorata so p , a tretata so -2pq i
gi sobereme trite ravenki dobivame:
2 2 2
(a1q - b1p) + (a2q – b2p) + .... (a2019q– b2019p) = 0 odnosno
a1q–b1p=a2q–b2p=...=a2019p–b2019q=0 od, kade dobivame deka
a1 a2 a p
= = ... = 2019 = .
b1 b2 b2019 q
95. Broevite a, b, c, d go delat intervalot [0,1] na najmnogu pet intervali me|u
koi postoi barem eden interval so dol`ina ne pomala od 0,2. Da go ozna~ime
toj interval so I. Neka x e sredinata na toj interval. Od vaka izlo`enata
konstrukcija, jasno e deka broevite |x-a|, |x-b|, |x-c| i |x-d| se pogolemi ili
ednakvi na 0,1 i u{te pove}e, najmalku dva od niv se ednakvi na 0,1. Ottuka
1 1 1 1
sleduva deka + + + < 40 .
x-a x-b x-c x-d

115
Odgovori, upatstva i re[enija

Tema 3: Algebarski racionalni izrazi


2 2 2 2 2 2 2
96. (x +1)(2x +x+2). 97. (a+1)(a +a+1)(a -a+1). 98.(a+b+c)(a +b +c -ab-ac-bc).
2 44 33 22 11
99. (x +8x+10)(x+2)(x+6). 100. (a+b)(b+c)(c+a). 103. Neka A=x +x +x +x +1 i
5
4 3 2 x −1 5
B= x +x +x +x+1, toga{ B = , odnosno x -1=(x-1)·B(x). Va`i
x −1
4 40 3 30 2 20 10
A(x)-B(x)=x (x -1)+x (x -1)+x (x -1)+x(x -1). Pritoa:
40 5 8 5
X -1=(x ) -1=(x -1)·P(x)=(x-1)·B(x)·P(x)
30 5 6 5
X -1=(x ) -1=(x -1)·Q(x)=(x-1)·B(x)·Q(x)
20 5 4 5
X -1=(x ) -1=(x -1)·R(x)=(x-1)·B(x)·R(x)
10 5 2 5
X -1=(x ) -1=(x -1)·S(x)=(x-1)·B(x)·S(x)
4 3 2
A(x)-B(x)=x ·(x-1)·B(x)·P(x)+x ·(x-1)·B(x)·Q(x)+x ·(x-1)·B(x)·R(x)+x·(x-1)·B(x)·S(x);
3 2
A(x)=B(x)+x·(x-1)·B(x)·[x P(x)+x Q(x)+xR(x)+S(x)]
3 2
A(x)=B(x) ·{1+ x·(x-1)·[x P(x)+x Q(x)+xR(x)+S(x)]}
Sleduva deka polinomot A(x) e deliv so polinomot B(x). 104. 2x+1.
1
105. P(8)= . Go formirame polinomot F(х)=хÿР(х)-1 koj e polinom so stepen 7.
4
1
Toga{ za n =1, 2, ..., 7 imame F(n)=nP(n)-1=nÿ -1=0, od kade sleduva deka
n
n=1, 2, …, 7 se nuli na polinomot F(x). Toga{
F(х)=ах(х-1)(х-2)(х-3)(х-4)(х-5)(х-6)(х-7) za nekoja konstanta а. Od edna strana
F(0)=0·Р(0)-1=-1, a od druga strana F(0)=а(-1)(-2)(-3)(-4)(-5)(-6)(-7). Od tuka
1 1
а= . Zna~i, F(х)= (х-1)(х-2)(х-3)(х-4)(х-5)(х-6)(х-7), pa F(8)=1, a od ovde pak
7! 7!
F(8) + 1 1
P(8) = = .
8 4
2 n-1 n
106. a2=-588. Neka P(x)=a0+a1x+a2x +…+an-1x +anx …(1). Od P(0)=1 dobivame
deka a0=1. Jasno e deka an=(-1)·2·(-3)·…·14·(-15)=15!. Nuli na dadeniot
polinom se x1=1, x2= − 1 , x3= 1 , …, x14= − 1 ,x15= 1 …(2). Od Vietovite
2 3 14 15
formuli imame x2x3x4·…·x14x15+x1x3x4·…·x14x15+…+x1x2x3·…·x13x15+x1x2x3·…·x13x15
a
= 1
…(3). Od (2) i (3), dobivame
a
n
1 1 1 1 a
− + ... − + = 1
2 ⋅ 3 ⋅ 4 ⋅ ... ⋅ 14 ⋅ 15 1⋅ 3 ⋅ 4 ⋅ ... ⋅ 14 ⋅ 15 1⋅ 2 ⋅ 3 ⋅ ... ⋅ 13 ⋅ 15 1⋅ 2 ⋅ 3 ⋅ ... ⋅ 13 ⋅ 14 15! .
1 − 2 + 3 − 4 + ... − 14 + 15 a1
Ottuka = , pa a1=-8. Zamenuva}i gi vrednostite na a0 i
15! 15!
2
a1 vo polinomot (1), dobivame P(x)=1-8x+a2x +Q(x)…(4) i
2
P(-x)=1+8x+a2x +R(x) …(5), kade Q(x) i R(x) se polinomi od trinaestti stepen.
2 2
Mno`ej}i gi ravenkite (4) i (5) imame P(x)·P(-x)=1-64x +2a2x +M(x) …(6), kade

116
Odgovori, upatstva i re[enija
M(x) e isto taka polinom od trinaestti stepen. Od druga strana
P(-x)=(1+x)(1-2x)·…·(1-14x)(1+15x), pa toga{
2 2 2 2 2 2 2 2 2 2 2
P(x)·P(-x)=(1-x )(1-2 x )(1-3 x )·…·(1-15 x )=1-(1 +2 +…+15 )x +N(x) …(7), kade
2
N(x) e polinom od trinaestti stepen. Izedna~uvaj}i gi koeficientite pred x
2 2 2
dobivame 2a2-64=-(1 +2 +…+15 ). Od poslednoto ravenstvo imame deka
2 2
a2=-588. 107. x +y =19. 109. 110. 110. 10.
2 2 2 2 2 2 2 2
111. Vrednosta na izrazot e 4. Upatstvo: a =x +y z +2; b =y +z x +2;
2 2 2 2 2 2 2 2 2 2 2 2 2
c =z +x y +2 i abc=x y +y z +x z +x +y +z +2. 112. 198. 113. Upatstvo:
3 3 3
ravenstvoto se zapi{uva kako (x -yz) +(y -zx) +(z -xy) =0. 114. a + b + c = 7
2 2 2 2 2 2

b3 c 3 a3
.
14
115. − . 116. xyz=4. Upatstvo: Iskoristi go ravenstvoto
25
1
3 2 2 2 3 3
(x+y+z) -3(x+y+z)·(x +y +z )+2·(x +y +z )=6xyz. 117.
3
.
2
3 1
118. a) 3; b) . Od a+b+c=1 i ab+bc+ca= sleduva deka
4 3
2 2 2 2 2 2 2
(a-b) +(b-c) +(c-a) =2(a +b +c -ab-bc-ca)=2[(a+b+c) -3(ab+bc+ca)]=0
1
odnosno a=b=c= .
3

119. 2. Upatstvo: ozna~ete


x+y x−y
+ = k i doka`i deka
k 2 2⋅ x + y
+ =
4 4
.
( )
4 4
x−y x+y 2 k x −y
2 2
120. Najmalata vrednost na polinomot e 9. P(x)=(x -5x+4)(x -5x+6)+10
2 2 2 2 2 2
=(x -5x) +10(x -5x)+34, odnosno P(x)=(x -5x) +10(x -5x)+25+9. Zna~i
2 2
P(x)=(x -5x+5) +9. Najmalata vrednost na polinomot }e se postigne vo
2
slu~ajot koga x -5x+5=0, odnosno najmalata vrednost na P(x) e 9. 121. -1.
2 n
Upatstvo: Zbirot na koeficientite na eden polinom Q(x) =a0+a1x+a2x +…+anx
se dobiva koga vo samiot polinom zamenime x=1. 122. (1-x1)(1-x2)...(1-xn)=n+1.
123. Neka P(a) = P(b) = P(c) = P(d) = 4 , toga{ polinomot f(x) = P(x) - 4 gi ima a, b, c,
d, kako nuli. Zna~i, f(x)=P(x)-4=(x-a)(x-b)(x-c)(x-d)g(x), kade g(x) e polinom so
celi koeficenti. Da go pretpostavime sprotivnoto tvrdewe, odnosno neka
postoi cel broj m takov {to P(m) = 7 . Toga{ f (m) = P(m) - 4 = 7 - 4 = 3
= (m - a)(m - b)(m - c )(m - d)g(m) . Zna~i celite broevi m-a, m-b, m-c, m-d, g(m) se
deliteli na brojot 3. Od ovde sleduva deka tie mora da bidat ednakvi so ± 1
ili ± 3 , pri {to samo eden od niv mo`e da bide ± 3. Toga{, tri od broevite
m-a, m-b, m-c, m-d mora da bidat ± 1 . Zna~i, barem dva od ~etirite broevi
treba da bidat ednakvi, {to e sprotivno so uslovot na zada~ata.
124. R(x)=-2550x+2500. Neka S(x) e koli~nikot, a R(x) ostatokot {to se dobiva
koga P(x) se deli so Q(x). Zna~i, P(x)=Q(x)·S(x)+R(x) kade R(x)=a·x+b. Imame:
P(1)=Q(1)·S(1)+R(1)=0·S(1)+R(1)=R(1);
P(-1)=Q(-1)·S(-1)+R(-1)=0·S(-1)+R(-1)=R(-1);
67−1
8 647 −1
4
8 6474−1 48 4
1− 2 ⋅ 1+ 3 ⋅1− 4 ⋅1+ ... + 99 ⋅ 1− 100 ⋅ 1 = R(1) = a + b …(1);

117
Odgovori, upatstva i re[enija
1+2+3+…+99+100=R(-1)=-a+b…(2). Od ravenstvata (1) i (2) se dobiva sistemot
a + b = −50 , ~ie re{enie e a=-2550 i b=2500.

− a + b = 5050
n n-1
125. Neka P(x)=anx +an-1x +…+a1x+a0, kade aiœZ za i=0, 1, 2, … ,n i neka
x1, x2œZ, x1≠x2 se takvi {to rastojanieto me|u to~kite (x1,P(x1)) i (x2,P(x2)) e
k k
x1 − x 2 k-1 k-2 k-2 k-1
cel broj. Da zabele`ime deka =x1 +x1 ·x2+…+x1·x2 +x2 œZ za x1,
x1 − x 2
k k
x1 − x 2
x2œZ, x1≠x2 . Neka bk=ak· œZ, za k=1, 2, …, n. Sleduva deka
x1 − x 2
n n n -1 n -1 2 2
P(x1 )−P(x2 ) x1 − x 2 x1 − x2 x1 − x 2
=a ⋅ +a ⋅ + ... + a ⋅ +a
x1 − x 2 n x1 − x 2 n -1 x1 − x 2 2 x1 − x 2 1

=bn+bn-1+…+b2+b1=mœZ. Sega d[(x1,P(x1)), (x2,P(x2))]


2
 P(x1 ) − P(x2 ) 
= (x1 - x 2 ) 2 + (P(x1 ) + P(x2 ))2 = x1 − x 2 ⋅ 1 +   =

 x1 − x 2 
2 2 2
= x1 − x 2 ⋅ 1 + m . Od uslovot na zada~ata x1 − x 2 ⋅ 1 + m œZ, zna~i 1+m e
2 2 2 2
poln kvadrat. No, ako m≠0 toga{ m < 1+ m < (1 + m ) , pa 1+m ne e poln
kvadrat, od kade zaklu~uvame deka mora m=0, odnosno P(x1)=P(x2). Od
poslednoto ravenstvo sleduva deka otse~kata {to gi svrzuva to~kite (x1,P(x1))
i (x2,P(x2)) e paralelna so x–oskata.
126. P(x)=c(x+1/2)(x+1/2 )…(x+1/2 ), c-proizvolen realen broj.
2 2018

ay − bx cx − az bz − cy
127. k=18. 128. 9 . 129. 2 i -1. 131. Neka = = =t.
c b a
y x c x z b z y a
Sleduva deka - = t; − = t i − = t.
b a ab a c ac c b bc
2 2 2
Sobiraj}i gi poslednite tri ravenstva dobivame deka c + b + a t=0. Zna~i
abc
x y z
ay-bx=cx-az=bz-cy=0, od kade pak = = .
a b c
2 2 2 3 3 3
133. Upatstvo: (x +y +z -xy-xz-yz)(x+y+z)=x +y +z -3xyz.
(a - b) ⋅ a 2 + ab + b 2 + p = 0 ( )

135. Ako gi odzememe ravenkite dobivame (b - c ) ⋅ b 2 + bc + c 2 + p = 0 , od kade ( )
 2 2
(c - a) ⋅ c + ca + a + p = 0 ( )
2 2 2 2 2 2
sleduva a +ab+b =b +bc+c =c +ca+a (a≠b≠c≠a). Od prvoto ravenstvo se dobiva
(a-c)(a+c)+b(a-c)=0 odnosno (a-c)(a+b+c)=0, pa bidej}i c≠a sleduva deka
a+b+c=0. 137. Od zada~a 102 a) imame deka

118
Odgovori, upatstva i re[enija

3 3 3 1 2 2 2
a +b +c -3abc= (a+b+c)·[(a-b) +(b-c) +(c-a) ]...(1). Od dadenata pro{irena
2
a + bx a + bx + b + cx + c + ax (a + b + c)(1 + x)
proporcija dobivame deka = = ,
b + cy b + cy + c + ay + a + by (a + b + c) + (1 + y)
 a + bx 1 + x 
odnosno (a+b+c)  −  =0. Ottuka mo`ni se slednive dva slu~ai:
 b + cy 1 + y 
3 3 3
I. a+b+c=0, pa od (1) dobivame a +b +c =3abc.
a + bx 1 + x
II. = , od kade sleduva deka a=b=c pri {to povtorno va`i
b + cy 1 + y
3 3 3
ravenstvoto a +b +c =3abc. 146. Upatstvo: ravenstvoto se transformira vo
3 3 3 3 3 3
ravenstvo x z-x y+z y-z x+y x-y z , odnosno (x-y)(x-z)(z-x)(x+y+z)=0.
147. Upatstvo: Dokaæi deka (xn + y n ) ⋅ (x n + zn ) ⋅ (y n + zn ) = 0.
148. Prvata dropka od ravenstvoto ja pro{iruvame so z, a vtorata so xz. Na toj
na~in dobivame ist imenitel vo site tri dropki, odnosno ravenstvoto dobiva
z xz 1
oblik + + = 1, {to treba{e i da se doka`e.
1 + z + xz 1 + z + xz 1 + z + zx
149. Upastvo: Dadenoto ravenstvo se transformira vo vidot
2 2 2 2 2 2
(a -b ) (c -d )+2(ab-cd) =0.
150. Dadenoto ravenstvo se transformira vo vidot 1 [(a-b) +(b-c) +(c-a) ]
2 2 2

2
2 2 2
=-3(x +y +z ). Bidej}i levata strana e pozitivna, a desnata strana od
2 2 2 2 2 2
ravenstvoto negativna sleduva deka (a-b) +(b-c) +(c-a) =0 i x +y +z =0.
Ottuka a=b=c i x=y=z=0.
ax 3 + by 3 + cz3 3 ax 3 + ax 3 + ax 3 3  1 1 1
152. 3 = = a ⋅ x = 3 a ⋅ x ⋅  + + 
3 3 x y z
 x x  3
b 3
c  3
= 3 a ⋅ 1 + +  = 3 a ⋅ 1 + + = a + 3 b + 3 c.
 y z  3
a 3
a 

1 1 1 1 1 1

153. (ma) +(nb) +(pc) =   ÿa+   ÿb+   ÿc=   ÿa+   ÿb+   ÿc


1/2 m 2
1/2 n 2
1/2 p 2 m 2 m 2 m 2
a b c a a a
1 1
 m  2
1
=   ÿ(a+b+c)=   ⋅ (a + b + c) ⋅ (a + b + c) 2
m 2
a  a  
1
1
=  m + m ⋅ b + m ⋅ c  ⋅ (a + b + c) 2 = (m + n + p ) 2 ⋅ (a + b + c ) 2 .
2 1 1

 a a
154. Voveduvame smeni x-y=a, y-z=b, z-x=c, pri {to va`i a+b+c=0…(1). So
ovie smeni, po~etnoto ravenstvo mo`e da se zapi{e vo oblikot:
2 2 2 2 2 2 2 2 2
a +b +c =(a-b) +(b-c) +(c-a) =2(a +b +c -ab-ac-bc), odnosno
2 2 2 2 2 2
a +b +c =2(a +b +c )-2(ab+ac+bc)...(2). Bidej}i od ravenstvoto (1) va`i
2 2 2 2
a +b +c =(a+b+c) -2(ab+ac+bc)=-2(ab+ac+bc), zamenuvame vo (2) i dobivame
2 2 2
deka a +b +c =0…(3). Od (1) i (3) sleduva deka a=0, b=0, c=0. Vra}ajki se vo

119
Odgovori, upatstva i re[enija
zamenata dobivame deka x=y=z. 155. Upatstvo: Doka`ete go ravenstvoto
 a b c   1 1 1  a b c
 + + ⋅ + + = 2
+ 2
+ 2
. 156 a+b+c.
 b − c c − a a − b   b − c c − a a − b  (b − c ) (c − a) (a − b)
2 2
158. Kvadriraj}i go neravenstvo |a-b|¥|c|, dobivame (a-b) -c ¥0. So
razlo`uvawe dobivame (a-b-c)·(a-b+c)¥0 odnosno (a-b-c)(b-c-a)§0...(1).
Analogno gi dobivame i neravenstvata (b-c-a)(c-a-b)§0...(2) i
(c-a-b) (a-b-c)§0...(3). Mno`ej}i gi neravenstvata (1), (2) i (3) dobivame
2 2 2
(a-b-c) (b-c-a) (c-a-b) §0, a bidej}i levata strana vo poslednoto
neravenstvo e nenegativna, sleduva deka a-b-c=0 ili b-c-a=0 ili c-a-b=0.
Kone~no, dobivame a=b+c ili b=c+a ili c=a+b. 159. Upatstvo: Primeni go
principot na matemati~ka indukcija. 160. Dadenoto ravenstvo mo`e da se
2 2 2 2
zapi{e vo oblikot a +b +c =2ab+2ac+bc, odnosno (a+b+c) =4(ab+ac+bc). Ottuka
sleduva deka ab+ac+bc mo`e da se zapi{e kako poln kvadrat. Re{avaj}i ja
ravenkata kako kvadratna ravenka po nepoznata s, dobivame deka
c = (a + b) ± 2 ⋅ ab . Bidej}i c e priroden broj sleduva deka ab e poln kvadrat.
Analogno se poka`uva deka bc i ca isto taka mo`e da se zapi{at kako polni
kvadrati.
161. Neka z=x+y+2m+1 kade m = xy + x + y .
2 2
Toga{ xy+z=xy+x+y+2m+1=m +2m+1=(m+1) ;
2 2 2 2
xz+y=x(x+y+2m+1)+y=x +xy+2mx+x+y=m +2mx+x =(x+m) ;
2 2 2 2
yz+x=y(x+y+2m+1)+x=y +xy+2my+x+y=m +2my+y =(y+m) .
163. a bc + b ac + c ab ≤ 12 . 164. 8.
166. (x,y)œ{(0,0),(0,1),(1,0),(1,1)}
2022 2022 2021 2021 2020 2020
(x +y )-2(x +y )+(x +y =0,
2022 2021 2020 2022 2021 2020
(x -2x +x )+(y -2y +y )=0,
2020 2 2020 2
x (x -2x+1)+y (y -2y+1)=0,
2020 2 2020 2
x (x-1) +y (y-1) =0,
x(x-1)=0 ⁄ y(y-1)=0,
(x=0 ¤ x=1) ⁄ (y=0 ¤ y=1).

Tema 4: Algebarski neravenstva


1 2 2018
167. Neka S = + + ... + . Toga{ S = 1 + 1 + ... + 1
2! 3! 2019! 2! 3! 2019!
2 3 2019 1 1 1 1
= + + ....... + = 1 + + + .... + odnosno S + = 1.
2! 3! 2019! 2! 3! 2018! 2019!
1
Kone~no S = 1 − .
2019!

120
Odgovori, upatstva i re[enija

1 3 5 2 4 99 98 n n −1
168. Neka x= ⋅ ⋅ ⋅ ... ⋅
⋅ ⋅ ... ⋅ i y=
⋅ 1 . Bidej}i > za
2 4 6 3 5 100 99 n+1 n
1
sekoj nœN, sleduva deka x<y. Od poslednovo i od x·y = sleduva
100
2 1 1
x <xy= odnosno x < .
100 10
1 1 1
169. a) Da ozna~ime S = + + ... + . So proverka se poka`uva deka za n§5
n 2 2
2 3 n2
po~etnoto neravenstvo e zadovoleno. Neka n¥6. Da zabele`ime deka
1 1 1 1 1 1  1 1  1 1  1 1
+ + ... + < + + ... + =  −  +  −  + ... +  − 
6 2
7 2
n 2 5 ⋅ 6 6 ⋅ 7 (n − 1) ⋅ n  5 6   6 7   n − 1 n 
1 1 1 1 1669 1 2389 2400 2
= − < . Toga{ Sn < S5 + = + = < = .
5 n 5 5 3600 5 3600 3600 3
1 1 1
b) Za k>1 va`i = − . Toga{
(k - 1) ⋅ k ⋅ (k + 1) 2 ⋅ (k − 1) ⋅ k 2 ⋅ k ⋅ (k + 1)
1 1 1 1 1 1 1
3
+ 3 + 3 + ... + 3 < 3-2 + 3 + ... + 3 =
2 3 4 n 2 3 −3 n −n
1 1 1 1 1 1
= + + ... + = − <
1⋅ 2 ⋅ 3 2 ⋅ 3 ⋅ 4 (n − 1) ⋅ n ⋅ (n + 1) 4 2 ⋅ n ⋅ (n + 1) 4
 1 1 1  t t  1 
170. Neka t = 1 + + + ... +  > 1 . Toga{ a = ; b= t −  i
 2 3 2019  2019 2018  2019 
1 t 1− t
a-b = − = < 0 , odnosno a<b.
2018 ⋅ 2019 2018 ⋅ 2019 2018 ⋅ 2019
 1  1 1 1 1 4 1
171. 1 +  ⋅ 1 +  = 1 + + + ≥ 1+ + = 9 . Ravenstvo va`i za
 x  y x y xy x + y  x + y 2
 
 2 
1
x=y= .
2
172. Transformiraj}i ja levata strana na neravenstvoto i primenuvaj}i go
neravenstvoto me|u aritmeti~ka i geometriska sredina dobivame
 1  1  1 xy + xz + yz x + y + z 1
1 +   1 +   1 +  = 1 + + +
 x   y  z xyz xyz xyz
3
33 x 2 y 2z2 33 xyz 1 1 1  1 
≥ 1+ + + ≥ 13 + 3 ⋅ 1+ 3 ⋅1 + 
2 
 3 xyz 
xyz xyz xyz (3 xyz )
2 3 xyz
 
3 3
 
= 1 +
1  ≥ 1 + 3 
 = (1 + 3 ) = 64.
3
 
 3 xyz   x + y + z 

121
Odgovori, upatstva i re[enija
173. Neravenstvoto }e go doka`eme so principot na matemati~ka indukcija.
Od (a 1 − a2 )
2
≥ 0 sleduva a 1 + a 2 ≥ 2 a 1 ⋅ a 2 . Neravenstvoto e to~no za n=2.
Neka toa e ispolneto za n=k≥2. ]e doka`eme deka neravenstvoto va`i i za
n=k+1. Neka a1≥a2≥a3≥….ak+1≥0.
Ako ak+1=0 toga{ neravenstvoto e ispolneto. Neka ak+1∫0. Postojat broevi
k+1 k k k
b1≥b2≥…bk>0 i t≥1 za koi a1=t b1b2…bk, a2=b1 , a3=b2 ,…,ak+1=bk . Imame
k+1 k k k+1 k+1
a1+a2+…+ak+1=t b1b2….bk+b1 +…bk ≥t b1b2bk+k k (b1k b 2 k …bk k ) =(t +k)b1b2…bk.
Od druga strana, (k+1) k +1 a 1a 2 ...a k +1 =(k+1)t b1b2…bk. Ostanuva da se doka`e deka
k+1 k+1 2 k
t +k≥ (k+1)t odnosno t +k-kt-t=(t-1)(t+t +…+t -k) ≥0 koe e o~igledno.
Ravenstvo se dostignuva za b1=b2=….=bk i t=1 odnosno za a1=a2=…=ak=ak+1.
174. Poradi simetrija dovolno e da pretpostavime deka a¥b¥c>0. Ako a¥b+c
toga{ neravenstoto e ispolneto bidej}i levata strana e negativna, a desnata
pozitivna. Ako а<b+c, toga{ levata strana e pozitivna i zatoa
(a + b - c)(b + c - a)(c + a - b) = (a + b - c )(b + c - a)(c + a - b)(a + b - c)(b + c - a)(c + a - b)
 
= a 2 -  b - c 2  ⋅ b 2 - (c - a )2  ⋅ c 2 - (a - b)2  ≤ a 2 b 2 c 2 = abc . Ravenstvoto
      
se dostignuva ako i samo ako а=b=с.
2 2 2
175. Neka x e proizvolen realen broj. Od (a1-xb1) +(a2-xb2) +….+(an-xbn) ≥0
2 2 2 2 2 2 2
sleduva deka (b1 +b2 +…+bn )·x -2(a1b1+…anbn)·x+a1 +a2 +…+an ≥0.
Bidej}i kvadratniot trinom od levata strana e sekoga{ pozitiven,
sleduva deka diskriminantata e negativna odnosno
2 2 2 2 2 2 2
(a1b1+a2b2+…+anbn) -(a1 +a2 +…+an )·(b1 +b2 +…+bn )§0.
a b a
Ravenstvo se dostignuva ako i samo ako 1 = 2 = ... = n .
b1 b2 bn
176. Neka а, b, c, d se posledovatelni ~lenovi na geometriskata progresija i
a c
neka a>b>c>d. Od = go dobivame slednovo ad=bc ñ ac-ad=ac-bc
b d
a−b a a
ñ a(c-d)=c(a-b) ñ = . Od a>c sleduva deka > 1 odnosno a-b>c-d,
c−d c c
{to e pak ekvivalentno so po~etnoto neravenstvo.
2 2 2
a b c a 2 − ab + b 2 b 2 − bc + c 2 c 2 − ca + a 2
178. Upatstvo: + + = + + .
b c a b c a

180. a²(b + c) + b²(a + c) + c²(a + b) = a²b + ab² + a²c + ac² + b²c + bc²
(a + b)(b + c)(c + a) (a + b)(b + c)(c + a)
(a + b)(b + c)(c + a) - 2abc 2abc 2abc 3
= = 1− ≥ 1− = ,
(a + b)(b + c)(c + a) (a + b)(b + c)(c + a) 8abc 4
{to treba{e da se doka`e.
Pritoa koristevme (a + b)(b + c)(c + a) ≥ 2 ab 2 bc 2 ac = 8abc

122
Odgovori, upatstva i re[enija
183. Od neravenstvoto me|u sredinata od stepen 3 i aritmeti~kata sredina za

a3 + b3 + c 3 a+b+c 3
broevite a, b i c, dobivame ≥ {to e ekvivalentno so
3 3
desnoto neravenstvo. Levoto neravenstvo e vsu{nost neravenstvoto pome|u
aritmeti~kata i geometriskata sredina za broevite a, b i c.
3 3 2 2 2 2
184. a + b = (a + b)(a + b − ab) ≥ (a + b)ab = a b + ab
b + c = (b + c)(b + c − bc) ≥ (b + c)bc = b c + bc ,
3 3 2 2 2 2

3 3 2 2 2 2
c + a = (c + a)(c + a − ca) ≥ (c + a)ca = c a + ca .
So sobirawe na neravenstvata se dobiva
3 3 3 2 2 2 2 2 2
2(a + b + c ) ≥ a b + a c + b a + b c + c a + c b t.e.
3 3 3 2 2 2
2(a + b + c ) ≥ a (b + c) + b (a + c) + c (a + b) odnosno
3 3 3 2 2 2 2 3 2 3 2 3
2(a + b + c ) ≥ a (1 − a) + b (1 − b) + c (1 − c) = a − a + b − b + c − c {to e
3 3 3
3 3 3 2 2 2 a +b +c 1
ekvivalentno so 3(a + b + c ) ≥ a + b + c . Kone~no 2 2 2
≥.
a +b +c 3
3 3 3 2 2 2
185. Neka a=x , b=y , c=z . Od (x-y) ¥0 sleduva deka x -xy+y ¥xy odnosno
3 3 2 2 3 3 3 3
x +y =(x+y)(x -xy+y )¥(x+y)xy. Analogno, x +z ¥(x+z)xz i y +z ¥(y+z)yz. Toga{
1 z z z z
= 3 3
§ = = . Analogno,
1 + a + b z + z(x + y ) z + z(x + y)xy z + xyz ⋅ x + xyz ⋅ y x+y+z
1 x 1 y
§ i § . So sobirawe na poslednite tri
1+ b + c x + y + z 1+ c + a x+y+z
neravenstva, se dobiva baranoto neravenstvo.
1 1 1
186. So smenata x= , y= , z= neravenstvoto se transformira vo
a b c
2 2 2 2 2 2
x +y +z ¥xy+xz+yz {to e pak ekvivalentno so (x-y) +(y-z) +(x-z) ¥0.
3 3 2 2 5 5 2 2
187. Od (a -b )(a -b )¥0 (zo{to?) sleduva a +b ¥a b (a+b). Ottuka
ab ab 1 abc c
≤ = = = …(1).
a5 + b5 + ab a2b2(a + b) + ab ab(a + b) + 1 ab(a + b + c) a + b + c
bc a
Analogno se dobivaat neravenstvata 5 5
≤ …(2) i
b + c + bc a+b+c
ca b
5 5
≤ …(3). So sobirawe na (1), (2) i (3) se dobiva baranoto
c + a + ca a+b+c
neravenstvo.
188. Od c-a=(c-b)+(b-a) sleduva
a−b b−c c −a  1 1   1 1 
+ + = (a − b ) −  + (b − c ) − 
a+b b+c c+a a +b c +a b + c c + a

123
Odgovori, upatstva i re[enija

=
(a − b )(b − c )(a − c ) . Od neravenstvata a-b < a+b , b-c < b+c i
(a + b )(b + c )(c + a )
c - a < c + a se dobiva baranoto neravenstvo.
3 2
189. Upatstvo: Doka`i deka va`i neravenstvoto a +1¥a +a.
191. Od neravenstvo me|u aritmeti~ka i geometriska sredina sleduva
1  (a + c ) (b + a )2 + (c + b ) 2  ≥ a ⋅ c + b ⋅ a + c ⋅ b odnosno
2
neravenstvoto  +
4  a + c b+a c + b  a + c b + a c + b
1 a⋅c b⋅a c ⋅b
⋅ (a + c + b + a + c + b ) ≥ + = , od kade {to se dobiva
4 a+c b+a c+b
2 2 2
a⋅c b⋅a c ⋅b 1 a + ac b + ba c + cb
+ + ≤ . Od + + = a + b + c = 1 sleduva
a+c b+a c+b 2 a+c b+a c+b
2 2 2
a b c  ac ba cb  1 1
+ + = 1−  + +  ≥ 1− = .
a+c b+a c +b a+c b+a c +b 2 2
ab 2 bc 2 ca 2 1
192. ]e go doka`eme neravenstvoto + + ≤ .
2 2 2 2
a +b b +c c + a2
2 2
a 2 + b 2 2ab 2 ab 2 b b 2 + c 2 2bc 2 bc 2 c
2
≥ 2
= ⇒ 2 2
≤ ; 2
≥ 2
= ⇒ 2 2

ab ab b a +b 2 bc bc c b +c 2
2 2 2
c +a 2ca 2 ca a
2
≥ 2
= ⇒ 2 2
≤ .
ca ca a c +a 2
So sobirawe na poslednite tri neravenstva imame
ab 2 bc 2 ca 2 a + b + c 1 . Ottuka,
2 2
+ 2 2
+ 2 2
≤ =
a +b b +c c +a 2 2
a3 b3 c3
+ 2 + 2
a2 + b2 b + c2 c + a2
ab 2 bc 2 ca 2
= a− 2 + b − + c −
a + b2 b2 + c2 c2 + a2
2 2
ab bc ca 2
= a+b+c −( 2 + + )
a + b2 b2 + c2 c2 + a2
ab 2 bc 2 ca 2 1 1
= 1− ( 2 + + ) ≥ 1− = .
a + b2 b2 + c 2 c 2 + a2 2 2
193. Upatstvo: Iskoristi go neravenstvoto 1 + 1 ≥ 4 .
x y x+y
 1 1 1 x y y z z x
194. x+y+z=(x+y+z)  + +  = 3 +  +  +  +  +  +  ≥ 9 .
x y z y x z y x z
1 1 1
Od + + = 1 sleduva xy + xz + yz − xyz = 0.
x y z
Zna~i, (x-1)(y-1)(z-1)=(x+y+z)-(xy+xz+yx-xyz)-1 ≥ 9-0-1=8.

124
Odgovori, upatstva i re[enija
195. Mno`ej}i go neravenstvoto so izrazot (x+1)(y+1)(z+1) se dobiva
2 2 2
ekvivalentnoto neravenstvo (x -1)(z+1)+(y -1)(x+1)+(z -1)(y+1)¥0 koe po
sreduvaweto se transformira vo slednoto neravenstvo
2 2 2 2 2 2
(x z+y x+z y)+(x +y +z )¥x+y+z+3 ...(1).
So primena na neravenstvoto me|u aritmeti~ka i geometriska sredina
2 2 2
dobivame deka va`i x z+y x+z y¥3...(2).
2 2 2 2 2 2 2 2 2 2 2 2 2 2 2 2
Od (x+y+z) =x +y +z +2xy+2yz+2zx§x +y +z +x +y +y +z +z +x =3(x +y +z ),
dobivame x +y +z ¥ (x + y + z ) =(x+y+z)· (x + y + z) ¥(x+y+z)· 3 xyz =x+y+z ...(3). Od
2 2 2 2

3 3
neravenstvata (2) i (3) sleduva neravenstvoto (1).
2 2
3 3 2 2 x y
196. Upatstvo: Od x +y =(x+y)(x -xy+y )¥xy(x+y) sleduva deka + ≥ x+y.
y x
1 1
197. Od 0§x§1 imame 1§x+1§2, od kade sleduva 1¥ ¥ .Od x¥0 imame
x +1 2
x x y y z z
x¥ ¥ . Analogno va`at neravenstvata y¥ ¥ i z¥ ¥ . Toga{
x +1 2 y +1 2 z +1 2
1 x y z x y z
1= (x+y+z)= + + § + + §x+y+z=2.
2 2 2 2 x +1 y +1 z +1
1 1 1
199. Od ≥ x + y + z sleduva deka xy + xz + yz ≥ x + y + z ...(1).
+ +
x y z
Kvadriraj}i go neravenstvoto (1) dobivame
(xy)2 + (xz)2 + (yz)2 + 2(x + y + z) ≥ x 2 + y 2 + z 2 + 2(xy + xz + yz)
 
ñ (xy)2 + (xz)2 + (yz)2 + 2(x + y + z) ≥ x 2 + y 2 + z2 + 2 1 + 1 + 1 
x y z
2 2 2 2 2 2
ï (xy) + (xz) + (yz) + 2(x + y + z) ≥ x + y + z + 2(x + y + z)
2
ï (xy)2 + (xz)2 + (yz)2 ≥ x 2 + y 2 + z 2 ...(2). Delej}i go neravenstvoto (2) so (xyz)
1 1 1
dobivame 2
+ 2
+ ≥ x 2 + y 2 + z2 . So mno`ewe na (1) i (2) se dobiva
x y z2
3
(xy)3 + (xz)3 + (yz)3 ≥ x 3 + y 3 + z 3 . Delej}i go poslednoto neravenstvo so (xyz)
1 1 1
se dobiva + + ≥ x3 + y3 + z3 .
3 3
x y z3
2 2 2 2 3 3
200. Upatstvo: (x +y ) – xy(x+y) =(x -y) (x -y ).
12 9 4 4 5 12
201. Ako 0<x<1 toga{ x -x +x -x+1=(1-x)+x (1-x )+x ¥0. Ako x¥1 toga{
12 9 4 9 3
x -x +x -x+1=(x +x)(x -1)+1¥1>0.
2 2
202. Voveduvaj}i smena x =a, y =b dobivame
x6 y6  4 a3 b3 (a − b )a 2 + (b − a )b 2
2
+ 2 -  x + y 4  = + − a2 − b2 =
y x   b a b a

= (a − b )
 a2

b 2  (a − b ) a 3 −b 3
=
( ) 3 3
≥ 0 (izrazite (a-b) i (a -b ) imaat ist znak).
 b a  ab
 

125
Odgovori, upatstva i re[enija

xy yz xz
203. Neka a= 2
, b= 2
i c= . Od abc=1, sleduva deka barem dva od
z x y2
broevite a, b i c se vo (-¶,1] iли [1,+¶). Bез губење на општоста, da
pretpostavime deka se toa broevite a i b.
1 1 1
Po~etnoto neravenstvo e ekvivalentno so 2 + 2 + 2 +3¥2(a+b+c), odnosno
a b c
1 1 2 2
so ( - ) +2(a-1)(b-1)+(ab-1) ¥0, koe e ispolneto poradi prethodnata
a b
diskusija.
2
204. Da pretpostavime deka а¥b (analogno se poka`uva koga b¥a). Toga{ b §ab
2 2 2 2 2 2 2 2
i a ¥ab. Sleduva deka a ¥a +b -ab¥b odnosno a ¥c ¥b , od kade imame a¥c¥b.
Zna~i a-c¥0 i b-c§0, od kade sleduva baranoto neravenstvo. 208. So
koristewe na neravenstvoto me|u aritmeti~kata i geometriskata sredina
1 1 1 1 1 1
dobivame + + ≥ + + …(1).
1 + 2ab 1 + 2bc 1 + 2ca 1 + a + b
2 2 2
1+ b + c 2
1+ c + a2
2

Od neravenstvoto me|u aritmeti~kata i harmoniskata sredina dobivame


1 1 1 3
+ + ≥3 …(2).
2
1+ a + b 2 2
1+ b + c 2 2
1+ c + a 2  2 2   2 2  2 2
1 + a + b  + 1 + b + c  + 1 + c + a 
     
Od (1) i (2) go dobivame baranoto neravenstvo.
209. Ako poznatoto neravenstvoto x2 + y2 + z2 ≥ xy + xz + yz go iskoristime
2 2 2
dvapati, dobivame a 4 + b 4 + c 4 ≥ (ab) + (ac ) + (bc ) ≥ a 2 bc + abc 2 + ab 2 c
= abc(a + b + c) .
212. Dadenoto neravenstvo se transformira vo oblikot
1 1 1
odnosno
ab
+
cd

(a + c ) ⋅ (b + d)
.
+ ≤
1 1 1 1 1 1 a+b c+d a+b+c+d
+ + +
a b c d a+c b+d
Mno`ej}i go poslednoto neravenstvo so -1 i dodavaj}i go brojot b+d, dobivame

b −
ab  
 + d −
cd 
 ≤b+d−
(a + c ) ⋅ (b + d) , odnosno b 2 + d 2 ≤ (b + d)2 .
 a+b  c + d a+b+c+d a+b c +d a+b+c +d
Ostanuva samo da go doka`eme neravenstvoto
 
2
  
2
 b  +  d   ⋅  a + b + c + d  ≤ (b + d)2 koe sleduva od
2 2

 a + b     
  c + d   
neravenstvoto na Ko{i-Bunjakovski-[varc.

Zabele{ka: Va`i i slednoto obop{teno neravenstvo


1 1 1 1
+ + ... + ≤ .
1 1 1 1 1 1 1 1
+ + + +
a1 b1 a2 b2 an bn a1 + a2 + ... + an b1 + b2 + ... + bn
5
213. Za sekoj realen broj a od intervalot [0,1] va`i a § a. Od x, yœ[0,1] sleduva
5 5 5 5 5
deka x §x, y §y i |x-y|§1. Zatoa, x +y +(x-y) §x+y+|x-y|.
5 5 5
i) Neka x¥y. Toga{, x +y +(x-y) §2x§2;

126
Odgovori, upatstva i re[enija
5 5 5
ii) Neka x<y. Toga{, x +y +(x-y) §2y§2.
214. Za prviot ~len od levata strana na neravenstvoto imame
x9 + y9
= x3 + y 3 −
(
2x 3 y 3 x 3 + y 3 )≥ x 3
+ y3  x 6 + x 3 y 3 + y 6 ≥ 33 x 9 y 9 = 3x 3 y 3  .
 
6
x +x y +y 3 3 6 6
x +x y +y 3 3 6
3  

x9 + y9 x3 + y3 9 9
Zna~i ≥ . Analogno, 6 y 3+ 3z 6 ≥ 1 y 3 z 3 i ( )
x6 + x3y3 + y6 3 y +y z +z 3
z9 + x 9
1 3
6
z +z x +x 3 3
3 6
( )
z + x3 . So sobirawe na trite neravenstva i so primena na

neravenstvoto me|u aritmeti~ka i geometriska sredina, dobivame


x9 + y9 y 9 + z9 z9 + x 9 2
6 3 3 6
+ 6 3 3 6
+ 6 3 3 6
≥ x3 + y3 + z3
3
( )
x +x y +y y +y z +z z +z x +x
2 3 3 3 3
≥ ⋅ 3 x y z = 2 . 215. Upatstvo: Ix+y-zI+Ix-y+zI ¥Ix+y-z+x-y+zI=2IxI.
3
2
 2 + 1
216. Za n=2 tvrdeweto e to~no bidej}i 2! <   . Da pretpostavime deka
 2 
k
 k + 1
tvrdeweto va`i za n=k, odnosno k! <   . Taka, za n=k+1 imame
 2 
k k +1
(k + 1)! < (k + 1) ⋅ k! < (k + 1) ⋅  k + 1 k + 2
= 2  ⋅
1
k +1
...(1).
 2   2   1 
1 + 
 k + 1
k +1
 1  1
Od neravenstvoto na Bernuli sleduva 1 +  > 1 + (k + 1) ⋅ =2
 k + 1 k +1
1 1
odnosno k +1
< ...(2). Od neravenstvata (1) i (2) sleduva
 1  2
1 + 
 k + 1
k +1 k +1
(k + 1)! < 2 k + 2  ⋅
1 k + 2
=  .
 2  2  2 
n n n
217. Bez gubewe na op{tosta da pretpostavime deka a ≥ b ≥ c . Od
neravenstvoto a2 + b2 + c 2 ≥ ab + ac + bc sleduva deka a2 − bc + b2 − ca + c 2 − ab ≥ 0 .
n
Mnoæej}i go poslednoto neravenstvo so c, dobivame deka
c n (a 2 − bc ) + c n (b 2 − ca) + c n (c 2 − ab ) ≥ 0 . Od
n n n
a ≥b ≥c i od prethodnoto
( ) (
neravenstvo sleduva an a2 − bc + bn b2 − ca + cn c2 − ab ≥ 0 {to e pak ekvivalentno ) ( )
so baranoto neravenstvo.
218. Dadenoto neravenstvo transformiraj go vo oblikot
(c-a)·lga+(a-b)·lgb+(b-c)·lgc≤0. Od a≤b≤c sleduva deka lga≤lgb≤lgc i c-a≥a-b. So
primena na neravenstvoto na ^ebi{ev za prvite dva ~lena se dobiva

127
Odgovori, upatstva i re[enija

1
(c-a)·lga+(a-b)·lgb+(b-c)·lgc≤ (c-a+a-b)·(lga+lgb)+(b-c)·lgc
2
1 ab 2 ab
= (c-b)·lgab-(c-b)·lgc=(c-b)·lg ≤0 (a≤c i b≤c fl ab ≤c fl ≤1).
2 c c
219. Poradi simetrija mo`eme da pretpostavime deka x¥y¥0.
a a b b
Neka a i b se proizvolni pozitivni celi broevi. Toga{ x ¥y , x ¥y i
a a b b a+b a+b a b b a
(x -y )·(x -y )¥0. Od poslednoto neravenstvo sleduva x +y ¥x y +x y i
a+b a+b
dodavaj}i go izrazot x +y na dvete strani od neravenstvoto, dobivame
a+b a+b a a b b
deka 2(x +y )¥(x +y )·(x +y ). Za a=2 i b=3 va`i neravenstvoto
5 5 2 2 3 3
2(x +y )¥(x +y )·(x +y )…(1). Za a=5 i b=4 va`i neravenstvoto
9 9 5 5 4 4
2(x +y )¥(x +y )·(x +y )…(2). Od neravenstvata (1) i (2) sleduva
9 9 5 5 4 4 2 2 3 3 4 4
4(x +y )¥ 2(x +y )·(x +y )¥ (x +y )·(x +y )·(x +y ).
220. Upatstvo: Primenete go dvapati neravenstvoto na Ko{i.
221. Bez gubewe na op{tosta, poradi simetrija , pretpostavuvame deka
 3(x − y )2 3(y − z )2 3(z − x )2  3(z − x )2
x ≤ y ≤ z .Toga{ max  , , = i dovolno e da
 4 4 4  4

3(z − x )2
doka`eme deka x 2 + y 2 + z2 − xy − xz − zx ≥ . So nekolku elementarni
4
transformacii dobivame (2y − x − z )2 ≥ 0 . Poslednoto neravenstvo e ispolneto
za site x, y, z.
222. Bez gubewe na op{tosta da pretpostavime deka a§b§c. Neka
2 2 2 2, 2 2}
x=b-a, y=c-b i m=min{x,y}. Jasno x,y¥0 i m =min{x ,y }¥min{(a-b) (b-c) ,(c-a) .
Toga{
1 2
2
( 1
2
) [ 2 2 1
2
2
] [
2
a + b 2 + c 2 = a 2 + (a + x ) + (a + x + y ) = (a + y ) + 2(a + x ) + 2xy ]
[ 2 2 2
]
≥ xy ≥ m 2 ≥ min (a − b) , (b − c ) , (c − a ) , {to i treba{e da se doka`e.
1− 2013 kp
kp
223. Za p¥0 i k>0 va`i kp¥0, 2013 ¥2013 =1, 1-2013 §0, 2013 kp
0
≤ 1.
kp
( )
So zamena vo poslednoto neravenstvo za k=1,2,3,…,2013 i so sobirawe na
dobienite neravenstva se dobiva baranoto neravenstvo.
224. Bidej}i logab>logaa=1 sleduva deka loga(logab)>logb(logab). Od
logca<logcc=1 sleduva deka logc(logca)>logb(logca). Zna~i,
loga(logab)+logb(logbc)+logc(logca)>logb(logab)+logb(logbc)+logb (logca)
=logb(logab·logbc·logca )=logb1=0.
226. Levata strana od neravenstvoto ja ozna~uvame so S. So primena na
neravenstvo me|u aritmeti~ka i geometriska sredina dobivame
S 3
≥ (log a bc )r ⋅ (log b ca )r ⋅ (log c ab )r
3
r
[( )( )(
= log a b + log a c ⋅ log b c + log b a ⋅ log c a + log c b )] 3

r
 1 1 13
( ) ( ) (
≥ 2 ⋅ loga b ⋅ loga c 2 ⋅ 2 ⋅ logb c ⋅ logb a 2 ⋅ 2 ⋅ logc a ⋅ logc b 2 
 
)
 

128
Odgovori, upatstva i re[enija

r r

= [
2 r ⋅ log b ⋅ log c ⋅ log c ⋅ log a ⋅ log a ⋅ log b
a a b b c c
] 6 r 3 r
= 2 ⋅ (1 ⋅ 1 ⋅ 1) = 2 . Od ovde
r
sleduva deka S¥3·2 .
227 Упатство: Koristete go neravenstvoto na Ko{i-Buwakovski-[varc i
neravenstvoto me|u aritmeti~ka i geometriska sredina.
228. Ako ai=0 za nekoj i=1,2,…,n ili bi=0 za nekoj i=1,2,…,n toga{ dokazot e
trivijalen. Zatoa, neka site broevi se strogo pozitivni. Toga{ pozitivni se i
a1 a2 an b1 b2 bn
broevite , , ..., i , , ..., . So
a 1 + b1 a 2 + b 2 an + bn a1 + b1 a2 + b2 an + bn
primena na neravenstvoto na Ko{i se dobiva

a1 a2 an 1  a1 a2 an  …(1)
⋅ ... ≤  + + ... + 
an + bn 
n
a1 + b1 a 2 + b 2 a n + b n n  a1 + b1 a 2 + b 2

b1 b2 bn 1  b1 b2 bn 
n ⋅ ⋅ ... ⋅ ≤  + + ... +  …(2). So sobirawe
a1 + b1 a2 + b2 an + bn n  a1 + b1 a2 + b2 an + bn 
na neravenstvata (1) i (2), a potoa mno`ej}i so n (a 1 + b1 )(a 2 + b 2 ) ⋅ ... ⋅ (a n + b n )
se dobiva baranoto neravenstvo.
229. Od neravenstvoto me|u harmoniskata i aritmeti~kata sredina imame
s s s
+ + ... +
n s − a1 s − a 2 s − an

1 1 1 n
+ + ... +
s s s
s − a1 s − a 2 s − an
n2 s s s
≤ + + ... +
s − a1 s − a 2 s − a n s − a1 s − a 2 s − an
+ + ... +
s s s
n2 s s s
≤ + + ... +
a a a  s − a1 s − a 2 s − an
n −  1 + 2 + ... + n 
 s s s 
s s s n2
+ + ... + ≥
s − a1 s − a 2 s − an n − 1 − n
 s   s   s  n2
 − 1 +  − 1 + ... +  − 1 ≥ −n
 s − a1   s − a 2   s − an  n −1
a1 a2 an n .
+ + ... + ≥
s − a1 s − a 2 s − an n − 1
Zabele{ka: Za n=3 se dobiva poznatoto neravenstvo na Nesbit.
231. Upatstvo: Koristete go neravenstvoto na Jensen i funkcijata f(x)=xlgx.

129
Odgovori, upatstva i re[enija

Tema 5: Geometriski neravenstva

232. P∆ABC =
a ⋅ ha
= r ⋅s =
a+b+c  b c
⋅ r toga{ ha =  1 + +  ⋅ r …(1). Analogno gi
2 2  a a
a c a b 
dobivame ravenstvata h b =  + 1 +  ⋅ r …(2) i h c =  + + 1 ⋅ r …(3). So
b b c c 
sobirawe na ravenstvata (1), (2) i (3) imame
  b a   c a   c b 
h a + h b + h c = 3 +  +  +  +  +  +   ⋅ r ≥ (3 + 2 + 2 + 2 ) ⋅ r = 9r .
  a b   a c   b c 
b2 + c 2 a2 b 2 + c 2 (b + c)2
imeme deka.
2
233. Od ravenstvoto t a = - i od ≥
2 4 2 4
2
ta ≥
(b + c )2 − a2 = (b + c + a )(b + c - a) = s(s − a) . Analogno gi imame
4 4
neravenstvata t b ≥ s(s - b ) i t c ≥ s(s - c ) . Sobiraj}i gi poslednite
2 2

neravenstva imame deka t a + t b + t c ≥ s(s - a ) + s(s - b ) + s(s - c ) = s 2 .


2 2 2

241. Od S > S1 + S2 + S3 i od neravenstvoto me|u aritmeti~ka i harmoniska


sredina na broevite S1, S2 i S3 dobivame:
1 1 1
+ +
S S1 + S 2 + S 3 3 3 S1 S2 S3 9 1 1 1 .
> ≥ ; < ; < + +
3 3 1 1 1 S 3 S S1 S2 S3
+ +
S1 S2 S3
2 2 2 2 2 2 2 2 2
242. Od ravenstvata t a 2 = b + c - a , t b 2 = a + c - b i tc2 = a + b - c
2 4 2 4 2 4
imame deka  t a 2 + t 2 + t c 2  ⋅  h a 2 + h 2 + h c 2  ≥ 3 ah a + bh + ch c 2 = 3 (6P )2 = 27P 2 .
( )
 b   b  4 b 4
1 1 4
243. ]e doka`eme deka + ≥ za m>0 i n>0.
m n m+n
2 2 2 2
Od (m-n) ¥0 sleduva deka m +n ¥2mn odnosno (m+n) ≥ 4mn. Delej}i go
m+n 4 1 1 4
neravenstvoto so (m+n)mn dobivame ≥ odnosno + ≥ . So
mn m+n m n m+n
1 1 4 4
primena na ova neravenstvo dobivame + ≥ = . Analogno
s−a s−b s−a+s−b c
1 1 4 1 1 4
imame deka + ≥ i + ≥ . Sobiraj}i gi poslednite tri
s−b s−c a s−c s−a b
neravenstva i kratej}i so 2 se dobiva baranoto neravenstvo.

130
Odgovori, upatstva i re[enija

244. Od a 2 + b 2 + c 2 ≥ ab + bc + ca se dobiva neravenstvoto


 1 1 1 
a 2 + b 2 + c 2 ≥ 2P  + +  ...(1).
 No, od neravenstvoto
 sinα sinβ sinγ 
1 1 1 1 i koristej}i go neravenstvoto
+ + ≥ 3⋅3
sinα sinβ sinγ sinα ⋅ sinβ ⋅ sinγ
3 3 1 1 1 8
sinα ⋅ sinβ ⋅ sinγ ≤ dobivame + + ≥ 3⋅3 = 2 ⋅ 3 ...(2). Od
8 sinα sinβ sinγ 3⋅ 3
neravenstvata (1) i (2) se dobiva baranoto neravenstvo.
248. Upatstvo: koristete ja kosinusna teorema i neravenstvoto
a + b + c ≥ 4 3P .
2 2 2

249. Upatstvo: Od a>Ib-cI, kvadriraj}i go neravenstvoto se dobiva


2 2 2
a +2bc>b +c .
250. ]e doka`eme deka PPBQO=2 P∆MPO ⋅ P∆OQN . Bidej}i PPBQO=2PóPBO imame
PPBQO 2P∆PBO PB OQ
= = = . Od sli~nosta na triagolnicite MPO i OQN
2P∆MPO 2P∆MPO MP MP
OQ P∆OQN P∆OQN
sleduva = . Toga{ PPBQO= 2P∆MPO ⋅ =2 P∆MPO ⋅ P∆OQN ...(1).
MP P∆MPO P∆MPO
Analogno se dobivat ravenstvata PNCED=2 P∆OQN ⋅ P∆FOC ...(2) i

PAMOF =2 P∆MPO ⋅ P∆FOE ...(3).

Od (1), (2) i (3) gi dobivame ravenstvata z = 2 ab , x = 2 bc , y = 2 ac ...(4).


Od relacijata me|u aritmeti~ka i geometriska sredina na tri broja imame
a b c abc
+ + ...(5). Zamenuvajki gi ravenstvata (4) vo neravenstvoto (5)
≥ 3⋅3
x y z xyz
se dobiva baranoto neravenstvo.
252. Lesno se proveruva deka abcIp-qI=I(b-c)(c-a)(a-b)I ...(1). Od toa {to
Ib-cI<a, Ic-aI<b i Ia-bI<c sleduva neravenstvoto Ib-cI·Ic-aI·Ia-bI<abc …(2).
Zamenuvaj}i go ravenstvoto (1) vo neravenstvoto (2) se dobiva abc·Ip-qI<abc,
od kade sleduva Ip-qI<1.
α β γ 1 α −β α + β γ
253. a) Neka S = sin sin sin = cos − cos sin . Od toa {to
2 2 2 2 2 2  2
α−β α+β γ 1 γ γ
0 < cos ≤ 1 i cos = sin sleduva S ≤  1 − sin sin . Funkcijata
2 2 2 2 2 2
1 γ 1
f (x ) = (1 − x )x = − x + x dostiga maksimum za x =
2
. Zna~i, ako sin =
2 2 2

131
Odgovori, upatstva i re[enija

1 α−β γ 1
toga{ P ≤ . Ravenstvoto se dostignuva za cos = 1 i sin = odnosno
8 2 2 2
α = β = γ = 60° .
α β γ
254. Od ravenstvoto cosα+cosβ+cosγ=1+4sin sin sin i neravenstvoto
2 2 2
α β γ 1 1 3
sin sin sin § dobivame cosα+cosβ+cosγ≤1+4 = . Od toa {to
2 2 2 8 8 2
α β γ
sin sin sin >0 imame cosα+cosβ+cosγ>1.
2 2 2
a b c
256. Od sinusna teorema imame deka R= = = …(1). Sega od
2sinα 2sinβ 2sinγ
abc
formulate za plo{tina na triagolnik imame = P = rs od kade abc=4rRs
4R
…(2). Da zabele`ame deka za sekoi pozitivni realni broevi x i y va`i

)⋅  1 − 1  ≤ 0 odnosno
2 2
x y
neravenstvoto x − y ( 2 2
≥ x + y ...(3).
+
x y y x
Koristej}i gi ravenstvata (1), (2) i kosinusna teorema
dobivame
2 2
b +c −a (2
)
2bc = 2R  b + c − a  = R  b + c − a  …(4). Analogno gi
2 2 2 2 2
cosα
=    
sin α
2
a
2R
2
( )
abc  a a  2rs  a a 

R a  2 2
 2 2

dobivame ravenstvata cosβ = R  a + c − b  …(5) i
cosγ b
  2
=  + − c 
sin β 2rs  b
2
b  sin γ 2rs  c c 
…(6). Sobiraj}i gi ravenstvata (4), (5), (6) i koristej}i go neravenstvoto (3)
cosα cosβ cosγ R  a 2 b 2 c 2 b 2 a 2 c 2 
dobivame + + = + + + + + − a − b − c
sin α sin β sin γ 2rs  b
2 2 2  a b c c a 

R R
≥ (a + b + c ) = .
2rs r
257. ]e doka`eme deka a·cos α +b·cos β §c...(1). Od toa {to
c=a·cos β +b·cos α , zamenuvaj}i vo neravenstvoto (1) dobivame deka treba da se
doka`e neravenstvoto a·cos α +b·cos β §a·cos β +b·cos α odnosno
(a-b)·(cos α -cos β )§0...(2). Od a≥b sleduva deka cos α §cos β odnosno a-b≥0,
cos α -cos β §0 pa zna~i sleduva neravenstvoto (2). Ako a§b sleduva
cos α ¥cos β odnosno a-b§0, cos α -cos β ¥0, pa povtorno va`i neravenstvoto
(2). Analogno va`at neravenstvata a·cos α +c·cos γ §b...(3) i
b·cos β +c·cos γ §a...(4). Sobiraj}i gi neravenstvata (1), (3), (4) se dobiva
baranoto neravenstvo.

132
Odgovori, upatstva i re[enija
258. Od sinusna teorema za triagolnikot ABC gi imame ravenstvata
abc
a=2Rsin α , b=2Rsin β , c=2Rsin γ . Od r ⋅s = dobivame deka
4R
r abc
= . Zamenuvaj}i gi ravenstvata od sinusna teorema i
R 2
2R ⋅ (a + b + c)
α β γ
koristej}i go ravenstvoto sinα + sinβ + sinγ = 4 ⋅ cos cos cos dobivame
2 2 2
α α β β γ γ
3 2 ⋅ 2sin cos ⋅ 2sin cos ⋅ 2sin cos
8R sinα ⋅ sinβ ⋅ sinγ
r
= = 2 2 2 2 2 2 = 4sin α sin β sin γ .
R 4R 3 ⋅ (sinα + sinβ + sinγ ) α β γ 2 2 2
4cos cos cos
2 2 2
α β γ
Koristej}i go pak ravenstvoto cosα + cosβ + cosγ = 4sin sin sin + 1 dobivame
2 2 2
r r +R
= cosα + cosβ + cosγ - 1 , odnosno cosα + cosβ + cosγ =
. Bidej}i
R R
cosα > 0, cosβ > 0 , cos γ > 0 (triagolnikot ABC e ostroagolen) mo`eme da go
koristime neravenstvoto me|u aritmeti~ka i harmoniska sredina za tri
~lena, pri {to se dobiva neravenstvoto
1 1 1
+ +
cosα cosβ cosγ 3
≥ od kade sleduva
3 cosα + cosβ + cosγ
1 1 1 1 9R
+ + ≥9⋅ = .
cos α cos β cos γ cos α + cos β + cos γ r + R
259. ®COB e centralen agol pri {to va`i ravenstvoto ®COB=2α odnosno
®BOA1=α. Od pravoagolniot triagolnik OA1B go dobivame ravenstvoto
1 1 ...(1). Analogno se dobivat i ravenstvata 1 1 ...(2);
= =
cosα OA 1 cosβ OB1
1 1
= ...(3). Sobiraj}i gi ravenstvata (1), (2) i (3) dobivame
cosγ OC1
1 1 1 1 1 1 9 (koristej}i go pritoa
+ + + = + ≥
OA1 OB1 OC1 cosα cosβ cosγ cosα + cosβ + cosγ
neravenstvoto me|u aritmeti~ka i harmoniska sredina). Od neravenstvoto
3
cos α + cos β + cos γ ≤ sleduva baranoto neravenstvo.
2

260. Od formulite za plo{tina na triagolnik imame deka


rs = s ⋅ (s − a ) ⋅ (s − b ) ⋅ (s − c ) od kade sleduva ravenstvoto
2
r =
(s − a ) ⋅ (s − b ) ⋅ (s − c ) …(1). Koristej}i ja vrskata me|u aritmeti~ka i
s
geometriska sredina na tri broja go imame neravenstvoto

133
Odgovori, upatstva i re[enija

s
=
(s − a ) + (s − b ) + (s − c ) ≥ 3 (s − a ) ⋅ (s − b ) ⋅ (s − c ) …(2). Koristej}i ja
3 3
Pitagorova teorema, ravenstvoto (1) i neravenstvoto (2), dobivame
2 2 2
[
VA ⋅ VB ⋅ VC = r + (s − a ) ⋅ r + (s − b ) ⋅ r + (s − c )
2 2
][ 2 2
][ 2 2
]
=
(s − a) ⋅ bc ⋅ (s − b) ⋅ ca ⋅ (s − c ) ⋅ ab = (abc )2 ⋅ (s − a)(s − b)(s − c )
s s s s3

≤ (abc ) ⋅
2 s3 (abc ) 2 abc
3 3
= 3
. Zna~i, VA ⋅ VB ⋅ VC ≤ .
3 ⋅s 3 3 3
a b c
261. Spored sinusnata teorema imame deka = = = 2R . Dadenoto
sinα sinβ sinγ
neravenstvo e ekvivalentno so neravenstvoto
2 2 2 2 2
2R (2sin α + 2sin β + 2sin γ ) ≤ 9R , odnosno cos2 α + cos2 β + cos2 γ ≥ − 1,5 .
Ostanuva da go doka`eme poslednoto neravenstvo. Neka to~kata O e centarot
na opi{anata kru`nica toga{ ®AOB= 2γ , ®BOC= 2α i ®COA= 2β . Imame:
→ → → →2 →2 →2 → → → → → →
(OA + OB+ OC) 2 ≥ 0 ⇔ OA + OB + OC + 2 OA⋅ OB+ 2 OA⋅ OC+ 2 OB⋅ OC ≥ 0 ⇔
R 2 + R 2 + R 2 + 2R 2 cos2 α + 2R 2 cos2 β + 2R 2 cos2 γ ≥ 0 ⇔
cos2 α + cos2 β + cos2 γ ≥ − 1,5.
k + KC > b k < b + KC
262. Imame deka  ...(1),  ...(2).
k + KB > c k < c + KB
1
Od (1) sobiraj}i gi neravenstvata dobivame k > (b + c - a ) .
2
1
Analogno nao|ame deka l > (c + a - b) , m > 1 (a + b - c ) . Sobiraj}i gi poslednite
2 2
k +l+m 1
tri neravenstva dobivame > .
a+b+c 2
1 1
Na sli~en na~in od (2) dobivame k< (a + b + c ) , l< (a + b + c ) ,
2 2
1 k +l+m 3
m< (a + b + c ) od kade se dobiva < .
2 a+b+c 2

263. Bez gubewe na op{tosta neka a§b§c i neka I e


prese~nata to~ka na simetralite na aglite vo
triagolnikot ABC (crt. 65). Toga{
sa sb sc sa sb s + s AI + IB
+ + > + ≥ a b
> > 1.
ta tb tc ta tb c c
Crte` 65

134
Odgovori, upatstva i re[enija
264. Neka a, b i c se dol`inite na stranite BC, CA i AB, a ha, hb i hc visinite
spu{teni od temiwata A, B i C. So C1 ja
ozna~uvame prese~nata to~ka na opi{anata
kru`nica k okolu triagolnikot ABC so
centar vo to~ka O so pravata CO. Neka
AH = x , BH = y i CH = z . Od toa {to
~etiriagolnikot AC1BH e parallelogram
(zo{to?) imame HC1 = 2HD . Od
triagolnikot CHC1 imame 2HD + z ≥ 2R .
Analogno gi dobivame neravenstvata
2HE + x ≥ 2R i 2HF + y ≥ 2R . Sobiraj}i gi
poslednite tri neravenstva imame
2HD + 2HE + 2HF ≥ 6R - (x + y + z) …(1). Od
ravenstvata aha+bhb+chc=6P i
aHN + bHT + cHM = 2P Crte` 66
(kade to~kite N, T i M se podno`jata na visinite ha, hb i hc, soodvetno) imame
a(h a − HN) + b(h b − HT ) + c(h c - HM) = 2P odnosno ax+by+cz=4P …(2).
Od druga strana, sobiraj}i gi neravenstvata c = AM + MB = xsinα + ysinβ ,
a = BE + EC = ysinγ + zsinβ i b = CT + TA = zsinα + xsinγ dobivame
(y + z)sinα + (x + z)sinβ + (x + y)sinγ = 2s odnosno (y + z)a + (x + z)b + (x + y)c = 4sR
…(3). Od (2) i (3) imame (x+y+z)(a+b+c)=4sR+4P odnosno x+y+z=2R+2r …(4).
Zamenuvaj}I (4) vo (1) imame 2(HD + HE + HF) ≥ 3R + R - 2r …(5). Od poznatoto
ojlerovo ravenstvo OV = R(R − 2r) (kade V e centarot na vpi{anata
kru`nica vo triagolnikot ABC) imame R¥2r …(6). Od (5) i (6) sleduva
baranoto ravenstvo. Ravenstvo va`i koga HªOªV, odnosno ABC e ramnostran
triagolnik.

Tema 6: Stepeni i koreni


265. x=a ili x=b ili x=c. Ravenkata se zapi{uva vo vidot:
3 2 2 2
x -bx -ax +abx-cx +bcx+acx-abc=0
2 2
x(x -bx-ax+ab)-c(x -bx-ax+ab)=0
2
(x -bx-ax+ab)·(x-c)=0
(x(x-b)-a(x-b))·(x-c)=0
(x-a)·(x-b)·(x-c)=0
(x-a=0) ili (x-b=0) ili (x-c=0). Kone~no, x=a ili x=b ili x=c. 267. 2 .
269. Dadeniot izraz se zapi{uva vo oblikot
2 3 12n 2 3 4 5 6 7 8 12n-3 12n-2 12n-1 12n
7+7 +7 +...+7 =(7+7 +7 +7 )+(7 +7 +7 +7 )+...+(7 +7 +7 +7 )
1 2 3 5 1 2 3 12n-3 1 2 3
=7(1+7 +7 +7 ) +7 (1+7 +7 +7 )+...+7 (1+7 +7 +7 )
1 2 3 5 12n-3 . .
=(1+7 +7 +7 )(7+7 +...+7 )=2800 Q=100 (28Q).
2 3 12n
Zna~i, brojot 100 e delitel na brojniot izraz 7+7 +7 +...+7 .

135
Odgovori, upatstva i re[enija
n+2 2n+1 n 2n n+2 2n+1
270. Od 11 +12 =121·11 +12·12 dobivame 11 +12 =
n 2n n n n 2n n n 2n n
=121·11 +12·12 +12·11 -12.11 =133·11 +12·12 -12·11 =133·11 +12(12 -11 )=
n n n n n n
=133·11 +12·(144 -11 )=133·11 +12·(144-11)Q=133·11 +12·133Q=133(11 +12Q).
Ottuka zaklu~uvame deka 133 e delitel na dadeniot izraz.
n
271. Od uslovot na zada~ata imame deka 3 +m=10k...(1). Toga{,
n+4 n n n
3 +m=81·3 +m=81·3 +81m-81m+m=81·(3 +m)-80m. Zamenuvaj}i go ravenstvoto
(1) na desnata strana od poslednoto ravenstvo, dobivame deka
n+4
3 +m=81·10k-80m =10(81k-8k). Ottuka zaklu~uvame deka 10 e delitel na
n+4
izrazot 3 +m.
(
272. pp + qq = pp - 1 + qq + 1 ) ( )
(
= (p - 1) p
p-1
+p
p-2
+ ... + p + 1) + (q + 1)(q
q-1
-q
q-2
+ ... - q + 1)
= (p - 1)(2m + 1) + (q + 1)(2n + 1) .
Od uslovot na zada~ata va`i
p=q+2, odnosno p-1=q+1. Zna~i
p + q = (p - 1)(2m + 2n + 2) = 2(p - 1)(m + n + 1) = [(p - 1) + (p - 1)](m + n + 1)
p q

= (p - 1+ q + 1)(m + n + 1) , odnosno pp + qq = (p + q)(m + n + 1) .


n n
274. 25 и 76. 275. Neka broevite od oblik 2 -1 i 2 +1 se i dvata delivi so 7.
n n 3
Toga{ 2 =7k+1 …(1) и 2 =7j-1 …(2). Mno`ej}i gi dvete strani so 2 dobivame
n+3 n+3 n+3 n+3
2 =56k+8 и 2 =56j-8, odnosno 2 =7(8k+1)+1=7ÿA+1 и 2 =7(8j-1)-1=7B-1.
n
Zabele`uvame deka ako broevite od vid 2 mo`e da se pretstavat vo vid (1) и
n+3
(2) toga{ i broevite 2 mo`at da se pretstavat vo istiot vid. Zatoa e
0 1 2 0
dovolno da se razgledat samo stepenite 2 =1, 2 =2 и 2 =4. No, samo 2 =1 mo`e
0 .
da se zapi{e vo vidot 2 =1=7 0+1. Sleduva deka samo broevite od vidot
n n
2 -1, kade n=3m, se delat so 7 i nitu eden od broevite od vidot 2 +1.
276. n1+n2+…+nr=19. 277. 9. 278. 7.
n n n
279. 43. Neka 2003=100ÿa+3, za a=20. Toga{ 2003 =(100a+3) =100ÿP(a)+3 , {to
zna~i deka poslednite dve cifri ke bidat isti so poslednite dve cifri na
n n
stepenot 3. Dvocifrenite zavr{etoci na 3 se
01,03,09,27,81,43,29,87,61,83,49, 47,41,23,69,07,21,63,89,67,01 i se povtoruvaat
n n
posle sekoj 20-ti ~len. Od isti pri~ini, 2005 ≡5 (mod20)≡5(mod20). Spored
20005 n 5
toa 2003 ≡3 (mod20) od kade {to se dobiva deka poslednite dve
cifri se 43. 280. 01. 282. 16. Bidej}i 2 ÿ503 =2012, mo`eme da go najdeme
2 1

ostatokot po modul 2012 preku ostatocite po modul 4 i po modul 503. Jasno e


deka 2 ª 0 (mod 4). Od malata teorema na Ferma imame a ª 1(mod 503), za
2012 502
2012 502 4 4 4
NZD(a,503)=1. Zna~i 2 ª(2 ) ÿ2 ª 2 (mod503)=16(mod503).
284. n=2. Ja koristime formulata
2n+1 2n+1 2n 2n-1 2n-2 2 2n-1 2n
a +b =(a+b)(a -a b+a b -...-ab +b ) od kade dobivame
2013 2013 2012 2011 2010 2 2012 2011 2010
3 +1 =(3+1)(3 -3 +3 -...-3+1)=2 (3 -3 +3 -...-3+1).
Za brojniot izraz vo poslednata zagrada }e poka`eme deka e neparen broj.
2012 2011 2010 2009 2 1
Navistina, 3 -3 +3 -3 +...+3 -3 +1
2011 2009 2011 2009 2
=3 (3-1)+3 (3-1)+...+3(3-1)+1=2(3 +3 +...+3)+1=2m+1. Toga{ 2 e
2013
najgolemiot delitel od ovoj oblik za 3 +1.
2 n 11 8
285. 12. Za n=7 direktno se proveruva. Neka k =2 +2 +2 . Za n¥8,
2 8 n-8 3 n-8 3 2
k =2 (2 +2 +1). Toga{ 2 +2 +1=m , odnosno e poln kvadrat, od kade
2 n-8 n-8
m -9=2 . Ponatamu imame (m-3)(m+3)=2 . Taka imame deka (m-3) i (m+3) se
stepeni so osnova 2. Nivnata razlika e 6 poradi {to mora m+3=8, od kade
n-8
m=5. Toga{ 2 =6, pa dobivame n=12.

136
Odgovori, upatstva i re[enija
m m m
286. Da pretpostavime deka n>m>0. Sega, od a +b =c sleduva deka 0<a<c,
n-m n-m n-m n-m
0<b<c. Spored toa 0<a <c i 0<b <c .
n n n m n-m m m n-m m n-m m n-m m n-m m n-m n n n
Zna~i, a +b =c =c ·c =(a +b )·c =a ·c +b ·c >a ·a +b ·b =a +b =c , {to
pretstavuva kontradikcija. Isto taka, se doa|a do kontradikcija dokolku
pretpostavime deka m>n>0. Zaklu~uvame deka m=n.

287. Dadeniot izraz, se razlo`uva i se zapi{uva vo oblikot


 23   23 
S =  400...0 23
1233  +  200...0
1233 + 200...0
43
1233 
1233 − 200...0
 n n   n n 
 
.
123 3 ⋅  200...0
1233 − 1
23 20
= 600...0
123 6 ⋅ M + 200...0
n n  n 
n n
No, a -b pri paren pokazatel n se deli so (a-b)(a+b). Sleduva deka
S = 600...0 23 .
123 6 ⋅ M + 200...0
123 3 ⋅ 200...0 123 2 ⋅ 200...0
123 4 ⋅ N
n n n n

Od toa {to zbirot na cifrite na brojot 200...0


1234 e 6, sleduva deka brojot e
n

123 4 = 3k . Na toj na~in dobivame deka


deliv so 3 i mo`e da zapi{eme 200...0
n

 
123 3 ⋅ k ⋅ N  ,
123 6 M + 200...0
23 23
S = 600...0
123 6 ⋅ M + 200...0
123 3 ⋅ 600...0
123 6 ⋅ k ⋅ N = 600...0
n n n n  n 
odnosno S se deli so brojot 600...06 .
123
n

289. 1+ 10( ) = 1+ 100 ⋅ 10 + ... + 10 i (1- 10 )


100 100 100
= 1 - 100 ⋅ 10 + ... + 10
100
.
    99 
10 (1+ 10) - (1- 10)
100 100 3
Zna~i,  10 + ... + 10  e cel broj.
   = 10 200 ⋅  10 +    
290. a) 4; b) 2.
1 1
291. (10 + 6 3 ) 3 (
− 2 10 + 6 3 )

3 = 3 1+ 3 3 + 9 + 3 3 − 2
3
1
1+ 3 3 + 9 + 3 3
3
= 13 + 3 3 + 3 3 +
2
( 3) 3

2
3 3
1 +3 3 +3 3 +
2
( 3) 3

=
3
(1 + 3 ) 3

2
= 1+ 3 +
2
= 1+ 3 +
2

1− 3

(1 + 3 )
3 3 1+ 3 1+ 3 1− 3

2(1 − 3 ) 2−2 3 2−2 3


= 1+ 3−
(1 + 3 )(1 − 3 ) = 1 + 3−
1 − 2
( 3) 2
= 1+ 3 −
−2

2−2 3 2 2 3
= 1+ 3 + = 1+ 3 + − = 1+ 3 + 1− 3 = 1+ 1= 2
2 2 2

137
Odgovori, upatstva i re[enija

292. 2 . 293. a) 3. Neka α = 3 9 + 4 5 , α = 3 9 - 4 5 . Toga{ α13 + α 2 3 = 10 i


1 2

α1 + α 2 = 1 . Od ravenstvoto
2
+ α1
3
α2
3
(
= (α1 + α2 ) ⋅ (α1 + α2 ) − 3α1 ⋅ α2
2
) se dobiva
ravenkata 10=x(x -3), ~ie realno re{enie e samo brojot 3.
( )
3
b) 4. Upatstvo: 20 + 14 2 = 2 + 2 ; 20 − 14 2 = 2 − 2 . ( )
3

294. Neka a = 2 + 2 + ... + 2 ,toga{ a = 2 + 2 + ... + 2 = 2 + a . Na toj na~in


1442443 1442443
n −1 koreni n koreni

2− 2+a 1
neravenstvoto dobiva oblik > . Mno`ej}i go neravenstvoto so
2−a 4
4(2-a), za 2-a>0, imame 8 − 4 ⋅ 2 + a > 2 − a odnosno 6 +a > 4⋅ 2+a .
Kvadriraj}i go poslednoto neravenstvo, dobivame deka treba da se doka`e
2 2
neravenstvoto a -4a+4>0, odnosno (a-2) >0, koe sekako va`i.
295. Da pretpostavime deka takvite prirodni broevi x i y postojat.
Kvadriraj}i ja ravenkata dobivame deka izrazot

x + x + x + ... + x = y 2 − x e cel broj (so 2018 koreni). Povtoruvaj}i ja


postapkata u{te 2016 pati, dobivame deka x+ x e cel broj. Neka
2
x + x = a . Toga{ x + x = a , od kade sleduva deka x = b e cel broj. Zna~i
2 2 2 2 2 2 2
b +b=a . No, b <b +b<(b+1) , a pome|u b i (b+1) ne postoi poln kvadrat na cel
broj, so {to dobivme kontradikcija.
296. x=0, y=2012. Kvadriraj}i ja dadenata ravenka, pritoa vodej}i smetka deka
y¥2012, ja dobivame ravenkata x + x = (y − 2012) 2 . Od toa {to x e cel broj
sleduva deka i x e cel broj. Ja voveduvame smenata x = t i ja razgleduvame
2 2 2
ravenkata t +t=(y-2012) =0, ~ija diskriminanta e D=1+4(y-2012) . Za ravenkata
da ima celi re{enija, potrebno e diskriminantata da bide poln kvadrat. So
2
ogled na toa deka D=1+u , kade u=2(y-2012), odnosno broj koj e za eden pogolem
od poln kvadrat, jasno e deka D ne e poln kvadrat osven za u=0. Toga{, y=2012
a−b
od kade se dobiva x=0. 297. x= ± 2 . 298. x1=0,x2=1. 299. x= . 300. x = ±2 .
2
43
301. x≥5. 302. x1 = 4 , x2 = − .
12
303. Najgolemata vrednost e 21 ; najmalata 7 . Zamenuvaj}i 4a+1 = x ≥ 0 ,
4b+1 = y ≥ 0 , 4c +1 = z ≥ 0 , dobivame deka x + y + z = 4(a + b + c) + 3 = 7. Ozna~uvame
2 2 2

S= 4a+1+ 4b+1+ 4c+1 pa S2 = x2 + y2 + z2 +2⋅ (xy+ xz+ yz) ...(1) .


Toga{ S2 = 7 + 2 ⋅ (xy + xz + yz) ≥ 7 , od kade sleduva deka S ≥ 7 (ravenstvo se
2 2 2 2
x +y x +z
postignuva na primer za x=y=0 i z = 7 ). Od toa {to xy ≤ , xz ≤ i
2 2

138
Odgovori, upatstva i re[enija
2 2
y +z 2 2 2
yz ≤ , dobivame deka xy+ xz+ yz§ x + y + z . Zna~i, od (1) sleduva deka
2
2
( 2 2 2
)
S ≤ 3 x + y + z = 3 ⋅ 7 = 21, odnosno S ≤ 21. 304. 0; 3. 305. 169.
306. a+b=13. Ravenkata se transformira vo ravenka
3
3x − 4 − 3 x − 2 = 3 7x − 8 − 3 5x − 6 .
Zamenuvaj}i 3 3x − 4 = a, 3 5x − 6 = b, 3 x − 2 = c i 3 7x − 8 = d dobivame sistem
a − c = d − b .
 3 3 3 3
a − c = d − b
Stepenuvaj}i ja prvata ravenka (so pokazatel 3), od sistemot imame
a ⋅ c ⋅ (a − c) = d ⋅ b ⋅ (d − b) , odnosno a ⋅ c = d ⋅ b . Vra}aj}i gi smenite imame
3
3x − 4 ⋅ 3 x − 2 = 3 7x − 8 ⋅ 3 5x − 6 . Od ovde ja imame ravenkata 4x2 − 9x + 5 = 0
~ii re{enija se x 1 = 1 i x 2 =
5 5 9
. Nivniot zbir e 1 + = . Zna~i odgovorot
4 4 4
na zada~ata e 13.
307. [x] =25. Ravenkata
2
x +25x+25=10x x + 1 se transformira vo ravenka
25 25 x +1 x +1 x +1
1+ + 2
=10 2
, odnosno vo 1+25· =10 . Stavaj}i smena
x x x x 2
x2
x +1 2 2 1.
=y ja dobivame ravenkata 1+25y =10 y , ~ii re{enija se y = ±
x2 5
Vra}aj}i se vo smenata dobivame ravenka x +2 1 = 1 odnosno x -25x–25=0,
2

x 25
~ie negativno re{enie se otfrla. Zna~i 25<x<26, odnosno [x ] =25. 308. 12.
309. (1,2,3); (4,8,6). Dadenoto ravenstvo se transformira vo oblikot
2
 
102n - 1 10n - 1 2  10n - 1 92
x⋅ -y⋅ =z ⋅  . Mno`ej}i ja poslednata ravenka so
9 9  9  10n − 1
 
dobivame  9x − z 2  ⋅ 10 n = 9y − 9x − z 2 …(1). Od uslovot na zada~ata postojat
 
n1∫n2, za koi e ispolneto ravenstvoto (1), odnosno 9x - z2 ⋅ 10n1 = 9y - 9x - z2 i ( )
(9x - z ) ⋅ 10 2 n2
= 9y - 9x - z2 . Odzemaj}i gi poslednite dve ravenstva, dobivame
(9x - z )⋅ (10 2 n1
- 10 n 2 = 0 ) ili
9x - z 2 = 0 ...(2) . Od (1) sleduva deka i
9y - 9x - z = 0 ...(3) . Ravenstvata (2) и (3) se ispolneti istovremeno, pa od niv
2

2
y z
se dobiva x= =   . Kako 0<x,y,z<9 dobivame deka re{enie se trojkite
2 3
(1,2,3) i (4,8,6).

139
Odgovori, upatstva i re[enija

Tema 7: Geometriski figuri vo ramnina


310. So x }e go ozna~ime rastojanieto od presekot D na pravite A1B i AB1 do
pravata p. Neka q e prava niz D, paralelna
so p koja ja se~e AA1 vo A2. Toga{ od
sli~nosta na triagolnicite A2DA1 i ABA1
A A AD a − x A 1D
imame 1 2 = 1 odnosno = ...(1).
A 1A A 1B a A 1B
Od sli~nosta na triagolnicite A1AB i DCB
A A AB a AB
sleduva 1 = 1 odnosno = 1 …(2).
DC DB x DB
Crte` 67

AA 1 B1B
Od sli~nosta na triagolnicite ADA1 i B1DB sleduva = odnosno
A 1D BD
a b
= ...(3). Od (2) i (3) sleduva a ⋅ BD = x ⋅ A1B = b ⋅ A1D odnosno
A 1D BD
A 1D x a-x x
= …(4). Od (1) i (4) sleduva = i
A 1B b a b
1 ab
ottuka x = = . Zna~i x ne zavisi od
1 1 a+b
+
a b
izborot na to~kite A i B.
Crte` 68
311. I re{enie: Od apolonievata teorema za triagolnikot ABC gi imame
 a  2   b  2 
ravenstvata b 2 + c 2 = 2   + t a 2  i a 2 + c 2 = 2   + t b 2  , odnosno
 2    2  
2 2 2 2 2 2 2 2
2b +2c -a =4ta ...(1) i 2a +2c -b =4tb ...(2). Agolot ATB e prav ako i samo ako
2 2 2
va`i ravenstvoto AS + BS = AB , od kade se dobiva ravenstvoto
2 2 2
4t a + 4t b = 9c ...(3). Zamenuvaj}i gi (1) i (2) vo (3) se dobiva baranoto
ravenstvo.
II re{enie: Neka BC =a, ΑC =b, AB =c .Neka te`i{nite linii povle~eni od
temiwata A i B se me|usebe normalni (crt. 68). Toga{ treba da doka`eme deka
a +b =5c ...(1). Od toa {to agolot kaj temeto S e prav agol imame deka:
2 2 2

2 2 2 2 2
2 2 2 b 1  2  4t + 16t a
NA = NS + AS ,   =  t b  +  t a  , b 2 = b ;
2 3  3  9
2 2 2 2 2
2 2 2 a 1  2  4t + 16t b
MB = MS + BS ,   =  t a  +  t b  , a 2 = a ;
 
2  3   3  9

140
Odgovori, upatstva i re[enija
2 2 2 2
2 2 2 2 2  4t + 4t b
AB = AS + BS , c =  t a  +  t b  , c 2 = a
2
.
 3   3  9
Od ovie ravenstva sleduva ravenstvoto (1).
2 2 2
Obratno, neka va`i ravenstvoto a +b =5c . Treba da doka`eme deka agolot
kaj temeto S e prav agol. Neka ®MSB= α toga{ ®ASB=180±- α.
2 2 2
b 1  2  1 2
Od triagolnikot NAS imame   =  t b  +  t a  − 2 t b t a cosα ,
2
   3   3  3 3
od triagolnikot MBS imame
2 2 2
a 1  2  1 2
  =  t a  +  t b  − 2 t a t b cosα ,
2 3  3  3 3
od triagolnikot ABS imame
2 2
2  2  2 2
c 2 =  t a  +  t b  − 2 t a t b cos(180 − α) .
3  3  3 3
2 2 2
Od ravenstvoto a +b =5c se dobiva
ravenstvoto 8tatbcosα=0, a ottuka cosα=0,
о
od kade dobivame deka agolot α=90 . Crte` 69
312.. Od triagolnicite ABF i AEC, (crt. 69) spored pitagorova teorema gi
2 2 2 2 2 2
dobivame ravenstvata (ta-x) =c -y , (ta+x) =b -y . Sobiraj}i gi ovie dve
2 2 2 2 2
ravenstva dobivame deka 2ta =b +c -2(x +y ) ...(1).
Primenuvaj}i ja povtorno pitagorova teorema za triagolnikot CED go
2
 a a 2
2 2
dobivame ravenstvoto x + y =   = ...(2). Zamenuvaj}i ja ravenkata (2) vo
 2 4
1 2 2 2
ravenkata (1) dobivame deka t a = ⋅ 2b + 2c − a .
2
313. Neka a+b=2p, b+c=2q i c+a=2r. Toga{ a+b+c=p+q+r, a=p-q+r, b=p+q-r,
ab bc ca a+b+c
c=-p+q+r. Zamenuvaj}i vo dadenoto ravenstvo + + =
a+b b+c c +a 2
p2 - (q - r)2 q2 - (r - p)2 r 2 - (p - q)2 1
dobivame + + = (p + q + r) , a ottuka
2p 2q 2r 2
(p - q) 2 (q - r) 2 (r - p) 2
+ + = 0 . Bidej}i p>0, q>0 i r>0 sleduva deka p=q=r, {to
r p q
zna~i a=b=c.
314. Od to~kite C i D povlekuvame pravi paralelni
so pravata p, (crt. 70) koi ja se~at stranata AB vo
to~kite N i M . Od sli~nosta na triagolnicite AB1C1
AC AN AB AN BN
i ANC sleduva = . Va`i = + .
AC 1 AB 1 AB 1 AB 1 AB 1

Crte` 70

141
Odgovori, upatstva i re[enija

Sega, BD ⋅ AC + CD ⋅ AB = BD ⋅ AN + CD ⋅ AN + CD ⋅ BN
AC1 AB1 AB1 AB 1 AB1
AN BN AN BN
(
= BD + CD ⋅ ) AB + CD ⋅
AB
AB1 AB1
= BC ⋅
...(1). + CD ⋅
1 1
Od talesova teorema za proporcionalni otse~ki za agolot CBN dobivame
CD NM
= , od kade va`i CD ⋅ BN = BC ⋅ NM …(2). Zamenuvaj}i (2) vo (1) dobivame
BC BN

BD ⋅
AC
+ CD ⋅
AB
= BC ⋅
AN
+ BC ⋅
NM
= BC ⋅
(AN + NM) = BC ⋅ AM …(3).
AC 1 AB 1 AB 1 AB 1 AB 1 AB 1

AM AD
Od talesova teorema za agolot MAD imame = . Zamenuvaj}i vo
AB 1 AD 1
ravenstvoto (3) se dobiva baranoto ravenstvo.
315. Neka x = MA1, y = MB1, z = MC1 . Toga{, ax+by+cz=2PóBMC+2PóAMC+2PóAMB
a b c a b c
=2PóABC. Sega,  + +  ⋅ 2P∆ABC =  + +  ⋅ (ax + by + cz)
 x y z  x y z
x y y z z x
= a 2 + b 2 + c 2 + ab +  + bc  +  + ca +  ≥ a 2 + b 2 + c 2 + 2ab + 2bc + 2ca
y x z y x z
pri {to ravenstvo se dostiga koga x=y=z, odnosno M e centar na vpi{anata
kru`nica vo triagolnikot ABC.
316. Ja prodol`uvame stranata AC niz A do to~ka N, taka {to AN = AK .Toga{
AN : CN = 1 : 5 = BL : CB , od kade sleduva deka AL i BN se paralelni. Neka AL
ja se~e BK vo to~ka M. Bidej}i A e sredina na otse~kata KN i AL || BN, sleduva
deka AM e sredna linija vo triagolnikot KNB. Zatoa M e sredina na BK.
317. Da pretpostavime deka triagolnikot ABC ne e ramnostran (a∫b). Bidej}i
a+ha=a+bsinγ i b+hb=b+asinγ toga{ (a-b)ÿ(1-sinγ)=0 od kade sleduva deka
sinγ=1 odnosno γ=90±. No, toga{ c e hipotenuza i c∫a, pa pri analogno
rasuduvawe dobivame deka sin β =1, odnosno β =90±, od kade se dobiva
protivre~nost na pretpostavkata.
318. a) Neka rastojanijata od to~kite A i O do pravata BC se h i h1
(crt. 71). Toga{ P∆ OBC : P∆ABC = h1 : h = OA 1 : AA 1 …(1). Analogno se dobivat
ravenstvata P∆OAC : P∆ABC = OB1 : BB1 ...(2) i P∆OAB : P∆ABC = OC1 : CC1 …(3).
Koristej}i go ravenstvoto P∆OBC + P∆OAC + P∆OAB = P∆ABC i ravenstvata (1), (2) i
(3) se dobiva baranoto ravenstvo.

142
Odgovori, upatstva i re[enija

Crte` 71 Crte` 72

b) Neka rastojanijata od to~kite B i C do pravata AA1 se db i dc (crt. 72). Toga{


P∆ ABO : P∆ACO = db : dc = BA1 : A1C ...(1). Analogno gi dobivame ravenstvata
P∆ ACO : P∆BCO = AC1 : C1B …(2) i P∆BCO : P∆ABO = CB1 : B1A …(3). Ako se pomno`at
ravenstva (1), (2) i (3) se dobiva baranoto ravenstvo .
319. Perimetarot na triagolnikot otse~en so pravata paralelna so BC e
ednakov so zbirot na rastojanijata od A do dopirnite to~ki na AB i AC so k.
Spored toa L1 + L 2 + L 3 = L . Od sli~nosta na triagolnicite se dobiva
r1 L1 r2 L2 r3 L3
= ; = ; = . Sobiraj}i gi poslednite tri ravenstva se dobiva deka
r L r L r L
r1+r2+r3=r.
320. Da pretpostavime deka trite to~ki se kolinearni. Neka S e to~ka na AC
→ →
takva {to PQ i BS se kolinearni (crt. 73).
→ →
AR AQ
Od talesova teorema imame deka = …(1)
→ →
RB QS
→ →
BP SQ
i = …(2). Od (1) i (2) dobivame
→ →
PC QC
→ → → → → → → → →
AR BP CQ AQ SQ CQ − QA QS CQ
⋅ ⋅ = ⋅ ⋅ = ⋅ ⋅ = −1 .
→ → → → → → → → →
RB PC QA QS QC QA QS CQ QA
Crte` 73
→ → →
AR BP CQ
Obratno, neka → ⋅ ⋅ = −1 . Da pretpostavime deka presekot na PQ i AB e
→ →
RB PC QA
→ → → → → → →
AR' BP CQ AR' PC QA AR
to~ka R’. Toga{ → ⋅ → ⋅ → = −1 , ottuka → = − → ⋅ → = → . Zna~i, R’=R od
R' B PC QA R' B BP CQ RB
kade zaklu~uvame deka to~kite P, Q i R se kolinearni.

143
Odgovori, upatstva i re[enija
321. Neka R i Q se sredini na otse~kite AC i AB, R –centarot na kru`nicata
AC BC
k1, a= , b= , x-radius na k1, y-radius na
2 2
k2 (crt. 74). Jasno e deka PR =a+x, QR =a+b-x
i PQ =b. Vo triagolnikot PQR ja
povlekuvame visinata RH. Od toa {to
rastojanieto od R do CD e h sleduva deka
PH = PC − CH =a-x. Zna~i, QH = QP − PH
=|b+x-a|.
Od triagolnikot PQR go dobivame ravenstvoto
2 2 2 2 ab
(a+x) –(a-x) =(a+b-x) –(b+x-a) od kade x = . Crte` 74
a+b
ab
Analogno za radiusot y na kru`nicata k2 se dobiva izrazot y = . Zna~i
a+b
kru`incite k1 i k2 imaat ednakov radius.
322. Spored teoremata na Ptolomej za ~etiriagolnikot ABCD va`i
ravenstvoto: AP ÿ BC = AB ÿ CP + AC ÿ BP ...(1). Od toa {to AB = BC = AC , po
kratewe na ravenstvoto (1), se dobiva baranoto ravenstvo.
323. Neka HLM e ramnostran triagolnik. Toga{
MH = ML , pa L le`i na kru`nicata opi{ana
okolu triagolnikot ABH, odnosno ®ALB=90±
(crt. 75). Toga{ ®LHA=®LBA i ®LBH=®LAH.
Bidej}i ®LBA=®LBH sleduva ®LHA=®LAH,
odnosno triagolnikot AHL e ramnokrak. Od
LA = LH = LM = AM sleduva deka triagolnikot
1
LAM e ramnostran, pa ®LHA=60±. Od ®LAH=
2
®LMH (zo{to?) sleduva ®LAH=30±, pa ®HAB=30±. Crte` 75
Toga{, ®ABH=90±-30±=60±. Zna~i, ®BAC=®CBA=60±, pa ABC e ramnostran
triagolnik.
324. Bidejki G e centarot na kru`nicata okolu triagolnikot CDA imame deka
AG = DG = CG. Od isti pri~ini
AE = EB = EC . No od toa {to AG = CG i
AE = EC sleduva deka E i G se na isto
rastojanie od temiwata A i C. Zna~i
EG le`i na simetralata na stranata
AC. No P ∈ EG zna~i AP = CP .
Analogno se doka`uva deka BP = DP .
325. Neka ®BOA=α, ®OAB=β i ®OAC=β’
(crt. 76).

Crte` 76

144
Odgovori, upatstva i re[enija

Od teoremata za vnatre{en agol imame ®O2O1A=®O1OA+®O1AO


α+β 1
= = ®CBA…(1).
2 2
Od toa {to AO2 e simetrala na agolot CAD dobivame ®AO2O1=®O2AD-®O2OA
1 α 1 1
= (180±-β’)- = (180±-β’-α)= ®BCA…(2).
2 2 2 2
Od (1) i (2) sleduva ®CBA=®BCA, odnosno triagolnikot ABC e ramnokrak.

326. Jasno e deka AP = AN (tangentni


otse~ki). Od AP + AN = AC + CP + AB + BN
= AC + CA 1 + AB + BA 1 = AB + AC + BC = 2s ,
sleduva deka AP = AN = s . Sli~no,
BM = BQ = s . Od AN = BM sleduva BN = AM ,
odnosno AB1 = BA1 ...(1) (crt. 77).

Crte` 77
Analogno se dobivat ravenstvata BC = CB ...(2) i AC = CA ...(3). Od (1), (2)
1 1 1 1

AC BA CB
1 1 1
i (3) se dobiva ⋅ ⋅ = 1.
CB A C B A
1 1 1
Spored teoremata na ^eva sleduva deka
pravite AA1, BB1, CC1 se se~at vo edna to~ka
.
327. Upatstvo: Koristete ja teoremata na
^eva. Crte` 78
329.Upatstvo: ®EOD=120°± i O e sredina na lakot
ED od kru`nicata opi{ana okulu to~kite E, O, D,
B (crt. 78).
330. Upatstvo: Primenete ja talesova teorema za
proporcionalni otse~ki.
334. Neka to~kite N, Q i M se podno`jata na
visinite ha, hb i hc spu{teni od temiwata A, B i
C (crt. 79).
Od sli~nosta na triagolnicite BCQ i CA1B
2R 1 a a2
dobivame = odnosno R1 = …(1).
a hb 2hb

Crte` 79

145
Odgovori, upatstva i re[enija
Analogno, od sli~nosta na triagolnicite CAM so AB1C i ABN so BC1A gi
b2 c2
dobivame ravenstvata R 2 = …(2) i R 3 = …(3). Mno`ej}i gi
2h c 2h a
a 2b 2 c 2 a3b 3c 3
ravenstvata (1), (2) i (3) dobivame R 1R 2 R 3 = =
8h a h b h c ah bh ch
64 ⋅ a ⋅ b ⋅ c
2 2 2
a 3b 3 c 3 (abc)3
= 3
= 3
= R3 .
64P (abc)
64 ⋅
64R 3
336. Neka AO ja se~e k1 vo to~ka G, a BO ja se~e k2 vo to~ka F i neka pravata AB
gi se~e k1 i k2 vo to~kite D, odnosno E (crt. 80).
Od skladnosta na triagolnicite ADG i BEF
sleduva AD = BE . Od ednakvosta na
perifernite agli nad isti lak sleduva pak deka
®AMD=®AGD=γ=®EFB=®ENB=®ACB.
Zna~i, pravite DM i BC se paralelni od kade
sleduva deka ®MDA=®NBE (kako soglasni agli).
Zaklu~uvame deka triagolnicite ADM i EBN se
skladni pri {to rastojanijata od to~kite M i N
do pravata AB se ednakvi.
Crte` 80 Spored toa, pravite MN i AB se paralelni.
337. P=r·R. Od triagolnicite ABO1, ACO1 i BO1C (crt. 81) dobivame deka
c ⋅r b ⋅r a ⋅r
P = P∆ABO + P∆ACO − P∆BO C = + − = (s − a) ⋅ r
1 1 1 2 2 2
...(1). Jasno e deka BQ = a + R . Od druga strana
BQ = BM i BQ + BM = BC + CO + BA + AM
= BC + CB1 + BA + AB 1 = 2s , pa BQ = s . Zna~i,
s = a + R , odnosno s − a = R . Ottuka, zaedno so (1),
sleduva P=r·R.
Crte` 81
3 2 a 2 2 2 b2 2
339. . Upatstvo: Da se koristi deka 2t a = b 2 + c 2 − , 2t b = a + c − ,
4 2 2
2 c2
2t c = a2 + b2 − .
2
5
342. cm. Neka CM e te`i{nata linija, CS
2
simetralata na agolot, a CV visinata
spu{tena od temeto kaj praviot agol C vo
pravoagolniot triagolnik ABC (crt. 82). Neka
O e centarot na vpi{anata kru`nica i K e
proekcijata na O vrz hipotenuzata.

Crte` 82

146
Odgovori, upatstva i re[enija

a⋅b a + b + c
Jasno, c=5cm. Od P = = ⋅ r dobivame deka r = OK = 1cm . Isto
2 2
a⋅b c ⋅h 12
taka, od P = = sleduva h = cm .
2 2 5
c 5
U{te MA = MC = MB = = cm . Od pravoagolniot triagolnik CMV dobivame
2 2
2 2 2 25 144 49 7
VM = CM − CV = − = , odnosno VM = cm . Spored zada~a
4 25 100 10
340, CS e simetrala na agolot VCM i zatoa VC : VS = CM : MS , pa ottuka
VC ⋅ MS 24  24  7
VS = = MS . Toga{, od VS + SM = VM sleduva 1 +  ⋅ MS = ,
CM 25  25  10
5
odnosno MS = cm . Od sli~nosta na triagolnicite VCS i KOS imame
14
24 5
VS VC ⋅ ⋅1
VS ⋅ OK 1
= , pa KS = = 25 14 = cm .
KS OK VC 12 7
5
2
1 5 
Toga{, OM = (KS + SM) 2
+ OK
2
=  +  + 12 =
5
cm .
 7 14  2
MA1 MB1 MC1
344. Neka = x, = y, = z . Od zada~a 318 a) sleduva deka
AA1 BB1 CC1
MA 1 MB1 MC1 x+y+z 1 1
+ + = x + y + z = 1 i zatoa 3 xyz ≤ = , odnosno xyz ≤ .
AA 1 BB1 CC1 3 3 27

1
Pa, najgolema vrednost se dostignuva koga x=y=z= odnosno
3
MA 1 MB1 MC1 1 {to zna~i deka to~kata M e te`i{te vo triagolnikot
= = =
AA 1 BB1 CC1 3

ABC.
2 2
345. Od uslovot na zada~ata imame AB = 2 ⋅ AC (crt. 83).
Prodol`uvaj}i gi otse~kite NA i NB do
presek so CD gi dobivame to~kite K i M,
KC CD DM
soodvetno. Nao|ame deka = = ...(1) .
AE EL LB
KC AC
Od AKC ≅ MBD imame = i ottuka
BD DM
2
KC ⋅ DM = AC ⋅ BD = AC .
Crte` 83

147
Odgovori, upatstva i re[enija
2 2 2
Ponatamu, 2KC ⋅ DM = 2 AC = AB = CD ...(2) . Od odnosot (1) zamenet vo (2)
2
dobivame 2 ⋅ AE ⋅ BL = EL . Bidej}i AL + BE = AB + EL imame
2 2 2 2 2
AL + BE + 2AL ⋅ BE = AB + EL + 2AB⋅ EL = AB + 2AE⋅ BL + 2AB⋅ EL
2
= AB + 2( AE ⋅ BL + AB ⋅ EL ) .
Od AL ⋅ BE = ( AE + EL) ⋅ (BL + LE) = AE ⋅ BL + ( AE + EL + LB) ⋅ EL = AE ⋅ BL + AB ⋅ EL ,
2 2 2
sleduva deka AL + BE = AB .
7 10
346. . Neka R e na BC taka {to AR e simetrala na aglite BAC i PAQ.
75
AP PR 35x 35
Neka x = = toga{ imame deka PR = i RQ = . Sega, neka
AQ RQ x +1 x +1
AB
y= i neka B’ e simetri~na to~ka na B vo odnos na AR i P’ simetri~na
AC
to~ka na P vo odnos na AR. Bidej}i BP = B' P' , PR = P' R spored teoremata na
Menelaj primeneta za triagolnikot CRB’ i pravata P’Q se dobivaat
CQ RP' B' A CQ RP BA
ravenstvata 1 = ⋅ ⋅ = ⋅ ⋅ .
QR P' B' AC QR PB AC
35x
100 x + 1 100xy
Imame 1 = ⋅ ⋅y = , odnosno 100xy=21 …(1).
35 21 21
x +1
Od teoremata za simetrala na agol imame
35x
21 +
AB BR x + 1 = 56x + 21 ,
y= = =
AC RC 100 + 35 100x + 135
x +1
100xy + 135y = 56x + 21 , odnosno
135y = 56x ...(2) . Zamenuvaj}i (1) vo (2) go
AB 56 ⋅ 21 7 10
dobivame odnosot = = .
AC 100 ⋅ 135 75
n a ⋅ a nb ⋅ b n c ⋅ c
n a nb n c
347. + + = 2 + 2 + 2 Crte` 84
h a hb h c h a ⋅ a hb ⋅ b h c ⋅ c
2 2 2
P∆BCO P∆CAO P∆ABO P∆ABC
= + + = = 1 (crt. 84).
P∆ABC P∆ABC P∆ABC P∆ABC
1 a 1 s-a
348. Upatstvo: Koristi gi ravenstvata = i = .
ha 2rs ra rs

148
Odgovori, upatstva i re[enija

ra a r b r c r r r
v) = , b = , c = . Toga{ a + b + c
ha 2(s - a) h b 2(s - b) h c 2(s - c) ha hb hc

abc
a b c abc 4R
= + + ≥ 3⋅3 = 3⋅3
2(s - a) 2(s - b) 2(s - c) 8(s − a)(s − b)(s − c) 8(s − a)(s − b)(s − c)
4Rs
PRs Rs R R
= 3⋅3 2
= 3⋅3 = 3⋅3 . Koristej}i deka R¥2r, dobivame deka 3 ⋅ 3 ≥3
2P 2P 2r 2r
r r r
i zatoa a + b + c ≥ 3 .
ha hb hc
349. Imame deka ®ABD=®ACD=90±. Neka F e sredina na
otse~kata BE (crt. 85). Bidej}i BF = FE , imame deka i
®AFB=90±. No ®CBA=®CDA od kade sleduva deka
AC CD DA
óACD~óAFB. Taka = ...(1). Od teoremata
=
AF FB BA
na Ptolomej za ~etiriagolnikot ABCD dobivame
AD ⋅ BC = AC ⋅ BD + CD ⋅ AB . Crte` 85
Toga{, BA ⋅ BC = AF ⋅ BD + BF ⋅ AB , AF ⋅ BD = AB ⋅ (BC − BF) . Ottuka,
AF ⋅ BD = AB ⋅ CF koe e ekvivalentno so ravenstvoto AF ⋅ BD = AB(FE + CE ) ,
odnosno AF ⋅ BD = AB ⋅ FB + AB ⋅ CE . Od (1) sleduva AC ⋅ BD = AB ⋅ CD + AD ⋅ CE .
350. 20± .
CM AC
351. Spored talesova teorema voo~uvame deka = odnosno
BQ AC + CQ
BQ CQ
= 1+ . Od toa {to óACP~óQCB sleduva
CM AC
CQ BQ BQ BQ
deka = , odnosno = 1+ . Delej}i
AC AP CM AP
go poslednoto ravenstvo so BQ go dobivame
baranoto ravenstvo. 352. 75±.
353. 180±. ^etiriagolnicite AC1A1C, ABA1B1 i
C1BCB1 (crt. 86) se tetivni ~etiriagolnici od
kade imame deka ®A1C1C=®A1BB1=90±-γ,
®C1CA1=®BB1A1=90±-β.
Crte` 86
Spored toa óCC1A1~óB1BA1 od kade sleduva sli~nost na triagolnicite A1C2C1
i A1B2B (A1C2 i A1B2 se te`i{ni linii vo óA1C2C1 i óA1B2B), pa se dobiva deka
®C2A1C1=®B2A1B=φ. Spored toa, ®C2A1B2=®C2A1B-®B2A1B=(α+φ)-φ=α…(1).

149
Odgovori, upatstva i re[enija
Od óC1CB1~óAA1B1 sleduva deka óAB1A2~óC1B1A2, od kade dobivame deka
®AB1A2=®C1B1C2=ξ. Spored toa, ®A2B1C2=®AB1C2-®AB1A2=(β+ξ)-ξ=β …(2).
Od óBB1C1~óA1AC1 sleduva deka óBB2C1~óA1A2C1, od kade dobivame deka
®BC1B2=®A1C1A2=µ. Spored toa , ®B2C1A2=®BC1A2-®A1C1A2=(γ+µ )-µ=γ...(3).
So sobirawe na ravenstvata (1), (2) i (3) se dobiva baranoto ravenstvo.
→ → → 2 → → → 2 1 → → → →
354. Od AT = AP+ PT , AA 1 = AP+ PT , ( ( AB+ AC)) = AP+ PT sleduva
3 3 2
1 → → → →
( AB+ AC) = AP+ PT …(1). Analogno se dobivaat ravenstvata
3
1 → → → → 1 → → → →
(BA+ BC) = BP+ PT ...(2), (CA+ CB) = CP+ PT ...(3). Sobirajki gi (1), (2) i (3)
3 3
→ → → → → → → → →
dobivame 0 = AP+ BP+ CP+ 3 PT odnosno PA+ PB+ PC = 3 PT .
→2 →2 →2 → → → → → → →2
Toga{ PA + PB + PC + 2 PA ⋅ PB + 2 PA ⋅ PC + 2 PB⋅ PC = 9 PT , odnosno
2 2 2 2
PA + PB + PC + 2 PA ⋅ PB ⋅ cosϕ1 + 2 PA ⋅ PC ⋅ cosϕ 2 + 2 PB ⋅ PC ⋅ cosϕ 3 = 9 PT ...( 4) .
Od kosinusna teorema za triagolnicite ABP, ACP i BCP imame:
2 2 2
2PA ⋅ PB ⋅ cos ϕ 1 = PA + PB − AB
2 2 2
2PA ⋅ PC ⋅ cosϕ 2 = PA + PC − AC
2 2 2
2PB ⋅ PC ⋅ cosϕ 3 = PB + PC − BC .
Zamenuvaj}i gi ovie ravenstva vo (4) imame
2 2 2 2 2 2 2 2 2 2 2 2 2
PA + PB + PC + PA + PB − AB + PA + PC − AC + PB + PC − BC = 9 PT , odnosno
2 2 2 2 2 2
3 (PA + PB + PC 2 ) = 9 PT + AB + AC + BC i ottuka
2 2 1 2 2 2 2 2
PA + PB + PC = 3 PT + ( AB + AC + BC ) .
3
355. Da gi ozna~ime so K i L sredinite na
dijagonalite AC i BD, soodvetno, so M
sredinata na stranata DC i so E i F
prese~nite to~ki na stranite AD i BC,
odnosno AB i CD, soodvetno. ]e doka`eme
1
deka PóKLE = PABCD. (crt. 87). Imame
4
1 1
PDKLC=PóDKC+PóCKL= PACD+ PóACL
2 2
1 1
= PóACD+ [PóABC–(PóABL+PóCBL)].
2 2

Crte` 87

150
Odgovori, upatstva i re[enija

1 1 1
Bidej}i PóALB= PóALD , PóCBL = PóCDL, sleduva PDKLC= PóACD+ [PóABC– PABCD]
2 2 2
1 1 1 1
= PABCD– PABCD = PABCD. Zna~i, PDKLC= PABCD ...(1). Od toa {to otse~kata
2 4 4 4
MK e sredna linija za triagolnikot ACD sleduva PóDKE = PóDME , a od ML e
sredna linija za triagolnikot DBC sleduva PóCLE = PóCME.
Zatoa PóDKE + PóCLE = PóDCE.
Koristej}i go ravenstvoto (1) dobivame
PóKLE=PEDKLC–PóDKE–PóCLE
1 1
=PEDKLC–PóDCE=PDKLC= PABCD. Analogno se doka`uva deka PóKFL= PABCD. Zna~i
4 4
PóKLE=PóKLF a ottuka EE1 = FF1 , kade {to EE 1 e rastojanieto od E do pravata

KL, a FF1 e rastojanieto od F do pravata KL.


Neka P e sredina na otse~kata EF. Toga{ od EP = FP , EE1 = FF1 i
®EE1P=®FF1P=90± sleduva óEE1P@óFF1P, pa ®F1PF=®E1PE i poradi
kolinearnosta na F,P i E sleduva F1,P i E, se
kolinearni odnosno to~kite P, K i L se kolinearni.
356. Neka normalite se se~at vo to~kata P (crt. 88).
Lesno se dobivaat ravenstvata
2 2 2 2 2 2 2 2
AC 1 − C1B = AP − PB , BA 1 − A 1C = PB − PC i
2 2 2 2
CB 1 − B1A = PC − AP . Sobiraj}i gi poslednite
ravenstva go dobivame baranoto ravenstvo
2 2 2 2 2 2
AC1 + BA 1 + CB1 = B 1A + C1B + A1C …(1). Crte` 88

Neka za nekoi to~ki A1, B1, C1 od stranite na triagolnikot ABC e ispolneto


ravenstvoto (1). Od A1 i B1 izdigame normali koi se se~at vo P. Od to~kata P
spu{tame normala PX kon stranata AB. Toga{ va`i ravenstvoto
2 2 2 2 2 2
AX + BA 1 + CB1 = B1A + XB + A 1C ...(2)
2 2 2 2
Od (2) go odzemame (1) i dobivame: AX - AC1 = XB - C1B ,
2 2 2 2
AX - XB = AC1 - C1B . Od AX + XB = AB i AC 1 + C1B = AB , sleduva AX - XB

= AC 1 - C1B , pa AX - AC 1 = XB - C1B . Poslednoto ravenstvoto e mo`no samo


ako to~kite X i C1 se sovpa|aat.
357. Od P∆AOB+P∆BOC+P∆COA=P∆ABC (crt. 89), sleduva
1 (cd +ad +bd )= 1 (a+b+c)·r ...(1).
1 2 3
2 2
Primenuvaj}i ja teoremata na Ptolomej za
~etiriagolnicite OM3AM1, OM1BM2 i OM2CM3
imame: d1· AM3 + d3· AM1 = M 1M 3 ·R ...(2),
Crte` 89

151
Odgovori, upatstva i re[enija

d1· BM2 + d2 BM 1 = M 1M 2 ·R ...(3),


d2· CM3 + d3· CM 2 = M2 M3 ·R ...(4).
Sobirajki gi (1), (2), (3) i (4) imame:
c a b
d1·( AM3 + BM2 + )+d2·( BM 1 + CM3 + )+d3·( AM1 + CM2 + )
2 2 2
a+b+c
=( M 1M 3 + M 1M 2 + M2 M3 )·R+   ·r,
 2 
b a c c b a c a b
d1·( + + )+d2·( + + )+d3·( + + )
2 2 2 2 2 2 2 2 2
a b c
=( + + )·(R+r) i zatoa d1+d2+d3=R+r.
2 2 2

Crte` 90

358. Neka D, E, i F se podno`nite to~ki na normalite spu{teni od to~kata M


na opi{anata kru`nica na triagolnikot ABC do negovite strani (crt. 90).
O~igledno e deka DF i EF }e le`at na edna prava ako ®DEB=®AEF ...(1). ]e go
doka`eme ravenstvoto (1). Okolu ~etiriagolnikot AMEF mo`e da se opi{e
0
kru`nica bidej}i ®AEM =®AFM=90 . Zna~i, ®AEF=®AMF...(2). Analogno, okolu
~etiriagolnikot MEBD mo`e da se opi{e kru`nica, bidej}i ®MFC=®MDC=90±.
Sleduva deka ®DEB =®DMB ...(3).
Imame ®AMF =®AMB -®FMB =(180±-ϕ) -®FMB; ϕ= =®ACB.
Sli~no, ®DMB =®DMF-®FMB =(180±-ϕ)-®FMB, bidej}i MFCD e tetiven
~etiriagolnik. Zna~i ®AMF=®DMB ...(4).
Zamenuvaj}i (2) i (3) vo ravenstvoto (4) dobivame
deka ®AEF=®DEB {to treba{e da se doka`e.
Zna~i, D, E i F le`at na edna prava.
359. Neka dc e normalata spu{tena od centarot
na opi{anata kru`nica kon stranata c, a hc e
visinata spu{tena kon c (crt. 91).
dc OP CP − R
Toga{, = = , odnosno
hc CP CP
R hc − dc P∆ABC − P∆ABO
= = …(1).
CP hc P∆ABC
Crte` 91
R P∆ABC − P∆BCO R P∆ABC − P∆CAO
Sli~no, = …(2) i = …(3). So sobirawe na (1),
AM P∆ABC BN P∆ABC
R R R 3P∆ABC − P∆ABC
(2) i (3) dobivame + + = i ottuka sleduva baranoto
CP AM BN P∆ABC
ravenstvo.

152
Odgovori, upatstva i re[enija
360. Neka C1 e sredinata na stranata AB, a B1 e
sredinata na stranata AC. Toga{
c b
MC1 = AC1 = B1N = i PB1 = B1A = NC1 = . U{te,
2 2
®MC1N=90±+α=®NB1P (crt. 92). Zatoa,
óMNC1~óN PB1 i ottuka MN = NP .
Za agolot PNM dobivame:
®PNM=®PNB1+®B1NC1+®C1NM=®PNB1+α+®B1PN
=180±-®PB1N+α=180±-(90±+α )+α=90±.

Crte` 92

361. Neka V e centarot na vpi{anata, a O centarot na opi{anata kru`nica


okolu triagolnikot ABC (crt. 93). Treba da doka`eme deka OO1 = OO2 = OO3 .
Od ⊄CAV=⊄O3O2O (kako agli so zaemno
normalni kraci), ⊄BAV=⊄O2O3O (kako agli so
zaemno normalni kraci) i ⊄CAV=⊄BAV sleduva
⊄O3O2O=⊄O2O3O. Zna~i triagolnikot O2OO3 e
ramnokrak pa OO 2 = OO 3 . Analogno se doka`uva
deka triagolnikot O3OO1 e ramnokrak, od kade
sleduva deka OO1 = OO3 . Zatoa,
OO1 = OO2 = OO3 .

Crte` 93

362. Neka AC 2 = A 2 C 2 = x i C3 A1 = x1 , kade {to CC2 i C1C3 se visinite vo


triagolnicite ABC i A1B1C1, soodvetno. Od pravoagolnite triagolnici B1C1C3
i A1C1C3 (crt. 94) imame: a1²-(c1+x1) ²=b1²-x1², odnosno a1²=b1²+c1²+2x1c1 ...(1).
Triagolnicite A2CC2 i A1C1C3 se sli~ni, so koeficient na sli~nost
A C
k 1 = 2 . Triagolnicite BCA2 i B1C1A1 se sli~ni so koeficient na sli~nost
A 1C1
CA 2
k2 = . Zna~i, k1=k2=k pa va`at ravenstvata x = C 2 A 2 = k C 3 A 1 = kx 1 ,
C 1A 1
b = CA 2 = k C1A 1 = kb 1 ...(2) (od sli~nosta na triagolnicite A2CC2 i A1C1C3) i
a = CB = k C1B1 = ka 1 , c - 2x = A 2B = k A 1B1 = kc 1 ...(3) (od sli~nosta na
triagolnicite BCA2 i
B1C1A1). Mno`ej}i go
ravenstvoto (1) so k i
koristej}i gi (2) i (3)
dobivame:
2 2 2
ka1 =kb1 +kc1 +2kx1c1,
a·a1=b·b1+(c-2x)·c1+2xc1,
a·a1=b·b1+c·c1.
Crte` 94

153
Odgovori, upatstva i re[enija
363. Upatstvo: Triagolnikot AHC e sli~en so triagolnikot NOM.
364. Upatstvo: Iskoristi gi zada~ite 363 i 357.
365. Ja povlekuvame otse~kata VZ (crt. 95). Da
zabele`ime deka AC : CV =3:2= BC : ZC od kade
sleduva sli~nost na triagolnicite ABC i VZC (spored
priznakot SAS). Zna~i AB : VZ =3:2 od kade sleduva
deka VZ e paralelna so AB, pri {to se dobiva
sli~nost na triagolnicite ZVP i ABP, od kade sleduva
ednakvost na aglite VZP i BAZ.
Crte` 95
Od taa sli~nost sleduvat odnosite AP : PZ = AB : VZ =3:2.
2 5
Spored toa AZ = AP + PZ = AP + AP = AP , od kade dobivame AZ : AP =5:3.
3 3
Na sli~en na~in se dobiva deka AW : AR =5:3. Spored toa triagolnikot ARP e
sli~en so triagolnikot AWZ (SAS) i agolot ARP e ednakov so agolot AWZ,od
kade dobivame deka pravata RP e paralelna so pravata BC. Analogno, se
dobiva deka triagolnikot BQP e sli~en so triagolnikot BUV, a triagolnikot
CRQ e sli~en so CXY. Ottuka sleduva deka QP i CA se paralelni kako i RQ i
AB. Zna~i, stranite na triagolnicite ABC i PQR se paralelni pa zatoa tie se
sli~ni.
366. Koristej}i ja formulate za plo{tina na triagolnik go dobivame
ravenstvoto a ⋅ AA 1 b ⋅ BB 1 c ⋅ CC 1
+ + = 3P , kade P e plo{tinata na
2 2 2
triagolnikot ABC (crt. 96). Od ova ravenstvo sleduvat ravenstvata
( ) ( ) (
a ⋅ 2R1 + HA1 b ⋅ 2R2 + HB1 c ⋅ 2R3 + HC1
+ + = 3P
)
2 2 2
2aR1 + 2bR 2 + 2cR3 a ⋅ HA1 + b ⋅ HB1 + c ⋅ HC1
+ = 3P, aR 1 + bR 2 + cR 3 + P = 3P.
2 2
aR1 + bR 2 + cR3
Neka P∆AC B = P1 , P∆C BA = P2 i P∆A CB = P3 . ⇒ = 1...(1)
1 1 1 1 1 1
2P
Od sli~nosta na triagolnicite AC1B1, C1BA1, A1CB1 so triagolnikot ABC
2 2
P1 R1 R
sleduvat ravenstvata = 2 , P1 = 12 ⋅ P ...(2);
P R R
2 2
P2 R 2 R
= 2 , P2 = 22 ⋅ P ...(3) i
P R R
2 2
P3 R3 R
= 2 , P3 = 32 ⋅ P ...(4). Od istite sli~nosti
P R R
sleduvaat i ravenstvata B1C1 =
a
⋅ R1 ...(5),
R
b c
C1A 1 = ⋅ R 2 ...(6) i A 1B1 = ⋅ R 3 ...(7).
R R
Crte` 96

154
Odgovori, upatstva i re[enija
Koristej}i gi ravenstvata (5), (6), (7) i (1) dobivame
B C + C1A 1 + A 1B 1 aR 1 + bR 2 + cR 3 aR 1 + bR 2 + cR 3 P P
P∆A 1B1C1 = 1 1 ⋅r = ⋅r = ⋅ ⋅ r = ⋅ r ...(8).
2 2R 2P R R
Od ravenstvoto P1 + P2 + P3 + P∆A B C = P , koristej}i gi pritoa ravenstvata (2),
1 1 1

2 2
R1 R2 R32 P
(3), (4) i (8) imame
2
⋅P + 2
⋅P + 2
⋅P + ⋅r = P . Mno`ej}i go poslednoto
R R R R
R2 2 2 2 2 2 2 2
ravenstvo so sleduva R1 +R2 +R3 +R·r=R odnosno R1 +R2 +R3 =R·(R-r)
P
{to treba{e da se doka`e .

367. Konstruirame prava niz H paralelna so pravata BD, koja ja se~e CB vo K a


CD vo M (crt. 97). Od sli~nosta na triagolnicite EHK i EDB sleduva deka
EH EK AD EK
= i zatoa va`i = .
HD KB DB KB

Ottuka, sleduva deka AE i DK se paralelni pravi. Vo triagolnikot DKC,


to~kata H e
prese~na to~ka
na visinite DE
i KM. Zatoa, CH
e normalna so
DK, odnosno CH
e normalna so
AE.
Crte` 97 Crte` 98
β β
368. Od toa {to ®BMN=®BNM=90±- , sleduva ®AMN=90±+ (crt. 98). Od
2 2
α+γ  β β
triagolnikot AOC sleduva deka ®AOC=180±- =180±-  90 o −  = 90°+ .
2  2 2

Zatoa triagolnicite AOC i AMP se sli~ni. Od sli~nosta imame deka


AO CA AO AM
= odnosno . =
AM AP CA AP
Koristej}i go poslednovo ravenstvo i toa {to
α
®OAM=®OAC= , se dobiva deka óAMO~óAPC,
2
odnosno ®APC=®AMO=90±. Zna~i, CP i sa se
zaemno normalni.
Crte` 99

369. Jasno e deka ®AHC1=β i ®BHC1=α, kade {to C1 e podno`jeto na visinata


povle~ena od temeto C (crt. 99). Ako R1 e radiusot na opi{anata kru`nica
okolu triagolnikot ABH, toga{, za triagolnikot ABH va`i

155
Odgovori, upatstva i re[enija

c c c
2R1 = = = . Od
( o
sin(α + β ) sin 180 − γ )
sinγ
druga strana, za triagolnikot ABC va`i
c
= 2R i zatoa R1=R. Analogno, se
sinγ
doka`uva deka R2=R3=R, kade {to R2 i R3 se
radiusite na opi{anite kru`nici okolu
triagolnicite BCH i CAH, soodvetno.
370. 60º i 120º Upatstvo: Doka`i deka
~etiriagolnikot ABCD e romb (crt. 100). Crte` 100

371. Definirame rotacija so centar vo O i agol COA, pri koja to~kata C se


preslikuva vo A. Neka M se preslikuva vo K. Analogno, pri rotacija so centar
vo O i agol COB, C se preslikuva vo B. Neka to~kata N se preslikuva vo L.
Toga{ triagolnicite AOK i COM se skladni (spored
priznakot SAS). Na sli~en na~in se sli~ni i
triagolnicite BOL i CON (crt. 101). Bidej}i OK = OM,
OL = ON i ®KOL=®AOB-(®AOK+®LOB)
=®AOB-®MON=®MON, sleduva deka se skladni i
triagolnicite KOL i MON odnosno KL = MN.
Zna~i, LóMCN = CM+ MN + NC = AK + KL + LB ≥ AB.

Crte` 101

372. Neka bisektrisata CD ja se~e otse~kata EF vo


to~kata O (crt. 102). So m i n gi ozna~uvame
rastojanijata od to~kata O do stranite AC i AB
soodvetno. Za da doka`ime deka O e to~kata na
presek na simetralite na aglite vo triagolnikot
ABC, dovolno e da poka`eme deka m=n. Bidej}i CD
e simetrala sleduva deka rastojanijata od O do AC i
BC se ednakvi. Od uslovot na zada~ata imame CE +
CF = EA + AB + BF , od kade dobivame:
Crte` 102
1 1
PDCEF =PDCOE+PDCOF = m( CE + CF )= m( EA + AB + BF )
2 2
1
=PDAOE+ ÿ AB m+ PDBOF ...(1). Od druga strana imame:
2
1
PDCEF=PEABF=PAOE+PDAOB+PDBOF=PDAOE+ ÿ AB ÿn+PDBOF ...(2).
2
Od ravenstvata (1) i (2) se dobiva deka m=n.
373. Neka simetralata na agolot γ ja se~e opi{anata
kru`nica vo to~ka M (crt. 103). Crte` 103

156
Odgovori, upatstva i re[enija

γ
Toga{ MV = MA = 2R ⋅ sin …(1).
2
Od triagolnikot VPC ( VP = r ), dobivame deka
r
VC = ...(2). Mno`ej}i gi (1) i (2) imame deka
γ
sin
2
va`i ravenstvoto MV ⋅ VC = 2Rr . Neka pravata OV ja
se~e opi{anata kru`nica vo to~kite K i L. Toga{,
MV ⋅ VC = KV ⋅ VL od kade {to 2Rr = R + OV ⋅ R - OV ( )( )
…(3). Od ravenstvoto (3) se dobiva baranoto
ravenstvo. Da zabele`ime deka va`i neravenstvoto
R¥2r.
Crte` 104

374. Imaj}i go predvid stepenot na M, N i P vo odnos na opi{anata kru`nica


(crt. 104) imame:
AM ⋅ AM1 + BN ⋅ BN1 + CP ⋅ CP1
( ) (
= AM ⋅ AM + MM1 + BN ⋅ BN + NN1 + CP ⋅ CP + PP1 ) ( )
2 2 2
= AM + AM ⋅ MM1 + BN + BN ⋅ NN1 + CP + CP ⋅ PP1
2 2 2
= AM + MC ⋅ MB + BN + NA ⋅ NC + CP + PA ⋅ PB
a2 b2 c2
b2 + c 2 − 2 a2 + c 2 − 2 a2 + b2 − 2
= 2 +a + 2 +b + 2 +c
2 4 2 4 2 4
= a2 + b2 + c 2 .
375. Neka H ja deli AA1 na polovina, a BB1 vo odnos 2:1 (crt. 105). Od
sli~nosta na triagolnicite АА1В i СС1В sleduva AA 1 :с= CC 1 :а od kade
1
imame a· AA 1 =c· CC 1 , a·2 AH =c· CC 1 odnosno a· AH = c· CC 1 ...(1). Od
2
sli~nosta na triagolnicite АС1С i АВВ1 imame BB 1 :c= CC 1 :b od kade imame
b· BB 1 = c · CC 1 , odnosno
3BH
b·( BH + B 1H )=c· CC 1 , b· =c· CC 1 ...(2).
2
Bidej}i P∆ABH = P∆BA H i P∆ACH = P∆HA C , imame
1 1

P∆ABH + P∆ACH = P∆BA H + P∆HA C = P∆BCH , a ottuka


1 1

c ⋅ C1H b ⋅ B1H a ⋅ A 1H
, odnosno c ⋅ C1H + b ⋅
BH
+ = = a ⋅ AH .
2 2 2 2

Crte` 105

157
Odgovori, upatstva i re[enija
Zamenuvaj}i gi ravenstvata (1) i (2) vo poslednoto ravenstvo dobivame
1 1
c ⋅ C1H + c ⋅ CC 1 = c ⋅ CC 1 , odnosno 6C1H = CC1 . Toga{, 6C1H = CH + HC1 , i
3 2
ottuka sleduva CH : C 1H = 5 : 1 .
376. Neka Н1, Н2 i Н3 se ortocentrite na triagolnicite АНCНB, ВНАНС i СНBНA
soodvetno, a Н ortocentarot na triagolnikot АВС (crt. 106) .

]e poka`eme deka
to~kite М1, М2 i М3
koi se sredini na
otse~kite HBHC, HAHC i
HAHB soodvetno, se
centri na simetrija za
to~kite Н1, Н2, Н3 i
ortocentarot Н. Da ja
razgledame to~kata Н3.
Crte` 106
Bidej}i НВН3 ┴ ВС и АН ┴ ВС toga{ НВН3 || AH …(1).
Bidej}i HAH3 ┴ AC и BH ┴ AC toga{ HAH3 || BH …(2).
Od relaciite (1) и (2) sleduva deka ~etiriagolnikot ННАН3НВ e paralelogram
od kade sleduva deka Н3 e simetri~na na Н vo odnos na М3.
Analogno se poka`uva i za to~kite Н1 и Н2. Od toa {to M1M2, M2M3, M3M1 se
sredni linii vo triagolnicite НАНВНС, Н1Н2Н3, H2H3H i H3H1H, sleduva deka
H A HB =2 M1M 2 = H1H 2 , HBHC =2 M 2 M 3 = H 2H 3 и HAHC =2 M3M1 = H3 H1 , odnosno
triagolnicite НАНВНС i Н1Н2Н3 se skladni (spored priznakot SSS).
377. Neka A1, B1 i C1 se podno`nite to~ki na visinite povle~eni od temiwata
A, B i C, soodvetno (crt. 107). Od sli~nosta na triagolnicite AB1H i AA1C
dobivame AB 1 ÿ AC = AH ÿ AA 1 odnosno AB 1 ÿb= AH ÿha...(1). Od sli~nosta na
triagolnicite CB1H i CC1A dobivame CB 1 ÿb= CH ÿhc...(2).
So sobirawe na (1) i (2) dobivame bÿ( AB 1 + CB 1 )= AH
2
ÿha+ CH ÿhc odnosno b = AH ÿha+ CH ÿhc...(3). Analogno, gi
2
dobivame ravenstvata a = BH ÿhb+ CH ÿhc...(4) i
2
c = AH ÿha+ BH ÿhb...(5). So sobirawe na (3), (4) i (5) se
dobiva baranoto ravenstvo.
Crte` 107
378.Upatstvo: Doka`i deka vnatre{nite agli vo
α+β β+γ γ+α
triagolnikot XYZ se , i i
2 2 2
deka ®COY=®BOX=®AOZ=180±. 379. r =
a
4
3 −1 . ( )
380. Upatstvo: doka`i deka triagolnicite APD i
QAB se skladni. 381. Neka F le`i na CE taka
{to MF e paralelna so AB (crt. 108).
Crte` 108

158
Odgovori, upatstva i re[enija
Toga{ F e sredina na otse~kata CE. Zatoa, triagolnicite MFC i MFE se
skladni, pa ®CME=2·®FME=2·®BEM.

382. Neka O e presekot na dijagonalite na


paralelogramot ABCD, M e sredinata na AB, a N
na AD i neka CN i CM ja se~at BD vo Y i X,
soodvetno (crt. 109). CM i BO se te`i{ni linii
vo triagolnikot ABC.
Zatoa, X e te`i{teto na triagolnikot ABC.
Crte` 109
2 2 BD BD
Ottuka, BX = BO = ⋅ = . Sli~no, se
3 3 2 3
BD BD
poka`uva deka DY = i ottuka XY = .
3 3

Crte` 110
mn
383. . Upatstvo: niz N povlekuvame prava, paralelna so CD koja ja se~e
m+n
AC vo to~ka K (crt. 110). Toga{ ∆AMP~∆NKP i ∆AKN~∆ACD.

384. Da gi ozna~ime kracite na dadeniot triagolnik so a , a osnovata so c . Od


uslovot vo zada~ata sleduva deka ®CAB=®BCB= 80 o , ®CAE= 30 o , ®CBD= 20 o ,
®ADB= 40 o , ®AEB= 50 o , ®BDC= 140 o .
Ottuka sleduva deka triagolnicite BCD i EAB se
ramnokraki, pa BE = BA = c i neka DC = DB = d . Neka F e
to~ka od krakot CB taka {to DF i AB se paralelni i
neka prese~nata to~ka na AF i DB e G (crt. 111).
Toga{ ~etiriagolnikot ABFD e ramnokrak trapez pa
®BAG=®ABG= 60 o , odnosno triagolnikot ABG e
ramnostran, kako i triagolnikot GFD . Zna~i
BG = BA = BE = c i ottuka sleduva deka triagolnikot
180 o − 20 o
EGB e ramnokrak. Toga{ ®BGE= = 80 o i
2
®FGE= 180 o − 80 o − 60 o = 40 o . Crte` 111
o
Od ®FGE=®GFE= 40 sleduva EG = EF , a bidej}i triagolnikot GFD e
ramnostran imame DG = DE . Zatoa ~etriagolnikot GEFD e deltoid, pa DE go
1
prepolovuva agolot GDE , odnosno ®BDE= 60 o = 30 o .
2
385. a) Na dijagonalata BD konstruirame to~ka P taka {to aglite ACD i PCB
se ednakvi (crt. 112). Aglite CAD i CBD se nad ist lak toga{ i tie se

159
Odgovori, upatstva i re[enija
ednakvi, pa triagolnicite ADC i BPC se sli~ni.
BC AC
Dobivame = ili BC ⋅ AD = PB ⋅ AC ...(1). Aglite
PB AD
DCP i ACB se ednakvi, pa triagolnicite ACB i DCP se
sli~ni i va`i CD = AC ili CD ⋅ AB = PD ⋅ AC …(2).
PD AB
Sobiraj}i gi (1) i (2)
imame: BC ⋅ AD + CD ⋅ AB = PB ⋅ AC + PD ⋅ AC odnosno
BC ⋅ AD + CD ⋅ AB = AC ⋅ BD , {to treba{e da se doka`e. Crte` 112
2P∆ABD 2P∆BCD 2P∆ABD 2P∆BCD
+ +
AB ⋅ AD + BC ⋅ CD sinα sinγ
b) = = sinα sinα
AB ⋅ BC + AD ⋅ CD 2P∆ABC + 2P∆ACD 2P∆ABC 2P∆ACD
+
sinβ sinδ sinβ sinβ
2PABCD
sinβ AC
= sinα = = .
2PABCD sinα BD
sinβ

386. I re{enie:
re{enie:
Se konstruira to~kata C1 simetri~na na C vo
odnos na MN (crt. 113). Jasno e deka
MC 1 = C1D . Ako zamenime AB = a , toga{
2 2 2
MC1 = C1D = 5a2 a MD = 10a 2 .

2 2 2
Zna~i, MD = MC1 + C1D odnosno spored
pitagorova teorema ®MC1D=90˚, od kade
sleduva deka ®MDC1=45˚=α. Sega,
®NDC +®C DM+®MDC= α + β + γ = 90 o .
1 1 Crte` 113
I I re{enie:
re{enie:
1 5
Neka AB = 1 (crt. 114). Toga{ MC = 5 , MD = 10 . Imame sinβ = = ,
5 5
2 2 5 1 10 3 3 10
cosβ = = , sinγ = = i cosγ = = . Od formulata
5 5 10 10 10 10
sin(β + γ ) = sinβ ⋅ cosγ + cosβ ⋅ sinγ dobivame
2
sin(β + γ) = , od kade sleduva deka
2
β + γ = 45o . Bidej}i α = 45 o , dobivame deka

α + β + γ = 90 o .
Crte` 114

160
Odgovori, upatstva i re[enija
I I I re{enie:
re{enie:
Neka AB = 1 toga{ MB = 2 i MD = 10 . Neka ®BMC=β1, a ®CMD=γ1
(crt. 115). Od teoremata za ednakvost na nadvore{en agol na triagolnik so
zbirot na dvata nesosedni agli, imame β+β1=45±, γ+γ1=β. Sobiraj}i gi
poslednite dve ravenstva dobivame β+β1+γ+γ1=β+45±…(1).
MB ⋅ MD ⋅ sin(β 1 + γ 1 ) BD ⋅ AM
P∆MBD = = 5 ⋅ sin(β 1 + γ 1 ) = = 1 . Zna~i,
2 2
1
sin(β 1 + γ 1 ) = = sin β . Bidej}i β i β1+γ1 se
5
ostri agli, dobivame β1+γ1=β…(2).
Zamenuvaj}i (2) vo (1) dobivame β+γ+β=β+45±
odnosno β+γ=45±, od kade sleduva deka
α + β + γ = 90 o . Crte` 115
I V re{enie:
re{enie:
Jasno ®PAM=®MCA=β i ®DAN=®MDA=γ
(crt. 116) i ®NAP=90±-(β+γ).
1 1
Pravata AN ima ravenka y= x so k1= , a
3 3
pravata AP ima ravenka y=2x so k2=2.
k − k1
Toga{ tg(90 o − (β + γ ) = 2 . Zna~i
1 + k1 ⋅ k 2
ctg(β+γ)=1, pa β+γ=45±, odnosno
®NAP=45±=α, od kade se dobiva deka
α + β + γ = 90 o .

Crte` 116

V re{enie:
re{enie:
Neka AB = 1 toga{ MB = 2 , MC = 5 i MD = 10 (crt. 117).

Crte` 117
Triagolnicite DBM i MBC se sli~ni, so koeficient na sli~nost 2.

161
Odgovori, upatstva i re[enija

Zna~i, ®BMD=.®BCM=β. Od triagolnikot BMD sleduva β+γ+135±=180± od kade


sleduva α + β + γ = 90 o .
V I re{enie:
re{enie:
Okolu triagolnikot ADM opi{uvame kru`nica k
~ij centar e vo sredinata na otse~kata MD
(talesova teorema) (crt. 118).
Od ®MPD=90± i C sredina na BD sleduva
PC = BC = AB = CD . Zna~i triagolnicite MAB,
BCP, PCD se skladni. Bidej}i ®ABP=.®CBM (kako
nakrsni agli) i BP = MB i AB = BC skladni se i
triagolnicite ABP i CBM, od kade ®PAB=.®MCB=β. Crte` 118
Od toa {to ~etiriagolnikot APDM e tetiven sleduva deka 45±+β+γ+90±=180±,
pa α + β + γ = 90 o .
V I I re{enie:
re{enie:
→ → →
Neka ®BMD=φ (crt. 119). Kvadriraj}i go ravenstvoto BM+ MD = BD dobivame
→ → → → → → →
| BM | 2 +2 | BM | ⋅ | MD | ⋅cos(BM, MD)+ | MD | 2 =| BD | 2 .
2
Ottuka sleduva deka 8-4 5 cosφ=0 od kade cosφ= …(1).
5

Analogno, ako go kvadrirame ravenstvoto


→ → →
BC+ CM = BM dobivame
→ → → → →
| BC | 2 +2 | BC | ⋅ | CM | ⋅cos(180 o − β)+ | CM | 2 =| BM | 2 .
Zamenuvaj}i gi soodvetnite vrednosti vo
2
poslednoto ravenstvo dobivame cosβ= …(2).
5
Od (1) i (2) se dobiva φ=β, pa od triagolnikot BMD
sleduva 135±+β+γ=180±, odnosno α + β + γ = 90 o . Crte` 119
V I I I re{enie:
re{enie:

]e poka`eme deka β+γ=45±, od kade potoa sleduva to~nosta na tvrdeweto vo


zada~ata. Jasno e deka MB = 2 ,
MC = 5 i MD = 10 (crt. 120). Neka
PMCRS=Q2 i PMDLS=Q3. Proektiraj}i gi
ortogonalno ovie pravoagolnici na
ramninata ADLQ, dobivame
PACRQ = Q 2 ⋅ cosβ i PADLQ = Q 3 ⋅ cosγ
odnosno Q 2 ⋅ cosβ = 2 i
Q 3 ⋅ cosγ = 3 …(1).
Crte` 120

162
Odgovori, upatstva i re[enija
Proektiraj}i gi istite pravoagolnici na ramninite CRXW i DLON dobivame
PCRXW = Q 2 ⋅ cos(90 o - β) i PDLON = Q 3 ⋅ cos(90o - γ ) odnosno Q 2 ⋅ sin β = 1 i
Q 3 ⋅ sin γ = 1.
Od (1) i (2) sleduva deka Q2·Q3·sinβ·cosγ=3 i Q2·Q3·cosβ·sinγ=2. Sobiraj}i gi
poslednite dve ravenstva dobivame Q2·Q3·sin(β+γ)=5 od kade se dobiva deka
β+γ=45±
a BD
387. Od sinusna teorema za triagolnikot ABD sleduva = …(1), a od
sinϕ sinα
a BD
sinusna teorema za triagolnikot BCD, = …(2) (crt. 121). Od
sinψ sinγ
ϕ + ψ = 180 o sleduva sinϕ = sinψ i so zamena vo (1)
i (2) dobivame sinα = sinγ , odnosno
α+γ α−γ α+γ
cos ⋅ sin = 0 . Ako cos = 0 imame
2 2 2
α+γ
= 90 o + k ⋅ 180 o , pa α + γ = 180 o + k ⋅ 360 o . Imaj}i
2
predvid deka α + γ < 360 o sleduva α + γ = 180 o . Crte` 121

α−γ
Toga{ i β + δ = 180 o , {to e vo kontradikcija so ϕ + ψ = 180 o . Zna~i sin =0
2
α−γ
odnosno = k ⋅ 180 o , α = γ + k ⋅ 360 o pa α = γ .
2

388. 47±. Neka М e sredina na stranata ВС a N na АD


(crt. 122). Toga{ triagolnikot BMP e skladen so
triagolnikot CМР i triagolnikot ANP e skladen so
DNР od kade sleduva deka BP = CP i AP = DP . Od
AB = CD sleduva deka triagolnicite ABP i DCP se
skladni, pa ®ABP=®DCP. Neka φ=®CBP=®BCP. Toga{
®DCP=®ABP=φ+77±. Za aglite okolu temeto C va`i
®DCB+®BCP+®PCD=150±+φ+77±+φ=360± od kade
sleduva 2φ =133±. Zna~i, ®BPC=180±-2φ =47± . Crte` 122

389. 45± . 391. Neka M i N se te`i{tata na triagolnicite BCO i DAO,


soodvetno. Neka M’ i N’ se sredini na stranite AB i CD, soodvetno.
Pravite M’N’ i MN se paralelni (zo{to?).

163
Odgovori, upatstva i re[enija
Konstruirame kru`nici so dijametar
AB i CD . Normalite, spu{teni od
to~kata P na pravite BO i CO, se PX
i PY, soodvetno (crt. 123). Od
sli~nosta na triagolnicite BPX i
CPY sleduva BP ⋅ PY = CP ⋅ PX .
Bidej}i BP ⋅ PY e stepenot na P vo
odnos na kru`nicata so dijametar AB,
a CP ⋅ PX e stepenot na P vo odnos na
kru`nicata so dijametar CD, sleduva
deka to~kata P le`i na radikalnata
oska na ovie dve kru`nici.
Analogno, se doka`uva deka i
to~kata Q le`i na radikalnata oska,
odnosno PQ e radikalna oska na
kru`nicite so dijametar AB i CD.
Zatoa, PQ e normalna so M’N’ odnosno
so MN. Crte` 123

392. Od sli~nosta na ~etiriagolnicite


BNOM i ONCP sleduva BN: R = R : CP,
odnosno BN⋅ CP= R2 …(1) (crt. 124). Od
sli~nosta na ~etiriagolnicite AMOQ i
OPDQ imame AM⋅ DQ = R2 ...(2). Mno`ej}i
gi ravenstvata (1) i (2) se dobiva
baranoto ravenstvo.

Crte` 124

394. Upatstvo: Neka M i N se ortogonalni proekcii na L na stranite AB i AD.


Doka`i deka triagolnicite KML i DNL se skladni.

396. Od uslovot na zada~ate imame deka A 1A n+1 = A 2 A n+ 2 = ... = A n A 2n ...(1) .


Neka O e centarot na 2n-agolnikot i M e proizvolna to~ka. Od toa {to MO e
te`i{na linija za triagolnicite MA1An+1, MA2An+2, ..., MAnA2n toga{ imame
deka MO2 = 1  MA1 2 + MA n+1 2  − 1 A 1A n+1 2 = 1  MA 2 2 + MA n+2 2  − 1 A 2 A n+2 2
2  4 2  4
1 1
= ... =  MA n + MA 2n  − A n A 2n . Od ravenstvoto (1) a i mno`ej}i gi potoa
2 2 2

2  4
poslednite ravenstva so 2 dobivame deka
2 2 2
MA 1 + MA n+1 = MA 2 + MA n+ 2
2 2
= ... = MA n + MA 2n
2
.
Konkretno za n=2 imame kvadrat za koj va`i ravenstvoto
2 2 2 2
MA + MC = MB + MD .

164
Odgovori, upatstva i re[enija

397. Neka AM = BN =x, CN = DM =h (crt. 125).


Od triagolnikot ANC sleduva
AC 2 = AB 2 + 2 AB ÿx + x2 + h2. Od triagolnikot BMD
2 2 2 2
sleduva BD = AB –2 AB ÿx+x +h . Sobiraj}i gi
poslednite dve ravenstva se dobiva
AC 2+ BD 2=2 AB 2+2(x2+h2)=2 AB 2+2 BC 2. Crte` 125
399. Od triagolnikot AMN (crt. 126) imame:
2 2 2
ΑΝ = AM + MN − 2 AM ⋅ MN ⋅ cos ρ ...( 1) kade ®AMN=ρ. Od triagolnikot CMN
2 2 2
imame: CN = CM + MN − 2CM ⋅ MN ⋅ cos( π − ρ)
2 2 2
CN = AM + MN + 2AM ⋅ MN ⋅ cosρ...(2).
Sobiraj}i gi (1) i (2) dobivame
2 2 2 2
AN + CN = 2 AM + 2MN . Analogno se dobiva deka
2 2 2 2
AB + AD = 2AN + 2BN ...(3) i
2 2 2 2
BC + CD = 2CN + 2BN ...(4) . Crte` 126
Sobiraj}i gi (3) i (4) se dobiva:
2 2 2 2 2 2 2
AB + BC + CD + AD = 2AN + 2CN + 4BN ,
2 2 2 2 2 2 2
AB + BC + CD + AD = 4 AM + 4MN + 4BN ,
2 2 2 2 2 2 2
AB + BC + CD + AD = AC + 4MN + BD .
400. Od proporcionalnosta na otse~kite
AM DN
sleduva = ...(1) (crt. 127). Od toa {to
MB NC
∆AOM~∆CON i ∆MOB~∆NOD sleduva
AM MO MB AM NC
= = , pa = …(2).
NC ON DN MB DN

Crte` 127
2
 AM 
Mno`ej}i gi (1) i (2) dobivme 
 MB 
=1 odnosno
 
AM = MB . Od (1) se dobiva DN = NC .

401. Od ®ABB1 =®AA1B i ®AMB=®A1MB1 sleduva


deka triagolnicite ABM i A1MB1 se sli~ni, i
A 1B1 B1M
zatoa = ...(1).
AB AM

Crte` 128

165
Odgovori, upatstva i re[enija

C1D1 D1M A 1D1 D1M B1C1 B1M


Sli~no = ...(2), = ...(3) i = ...(4) (crt. 128).
CD CM AD AM BC CM
A 1B1 C1D1 B1M D1M
Od (1) i (2) dobivame ⋅ = ⋅ ...(5), a od (3) i (4) dobivame
AB CD AM CM
A 1D1 B1C1 D1M B1M
⋅ = ⋅ ...(6) .
AD BC AM CM
A 1B1 C1D1 A 1D1 B1C1
Od (5) i (6) sleduva ⋅ = ⋅ , a imaj}i predvid deka
AB CD AD BC
AB = AD = CD = BC se dobiva baranoto ravenstvo.
402. Neka to~kata P e presekot na pravite XZ i YW, a
to~kata O e centarot na vpi{anata kru`nica vo
~etiriagolnikot ABCD (crt. 129). Gi voveduvame slednite
oznaki: α=®WOX; β=®XOY; γ=®YOZ; δ =®ZOW. Od ovde
sleduva deka ®DCB=180±-γ i ®DAB=180±-α. Sobiraj}i gi
poslednite dve ravenstva dobivame deka
180±=®DCB+®DAB=β+ δ , od kade sleduva α+γ=180±.
Crte` 129
Ako ja koristime vrskata me|u centralniot i periferniot agol vo kru`nica gi
γ α
dobivame ravenstvata ®ZWP=®ZWY= i ®WZP=®WZX= .
2 2
α+γ
Zna~i, ®ZPW =180±-(®ZWP+®WZP)=180±- =180±-90±=90±.
2
403. Neka prese~nite to~ki na simetralite se P, Q, R i
Ѕ (crt. 130) . Jasno e deka ®SPQ=®CPB i ®QRS=®ARD. Od
druga strana pak, ®BPC=π-(®PBC+®BCP) i
®ARD=π-(®RAD+®RDA).
Spored toa ®QPS+®QRS=®BPC+®ARD
=2π-(®PBC+®BCP+®RAD+®RDA)=2π-π=π.

Crte` 130
Zna~i, ~etiriagolnikot PQRS e tetiven, odnosno
to~kite P, Q, R i S le`at na edna kru`nica.
404. Neka На, Нb, Hc i Hd se ortocentrite na
triagolnicite AKN, BKL, CLM i DMN, soodvetno, i
neka О e centarot na kru`nicata k. Bidej}i OK||NHa i
ON||KHa (crt. 131), sleduva deka ~etiriagolnikot
ONHaK e paralelogram. Spored toa KHa = ON ...(1).
Na ist na~in dobivame deka ~etiriagolnikot OKHbL e
paralelogram i KHb = OL ...(2). Crte` 131

166
Odgovori, upatstva i re[enija

Od ravenstvata (1) i (2) i od ®HaKHb=®NOL, dobivame deka triagolnicite


HaHbK i NLO se skladni i zatoa otse~kata НаНb e ednakva i paralelna so
otse~kata NL. Analogno, zaklu~uvame deka otse~kata НcНd e paralelna i
ednakva so otse~kata NL. Spored toa ~etiriagolnikot НаНbНсНd e
paralelogram.

405. Od tetivniot ~etiriagolnik AMNP sleduva


®NAM=®NPM i ®PAN=®PMN (crt. 132). Bidej}i
®ACB=®CAD=®NAP=®PMN, sleduva deka
triagolnicite ABC i PNM se sli~ni, a od
skladnosta na triagolnicite ABC i CDA sleduva i
sli~nosta na PNM i CDA.
Crte` 132
AC AB AD
Zatoa, = = ...(1). Od teoremata na Ptolomej za ~etiriagolnikot
PM PN NM
AMNP sleduva AM ⋅ NP + MN ⋅ AP = AN ⋅ MP ...(2). Od (1) i (2) dobivame
AC AB AD
AN ⋅ MP ⋅ = AM ⋅ NP ⋅ + MN ⋅ AP ⋅ i ottuka
PM PN NM
AN ⋅ AC = AM ⋅ AB + AP ⋅ AD .
406. Neka simetralata s na agolot ACD ja se~e BD vo F.
Toga{, ®QBC=®FCD, ®FCO=®DBQ, kako agli so zaemno
normalni kraci (crt. 133). Zna~i QB e simetrala na
agolot CBD.
Crte` 133
Ponatamu, ®BCP=®BDQ=45± od kade sleduva sli~nost na triagolnicite BCP i
DQ PC
BDQ. Od taa sli~nost sleduva ravenstvoto = , a od praviloto na
DB BC
PC PO DB
simetrala imame = . Zna~i, DQ = ⋅ PO
BC OB OB
odnosno, DQ = 2PO .
5
407. . Neka E e prese~na to~ka na prodol`enijata
16
na stranite AD i BC (crt. 134). Da zabele`ime deka
∆EDC~∆EBA, pa mo`eme da zamenime ED =x i EB =3x.
Crte` 134
Toga{, EA =x+10 i EC =3x-2 od sli~nosta na triagolnicite EDC i EBA
dobivame EA ≅ ED = EB ≅ EC , odnosno xÿ(x+10)=3x≅(3x-2). Re{enie na
ravenkata e x=2 (x=0 bi zna~elo EªD). Sega, spored teoremata na Menelaj
dobivame:
MP NC EA 3MP MQ NB ED MQ
1= ÿ ÿ = , 1= ÿ ÿ = .
PN CE AM 5PN QN BE DM 15QN

167
Odgovori, upatstva i re[enija

Zamenuvaj}i MN =y i od predhodnite dve ravenstva


5y 15y
imame deka MP = i MQ = . Toga{
8 16
5y PQ 5
PQ = MQ - MP = od kade dobivame deka = .
16 MN 16

Crte` 135

408. Jasno e deka ®DFG=®DEG kako agli nad isti lak, analogno ®FDE=®EGF
(crt. 135). Spored toa, va`i odnosot FD = EG …(1). Neka OH e normalna na DF
FC EC
i OJ e normalna na EG (HœDF, JœGE) toga{ FD = 2FH , EG = 2EJ …(2).

Zamenuvaj}i go ravenstvoto (2) vo ravenstvoto (1) dobivame FH = EJ od kade


FC EC
sleduva óFCH~óECJ, pri {to dobivame deka ®CHM=®NJC...(3).
^etiriagolnikot OCMH e tetiven bidej}i ®OHM+®MCO=180± od kade sleduva
deka ®CHM=®COM. Analogno dobivame deka ~etiriagolnikot OJNC e tetiven
od kade se dobiva ravenstvoto ®NJC=®NOC. Od ravenstvoto (3) dobivame
deka ®COM=®NOC, od kade sleduva skladnost na triagolnicite OCM i OCN
(spored priznakot ASA). Od taa skladnost sleduva deka to~kata C e sredina
na otse~kata MN.
409. 13 i 11. 411. Neka podno`jata na visinite kon stranite BC, CA i AB se H1,
H2 i H3, soodvetno (crt. 136). So k1, k2 i k3 }e gi ozna~ime kru`nicite so
dijametri HH1, HH2 i HH3, soodvetno. Neka vtorata prese~na to~ka na k1 i k2 e
C1, na k2 i k3 e A1, a na k3 i k1 e H3. Jasno ®HC1H1+®HC1H2=180± pa C1 le`i na
otse~kata H1H2. Analogno, A1 le`i na H2H3, a B1 na H3H1. Od tetivnosta na
~etiriagolnicite ABH1H2 i BCH2H3 sleduva
®HH2A1=®BH2H3=®BCH3=90±-®ABC i
®HH2C1=®BH2H1=®BAH1=90±-®ABC.
Zna~i, ®HH2A1=®HH2C1, pa pravoagolnite
triagolnici H2A1H i H2C1H se skladni,
odnosno A1HC1H2 e deltoid i zatoa A1C1 i HH2
se zaemno normalni. Bidej}i HH2 e normalna so
AC, sleduva deka A1C1 i AC se paralelni.
Analogno se doka`uva deka A1B1 i AB, odnosno
BC i B1C1 se paralelni. So toa sme doka`ale
deka triagolnicite ABC i A1B1C1 se sli~ni. Crte` 136
412. Upatstvo: Doka`i skladnost na triagolnicite C1BC i A1BA, kako i
9
skladnost na triagolnicite A1CA i B1CB. 413. ab . 414. .
2

168
Odgovori, upatstva i re[enija
415. Neka p∩q={M}, O1 i O2 se centrite na k1 i k2, soodvetno. Neka prese~nata
to~ka na O1M so k1, koja e
me|u O1 i M ja ozna~ime so P,
a na O2M so k2, Q. U{te, neka
i dopirnite to~ki na k1, k2, k3
i k4 so p se A, B, C i D,
soodvetno (crt. 137).
Od O1M i O2M simetrali na
aglite me|u p i q sleduva
®O1MO2=90± i zatoa
®O1MA+®O2MB=90±, pa
®MO1A=®O2MB. Ottuka,
triagolnicite AMO1 i BO2M
AM AO1
se sli~ni pa = ,
BO 2 BM
odnosno AM ⋅ BM = 1 …(1). Crte` 137
5
Od sli~nosta na MO1A i MO3C sleduva 1 = O1M = O1M
i ottuka O1M = .
4 O 3M 4 + 1 + O1M 3
2
5 4
Toga{, od pravoagolniot triagolnik AMO1 imame AM =   − 12 = . Od (1)
3 3
3
sleduva deka BM = .
4
1 MO 2
Od sli~nosta na MBO2 i MDO4 sleduva =
R MO 4
2
3 2
  +1
 
4 5 5
= , odnosno + 1+ R = R i imame
2 4 4
3 2
  + 1 + 1+ R
4
R=9.
Crte` 138

416. Od teoremata na Ptolomej imame AC ⋅ MB + AB ⋅ CM = AM ⋅ CB i bidej}i


AB = BC = CA sleduva MB + CM = AM , odnosno x+y=z (crt. 138).
2 2 2
Ottuka x +z +2xz=y …(1). Od triagolnikot BMC imame
2 2 2 2 2 2
CB =a =x +z -2xzcos120±=x +z +xz…(2). Od (1) i (2) sleduva
2 2 2 2 2 2 2 2
x +y +z =2x +2z +2xz=2(x +z +xz)=2a .

417. Neka E i F se sredini na stranite CA i CB (crt. 139). Imame


OE OF
CM ⋅ OE + CN ⋅ OF = 2P∆CMN = CM ⋅ CN ⋅ sinγ . Taka, + = 2sinγ koe
1 1
CN CM
2 2

169
Odgovori, upatstva i re[enija
ravenstvo e ekvivalentno so
OE OF AB
ravenstvoto + = …(1).
EP FQ OA
Neka D e to~ka od stranata AB za
BD OF
koja va`i = …(2).
OB FQ
Zamenuvaj}i (2) vo (1) dobivame
OE AB BD AD
= − = .
EP OA OB OA
Taka dobivame deka triagolnicite
OFQ i DBO se sli~ni, a takvi se i
triagolnicite OEP i DAO. Sega od
®EOP+®FOQ=®ODA+®BDO=180±
sleduva ®POQ+®FOE=180± odnosno
®POQ=180±-®FOE=®ACB.
Crte` 139

Tema 8: Linearni ravenki i neravenki


neravenki

1
418. x=2020. 419. aœ(1,+¶). 420. x=3 i 0≤x≤1. 421. x =; n ∈ {1,3,5, ..., 101}. Od
n
toa {to a = ± a sleduva deka desnata strana na ravenkata e od oblik
± (x − ±(x − ... ± (x − ±(x − 1))...)) . Od ovde pak sleduva deka desnata strana e od
oblik kx ± 1 , kade k e paren broj (Razmisli zo{to k e paren broj?) i
k ∈ [− 100,100] . Zna~i, ravenkata se transformira vo vidot x = kx ± 1 , ~ie
1
re{enie e od oblik x = , za n neparen broj pome|u 1 i 101. Jasno e deka x e
n
pozitiven broj (desnata strana vo ravenkata e pozitivna).

422. a=-1. Ako x e re{enie na ravenkata, toga{ od


(-x) + (-x) + a + (-x) − (-x) − a = x + x + a + x − x − a = 2 , sleduva deka i –x e isto
taka re{enie na dadenata ravenka. Za ravenkata da ima tri re{enija, mora
edno od re{enijata da bide 0. Zamenuva}i za x=0, dobivame
2 = a + − a = a + a = 2 a od kade a = 1 , odnosno a=≤1. Ako a=1 i x¥0, toga{ x=0,
pa ravenkata bi imala edno re{enie. Ako a=-1 i x¥0, toga{ dobivame deka
x=0 ili x=1. Bidej}i x=1 e re{enie, sleduva deka i x=-1 e re{enie, pa
ravenkata ima to~no tri re{enija.

170
Odgovori, upatstva i re[enija

1 1 1
423. 200. 424. x=ab+bc+ca, (a,b,cœN), za + + ≠0 ili
a+b b+c c+a
1 1 1
beskone~no mnogu re{enija za + + =0.
a+b b+c c+a
2 2
425. x = a + b ; x2=a+b (a+b∫0); x3=0 (a+b=0).
1 a+b
 1 1 1  1 1 1
426. x=a+b+c, za (a + b + c ) ⋅  + +  ≠ 4 i xœR za (a + b + c ) ⋅  + +  = 4 .
a b c a b c
427. 52. Spored uslovot na zada~ata imame deka xª1(mod3), xª2(mod5),
xª3(mod7). Od prvata kongruencija dobivame deka x=1+3·a...(1). Spored
vtorata kongruencija imame 1+3·aª2(mod5) ï 3·aª1(mod5) ï 3·aª6(mod5) ï
aª2(mod5) ï a=2+5·b...(2). Zamenuvaj}i ja ravenkata (2) vo ravenkata (1), go
dobivame ravenstvoto x=7+15·b...(3). Od tretata kongruencija imame
7+15·bª3(mod7) ï 15·bª3(mod7) ï 5·bª1(mod7) ï 5·bª15(mod7)
ï bª3(mod7). Ottuka b=3+7·c...(4). Zamenuvaj}i ja ravenkata (4) vo ravenkata
(3), dobivame x=52+3·5·7·c. Najmalata vrednost za x se dobiva koga c=0, pa
x=52.
428. 247, 499, 751. 429. x=10 ili x=52. Neka imalo vkupno n paketi, a vo sekoj
od niv se nao|ale po 3 jabolki. Toga{ brojot na paketi so po x jabolki bi bil
20-n. Od uslovot na zada~ata ja sostavuvame ravenkata 3n+x(20-n)=109, koja
se transformira vo (x-3)(20-n)=49 od kade pak x=10, n=3 ili x=52, n=19.
430. 1999. 431. 18. 432. 1904. 433. 13 i 31. 434. 15 dena. 435. 35m.
436. 5 ~lena. Neka n e brojot (koli~estvoto) na ispieni ~a{i (broj na lu|e vo
semejstoto), a x e koli~estvoto na ispieno mleko. Toga{ koli~estvoto na
ispieno kafe e n-x. Spored uslovot na zada~ata, ja imame ravenkata
x/4+(n-x)/6=1. Poslednata ravenka se doveduva vo oblikot x+2n=12 ...(1).
Bidej}i n e priroden broj, sleduva deka i x e priroden broj. Osven toa, x§n,
bidej}i koli~estvoto na ispieno mleko ne mo`e da bide pogolemo od vkupniot
broj ispieni ~a{i. Spored toa, ravenkata (1) ima tri re{enija i toa: n=6, x=0;
n=5, x=2; n=4, x=4. No, prvoto i tretoto re{enie odgovaraat na slu~aite koga
site piele samo kafe ili samo mleko. Zna~i, odgovorot e 5 lu|e. 437. 500
`iteli. 438. 23 godini. 439. 1,5 min.
440. 240 m. Neka x e dol`inata na vozot; V1-brzinata na vozot; V2-brzinata na
lu|eto. Od formulata S=Vÿt, imame
30 = V2 ⋅ t V 30 10 = V2 ⋅ t ′ V2 1 30
 ⇒ 2 =  ⇒ = = ⇒ x = 240m .
 x − 30 = V 2 ⋅ t V 1 x − 30 70 = V1 ⋅ t ′ V1 7 x − 30
1 2 2 3
x+ y x+ y
441. 108 ili 1107. 442. 9:35. Upatstvo: sostavi ja ravenkata 3 5 = 3 5 ,
17 27
kade x i y se delovite od prviot i vtoriot most koi se sodr`at vo noviot
most. 443. Vasko, Atanas, Blagoja.
444. 251. Da pretpostavime deka sme koristele n moneti od 1 denar, d moneti
od 2 denari i q moneti od 5 denari. Toga{ va`i ravenstvoto n+d+q=1000...(1).
Od uslovot na zada~ata, ja imame ravenkata n+2d+5q=2000...(2). Zamenuvaj}i ja
ravenkata (1) vo (2), dobivame deka d=1000-4q...(3). Zamenuvaj}i ja ravenkata

171
Odgovori, upatstva i re[enija
(3) vo (2), dobivame deka n=3q. Bidejki n, d i q se nenegativni celi broevi
sleduva deka qœ{0,1,2, …, 250}. 4q§1000. Zna~i postojat 251 na~in.
1 1 m 50
445. 2. Od 49<n<101 sleduva deka ≤ , a od 19<m<51 sleduva ≤ . Taka
n 50 n n
m+n m 50 50
imame = +1≤ +1≤ + 1 = 2 . Zna~i najgolemata vrednost na
n n n 50
baraniot izraz e 2.

Tema 9: Teorija na broevi


11 36 11 4 9 11 4 32 28 24 4
448. P(n)=n (n -1)=n ((n ) -1)=n (n -1)·(n +n +n +…+n +1). Ako n e paren
4
broj toga{ 2| P(n). Ako n e neparen broj toga{ n -1 e paren broj i zatoa 2| P(n).
Zna~i, za sekoj priroden broj n, 2|P(n). Ako 5|n toga{ 5|P(n). Ako 5Fn toga{ n=5k
± 1 ili n=5k ± 2, {to povlekuva n2=5A ± 1 odnosno n4=5B+1, za nekoj priroden
4
broj B. Zna~i, 5|n -1, od kade {to sleduva deka 5|P(n). Taka, za sekoj priroden
broj n va`i 5|n. Od 2|P(N), 5|P(n) i NZD(2,5)=1 sleduva deka 10|P(n), za sekoj
priroden broj n.
5555 2222 5555 5555 2222 2222 5555 2222
449. 2222 +5555 =(2222 +4 )+(5555 -4 )-(4 -4 )
2222 3333 3333 1111
=2226ÿM+5551ÿN-4 ÿ(4 -1). Но 2226=7ÿ318, 5551=7ÿ793 i 4 -1=64 -1 se
5555 2222
deli so 63. Sleduva deka 7|(2222 -5555 ).
451. an=n(n -5n +4)=n(n -4)(n -1)=(n-2)(n-1)n(n+1)(n+2). Zna~i an mo`e da se
4 2 2 2

zapi{e kako proizvod na pet posledovatelni celi broevi. Sleduva deka an e


deliv so 5!=120.
3 2
452. Upatstvo: Za n=2k+1; n +3n –n–3=8k(k+1)(k+2). 454. 10k+3; 10k+7.
(a + c)(b + c)
455. Od uslovot na zada~ata sleduva deka a+b+c+d= e priroden
c
broj, odnosno c|(a+c)(b+c). Mo`ni se slednive slu~ai:
i) a+c=k1c. Od a+c>c sleduva k1>1 i a+b+c+d=k1(b+c) e slo`en broj (proizvod na
dva prirodni broja pogolem od 1).
ii) b+c=k2c. Analogno, od b+c>c sleduva k2>1 i povtorno a+b+c+d=k2(a+c) e
slo`en broj.
iii) a+c=m1c1, b+c=m2c2 i c=c1c2 (c1,c2>1). Od a+c>c>c1 i b+c>c>c2 sleduva m1 i m2
se pogolemi od 1 od kade {to a+b+c+d=m1m2 e slo`en broj.
2008
456. Da. Pr: 1,2,3,6,12,24, ..., 3ÿ2 .
458. Od uslovot na zada~ata imame aª ± 1(mod5) или aª ± 2(mod5). Sleduva
4 4 4 4 4
deka a ª1(mod5), b ª1(mod5), c ª1(mod5), d ª1(mod5) и e ª1(mod5), odnosno
4 4 4 4 4
a +b +c +d +e ª5ª0(mod5).
101 101 101 101 100 200 100
459. 1050906. Da zabele`ime deka 30 =2 ·3 ·5 , a 20 =2 ·5 .
101 100
Delitelite na 30 koi se deliteli i na 20 mora da se deliteli i na brojot
101 100
2 ·5 . Takvi deliteli ima (101+1)·(100+1)=102·101=10302. Od druga strana,
101 3
brojot 30 ima (101+1) =1061208 deliteli. Zna~i postojat
101 100
1061208-10302=1050906 deliteli na 30 koi ne se deliteli na 20 .
2 2 3 3 2 2
460. Od 10|x +xy+y sleduva deka 10|x -y =(x-y)(x +xy+y ), {to povlekuva deka
2 2
x i y zavr{uvaat na ista cifra. Toga{ i broevite x , xy i y zavr{uvaat na
2 2
ista cifra i taa mora da bide 0, inaku vo sprotivno brojot x +xy+y ne bi bil

172
Odgovori, upatstva i re[enija
deliv so 10. Ottuka sleduva deka x i y se delivi so 10 {to povlekuva deka
2 2
x +xy+y e deliv so 100.
461. Neka x=ca+9a+81. Od toa {to 2015 e delitel na ab+9b+81 sleduva deka
ab+9b+81=2015k…(1), za nekoe kœZ. Prosti deliteli na 2015 se 5, 13 i 31 pri
{to nieden od niv ne e delitel na 81. Od ravenstvoto (1) sleduva deka 5, 13 i
31 ne se deliteli nitu na b.
Od bx=abc+9ab+81b=c(ab+9b+81)-9(bc+9c+81)+9(ab+9b+81) sleduva deka 2015
e delitel na bx, a bidej}i 2015 ne e delitel na b sleduva deka 2015 e delitel
na x.
464. ]e doka`eme deka broevite x, y i z davaat isti ostatoci pri delewe so 3.
Toga{ od uslovot na zada~ata }e sleduva deka brojot x+y+z se deli so 27. Ako
broevite x, y i z davaat razli~ni ostatoci pri delewe so 3, toga{ brojot
(x-y)(y-z)(z-x) ne se deli so 3, a brojot x+y+z se deli so 3. Sleduva deka barem
dva od trite broja x, y, z davaat isti ostatoci pri delewe so 3. No, toga{
brojot x+y+z=(x-y)(y-z)(z-x) se deli so 3, pri {to mora i tretiot broj da dava
ist ostatok pri delewe so 3, kako i drugite dva broja. 465. x=5.
466. n=9k, kœN. Neka A= 1313...13
1424 3 . Za da brojot A bide deliv so 63 treba da
2n cifri
bide deliv so 7 i so 9. Zbirot na cifrite na A e n+3n=4n.
Od toa {to NZD(4,9)=1, sleduva deka A se deli so 9 ako i samo ako n se deli
so 9. Zna~i n=9k. Od druga strana A = 13 + 13 ⋅ 10 2 + ... + 13 ⋅ 10 2n−2
100 n − 1
( )
= 13 1 + 100 + 100 2 + ... + 100 n−1 =13·
99
, od kade {to sleduva deka 7|A ako
n 3 n
i samo ako 7|100 -1. Bidej}i 100 ª1(mod7) sleduva deka za n=9k, 7|100 -1.
Zna~i, za n=9k, A e deliv so 9 i 7, odnosno so 63.
467. nœ{0,6,40,48,286,2008}. 468.1995. 469. 630. 470. 2178.
471. Sekoj priroden broj ima eden od oblicite 4k,4k+1,4k+2 ili 4k+3. Bidej}i
a, b i c se neparni broevi tie imaat eden od oblicite 4k+1 ili 4k+3. Spored
principot na Dirihle dva od broevite a, b i c imaat ist oblik. Bez gubewe od
op{tosta mo`eme da pretpostavime deka a i b imaat ist oblik. Spored toa
a=4k1+1, b=4k2+1 ili a=4k1+3, b=4k2+3.
Vo prviot slu~aj imame ab-1=4(4k1ÿk2+k1+k2), a vo vtoriot slu~aj
ab-1=4(4k1k2+3k1+3k2+2). Spored toa 4|(ab-1), odnosno eden od broevite
ab-1, bc-1, ca-1 e deliv so 4. 472. y=9, z=9, xœ{1,2,3,…,9}.
2
473. 784 i 144. Od uslovot na zada~ata imame deka (a − 1)(b − 1)(c + 2) = p i
2
abc =q . Bidej}i poln kvadrat zavr{uva na cifrite 1, 4, 5, 6, 9, 0, sleduva
deka edinstveni vrednosti koi mo`e da gi ima cifrata c se 4 ili 9. Zatoa }e
gi razgledame slednite slu~ai:
2 2
I. Za c=4 imame (a − 1)(b − 1)6 = p i ab4 = q . Od poslednite dve ravenstva
2 2
dobivame deka 100a+10b=p +104...(1) i 100a+10b=q -4...(2). Zamenuvaj}i ja
2 2
ravenkata (1) vo ravenkata (2) go dobivame ravenstvoto q -p =108, koe se
2 3
transformira vo oblikot (q-p)(q+p)=2 3 ...(3). Od ravenstvoto (3) se dobivaat
q − p = 1 q − p = 2 q − p = 4 q − p = 6
slednite pet sistemi:  ,  ,  ,  i
q + p = 108 q + p = 54 q + p = 27 q + p = 18

173
Odgovori, upatstva i re[enija

q − p = 9
 . Od vtoriot sistem dobivame q=28, odnosno abc = 784 , a od
q + p = 12
~etvrtiot sistem dobivame q=12, odnosno abc = 144 .
2 2
II. Za c=9 imame (a − 1)b1 = p i ab9 = q . Od poslednite dve ravenstva
2 2
dobivame deka 100a+10b=p +99...(3) i 100a+10b=q -9...(4). Zamenuvaj}i ja
2 2
ravenkata (3) vo ravenkata (4) go dobivame ravenstvoto q -p =108 ~ii
re{enija se isti kako vo slu~ajot I.
474. 550, 803. Od principot za delivost so 11 sleduva ravenstvoto
a+c-b=11k za nekoj cel broj k. Bidej}i a, b i c se cifri sleduva deka mo`ni
vrednosti za k se 1 i 0. Zna~i, razgleduvame dva slu~ai i toa:
2 2 2
I. Za, a+c-b=11...(1) imame 100a+10b+c=11(a +b +c ) odnosno
2 2 2 2 2 2
99a+11b+a+c-b=11(a +b +c ) od kade sleduva 9a+b+1=(a +b +c )...(3).
2 2 2 2
Zamenuvaj}i go ravenstvoto (a +b +c )=(a+b-c) -2ac+2ab+2bc...(4) vo
ravenstvoto (3) dobivame deka 9a+b+1=121-2(ac-ab-bc). Poslednoto
ravenstvo se transformira vo oblikot a-b=10a+2(ac-ab-bc)-120 od kade
zaklu~uvame deka a-b se deli so 2. Od druga strana, so ogled na toa {to
najgolema vrednost na a e 9 a najmala vrednost na b e 0 , dobivame deka
najgolema vrednost na a-b e 9. Zatoa, gi razgleduvame samo slu~aite koga
a-b=2, a-b=4, a-b=6 i a-b=8. Re{enie se dobiva samo vo slu~ajot koga a-b=8.
Vo toj slu~aj c=3, pa ravenkata (3) se sveduva na kvadratnata ravenka
2
a -13a+40=0, ~ii re{enija se a=8 i a=5. No, za a=5 se dobiva b=-3. Spored toa
a=8, b=0 i c=3, pa baraniot broj e 803.
II. Za a+c-b=0, od ravenstvoto (2) dobivame deka
2 2 2 2 2 2
99a+11b=11(a +b +c ), odnosno 9a+b=a +b +c ...(5). Od ravenstvoto (4) sleduva
2 2 2
ravenstvoto a +b +c =2ab+2bc-2ac ...(6). Zamenuvaj}i go ravenstvoto (6) vo (5)
dobivame deka b-a=2(ab+bc-ac)=10a, od kade zaklu~uvame deka i b-a=c se
deli so 2. So ogled na toa {to najgolema vrednost na b-a e 9, sleduva deka
dovolno e da gi razgledame slu~aite koga b-a=0, b-a=2, b-a=4, b-a=6 i b-a=8.
Re{enie se dobiva koga b-a=0, c=0. Zamenuvaj}i vo ravenstvoto (5) ja
2
dobivame kvadratnata ravenka a -5a=0, ~ii re{enija se 5 i 0. Za, a=5 go
dobivame brojot 550 a za a=0 zada~ata nema re{enie(Zo{to!).
475. 330. Jasno e deka a>0 i b>c. Od e¥0 sleduva deka d¥0. Za sekoj izbor na
trojkata (b,c,d) so b>c i d¥c postoi edinstven par (a,e) takov {to brojot abcde
gi zadovoluva uslovite od zada~ata.
Za c=0, bœ{1, 2, ..., 9}, dœ[0, 1, ..., 9}.
Za c=1, bœ{2, 3, ..., 9}, dœ{1, 2, ..., 9}.
...


Za c=8, bœ{9}, dœ{8,9}. Vkupniot broj na takvi petcifreni broevi e
9 9

∑ (9 − n)(10 − n) = ∑ (n
n =0 n=0
2
)
− 19n + 90 = 330 .

476. 142857. Neka abcdef e baraniot {estocifren broj. Bidej}i proizvodot na


toj broj so 6 e isto taka {estocifren broj, sleduva deka a=1. Bidej}i bcdef1 se
dobiva pri mno`ewe na 1bcdef so nœ{2, 3, 4, 5, 6} sleduva deka n=3 i f=7. Na toj

174
Odgovori, upatstva i re[enija

na~in gi dobivame broevite bcde71 i 1bcde7 . Pretposlednata cifra 7 vo


bcde71 , mo`e da se dobie pri mno`ewe na 1bcde7 so 3, samo ako e=5. Taka gi
dobivame broevite bcd571 i 1bcd57 . Na sli~en na~in, cifrata 5 vo bcd571
mo`e da se dobie pri mno`ewe na 1bcd57 so 3 samo ako d=8. Taka se dobivaat
broevite bc8571 i 1bc857 . Analogno se nao|a deka c=2 i b=4. Kone~no, baraniot
16 19 26 49
broj e 142857. 477. ; ; ; . 478. 12.
64 95 65 98
p q p q p-1 p-2 q-1 q-2
480. a) p +q =(p -1)+(q +1)=(p-1)(p +p +...+1)+(q+1)(q -q +...+1)
=(p-1)(2m+1)+(q+1)(2n+1). Od uslovot na zada~ata p=q+2, odnosno
p q
p-1=q+1. Sleduva p +q =2(p-1)(m+n+1)=(p-1+q+1)(m+n+1)=(p+q)(m+n+1). Od
p q
ovde sleduva deka p +q se deli so p+q. 481. Upatstvo: Doka`ete deka brojot
3(a+b) e deliv so 9 i deka a-b e deliv so 3. 483. 3471875.
1 ap 1 ap-1 1 a 1 a1
484. Da zabele`ime deka = , = , …, = p-r +1 , …, = i
a1 m a2 m ar m ap m
1 bq 1 bq-1 1 b 1 b1 1 1 1 1 1 1
= , = , …, = p-s+1 , …, = . Od, + + ... + = + + ... +
b1 n b2 n bs n bq n a1 a2 ap b1 b2 bq
ap ap-1 a1 bq bq-1 b
sleduva + + ... +
= + + ... + 1 odnosno
m m m n n n
a p + a p-1 + ... + a1 b q + b q-1 + ... + b 1
= …(1). Od (1) se dobiva m=n, koristej}i go
m n
pritoa ravenstvoto a1+a2+…+ap=b1+b2+…+bq.
485. Da zabele`ime deka 30k = (29 + 1)k = 29M + 1 ...(1) i

n-1 n-2 a(10 n − 1)


aa...a
123 = a(10 + 10 + ... + 1) = . Bidej}i a e cifra i 29 e prost broj,
9
n cifri
n n n
123 ako i samo ako 29|10 -1 ako i samo ako 29|3 (10 -1),
sleduva deka 29| aa...a
n cifri
n n n n n
odnosno 29|30 -3 . Od (1) sleduva deka 29|30 -3 ako i samo ako 29|3 -1. Od
28
NZD(3,29)=1 i 29F3, spored malata teorema na Ferma, sleduva deka 29|3 -1.
Ottuka, n=28.
486. Dovolno e da poka`eme deka zbirot na broevite na sme{nite taloni se
deli so 1001 = 13·77. Na sekoj sme{en bilet so broj A mu soodvetstvuva talon
so broj A’=999999-A. Se voo~uva deka i A’ e sme{en talon ~ij broj e razli~en
od A. Na toj na~in site broevi od sme{nite taloni mo`e da se razdelat vo
parovi (A,A’) pri {to zbirot na broevite na sekoj par e 999999, a ovoj broj e
deliv so 1001. Sleduva deka i zbirot od broevite na site sme{ni taloni se
deli so 1001. 487.18.
489. (x,y)œ{(1,2),(-5,2),(2,1),(2,-5)}. Upatstvo: Dadenata ravenka e ekvivalentna
so ravenkata (x+y+2)·(xy-x-y+2)=5.
490. Re{enie e sekoj par (x,x). Dadenata ravenka se zapi{uva vo oblikot
3 3
x −y x3 − y3
= 2021 . Od x2 + xy + y2 = = 2021≡ 2(mod3) sleduva deka brojot
x−y x−y

175
Odgovori, upatstva i re[enija
2 2
x +xy+y pri delewe so 3 dava ostatok 2. No, od druga strana
2 2 2 2 2
x +xy+y ªx -2xy+y =(x-y) ª0 ili 1(mod3), od kade zaklu~uvame deka pri x∫y
zada~ata nema re{enie.
494. Sekoj priroden broj mo`e da se pretstavi vo oblikot 6k + m , kade {to k e
priroden broj a m ∈ {0,1, 2, 3, 4, 5} . Ako p e prost broj pogolem od 3 toga{ toj e
neparen i zatoa nemo`e da bide od oblikot 6k, 6k + 2 i 6k + 4 . Isto taka
nemo`e da bide od oblik 6k + 3 bidej}i toga{ bi bil deliv so 3. Zna~i
ostanuva p da e od oblikot 6k ± 1 .
493. 7. 495. Upatstvo: Razgledaj gi ostatocite pri delewe so 3.
496. Da pretpostavime deka p∫q∫r∫p. Od uslovot na zada~ata imame deka
pqr=n(pq+pr+rp), a bidej}i p, q, r se zaemno prosti broevi (p, q i r se razli~ni
prosti broevi) sleduva deka n=kpqr, za nekoj priroden broj k. No toga{ gornoto
ravenstvo se transformira vo 1=k(pq+qr+rp) koe e nevozmo`no. Ako to~no dva
od prostite broevi se ednakvi me|usebe, bez gubewe od op{tosta mo`eme da
pretpostavime deka p=q∫r, ravenstvoto se transformira vo pr=n(p+2r). Od
NZD(r,p)=1 sleduva deka NZD(r,p+2r)=1, odnosno sleduva deka r e delitel na n.
Neka n=r·m za nekoj priroden broj m. Ravenstvoto se transformira vo
p=m·(p+2r) koe e nevozmo`no bidej}i m·(p+2r)¥p+2r>p. Spored toa zaklu~uvame
deka p=q=r, od kade se dobiva p=q=r=3 i n=1.
2
497. Od Heronovata formula imame 16P =L(L-2a)(L-2b)(L-2c)...(1) kade {to P e
plo{tina, a L perimetarot na triagolnikot ABC. Da pretpostavime deka P e
cel broj. Toga{ od (1) sleduva deka L e paren broj. Zna~i a, b i c se parni
broevi ili od niv eden e paren, a drugite dva se neparni broevi. Ako a=b=c=2
toga{ P = 3 odnosno P ne e cel broj. Ako a=2 i b∫c toga{ b - c ¥2 и ne va`i
neravenstvoto pome|u stranite na triagolnikot ABC. Ako a=2 i b=c toga{ se
2 2
dobiva P =b -1, odnosno (b-P)(b+P)=1, {to e nevozmo`no ako b и P se prirodni
broevi. 498. Ne, deliv e so 11.
499. 1 i 2. Zamenuvajki za n=1, gi dobivame broevite (11,5,2); za n=2 broevite
(29,11,5); za n=3 broevite (59,21,10); za n=4 broevite (101,35,17) i za n=5 se
dobivaat broevite (155,53,26). ]e doka`eme deka za n>3, pome|u trite broja
postoi broj deliv so 5. Brojot n mo`e da se zapi{e kako 5k, 5k≤1 ili 5k≤2.
2 2
Ako n=5k toga{ 6n +5=5(30k +1) ne e prost broj.
2 2
Ako n=5k≤1 toga{ 2n + 3=5(10k ≤4k +1) ne e prost broj.
2 2
Ako n=5k≤ 2 toga{ n +1 = 5(5k ≤4k+1) ne e prost broj.
Ostanuva mo`nosta eden od trite broja da e 5.
2 2 2
6n +5 ne mo`e da e 5 za nœN.2n +3=5 za n=1 i n +1=5 za n=2.
500. 3. 501. Ne postoi takvo p.
p p
502. 2 i 5. Upatstvo: 7 ª -13(modp) i 7 ª7(modp).
503. p=3. Uslovite na zada~ata se zadovoleni za p=3. Site prosti broevi
pogolemi od 3 se od oblik 6k-1 ili 6k+1. Ako p=6k-1 toga{ p+10=3(2k+3) e
slo`en broj. Ako p=6k+1 toga{ p+20=3(2k+7) e slo`en broj. Zna~i edinstveno
re{enie e p=3.
505. Da pretpostavime deka p ne e stepen na 2. Toga{ p=k(2n+1) kade {to
p k 2n+1 k
1§k<p. Brojot a +1=(a ) +1 e deliv so a +1 i pritoa od a>1 i k<p sleduva
k p p
deka 1<a +1<a +1, od {to zaklu~uvame deka a +1 ne e prost bro,j {to e
sprotivno na uslovot na zada~ata. Zna~i, pretpostavkata ne e to~na odnosno p
e stepen na 2. 506. p=5, q=3 .
507. Odgovorot e ne, Bidej}i 2n+1 e to~en kvadrat, sleduva deka i

176
Odgovori, upatstva i re[enija
2 2
8n+4=4(2n+1) e to~en kvadrat. Neka 8n+4=m i 3n+1=k , m,kœN. Jasno e deka
2 2
m>k. Toga{ 5n+3=m -k =(m+k)(m-k). Brojot 5n+3 e prost ako i samo ako m-k=1
2
odnosno m=k+1. No toga{ 5n+3=m+k=2k+1§k +2=3n+3 odnosno 5<3, {to
pretstavuva kontradikcija. 508. a) p=13. 510. 5.
511. I re{enie:
e{enie: Ako p e prost broj pogolem od 3 toga{ p=6k≤1, kœN i
2
p -1=12k(3k≤1).
2
i) Ako k e paren toga{ jasno 24|p -1;
2
ii) Ako k e neparen toga{ 3k≤1 e paren i povtorno 24|p -1.
2
II re{enie:
e{enie: ]e doka`e deka p -1 se deli so 8 i so 3 [NZD (3,8)=1].
2
Od p -1=(p-1)(p+1) i p e prost broj pogolem od 3 sleduva deka i dvata broja
p-1 i p+1 se posledovatelni parni broevi, pri {to edniot se deli so 4. Zna~i
nivniot proizvod e deliv so 8. Od trite posledovatelni broevi p-1, p i p+1
barem eden e deliv so 3. Kako p e pogolem od 3 sleduva deka eden od broevite
2
p-1 i p+1 e deliv so 3, pa i p -1 e deliv so 3. 512. p=3.
1 1 1 2 2 2 2
513. p=2. Ravenkata = 2 + 2 e ekvivalentna so ravenkata a b =p(b +a ).
p a b
2 2 2 2 2 2
Od p|a b sleduva deka p|a ili p|b odnosno p|a ili p|b. Bidej}i a b e to~en
2 2 2 2 2
kvadrat, sleduva deka p |a b , od kade zaklu~uvame deka p|(b +a ). Bidej}i p e
delitel na eden od broevite a i b sleduva deka p e delitel i na drugiot broj.
Zna~i p|a i p|b {to povlekuva deka a¥p i b¥p.
1 1 1 1 1 2
Toga{ = 2 + 2 § 2 + 2 = 2 , odnosno p§2. Jasno e deka p=2 go ispolnuva
p a b p p p
uslovot na zada~ata (a=2, b=2).
514. Upatstvo: 101010...101= 11 { ...1·(10100 -1099 +...-10+1). 515. Iskoristete
101
9 ⋅ (10 p
− 10 1 ) 8 ⋅ (10 2p
− 10 2
) + ... + 1 ⋅ (10
i primenete ja
9p
− 10 9
)
N = +
9 9 9
malata teorema na Ferma.
y y
518. Od x +1=z sleduva deka x i z imaat razli~na parnost. Bidej}i x i z se
y
prosti broevi, sleduva deka z=2 ili x=2. Ako z=2 toga{ x =1. Ovaa ravenka
nema re{enie vo mno`estvoto na prosti broevi (x=1). Ako x=2 toga{ po~etnata
y
ravenka se transformira vo 2 +1=z. Za y=2 se dobiva z=5. Ako y>2 toga{ y e
y y y y 1 y 2
neparen prost broj, 2 +1>3 i 3|2 +1 [2 +1=(2+1)(2 - -2 - +...+1)]. Zna~i, za y>2, z
ne e prost broj. Dobivame edno edinstveno re{enie na ravenkata e x=y=2 i
z=5.

Tema 10:
10: Kompleksni broevi

 π π
519. -4. 520. 1 + i 3 = 2 cos + isin ;
 3 3
 1 + i 3  = 2  cos π + isin π  = 6 + i 2 ;
   
 1  6 6 2 2

177
Odgovori, upatstva i re[enija

 1 + i 3  = 2  cos 7 π + isin 7 π  = − 6 − i 2 ;
   
 2  6 6  2 2
 5π 5π 
1 − i 3 = 2 cos + isin ;
 3 3 
 1 − i 3  = 2  cos 5 π + isin 5 π  = − 6 + i 2 ;
   
 1  6 6  2 2
 1 − i 3  = 2  cos 11π + isin 11π  = 6 − i 2 ;
   
 2  6 6  2 2
i 2

 6
A = 1+ i 3 + 1− i 3 =  .
− 6
− i 2

2  2 1+ i
+ 1 + i . Neka z = 2
521. toga{ z = i . Ako so Ѕ go ozna~ime

2  2  2
baraniot zbir, dobivame:
S = z + i + z ⋅ i + i2 + z ⋅ i2 + i3 + zi3 + ... + z ⋅ i1008 + i1009
( ) (
S = z ⋅ 1 + i + i 2 + .... + i1008 + i ⋅ 1 + i + i 2 + ... + i1008 )
S = (z + i) ⋅ [(1 + i − 1 − i) + (1 + i − 1 − i) + ... + (1 + i − 1 − i) + 1]

S = z+i =
1+ i
+i =
1 + i + 2i
=
1
+
( 2 +1 )i = 2 
+ 1+
2 
i.
2 2 2 2 2  2 
522. a) 6z6=1, 523. n=97.
z 2 2
524. -1. Neka = u = x + y ⋅ i . Toga{ u = u − 1 = 1. Sleduva deka x +y =1 i
w
2 2 1
(x-1) +y =1. So odzemawe na ovie dve ravenstva dobivame deka x = ,
2
673
1 3 2019  3
y = ± 3 ⋅ i , odnosno u = ± ⋅ i . Sega,  
z
 1 3  
= (− 1) = −1 .
673
2 =  ± ⋅ i
2 2 w  2 2  
 
525. Neka u=cosx+isinx, v=cosy+isiny, w=cosz+isinz. Od uslovot na zada~ata go
dobivame ravenstvoto u+v+w=0 …(1). Ako kompleksnite broevi u, v, w gi
pretstavime geometriski kako vektori, toga{ od (1) sleduva deka tie
formiraat triagolnik. Od u = v = w = 1 sleduva deka toj triagolnik e

ramnostran, {to povlekuva deka aglite me|u vektorite se ednakvi na ±
3
2π 2π
odnosno y = x ± , z=y± . Od poslednoto sleduva sin3x=sin3y=sin3z.
3 3
526. a = ± 2 , z1= 2 ·i, z2=- 2 ·i, z3=1+2·i, z4=1-2·i. 527. -2.

178
Odgovori, upatstva i re[enija

1 1 1 1
528. Upatstvo: Vovedete smeni x = , y= , z= i t= .
a b c d
1 + 2i
529. z2021=-2+i. So direktna presmetka dobivame zë=1-i, z1 = , z2=-2+i,
5
z3=zë=1-i. Od poslednoto sleduva deka zi+3k=zi, za iœ{0,1,2} i kœN. Ottuka
z2021=z2=-2+i.
532. Neka z=cosx+i·sinx. Od Moavrovata formula sleduva deka
n n
z =|z| ·(cosnx+i·sinnx)=cosnx+i·sinnx. Analogno se dobiva
n
z cosnx + i ⋅ sinnx 1
2n
1+z =2cosnx·(cosnx+i·sinnx). Kone~no = = .
2n 2cosnx ⋅ (cosnx + i ⋅ sinnx) 2cosnx
1+ z
533. Od uslovot na zada~ata imame deka z1 ⋅ z1 = z2 ⋅ z2 = z3 ⋅ z3 = ... = zn-1 ⋅ zn-1
= zn ⋅ z n = r 2 . Neka dadeniot izraz go ozna~ime so w. Toga{,

w=
(z ⋅ z ⋅ z
1 1 2 )( ) ( )(
+ z 2 ⋅ z 2 ⋅ z1 ⋅ z 2 ⋅ z 2 ⋅ z 3 ⋅ + z 3 ⋅ z 3 ⋅ z 2 ⋅ ... ⋅ zn -1 ⋅ zn -1 ⋅ zn + zn ⋅ zn ⋅ zn-1 ⋅ zn ⋅ zn ⋅ z1 + z1 ⋅ z1 ⋅ z n )
(
z1 ⋅ z1 ⋅ z2 ⋅ z 2 ⋅ ... ⋅ zn ⋅ zn ⋅ z1 ⋅ z 2 ⋅ ... ⋅ zn )
=
(r 2
)( ) (
⋅ z 2 + r 2 ⋅ z 1 ⋅ r 2 ⋅ z 3 ⋅ +r 2 ⋅ z 2 ⋅ ... ⋅ r 2 ⋅ z n + r 2 ⋅ z n-1 ⋅ r 2 ⋅ z1 + r 2 ⋅ z n)( )
2 ⋅ r 2 ⋅ ... ⋅ r 2 ⋅
r1 4243 (z1 ⋅ z 2 ⋅ ... ⋅ zn )
n − pati
(z1 + z 2 )⋅ (z 2 + z 3 )⋅ ... ⋅  z n − 1 + z n  ⋅ (z n + z1 )
=
z 1 ⋅ z 2 ⋅ ... ⋅ z n

=
(z1 + z 2 )⋅ (z 2 + z 3 )⋅ ... ⋅ (z n − 1 + z n )⋅ (z n + z1 )
z1 ⋅ z 2 ⋅ ... ⋅ z n

=
(z1 + z2 )⋅ (z2 + z3 )⋅ ... ⋅ (zn − 1 + zn )⋅ (zn + z1)
z1 ⋅ z 2 ⋅ ... ⋅ zn
w = w...(1) . Od (1) sleduva deka w e realen broj. 534. log2S=2
1008
. 535. 2022. 537.
ab + bc + ca
2
(
= (ab + bc + ca ) ⋅ ab + bc + ca = (ab + bc + ca ) ⋅ ab + bc + ca) ( )
= abab + abbc + abca + bcab + bcbc + bcca + caab + cabc + caca
2 2 2 2 2 2 2 2 2 2 2 2
= a ⋅ b + ac ⋅ b + bc ⋅ a + c a ⋅ b + b ⋅ c + ab ⋅ c + bc ⋅ a + ab ⋅ c + a ⋅ c
= 1 + ac + bc + c a + 1 + ab + bc + ab + 1
= aa + bb + c c + ac + c a + ab + ba + bc + cb
( )
= (a + b + c ) ⋅ a + b + c = (a + b + c ) ⋅ a + b + c = a + b + c . ( ) 2

539. Lesno se poka`uva deka ako edniot od broevite z1, z2, z3 e 0 toga{ i
preostanatite dva se ednakvi na 0. Zatoa pretpostavuvame deka z1, z2, z3 se
2
site razli~ni od 0. Od 0=z1z2+z2z3+z3z1=z1(z2+z3)+z2+z3=-z1 +z2+z3 se dobiva
2 3
z1 =z2z3, odnosno z1 =z1z2z3. Poradi simetri~nosta na ravenkite, analogno se
3 3 3 3 3
dobiva z2 =z1z2z3 i z3 =z1z2z3. Zna~i, z1 =z2 =z3 od kade {to sleduva deka
z = z = z .
1 2 3

179
Odgovori, upatstva i re[enija

540. Od uslovot na zada~ata imame deka z 1 ⋅ z1 = z 2 ⋅ z 2 = z 3 ⋅ z 3 = r 2 . Neka


z1 ⋅ z 2 + z 2 ⋅ z 3 + z 3 ⋅ z1
w= . Mno`ej}i gi broitelot i imenitelot so z1 ⋅ z 2 ⋅ z 3
z1 + z 2 + z 3
z 1 ⋅ z 2 ⋅ z1 ⋅ z 2 ⋅ z 3 + z 2 ⋅ z 3 ⋅ z 1 ⋅ z 2 ⋅ z 3 + z 3 ⋅ z1 ⋅ z 1 ⋅ z 2 ⋅ z 3
dobivame w =
(z1 + z 2 + z 3 ) ⋅ z1 ⋅ z 2 ⋅ z 3
4 4 4
r ⋅ z 3 + r ⋅ z1 + r ⋅ z 2 z1 + z 2 + z 3
w= 2 2 2
= r2 .
r ⋅ z 2 ⋅ z 3 + r ⋅ z 1 ⋅ z 3 + r ⋅ z1 ⋅ z 2 z1 ⋅ z 2 + z 2 ⋅ z 3 + z 3 ⋅ z1
2 1 2
Zna~i, w = r ⋅ od kade sleduva deka w ⋅ w = r . Od poslednoto sleduva
w
deka w = r .

541. 1+ z + z2 z œR ako z œR. Izrazot


= 1+ 2 ⋅
1− z + z 2
1− z + z 2
1− z + z 2
z 1 1
2
= − 1 + z ∈ R ako z + ∈ R . Poslednata relacija e ekvivalentna so
1− z + z z z

ravenstvoto z + = z +
1 1
z
2
( )(
odnosno z − z 1 − z = 0 , od kade dobivame deka )
z
z = z ili |z|=1. Bidej}i z ne e realen broj, sleduva deka |z|=1.
542. Neka z1=cosx+i·sinx, z2=cosy+i·siny i z3=cosz+i·sinz. Jasno e deka
|z1|=|z2|=|z3|=1. Od z1+z2+z3=(cosx+cosy+cosz)+i·(sinx+siny+sinz)=0 imame
2 2 2 2 1 1 1
z1 +z2 +z3 =(z1+z2+z3) -2(z1z2+z1z3+z2z3)= − 2z 1z 2 z 3 ⋅ ( + + )
z 3 z 2 z1
z3 z2 z1
= −2z 1z 2 z 3 ⋅ ( 2
+ 2
+ 2
) = −2z 1z 2 z 3 ⋅ ( z 1 + z 2 + z 3 )
z3 z2 z1
= −2z 1z 2 z 3 ⋅ ( z 1 + z 2 + z 3 ) = 0 ...(1).
2 2 2
Od druga strana z1 +z2 +z3 =cos2x+i·sin2x+cos2y+i·sin2y+cos2z+i·sin2z
=(cos2x+cos2y+cos2z)+i·(sin2x+sin2y+sin2z)...(2).
Od (1) i (2) sleduvaat baranite ravenstva.
2 2 2 2
543. 443. Od ravenstvoto (z1+z2+z3) =z1 +z2 +z3 +2(z1z2+z2z3+z3z1) sleduva
2 2 2
z1z2+z2z3+z3z1=-1. Od ravenstvoto (z1+z2+z3)[(z1 +z2 +z3 -z1z2-z2z3-z3z1)
3 3 3
=z1 +z2 +z3 -3z1z2z3 sleduva z1z2z3=1. Od Vietovite vrski imame deka broevite
3 2
z1, z2 i z3 se nuli na funkcijata f(x)=x -x -x-1.
n n n
Neka Sn=z1 +z2 +z3 . Toga{ od uslovot na zada~ata imame deka S1=1, S2=3 i
S3=7. Za n>0 imame deka Sn+3-Sn+2-Sn+1-Sn
n+3 n+2 n+1 n n+3 n+2 n+1 n n+3 n+2 n+1 n
=(z1 -z1 -z1 -z1 )+(z2 -z2 -z2 -z2 )+(z3 -z3 -z3 -z3 )
n 3 2 1 n 3 2 1 n 3 2 1
=z1 (z1 -z1 -z1 -1)+z2 (z2 -z2 -z2 -1)+z3 (z3 -z3 -z3 -1)
n n n
=z1 ·f(z1)+z2 ·f(z2)+z3 ·f(z3)=0. Zna~i, ja dobivame rekurentnata vrska
Sn+3=Sn+2+Sn+1+Sn. Taka dobivame deka S4=11, S5=21, S6=39, S7=71, S8=131,
S9=241 i S10=443.

180
Odgovori, upatstva i re[enija

Tema 11
11: Kvadratni ravenki i kvadratni funkcii

2
544. x=0 i x = − . 545. x1=-1, x2=12. 546. x=6, y=5. 547. Da pretpostavime deka
33
parot prirodni broevi (x,y) e re{enie na dadenata ravenka. Od ravenstvoto
2 2
x(x+1)=y sleduva deka y e prizvod na dva vzaemno prosti broja x i x+1.
2 2
Ravenstvoto e ispolneto samo ako x i x+1 se to~ni kvadrati. Za x=k i x+1=j ,
2 2
j -k =(x+1)-x=1. Zna~i (j-k)(j+k)=1 {to e nevozmo`no bidej}i j+k¥2 i
2 2
protivre~i na pretpostavkata parot (x,y) da e re{enie na ravenkata x +x=y .
548. m=0, n=3; m=3, n=3; m=3, n=6. 549. x=0, x=1. 550. a) 1, 3 ± 5i b) x1,2=-2≤ 6 .
2 2 2
551. 5, 34, 125. Upatstvo: (n+4) <n +25n+19<(n+13) .
2 2
552. -25, -6, 1, 5, 10, 15, 19, 26 i 45. n -20n+244=(n-10) +144. Zamenuvaj}i
2 2
n-10=m, za mœZ, dobivame deka m +144=q . Da pretpostavime deka q e
pozitiven cel broj. Toga{ 144=(q-m)(q+m) i od toa {to q>m imame deka
q-m>0, pa mora q+m>0. Neka a i b se re{enija na ravenkata aÿb=144. Od ovde
b−a b+a
dobivame deka q-m=a i q+m=b, odnosno m = i q= . Bidej}i m,qœZ,
2 2
sleduva deka a i b se so ista parnost, no so ogled na toa {to aÿb=144=122,
mora a i b da se parni. Neka a=2U i b=2V. Sega aÿb=4UÿV=144, odnosno UÿV=36.
2 2
Zna~i UÿV =3 ÿ2 ima 3ÿ3=9 deliteli, odnosno 9 vrednosti za a i b, a isto tolku
vrednosti za m i q, odnosno n.
.
553. Od vietovite formuli imame deka x1+x2=-p; x1 x2=q. Toga{
.
198=p+q=-(x1+x2)+x1 x2=(x1-1)(x2-1)-1, odnosno (x1-1)(x2-1)=199. Bidejki 199 e
prost broj, a x1-1 i x2-1 se celi broevi, gi dobivame sistemite x1 − 1 = 1 ili
x2 − 1 = 199
x 1 − 1 = −1 , ~ii re{enija se (2,200) i (0,198). Za re{enieto (2,200) dobivame

x 2 − 1 = −199
2 −8
p=-202, q=401. Za re{enieto (0,198), dobivame p=-198, q=0. 554. р= ; q = .
3 3
2
555. m=2. 556. q=1. Ravenkata x -x+q=0 ima realni koreni ako
1
diskriminantata D=1-4q¥0, odnosno q§ . Od Vietovite formuli imame
4
4 4 2 2 2 2 2 2 2 2
x1+x2=1, x1ÿx2=q. Toga{ x1 +x2 =(x1 +x2 ) -2x1 x2 =[(x1+x2) -2x1x2] -2(x1x2)
2 2 2 2
=(1-2q) -2q =2(q-1) -1. Funkcijata f(q)=2(q-1) -1, na intervalot (-¶,1] opa|a,
4 4
{to zna~i deka najmalata vrednost na izrazot x1 +x2 se dostignuva za q=1.
557. 0. 558. Bez gubewe na op{tosta da pretpostavime deka a>b. Toga{
r-a<r-b. Bidej}i r e koren na dadenata ravenka sleduva deka (r-a)(r-b)=-1;
a,b,rœZ. Edinstveno re{enie e r-a=-1 i r-b=1. Sobiraj}i gi poslednite dve
ravenstva dobivame deka a+b=2r.
560. Upatstvo: iskoristete gi neravenstvata (x1-2)(x2-3)<0; (x1-3)(x2-2)<0 i
Vietovite formuli.
2
561. Neka f(x) =ax +bx+c. ]e gi razgledame slednite dva slu~ai:

181
Odgovori, upatstva i re[enija

x = 0
1) Ako c=0 toga{ f(x) =0 ako i samo ako  . Od uslovot 2a+3b=0 dobivame
x = − b
 a
− b 2 i xœ(0,1).
=
a 3
2
2) Ako c∫0 toga{ f(0)=c, f   =
4 2 2 −c
. Dobivme deka na
a+ b+c = (2a + 3b) + c =
3 9 3 9 3
2
kraevite od intervalot [0, ], funkcijata f(x) prima razli~ni vrednosti po
3
znak, {to uka`uva na toa deka grafikot na kvadratnata funkcija ja se~e
x-oskata na intervalot (0,1), odnosno f(x) ima nula na (0,1).
563. Od uslovot za koeficientite, ravenkata mo`e da ja zapi{eme vo oblik
ax²+bx+c=x·1=x(a+b+c), odnosno, sreduvaj}i go izrazot, (x-1)·(ax-c)=0...(1). Neka
c
ravenkata ima koren x1œ(0,1). Od (1) i 0<x1<1 imame x1= <1, a ottuka c<a.
a
Toga{ 1=a+b+c<a+b+a=2a+b .
c
Obratno, neka 2a+ b>1, zna~i 2a+b>a+b+c. Dobivame a>c, od kade 0< <1 . Od
a
c
(1) sleduva deka x1= e koren na ravenkata i istiot zadovoluva uslov 0<x1<1.
a
564. Odzemaj}i gi dvata trinoma dobivame
2 2 2
ax +bx+c-((c-b)x +(c-a)x+(a+b))=(a+b-c)·(x +x–1)...(1).
Dadenite trinomi imaat zaedni~ki koren so trinomot (1). Sleduva deka ili
a+b-c=0 ili zaedni~kiot koren se sovpa|a so eden od korenite na trinomot
2
x +x–1, koi se iracionalni broevi. Vo prviot slu~aj a+b+2c=3c e deliv so 3.Vo
2 2
vtoriot slu~aj neka x0 e zaedni~ki koren za ravenkata ax +bx+c=0 i x +x–1=0.
2 2 2
Toga{ va`i ax 0 +bx0+c+c·(x0 +x0–1)=0 odnosno (a+c)x0 +(b+c)x0=0…(2).
Bidej}i x0 e iracionalen broj sleduva deka ravenstvoto (2) e mo`no samo ako
a+c=0 i b+c=0, odnosno a+b+2c=0 i e deliv so 3.
1 2 2
565. x1= . Od (x-x1)·(x-x2)=(x-x1)·(x-4x1)=x -5x1·x+4x1 i od uslovot za
4
2
koeficientite dobivame 3=2(4x1 +5x1).Od tuka se dobiva pozitivna vrednost
1
za x1= . 566. 6. 567. (0,0), (0,-1), (-6,8), (-6,9). 568. a=b=2; a=1, b=5; a=2, b=3.
4
2
569. p(3q-p ). 570. kœ(1,¶). 571. 40 km/~. 572. a+b+c=-3.
574. Ako b=0 toga{ d=0, c=-a, a-proizvolen realen broj. Ako b≠0, toga{ a=c=1
i b=d=-2. 578. 10. 579. a+b+c=5.
580. a=-4, b=2 Upatstvo: f(4)-f(2)§4; f(0)-f(2)§4; f(2)=b-4¥-2.
2 2
581. Upatstvo: Definiraj funkcija f(x)=x +bx+ac. f(c)=c +bc+ac=(a+b+c)·c<0.
Toga{ fukcijata f(x) ima presek so x-oska.
2 2 2 2
582. Dadenata ravenka e ekvivalentna so a x +abx+ac=0. Vo a x +abx+ac=0
2
stavame y=ax i dobivame y +by+ac=0. Ako x e racionalen koren toga{ i y e
2
racionalen koren. Bidej}i b i ac se celi koeficienti i y +by+ac=0 e ravenka

182
Odgovori, upatstva i re[enija
so celi koeficienti. Ako poslednata ravenka ima racionalen koren toga{ toj
mora da e cel broj (vidi zada~a 89).
Neka e toa y1, toga{ y2=-b-y1. Od Vietovite formuli imame y1+y2=-b i
y1·y2=ac. Mno`ej}i gi ravenstvata dobivame abc=-(y1+y2)y1y2. Bidej}i barem
eden od korenite (y1+y2), y1, y2 e paren broj, sleduva deka i abc e paren broj.
Toga{ barem eden od koeficientite a, b, c e paren broj.
2
583. p=2, q=5. Jasno e deka ako x e re{enie na ravenkata px -qx+p=0, toga{
2 m
x>0 (za x<0, levata strana px -qx+p>0). Neka x= , kade m i n se prirodni
n
broevi, NZD(m,n)=1. Zamenuvaj}i vo dadenata ravenka dobivame deka
2
m(qn-pm)=pn , od kade zaklu~uvame deka m|p. Zna~i m=1 ili m=p, pri {to i
1
n=1 ili n=p. Od ovde imame x=1 ili x=p ili x= . Ako x=1 toga{ q=2p, {to e
p
1
kontradikcija so toa deka q e prost broj. Ako x=p ili x= toga{ se dobiva
p
2
q=p +1. Od poslednoto ravenstvo dobivame deka mora p da bide paren broj.
1
Zna~i p=2 i q=5 . 584. c = .
10
585. -12. Upatstvo:sekoj izraz vo zagradite ima vrednost -1.
586. (-4,4), (2,1), (-1,-2), (1,-1), (0,-4), (0,-1).
587. 237. Od ravenstvoto (111)B=(aabbcc)6 dobivame deka
2 4 2 2
B + B + 1 = 6 ⋅ 7a + 6 ⋅ 7b + 7c ...(1). Zna~i B + B + 1 e deliv so 7. Ova va`i za
B = 7n + 2 i B = 7n + 4 . Zamenuvaj}i gi vo (1) gi dobivame slednite ravenki:
2 2
7n + 5n + 1 = 1296a+ 36b + c ...(2) i 7n + 9n + 3 = 1296a + 36b + c ...(3) .
Ravenkite (2) i (3) }e gi razgledame na intervalot [1296a;1296a+185]
(36b + c ≤ 36 ⋅ 5 + 5 = 185),(a ≠ 0) . Neka а=1 toga{ 2592 ≤ 7n2 + 5n + 1 ≤ 1481.
Ravenstvoto (2) e zadovoleno samo za n=14, od kade dobivame deka b=4 i c=3,
odnosno B=100. Za aœ{2,3,4,5}, ravenkata (2) nema re{enie vo broen sistem so
osnova 6. Razgleduvaj}i ja ravenkata (3), za а=2 dobivame
2592 ≤ 7n2 + 9n + 3 ≤ 2777. Ravenstvoto e zadovoleno samo za n=19 od kade b=3
i c=1, odnosno B=137. Za aœ{3,4,5}, ravenkata (3) nema re{enie vo broen sistem
so osnova 6. Zna~i baraniot zbir e 100+137=237.

Tema 12: Plo[tina na ram,ninski figuri


figuri

3600 2
588. a) P= , b) P=8. 591. P=14π cm . 592. 90.
289
593. a) Neka M, N, P se sredinite na stranite AB, BC, CD, soodvetno (crt. 140).
Ja prodol`uvame srednata otse~ka PN do to~ka K za koja PK = AB .

183
Odgovori, upatstva i re[enija

Toga{ MK = AN i CK = PB odnosno
triagolnikot CMK e formiran od te`i{nite
linii na triagolnikot ABC.
b) Ako so L ja ozna~ime sredinata na stranata
MK, toga{:
1 1 3
P∆CMK=2(P∆BCM-P∆MLB)=2( P∆ABC- P∆ABC)= P∆ABC.
2 8 4
Zna~i, P∆CMK:P∆ABC=3:4. Crte` 140

v) P∆ABC=30, S=S1+S2+S3+...
3 3 3
Od S1=30, S2:S1=3:4 sleduva S2= S1, ..., Sk= Sk-1, ... odnosno S2= S1;
4 4 4
 3
2 3  3 3  2 2
S3=   S1; S4=   S1, ... , S=S1ÿ  1 + +   + ...  =120. 594. P =
3 R 3
.
 4 4  4 4  4
 
a b
596. PóABC : Pó A 1B1C1 = 1 : 7. 598. + + 2 . 599. PóMBN = S1S3 (S1 + S2 ) (S2 + S3 ) .
b a 2
S2 (S2 - S1S3 )

Upatstvo: najdi ja plo{tinata na triagolnikot MON, odnosno PóMON = S1 ⋅ S3 i


S2
P∆ACM P∆AMN
iskoristete go odnosot = .
P∆ MBC P∆ MBN
601. Bez gubewe na op{tosta neka {rafiranite triagolnici imaat plo{tina 1
a triagolnicite BOZ,XCO i AOY neka imaat plo{tina x, y, z, soodvetno.
Od toa {to triagolnicite AZO i ZBO imaat ista visina va`i odnosot
1 : x = AZ : BZ . Analogno, za triagolnicita AZC i ZBC dobivame
(2 + z) : (1 + x + y) = AZ : BZ .
1+ y
Od poslednite dve ravenstva dobivame x= …(1). So analogno
1+ z
1+ z 1+ x
rasuduvawe doa|ame do ravenstvata y = …(2) i z =
…(3). Mno`ej}i
1+ x 1+ y
gi ravenstvata (1), (2) i (3) dobivame deka x·y·z=1. Sega, da pretpostavime deka
y¥1…(4) i y¥z. Od poslednoto neravenstvo va`i 1+y¥1+z odnosno
1+ y 1
x= ¥1. Zna~i, y¥1 i x¥1 toga{ z = §1§x. Taka, 1+z§1+x od kade
1+ z xy
1+ z
zaklu~uvame deka y = §1…(5). Od neravenstvata (4) i (5) zaklu~uvame
1+ x
2
deka y=1. Od ravenstvoto (3) dobivame x=z.Toga{ 1=xyz=x od kade x=1=z.
602. P = ( P1 + P2 + P3 ).
2

184
Odgovori, upatstva i re[enija
603. Od sli~nosta na triagolnicite JNH, MFG, DET i NTM so triagolnikot
P1 DE P2 SB P3 AM
ABC (crt. 141) sleduvat ravenstvata = , = , = i
P AB P AB P AB
P4 ES
= . Delej}i go ravenstvoto
P AB
AB = AM + SB + DE + MD + ES so AB
AM SB DE ES
dobivame 1= + + +2⋅
AB AB AB AB
P1 P2 P3 P4
odnosno 1 = + + + 2⋅ .
P P P P
Crte` 141
Mno`ej}i go poslednoto ravenstvo so P se dobiva baranoto ravenstvo.
604. Neka Pa, Pb i Pc se plo{tini na trite triagolnici koi gi sodr`at
temiwata A, B i C, a P-plo{tinata na ~etvrtiot triagolnik. Jasno e deka
P∆ABB1 + P∆BCC1 + P∆CAA 2 = P∆ABC - P + Pa + Pb + Pc .
Od toa {to P∆ABC = P∆AOB + P∆BOC + P∆COA = P∆ABB1 + P∆BCC1 + P∆CAA 2 sleduva
2 2
deka P=Pa+Pb+Pc, {to treba da se doka`e. 605. P = k − c . 606. PABC =3S.
4
1 2
CA ⋅ CB ⋅ sinγ
CQ ⋅ CN ⋅ sinγ 3 3 2
Упатство: P∆CQN = = = P∆ABC . 607. P=32.
2 2 9
608. Neka x, y i z se rastojanijata od to~kata S do stranite na triagolnikot
ABC (crt. 142). Jasno e deka zbirot ax+by+cz=2PóABC e konstanten broj.
Koristej}i go neravenstvoto me|u aritmeti~ka i geometriska sredina
3
dobivame (ax ) ⋅ (by ) ⋅ (cz ) ≤  ax + by + cz  , pri {to ravenstvo va`i ako
 3 
ax=by=cz i toga{ vrednosta na proizvodot (ax)·(by)·(cz) e najgolema, a poradi
konstantnosta na a, b i c sleduva maksimalnost na proizvodot xyz .
Sega, }e go doka`eme tvrdeweto ,,Ako
ax=by=cz toga{ to~kata S se nao|a vo
presekot na te`i{nite linii”.
Neka A1 e to~ka vo koja pravata AS ja se~e
P∆ABA P∆SBA
stranata BC. Toga{ 1
= 1
…(1) od
P∆ACA P∆SCA
1 1

P∆ABA P∆ACA
1 1
kade se dobiva = Crte` 142
P∆SBA P∆SCA
1 1

P∆ABA P∆ACA
odnosno 1
−1 = 1
−1.
P∆SBA P∆SCA
1 1

185
Odgovori, upatstva i re[enija

P∆ABS P∆ACS P∆ABS P∆SBA


1
Ponatamu, sleduva deka = odnosno = …(2). Od
P∆SBA P∆SCA P∆ACS P∆SCA
1 1 1

BA 1 P∆ABA P∆ABS cz
ravenstvata (1) i (2) dobivame = 1
= = = 1 {to zna~i deka
CA 1 P∆ACA P∆ACS by
1
AA1 e te`i{na linija . Analogno se doka`uva deka S le`i na ostanatite
te`i{ni linii.
2
609. Spored heronova formula imame deka P =s(s-a)(s-b)(s-c) odnosno
1 2 1 1 s
P = s(s-a)(s-b)(s-c)§ [ +(s-a)+(s-b)+(s-c)] (spored neravenstvoto na
3 3 4 3
Ko{i). Pri konstantna plo{tina zbirot s +(s-a)+(s-b)+(s-c) e minimalen koga
3
s-a=s-b=s-c= s od kade dobivame deka a=b=c= 2s {to zna~i deka
3 3
triagolnikot e ramnostran .
2
610. Spored heronova formula imame deka P =s(s-a)(s-b)(s-c) odnosno
3
P =s(s-a)(s-b)(s-c)§s·  (s − a ) + (s − b) + (s − c )  = s .
4
2
Ravenstvo va`i ako
 3  27
s-a=s-b=s-c od kade sleduva deka a=b=c. Zna~i od
site triagolnici so perimeter 2s, najgolema
plo{tina ima ramnostraniot triagolnik i taa
s2 a2 3
plo{tina ima vrednost = .
3 3 4
611. ^etiriagolnikot MZCY e pravoagolnik i
óHSY~óSMQ~óQKZ. Bidej}i АS e simetrala na
SY AY
agolot α imame deka = = cos α (crt. 143).
SM AM
QZ P∆HSY P
Sli~no = cos β . Sleduva deka = ∆QKZ
QM P∆SMQ P∆SMQ
2 2
=cos α+cos β=1 . Crte` 143

1
Zna~i PóHSY + PóQKZ = PóSMQ. Od ovde dobivame deka PóHKC = PYMZC = PóABC .
2

613. Neka delovite vo triagolnikot ABC imaat plo{tini P1, P2, P3 i P4


(crt. 144). Bidej}i triagolnicite AMC i MBC imaat ista visina (od temeto C)
sleduva (P1 + P4 ) : (P2 + P3 ) = AM : MB ...(1).
Analogno za triagolnicite BNA i NCA dobivame
(P1 + P2 ) : (P3 + P4 ) = BN : NC …(2).

186
Odgovori, upatstva i re[enija
Od (1) i (2) imaj}i go predvid uslovot
AM : MB = BN : NC dobivame
(P1 + P4 ) : (P2 + P3 ) = (P1 + P2 ) : (P3 + P4 ) , koe
ravenstvo se transformira vo ravenstvoto
(P4 − P2 )(P1 + P2 + P3 + P4 ) = 0 , od kade sleduva
deka P2=P4.

Crte` 144

614. Od toa {to AE e simetrala na triagolnikot ABC (crt. 145), a AF e


simetrala na triagolnikot ADC imame EC : BE = AC : AB =cosα= DA : AC
= DF : FC . Zna~i, EC · FC = BE · DF =( BC - EC )·( CD - CF ) odnosno
BC · CD = BC · CF + EC · CD .
Mno`ej}i gi dvete strani od poslednoto
1
ravenstvo so sin®BCD dobivame deka
2
P∆BCD=P∆BCF+P∆ECD...(1).
No, P∆BCD=PCEGF+P∆BEG+P∆DFG...(2),
P∆BCF=PGECF+P∆BEG...(3) i
P∆ECD=PGECF+P∆DFG...(4).
Crte` 145
Zamenuvaj}i gi ravenstvata (2), (3) i (4) vo ravenstvoto (1) se dobiva baranoto
4 36 + 25 3
ravenstvo. 615. . 616. P = .
13 4

617. Neka P∆VGF = P , P∆FEC = P1 i P∆BDG = P2


(crt. 146). Ako H e dopirnata to~ka na BC i
pripi{anata kru`nica, toga{ CE = CH i
BD = BH , odnosno CB = CH + HB = CE + BD ...(1).
CB ⋅ r
P + P∆VFC + P∆VGB = P∆BCV = .
2
Crte` 146
Zamenuvaj}i go (1) vo prethodnoto ravenstvo, dobivame
CE ⋅ r BD ⋅ r
P + P∆VFC + P∆VGB = + .
2 2
CE ⋅ r BD ⋅ r
Imaj}i predvid deka = P∆VEC = P∆VFC + P1 i = P∆VBD = P∆VGB + P2 ,
2 2
imame P + P∆VFC + P∆VGB = P∆VFC + P1 + P∆VGB + P2 , odnosno go dobivame
baranoto ravenstvo.

187
Odgovori, upatstva i re[enija

618. Jasno e deka AE = AD ,


BE = BF i CF = CD
(crt. 147). Imame deka
CD ⋅ CF ⋅ sinγ
P∆CDF 2
=
P∆ABC AC ⋅ BC ⋅ sinγ
2
2
a+b+c 
2  − c
=
CD
=  2 
AC ⋅ BC ab

=
(a + b − c )2 …(1).
4ab
Analogno, va`at
ravenstvata
P∆ BFE
=
(a + c − b)2 …(2) i
P∆ABC 4ac
P∆AED
=
(b + c − a)2 …(3).
P∆ABC 4bc
Crte` 147
P∆CDF + P∆BFE + P∆AED
Sobiraj}i gi ravenstvata (1), (2) i (3) dobivame
P∆ABC
a 3 + b 3 + c 3 − a 2b − b 2c − c 2 a - ab 2 − bc 2 − ca 2 + 6abc
= …(4).
4abc
CF VF
Od sli~nosta na triagolnicite CVF i JCM imame odnosno =
JM CM
CF ⋅ CM = VF ⋅ JM …(5), a od sli~nosta na triagolnicite BFV i JMB imame
BF VF
= odnosno BF ⋅ BM = VF ⋅ JM …(6).
JM BM
CF BM
Od (5) i (6) va`i CF ⋅ CM = BF ⋅ BM odnosno = …(7).
BF CM
Koristej}i go ravenstvoto CM + BM = CF + BF dobivame
 BM   CF 
CM ⋅  + 1 = BF ⋅  + 1 …(8).
 CM   BF 
   

(
 CM 
) 
Od (7) i (8) se dobiva CM − BF ⋅  BM + 1 = 0 , odnosno CM = BF od kade pak
 
sleduva ravenstvoto CF = BM .

188
Odgovori, upatstva i re[enija

Analogno, va`at ravenstvata CT = AD i BK = AE . Od


CT ⋅ CM ⋅ sinγ b+c-a a+c-b

P∆CTM
= 2 =
AD ⋅ BF
= 2 2 =
(b + c - a) ⋅ (a + c - b) …(9).
P∆ABC AC ⋅ BC ⋅ sinγ AC ⋅ BC ab 4ab
2

Analogno, se dobivat ravenstvata


P∆BKM
=
(b + c - a) ⋅ (a + b - c ) …(10) i
P∆ABC 4ac
P∆AKT
=
(a + c - b ) ⋅ (a + b - c ) …(11).
P∆ABC 4bc
Sobiraj}i gi ravenstvata (9), (10) i (11) dobivame
3 3 3 2 2 2 2 2 2
P∆CTM + P∆BKM + P∆AKT a + b + c − a b − b c − c a - ab − bc − ca + 6abc
= …(12).
P∆ABC 4abc
Od ravenstvata (4) i (12) imame
P∆CDF + P∆ BFE + P∆AED = P∆CTM + P∆ BKM + P∆AKT …(13). Od ravenstvoto
P∆CDF + P∆ BFE + P∆AED + P∆EFD = P∆CTM + P∆ BKM + P∆AKT + P∆KMT sleduva baranoto
ravenstvo, koristej}i go pritoa ravenstvoto (13).
621. Spored crte` 148 imame P2+S2+P3+P1+S3=S1+P+S4...(1) i
P3+S1+P2+S4+P1=S2+P+S3...(2). Sobiraj}i gi
ravenstvata (1) i (2) imame
2(P1+P2+P3)+S1+S2+S3+S4=2P+S1+S2+S3+S4 od
kade sleduva
P=P1+P2+P3.

Crte` 148

622. Spored crte` 149 imame


P1+P2+S2=P+S1+S3...(1). Crte` 149

Od P+S2=P+S1+S3=S/2 (S-plo{tina na trapezot ABCD) sleduva deka


S2=S1+S3...(2). Od (1) i (2) se dobiva baranoto ravenstvo.

623. P=24. Neka P∆AQH = P∆FCN = S , a PABCD = P


P P
(crt. 150). Toga{, PBFDH = i zatoa 10S = ,
2 2
odnosno S =
P . Sega, P P.
PMNPQ = 4S = 4 =
20 20 5
Zamenuvaj}i za P=120 dobivame deka
PMNPQ = 24 .
Crte` 150
626. 25:1. Zna~i, golemiot kvadrat e so strana a=2 (crt. 151).

189
Odgovori, upatstva i re[enija
2 2
Ako so 2x ja ozna~ime stranata na pomaliot kvadrat, imame (2x+1) +x =2,
2 2 2 2
4x +4x+1+x =2, 5x +4x-1=0, 5x +5x-x-1=0, 5x(x+1)-(x+1)=0,
1
(5x-1)(x+1)=0. Zna~i, 5x=1 odnosno x = ;
5
2
2 4
y= = = 25
2 4 .
2
  25
5

1
627. . 628. Upatstvo: Doka`i deka BE = DF i
4
8R 3 r 3
EC = FC odnosno ABCD e kvadrat. 629. P= . Crte` 151
(R 2 + r 2 )
630. a) Triagolnikot APB simetri~no go
preslikuvame vo triagolnikot APM vo odnos na
pravata AP, a triagolnikot ADQ go preslikuvame vo
triagolnikot AQM vo odnos na pravata AQ
(crt. 152). Analogno, triagolnikot CPB go
preslikuvame vo triagolnikot CPN, a triagolnikot
CDQ vo triagolnikot CQN. Pritoa se dobiva deka
®PMQ+®QNP=®PMA+®AMQ+®QNC+®CNP=180°.

Crte` 152

No, od toa {to óPMQ@óPNQ sleduva deka ®PMQ=®QNP=90°. Zna~i


triagolnikot PMQ e pravoagolen od kade se dobiva deka
2 2 2 2 2
PQ = PM + MQ = BP + QD ;
b) P∆PAQ + P∆BCP + P∆QCD = P∆PAQ + P∆NCP + P∆QCN = PAPCQ − P∆PQN
= PAPCQ − P∆PQM = P∆PCQ + P∆APM + P∆AQM = P∆PCQ + P∆APB + P∆AQD
631. PMNPQ = 2P . Jasno e deka ~etiriagolnikot MNPQ e kvadrat (crt. 153). Od
5
sli~nosta na triagolnicite C1QD i AMD sleduva deka MQ = 2QD . Spored
uslovot od zada~ata imame deka odnosot na katetite
vo pravoagolnite triagolnici C1QD, AMD, AD1M e
1:3. Spored toa, AM = 3D 1M i DM = 3 AM = 9D 1M .
No, od toa {to
DQ = AM = 3D 1M sleduva
QM = DM − DQ = 9D 1M − 3D 1M = 6D 1M . Od pitagorova
teorema za triagolnikot AD1D dobivame deka
2
a 10a .
DD1 =   + a 2 =
3 3
Crte` 153

190
Odgovori, upatstva i re[enija

Spored toa, sleduva deka


10a
= DD1 = DQ + QM + MD1 = 10D1M , odnosno
3
10a .
D1M =
30
10a
Od toa {to QM = 6D1M dobivame deka QM = .
5
2
 10a  10a 2 2a 2 2P
Sega, PMNPQ =   .
 5  = 25 = 5 = 5
 
632. Neka PABCD=P i O e presekot na dijagonalite
(crt. 154). Od uslovot na zada~ata imame
P P
PóABC=PóOAB+PóOBC= , PóBCD=PóOCD+PóOBC= .
2 2
Crte` 154
1 1
Sleduva deka PóOAB=PóOCD odnosno bcsin®AOB=
adsin®COD od kade se
2 2
dobiva deka bc=ad. Na sli~en na~in se dobiva deka ab=cd. Mno`ej}i gi
poslednite dve ravenstva se dobiva b=d. Analogno se poka`uva deka a=c.
Zna~i, dijagonalite na ~etiriagolnikot se prepolovuvaat, pa ~etiriagolnikot
2
16R
ABCD e paralelogram. 633. P = . 634. 25:6p.
25
2
635. Ako dadenoto ravenstvo go podelime so (sin j)/4 kade j e agolot me|u
dijagonalite, toga{ dadenoto ravenstvo od
plo{tini pominuva vo ravensvoto
(AO ⋅ BO) + (CO ⋅ DO) = (BO ⋅ CO) + (DO ⋅ AO) ,
2 2 2 2

 2 2  2 2
odnosno  AO − CO  ⋅  BO − DO  = 0 od kade se
   
dobiva deka AO = CO ili BO = DO .

Crte` 155
636. Neka AB = a, CD = b, MN = x (crt. 155). Toga{ od uslovot na zada~ata va`i
x+a x +b
ravenstvoto ⋅ NF = ⋅ CE a ottuka
2 2
x + a CE
= ⋅
x + b NF

Neka KœMN, CK||AD, LœAB, LN||AD. Toga{,


KN CE x −b CE x +a x −b
= , = . Zna~i, = ,
LB NF a−x NF x+b a−x
2 2
a +b
odnosno x = . Crte` 156
2

191
Odgovori, upatstva i re[enija

638. Vo ~etiriagolnikot ABCD ja povlekuvame dijagonalata BD (crt. 156).


Toga{ PóNDM=PóNDA. Od ovde sleduva deka PóANM=2PóABD…(1). Analogno se
dobiva deka PóPQC=2PóBCD…(2).
Ako ja povle~eme dijagonalata AC, toga{ gi dobivame ravenstvata
PóDMQ=2PóADC…(3). PóNPB=2PóABC…(4). Sobiraj}i gi ravenstvata (1), (2), (3) i (4)
dobivame PóANM+ PóDMQ+ PóPQC+ PóNPB=4PABCD, od kade sleduva deka
PMNPQ=5PABCD.
DD1 + QQ 1
639. Od ravenstvoto PP1 = sleduva
2
1
ravenstvoto P∆MNP = ⋅ (P∆NCD + P∆BMQ ) …(1) (crt. 157).
2
Analogno se dobiva i ravenstvoto
1
P∆QMP = ⋅ (P∆PND + P∆ABQ ) …(2).
2
Crte` 157
1
Od ravenstvata (1) i (2) sleduva deka PMNPQ = ⋅ (PABMQ + PNCDQ ) . Toga{
2
1
3PMNPQ = PABMQ + PNCDQP + PMNPQ i ottuka PMNPQ = PABCD .
3

641. Lesno se zabele`uva deka va`i ravenstvoto


PKPLQ = PABCD − P∆KBC − P∆BLC − P∆ALD − P∆AKD + P∆BPC + P∆AQD (crt. 158).
Od toa {to L ja deli stranata CD vo odnos m:n,
imame deka P∆BLC: P∆BDC= m: (m+ n) odnosno
m
P∆BLC = P∆BCD . Analogno va`at ravenstvata
m+n
m m
P∆ALD = P∆ACD , P∆KBC = P∆ABC i
m+n m+n
m
P∆AKD = P∆ABD .
m+n
Crte` 158

Toga{
m m
PABCD − P∆KBC − P∆BLC − P∆ALD − P∆AKD = PABCD − P∆ABCD − P∆ABCD = 0
m+n m+n
i zamenuvaj vo (1) dobivame deka
PKPLQ = P∆BPC + P∆AQD.
644. Neka to~kite E, F, G i H se sredinite na
stranite AB, BC, CD i DA, soodvetno (crt. 159). Od
toa {to EH e sredna linija za triagolnikot ABD
1 1
va`i PóAEH= PóABD. Analogno va`i PóCFG= PóBCD.
4 4
Crte` 159

192
Odgovori, upatstva i re[enija

1 1 1 1
Ponatamu, PóAEH+ PóCFG= PABCD…(1). Sli~no dobivame deka PóBFE+ PóCDA=
4 4 4 4
PABCD...(2).
Od ravenstvata (1) i (2) se dobiva PABCD=2PEFGH= EG · HF ·sinα kade α e
agolot megu pravite EG i HF. Bidej}i sinα§1, dobivame deka PABCD§ EG · HF .
→ → → → → → → →
Jasno e deka EG = EA + AD + DG i EG = EB + BC + CG .
→ 1  → →
Sobiraj}i gi poslednite dve ravenstva imame EG = ⋅  AD+ BC od kade
2  
 
1 1
sleduva deka EG § ·( AD + BC ). Analogno se dobiva HF § ·( AB + CD ).
2 2
1 1 2
Zna~i, PABCD§ ( AD + BC )( AB + CD ). Od neravenstvoto ab§ (a+b) va`i
4 4
1 1 2
PABCD§ ÿ ÿ( AB + BC + CD + DA ) =1.
4 4

645. PóAMB=PóBMC; PóAMD=PóMCD; PABMD=PBCDM.


Dokolku to~kata M se dvi`i po pravata
paralelna so pravata BD toga{ plo{tinite
na ~etiriagolnicite ABDM i BCDM ne se
menuvat (crt. 160). Ako MªO toga{
PABOD=PBCDO...(1). Analogno za to~kata N
imame deka PABCO=PCDAO …(2). Ponatamu, od
(1) va`i:
PóADO+PóABO=PóBCO+PóCDO i od (2) va`i:
PóABO+PóBCO=PóCDO+PóADO. Crte` 160
Od ovaa ravenstvo se dobiva deka PóADO=PóBCO=P1 i PóABO=PóCDO=P2. Ottuka
sleduva deka PATOS=PTBQO=PQCRO =PRDSO
P + P2
= 1 .
2

646. 16:1. Neka x e radiusot na najmalata


kru`nica k2, a R na najgolemata kru`nica
k, toga{ R/2 e radiusot na k1. Na crte` 161
t1 e zaedni~ka tangenta na k i k1, t2
zaedni~ka tangenta na k1 i k2 i t3 e
zaedni~ka tangenta na k i k2. Pravata p
minuva niz presekot na t1 i t3 i centarot na
k. Toga{ p e simetrala na agolot me|u t1 i
t3 odnosno ®DEO=®CEO.
Od druga strana, aglite DEO i EOC se
ednakvi kako naizmeni~ni agli od kade
sleduva deka ®EOC=®OEC.
Crte` 161

193
Odgovori, upatstva i re[enija

Zaklu~uvame deka óEOC e ramnokrak so vrv C. To~no e deka FO = FG (F e na


radikalnata oska na k1 i k2), ED = ET i CT = CG (tangentni otse~ki). Od EG
e simetrala na agolot FEC sleduva deka triagolnicite FGE i CGE se
skladni i ottuka FG = GC . Toga{, ET = 2CT .
2
Od toa {to óEGC~óETM ja dobivame proporcijata R: CT =2ÿ CT :x, 2ÿ CT =R·x.
R
Od druga strana óEGC~óMNL. Zna~i, va`i proporcijata R: CT =2ÿ CT :( -x),
2
2 R R
2ÿ CT =R·( -x), od kade dobivame deka x= . Kone~no, odnosot na
2 4
P R2π
plo{tinite na kru`nicite k i k2 e = = 16 .
P2  R  2
  ⋅π
4
1
(
647. Neka DD1, KK1 i CC1 se normali na AB. Toga{ KK 1 = DD1 + CC1 .
2
)
1 1
Neka AL = BL = a . P∆ABK = a ⋅ KK 1 = a ⋅ CC1 + a ⋅ DD1 = P∆BLC + P∆ALD .
2 2
Od druga strana, PóABK=PLQKP+PóAPL+PóLBQ; PóLBC=PóBCQ+PóLBQ i
PóALD=PóAPD+PóAPL. Zna~i, PLQKP=PóAPD+PóBCQ.
648. Od óECD@óECM sleduva PECD=PECM odnosno 2z+y=2y+x i 2z=x+y...(1)
(crt. 162). Od sli~nosta na petagolnicite ABCDE i
2 2
MNSOK imame PABCDE : PMNSOK = AB : OS ...(2).
Od sli~nosta na triagolnicite ABD i OSD imame:
PóABD:PóOSD= AB ²: OS ²...(3).
Od (2) i (3) imame PABCDE : PMNSOK =PóABD : PóOSD
odnosno (x + 5y + 5z) : x = (x + 3y + z) : y...(4).

Crte` 162
Od (1) sleduva y=2z – x, pa zamenuvajki vo (4)
dobivame (x+10z–5x+5z):x=(x+6z–3x+z):(2z–x)
x 5
odnosno 5z²-5xz+x²=0. Ottuka, z = + x,
2 10
5
y= x . Zna~i, P = PABCD = x + 5y + 5z
5
5x 5 x 7x 3 5 x
= x + 5x + + = + ,
2 2 2 2
7+3 5 
odnosno P =  x .
 2 
 
Crte` 163

194
Odgovori, upatstva i re[enija

649. Jasno e deka ®A+®B+®C=360é...(1) (crt. 163). Od uslovot na zada~ata


dovolno e da poka`eme deka P∆ABC = P∆AC1B + P∆BA1C + P∆CB1A . Za taa cel go
rotirame triagolnikot AC1B so centar vo B taka {to C1 da se sovpadne so A1.
Pri taa rotacijata, neka A se preslika vo to~ka A’.
Toga{, ®A’A1C=360é-®A’A1B-®BA1C
=360é-®BC1A-®BA1C. Zna~i, ®A’A1C=®AB1C. Od ravenstvoto (1) i od toa {to
agolot ne se menuva pri rotacija imame óA’A1C@óAB1C, odnosno
P∆A′A1C = P∆AB1C i AC = A' C od kade se dobiva óABC@óA’BC (SSS). Zna~i ,
P∆ABC = P∆A ′BC = P∆A ′A 1B + P∆BA 1C + P∆CA 1A ′ = P∆AC 1B + P∆BA 1C + P∆CB 1A .

650. Plo{tinata na ~etiriagolnikot KLMN e tretina od plo{tinata na


~etiriagolnikot ABCD (crt. 164). ]e doka`eme deka ND1 = D1C1 = C1M i
KA1 = A1B1 = B1L . Od sli~nosta na triagolnicite NPD i ACD sleduva deka
1
NP||AC i NP = AC . Od sli~nosta na triagolnicite QMB i ACB sleduva deka
3
2
MQ||AC i MQ = AC . Zna~I, NP||MQ i MQ = 2NP .
3
Spored toa óQMD1~óNPD1 od kade se dobiva deka
1 1
ND1 = D1M odnosno ND1 = MN .
2 3
1 1
Analogno, C M = MN od kade sleduva D C = MN .
1 3 1 1 3
Na sli~en na~in se doka`uva deka
1
KA1 = A1B1 = B1L = KL . Crte` 164
3
Zna~i plo{tinata na ~etiriagolnikot A1B1C1D1 e tretina od plo{tinata na
~etiriagolnikot KLMN od kade sleduva deka plo{tinata na ~etiriagolnikot
A1B1C1D1 e devetina od plo{tinata na ~etiriagolnikot ABCD .
651. I re{enie: Niz D povlekuvame prava p || MN , a niz E i F normali na
pravata p (crt. 165). Neka P i Q se
presecite na tie normali so
pravata p i neka {R} = FP ∩ CD , a
{S} = EQ ∩ AD . Da stavime
AB = a, AM = b, MB = c. Toga{ od
pravoagolniot triagolnik ABM
sleduva a 2 = c 2 + b 2 .
1± óKAD@óFRD, spored priznakot
ASA, bidej}i KD = FD , ®ADK=®RDF
i ®AKD=®RFD, kako agli so
normalni kraci. Ottuka sleduva
FR = KA = b, RD = AD = a ...(1).
Crte` 165

195
Odgovori, upatstva i re[enija
2± óMBA@óPDR, kako dva pravoagolni triagolnici so ednakvi hipotenuzi
( AB = RD ) i eden ostar agol (®MAB=®PRD, kako agli so paralelni kraci).
Ottuka sleduva RP = AM = b, PD = MB = c …(2)
3± Od (1) i (2) dobivame FP = FR + RP = 2b , pa poradi PD = c sleduva
c
P∆FPD = 2b ⋅ = bc .
2
4± Na sli~en na~in doka`uvame deka EQ = 2c, DQ = b, P∆DQE = bc.
5± Plo{tinata na pravoagolniot trapez FPQE e
FP + QE 2b + 2c 2
PFPQE = ⋅ PQ = (b + c) = (b + c) .
2 2
6± Na krajot ja nao|ame plo{tinata na óDEF
P∆DEF = PFPQE − (P∆FPD + P∆DQE ) = (b + c) 2 − 2bc = c 2 + b 2 . Bidej}i plo{tinata na
2 2 2
kvadratot ABCD e ednakva na a , odnosno c +b , sleduva deka PABCD = P∆DEF .
So toa tvrdeweto e doka`ano.
II re{enie: Neka CN = MB = x i
AM = BN = y (crt. 166). Imame deka
2
PABCD = AB
2 2
= MB + AM = x 2 + y 2 = PNCLT + PAMGK …(1).
Od P∆ CLD = P∆ BNC = P∆ AMB = P∆ KAD
dobivame PLDKGT = 2PABCD + 4P∆BNC
= 2(x 2 + y 2 + xy) .
No, od druga strana imame
LT + KG
PLKGT = ⋅ GT = (x + y) 2 .
2
Crte` 166
Sleduva P∆ KLD = PLDKGT − PLKGT = x 2 + y 2 . Taka, P∆FDE = P∆KLD = x 2 + y 2 …(2).
Od (1) i (2) sleduva PABCD = P∆FDE .

Tema 13: Elementi od stereometrija


3 2 3
652. V=140 cm . 653. P=8R . 654. 1:2:3. 655. 2 ÿа . Upatstvo: Se razgleduva
pravoagolniot triagolnik AC1D na prizmata ABCDA1B1C1D1 so osnova ABCD,
о 6
kaj koj ®AC1D=30 . 656. .
π
657. V = 12 3r . 3
Od ednakvosta na tangentnite otse~ki sleduva
QM = LM = OL = r (crt. 167). Od toa {to AM e te`i{na linija, dobivame deka

196
Odgovori, upatstva i re[enija

AQ = 2r . Od AT = AQ sleduva AT = 2r . Neka TN = NL = x . Spored pitagorova


teorema za pravoagolniot triagolnik AMN imame
2 2 2
AN = AM + MN , od kade go dobivame
2 2 2
ravenstvoto (2r+x) =(3r) +(r+x) . Od poslednoto
ravenstvo dobivame deka x=3r. Zna~i, visinata na
prizmata e 4r.
Neka a e stranata na ramnostraniot triagolnik
ABC. Od pravoagolniot triagolnik AMB imame
2
2 2 2 a
AM + MB = AB , odnosno (3r) 2 +   = a 2 , od
2
2 2
kade dobivame a =12r . Spored toa
2
a 3
V= ⋅ 4r = 12r 2 ⋅ 3 ⋅ r = 12 3r 3 .
4
Crte` 167

1
658. V= abc. Da go razgledime presekot na dadeniot paralelopiped so
2
ramninata opredelena so to~kite M, D1 i B1 (crt. 168). Bidej}I MD1 i BN se
paralelni, a BN e normalna so B1M,
dobivame deka triagolnikot MB1D1 e
pravoagolen.
1
Toga{ P∆MB1D1 = 1 MB1 ⋅ MD1 = bc. Neka O
2 2
e centarot na paralelogramot A1B1C1D1.
Ramninite na triagolnikot MB1D1 i
paralelogramot AA1C1C se se~at vo
pravata MO, zna~i pravata A1C ja se~e
ramninata na triagolnikot MB1D1 vo
nekoja to~ka P, koja le`i na pravata MO. Crte` 168
Bidej}i MO e sredna linija vo triagolnikot AA1C1, imame deka
1 1 1
A 1P = ( A 1C) = a . Pravata A1C e normalna so pravite B1M I D1M, zna~i
2 2 4
A1P e visina vo triagolnata piramida A1MB1D1. Toga{
1 1
VA1MB1D1 = P∆MB1D1 ⋅ A 1P = abc . Neka P A B C D = S , a H-visinata na
3 24 1 1 1 1
paralelopipedot, spu{tena kon osnovata A1B1C1D1. Toga{ imame deka
1 1 1 1 1
VABCDA1B1C1D1 = S ⋅ H ; V A 1MB 1D1 = ⋅ ⋅ S ⋅ H = S ⋅H = ⋅ V ABCDA 1B1C1D .
3 2 2 12 12 1

1 1
Sleduva deka, V ABCD A B C D =12· V A MB D = 12 ⋅ abc= abc.
1 1 1 1 1 1 1 24 2
6 2
659. P = a . Konstruirame ramnina niz dijagonalata BD1 ~ij presek so
2
kockata e paralelogramot MBND1.

197
Odgovori, upatstva i re[enija

BD 1 ⋅ MP
Ottuka, P∆ BD M = , MP -visina kon BD1. Plo{tinata na presekot }e
1 2
bide najmala koga visinata MP vo
triagolnikot MBD1 }e ima najmala vrednost.
To~kite A, A1 imaat ista polo`ba vo odnos na
BD1, kako i to~kite C, C1. Zna~i pravata MN
opredelena so to~kite M i N sredini na
stranite AA1 i CC1 e simetrala na
dijagonalata BD1.
Vo rombot MBND1 dijagonalite se zaemno
normalni, (O - prese~na to~ka na
1
dijagonalite) pa MN e baranata najmala
2
vrednost na MP (OªP) (crt.169).
Crte` 169

d a 2 BD ⋅ MP
Od MP = = , dobivame PMBND1 = 2P∆MBD1 = 2 ⋅ 1
2 2 2
a 2 6a 2
= BD1 ⋅ MP = 3a 2 ⋅ = .
2 2
7
660. . Neka presekot na ramninata i kockata e ~etiriagolnikot APMN
29
(crt. 170). Neka pravata CD ja se~e
ramninata vo to~ka R, a to~kata T
e sredinata na rabot CD.
Dol`inata na stranata na kockata
da ja ozna~ime so a, volumenot na
kockata so V, volumenot na
potse~enata piramidata APCDNM
so V1, a volumenot na ostanatiot
del od kockata so V2.
Crte` 170

RD DA a
Od sli~nosta na triagolnicite ARD i PRC sleduva = = = 2 od kade
RC CP a
2
dobivame deka RD = 2RC , pa RC = CD = a . Od sli~nosta na triagolnicite
a
a+
QTR i MCR dobivame
QT
=
RT
= 2 = 3 , od kade CM = 2 QT = 2 ⋅ a = a .
CM RC a 2 3 3 2 3
2a
Toga{, DN = 2 ⋅ CM = . Sega,
3

198
Odgovori, upatstva i re[enija

1 DA ⋅ DN 1 CP ⋅ CM 7a 3
V1 = VADNR − VPCMR = ⋅ ⋅ DR − ⋅ ⋅ RC = . Na kraj dobivame
3 2 3 2 36
3
7a
V1 V1
36 7
deka = 3
== .
V2 V − V1 3 7a 29
a −
36
663. Da zabele`ime deka site osum paralelopipedi
imaat zaedni~ka to~ka koja e prese~na to~ka na trite
ramnini (crt. 171). Sekoj dva od niv koj imaat to~no
edno zaedni~ko teme gi narekuvame par. Crte` 171

Takvi parovi ima ~etiri. Ako rabovite na eden


paralelopiped se 1/2+x, 1/2+y, 1/2+z toga{
rabovite na drugiot paralelopiped od parot se
1/2-x, 1/2-y, 1/2-z. Proizvodot od volumenite
na eden par go zadovoluva neravenstvoto
1 2 1 2 1 2 1
V1 ⋅ V2 =  − x  ⋅  − y  ⋅  − z  ≤ .
4  4  4  64
No toga{ barem eden od tie volumeni ne e
pogolem od 1/8. Bidej}i osum paralelopipedi
se rasporedeni vo ~etiri para toga{ vo sekoj
par ima barem eden paralelopiped so volumen
ne pogolem od 1/8. Crte` 172
3
3 2 a 2 π
664. P= a ; V= . 665. .
4 32 4

666. P=40. Upatstvo: Triagolnicite ADB i CBD


se skladni bidej}i imaat ednakvi agli i po
edna zaedni~ka strana (crt. 172).
Triagolnicite ABD i BAC se skladni spored
priznakot SAS, pa sleduva deka spored
priznakot SSS skladni se triagolnicite ACD
i ABC.
667. P=3. Ako prese~na ramnina minuva niz
sredinite na dva sprotivni raba, toga{ go deli
na dva dela so ednakov volumen. Neka K e
prese~nata to~ka na ramninata so rabot AB, a P
i Q sredinite na rabovite AD i BC (crt. 173).
Crte` 173
5
Toga{ VPMQKAC = 2 . Od druga strana, imame deka VPMQKAC = VPMQKA + VMCQA ,
CM CQ 3 1 3
VPMQKA = ⋅ VABCD = ⋅ ⋅ VABCD = ;
CD CB 5 2 2

199
Odgovori, upatstva i re[enija

5 3
VPMQKA = VPMQKAC - VMCQA = - = 1. Bidej}i h=1 e rastojanieto od temeto A
2 2
1
do prese~nata ramnina, toga{ VPMQKA = PPMQK ⋅ h i ottuka PPMQK = 3.
3

668. 3:1. Neka Р e plo{tinata na osnovata


АBCD, Н e visina na paralelopipedot
spu{tena kon osnovata i V e volumenot na
paralelopipedot (crt. 174).
1 1 1 1
Toga{ VB1ABC = P∆ABC ⋅ H = ⋅ P ⋅ H = V .
3 3 2 6
Analogno se doka`uva deka
1
VAA1B1D1 = VCB1C1D1 = VD1ADC = V. Crte` 174
6
4 1
Zna~i, VACB1D1 = V - VB1ABC - VAA1B1D1 - VCB1C1D1 - VD1ADC = V - V = V односно
6 3
V : V ACB 1D1 = 3 : 1 .
1
669. Osnovata ima plo{tina P = a 2 b 2 + a 2 c 2 + b 2 c 2 (primeni heronova
2
abc 1
formula). Od V = abc = OH ⋅ P , dobivame OH= = .
6 3 2 2 2 2 2 2 1 1 1
a b +a c +b c
+ +
a2 b2 c2
670. Neka CDEF e prese~nata ramnina a EF sredna
linija vo triagolnikot ABP (P vrv na piramidata)
(crt. 175). To~kata F ja povrzuvame so A i D, toga{
volumenot na piramidata ABCDF e polovina od
volumenot na celata piramida, a volumenot na
piramidata AEFD e 4 pati pomal od volumenot na
piramidata ABDP (od D se povlekuva visina kon
1
triagolnikot ABP). P∆AEF = P∆ABP .
4
Crte` 175
1 1 1 1 5
Zna~i, VAEFD =
VABDP = V , VABCDEF = VABCD + VAEFD = V+ V = V,
4 8 2 8 8
3
odnosno VCDEFP = V . Zna~i baraniot odnos e 3:5.
8
2 3
671. V = a .
6

200
Odgovori, upatstva i re[enija

1
672. V1= V . Neka H e visinata na piramidata
8
A1A2A3S (crt. 176), P∆ A A A = B , P∆ MNL = B1 i
1 2 3

P∆L 1L 2L 3 = B 2 . Od sli~nosta na triagolnicite


A1A2S i MNS imame B:B1=4:1, odnosno B=4B1 …(1).
Od zada~a 606 imame B1=3B2...(2).
B
Od (1) i (2) dobivame deka B 2 = . Taka,
12
H B H 1 B ⋅H 1
V2 = B 2 ⋅ = ⋅ = ⋅ = V.
2 12 2 8 3 8

Crte` 176
673. V=36. Neka О e centarot na sferata, М presekot na te`i{nite linii vo
triagolnikot ABC i neka sferata go dopira rabot CD vo to~ka P, bo~niot yid
ABD vo to~ka Q i ABC vo to~ka М (od uslovot na zada~ata) (crt. 177). Bidej}i
ABC e ramnokrak triagolnik sleduva deka СК e normalna so AB (К sredina na
AB) odnosno СК e normalna so АD, od kade sleduva deka pravata СК e
normalna na ramninata ABD. No, pravata ОQ e isto taka normalna na
ramninata ABD, zna~i ~etiriagolnikot КМОQ e kvadrat ( OM= OQ).
Imame: KM = OQ=2; CM=2 KM =4; CP = CM =4; DQ = DP = CD - PC =9-4=5. Od
toa {to pravata СК e normalna na ramninata ABD dobivame deka СК e
normalna so DК. Spored pitagorova teorema
2 2
DK = CD - CK =3 5 .
Neka AK = KB = x i neka F e ortogonalna
proekcija na to~kata Q na АD.
2 2 2 2
Toga{ DF = DQ - FQ = DQ - AK = 25 - x 2 i
2 2
AD = DK - AK = 45 - x 2 . Od AD = AF + FD =
QK + FD sleduva 45 - x 2 =2+ 25 - x 2 i zatoa
AB ⋅ AD
х=3. Zna~i, AB =6= AD i VABCD = ⋅ KC = 36 .
3⋅2
Crte` 177
674. ]e doka`eme deka plo{tinata na sekoja bo~na
strana VAB, VBC, VAC, e geometriska sredina od
plo{tinata na triagolnikot ABC i plo{tinata na
ortogonalnata proekcija na toj yid na osnovata,
odnosno Ρ 2 ∆ BCV = Ρ ∆ABC ⋅ Ρ ∆ BCH …(1), od kade
2
 BC ⋅ VM 
  = BC ⋅ AM ⋅ BC ⋅ HM (crt. 178).
 2  2 2
 
Crte` 178

201
Odgovori, upatstva i re[enija
Zna~i, treba da poka`eme deka va`i
2
ravenstvoto VM = AM ⋅ HM …(1’).

Od pravoagolniot triagolnik AMV imame


deka VM : AM = HM : VM od kade imame deka
va`i ravenstvoto (1’) a so toa zna~i deka
va`i i ravenstvoto (1). Analogno, se dobivat
ravenstvata: Ρ 2 ∆ ABV = Ρ ∆ABC ⋅ Ρ ∆ABH ...(2) i
Ρ 2 ∆ACV = Ρ ∆ABC ⋅ Ρ ∆ACH ...(3).
So sobirawe na (1), (2) i (3) dobivame
P2∆ABV + P2∆ACV + P2∆BCV
= P∆ABC ⋅ (P∆ABH + P∆ACH + P∆BCH ) odnosno
Ρ 2 ∆ABV + Ρ 2 ∆ACV + Ρ 2 ∆ BCV = Ρ 2 ∆ABC .

Crte` 179

675. a) Neka S1, S2, S3 i S4 se sredini na stranite AB, BC, CD i DA na


piramidata ABCDV, to~kite T1, T2, T3 i T4 te`i{tata na bo~nite strani CDV,
ADV, ABV i BCV, a O1, O2, O3 i O4 gi delat otse~kite S1T1, S2T2, S3T3 i S4T4
soodvetno vo odnos 3:2 (crt. 179).
→ → → → → → → → 3 →
Toga{, VS1 = VO1 + O1S1 i VT1 = VO1+ O1T1 od kade VS1 = VO1− O1T1 ...(1) i
2
→ → → → → →
O1T1 = VT1− VO1 ...(2). Zamenuvaj}i (2) vo (1) dobivame 2 VS1+ 3 VT1 = 5 VO1 ...(3).
→ 2 → 3 → 2 1 → → 3 1 → →
Od ravenstvoto (3) imame VO1 = VS1+ VT1 = ⋅ ( VA+ VB) + ⋅ ( VC+ VD)
5 5 5 2 5 3
1 → → → →
= ( VA + VB + VC+ VD) .
5
→ → → 1 → → → →
Analogno, dobivame VO2 = VO3 = VO4 = ( VA+ VB+ VC+ VD) . Na toj na~in
5
→ → → →
VO 1 = VO 2 = VO 3 = VO 4 {to zna~i deka to~kite O1, O2, O3 i O4 se sovpa|aat,
odnosno otse~kite S1T1, S2T2, S3T3 i S4T4 se se~at vo edna to~ka koja gi deli vo
odnos 3:2.
b) Neka E, F, G i H se sredini na otse~kite S1T1, S2T2, S3T3 i S4T4, soodvetno
→ → → 1 → → 1 → →
(crt. 180). Toga{, EF = VF− VE = ( VS2 + VT2 ) − ( VS1+ VT1)
2 2
1 1 → → 1 → →  1 1 → → 1 → → 
=  ( VB+ VC) + (VA+ VD) −  (VA+ VB) + (VC+ VD)
2 2 3  2 2 3 
1 → 1 → 1 → → 1 → → 1 → →
= VC − VA = ( VC − VA ) = AC . Analogno, dobivame HG = AC = EF i
12 12 12 12 12

202
Odgovori, upatstva i re[enija
→ 1 → →
FG = BD = EH . Bidej}i to~kite E, F, G i H
12
ne le`at na ista prava, sleduva deka
~etiriagolnikot EFGH e paralelogram.
v) P2:P1=1:72. Neka ψ e agolot me|u
dijagonalite AC I BD. Neka P1 e plo{tinata na
osnovata, a P2 plo{tinata na paralelogramot
1
EFGH. Toga{, P1 = AC ⋅ BD ⋅ sinψ .
2
→ 1 → → 1 →
Od toa {to EF = AC , EH = BD i agol me|u
12 12
EF i EH e ψ ili 180 o - ψ dobivame
1 1 1
P2 = EF ⋅ EH ⋅ sinψ = AC ⋅ BD ⋅ sinψ = ⋅ P1 .
12 12 72
Zna~i, P2:P1=1:72.
Crte` 180
676. 60±. Neka ACC’A’ i ABC’D’ se dadenite preseci (crt. 181). Ortogonalnata
proekcija na trapezot ABC’D’ na ramninata AA’C’ }e bide ~etiriagolnikot
AQC’Q’.
Od teoremata za ortogonalna proekcija na mnoguagolnik
imame deka PAQC’Q’=PABC’D’ ⋅ cosα …(1), kade α e agolot me|u
prese~enite ramnini.
1
Osven toa, imame deka PAQC’Q’= PACC’A’...(2).
2
1
Zamenuvaj}i (2) vo (1) se dobiva cos α = , (PACC’A’= PABC’D’)
2
od kade sleduva deka α=60±.
Crte` 181
677. Neka A1A2…An-1AnP e piramida so vrv P. Ako okolu taa piramida mo`e da
se opi{e sfera, toga{ ramninata na osnovata A1A2…An-1An se se~e so taa
sfera ~ij presek e kru`nica, vo koja e opi{an mnoguagolnikot A1A2…An. Neka
okolu mnoguagolnikot A1A2...An na piramidata A1A2…An-1AnP mo`e da se opi{e
kru`nica so centar O.
Geometriskoto mesto na to~ki vo prostorot, ednakvo oddale~eni od temiwata
na mnoguagolnikot A1A2…An e pravata a, koja minuva niz to~kata O i e
normalna na ramninata na osnovata A1A2…An. Geometriskoto mesto na to~ki vo
prostorot, ednakvo oddale~eni od to~kite P i A1, e ramninata Σ , normalna
na otse~kata PA1 i minuva niz nejzinata sredina. Ako pretpostavime deka
pravata a i ramninata Σ se paralelni toga{ pravata a e normalna na pravata
PA1, {to zna~i deka PA1, le`i vo ramninata na osnovata A1A2…An {to e
nevozmo`no bidej}i P ne le`i vo taa ramnina. Zna~i, a i Σ se se~at vo nekoja
to~ka O1, ednakvo oddale~ena od temiwata P, A1, A2, …, An. Sleduva deka O1 e
centarot na sferata opi{ana okolu taa piramida.

203
Odgovori, upatstva i re[enija
678. Neka M i N se te`i{tata na bo~nite
yidovi BCV i ACV. Vo triagolnikot VFR,
kade {to FR e sredna linija vo
triagolnikot ABC (crt. 182), va`i deka MN
i FR se paralelni i zatoa
VN : NF = VM : MR = 2 : 1. Od triagolnikot
VFR imame deka NM : FR = 2 : 3 ,
AB
NM : = 2 : 3 i NM : AB = 1 : 3 .
2
Od paralelnosta na NM i AB sleduva deka
triagolnicite ABS i MNS se sli~ni odnosno
AS : SM = BS : SN = AB : SM = AB : NM = 3 : 1.

Crte` 182
679. 1:27. 680. P=65. Od uslovot na zada~ata imame deka
 ab
 2 = 15
  ab = 30
 ac 
 = 20 ⇔  ac = 40 ⇔ abc = 120 10 .
 2  bc = 120
 bc 
 2 = 60

Zna~i, delej}i go poslednoto ravenstvo so trite


prethodni ravenstva dobivame deka
a = 10, b = 3 10 i c = 4 10 . Koristej}i ja
pitagorovata teorema, gi nao|ame stranite na
triagolnikot so sina tkaenina ~ii vrednosti se 10,
170 i 5 10. Crte` 183
 x 2 + y 2 = 170
Od crte` 183 imame  , dobivaj}i pritoa
 x 2 − 20 x + y 2 + 100 = 250

deka x=1 i y=≤13, zna~i P = 10 ⋅ 13 = 5 ⋅ 13 = 65 .


2
Baranata plo{tina e 65. Zabele{ka:
Zabele{ka:
152 + 202 + 602 = 652 . Kvadratot od plo{tinata na
najgolemiot yid e ednakov na zbirot od kvadratite
na plo{tinite na pomalite yidovi.
681. I re{enie:
re{enie: Neka h1, h2 i h3 se rastojanijata od M
do bo~nite yidovi, a ha, hb i hc se rastojanijata od
temiwata do bo~nite yidovi (crt. 184).
Neka VABCD = V; VBCDM = V1; VACDM = V2 ; VABDM = V3
V1 V2 V3
toga{ V1 + V2 + V3 = V , + + = 1.
V V V

Crte` 184

204
Odgovori, upatstva i re[enija

PBCD
h1
V1 h V2 h 2 V3 h 3 h h h
Od = 3 = 1, = , = sleduva 1 + 2 + 3 = 1 ⋅
V PBCD ha V hb V hc h a hb h c
ha
3
Triagolnicite M1PM i A1DA se sli~ni (®M1MP=®A1AD kako agli so paralelni
h1 MP h 2 MQ h3 MR
kraci, ®MM1P=®AA1D=90±), pa = . Sli~no, = i = .
ha AD hb BD hc CD
MP MQ MR
Sobiraj}i gi poslednite tri ravenstva dobivame = 1. + +
AD BD CD
II re{enie:
re{enie: Od sli~nosta na triagolnicite ALD i MLP se dobiva ravenstvoto
MP ML MQ MT
= …(1) (crt. 185). Analogno, gi dobivame ravenstvata = …(2) i
DA AL DB BT
MR MS
= …(3). Sobiraj}i gi ravenstvata (1), (2) i (3)
DC CS
MP MQ MR ML MT MS
dobivame deka + + = + + …(4).
DA DB DC AL BT CS
ML MT MS
Zamenuvaj}i go ravenstvoto =1 + +
AL BT CS
( vidi zada~a 318 a) ) vo ravenstvoto (4) se dobiva
baranoto ravenstvo.
Crte` 185
1 2 → → → → → → → → →
682. PT = a + b2 + c 2 . Od PT = PA+ AT , PT = PB+ BT i PT = PC+ CT
3
→ → → → → → → → → →
(crt. 186) sleduva 3 PT = PA+ PB+ PC+ ( AT+ BT+ CT) = PA+ PB+ PC pa

→ 1 → → → → →2 1  → → →
2 2
1 → → →
2 2
→→ →→ →→
PT = ( a + b + c) . PT = PT = a+ b+ c = a + b + c +2 a⋅ b +2 a⋅ c +2 b⋅ c
3 9  3
 
1 2
= a + b2 + c 2 .
3

3
R a 3 6
683. V= . R=
kade R e radiusot na
4 3
kru`nicata opi{ana okolu ramnostran
triagolnik ABC.
Toga{ za visinata imame a 2 − R 2 = 2R i
P∆ABC ⋅ DM R 3 6
zatoa V = = .
3 4
Crte` 186

205
Odgovori, upatstva i re[enija
684. 4V. Neka AMBK e osnovata na piramidata PAMBK, kade α e agolot me|u
dijagonalite AB i MK, H e visinata na piramidata PAMBK (crt.187). Neka
PC : PK = PD : PM = CD : KM = x . Toga{, ako rastojanieto me|u pravite AB i CD
d
go ozna~ime so d= OQ, va`i PQ : PO = x odnosno (H-d):H=x i ottuka H = .
1− x
Sega, V=VABCD=(VCOQA+VCOQB)+(VDOQA+VDOQB)=( VCOC1A + VCOC1B )+( VDOD1A + VDOD1B ).
P∆ABC1 ⋅ CC 1 P∆ABD1 ⋅ DD 1 P∆AD1BC1 ⋅ QO
Toga{, V= VABC1C + VABD1D = + =
3 3 3
AB ⋅ CD ⋅ sin α
(CC 1 )
= QO = DD 1 . Zna~i, V =
2⋅3
⋅ d od kade sleduva

AB ⋅ CD ⋅ d ⋅ sinα = 6V …(1). Ponatamu imame deka


1 1 1 CD
V AMBKP = PAMBK ⋅ H = AB ⋅ KM ⋅ sinα ⋅ H = AB ⋅ ⋅ sinα ⋅ H
3 6 6 x
1 CD d
= AB ⋅ ⋅ sinα ⋅ .
6 x 1− x
Od ravenstvoto (1) sleduva deka
V V
VAMBKP = =
x(1 − x ) 1 1
− + x − x2
4 4
V V
= 2
≥ = 4V , pri {to ravenstvo va`i
1  1 1
− x −  4
4  2
koga x=1/2 odnosno to~kata C e sredina na rabot
PK. 685. 12.

Crte` 187
687. Neka B e plo{tinata na zidovite na pravilniot tetraedar ABCD so
volumen V i neka A1, B1, C1 i D1 se podno`nite to~ki na normalite povle~eni
od to~kata O, a A2, B2, C2 i D2 se
podno`nite to~ki na normalite
povle~eni od to~kata P na
zidovite BCD, CDA, ABD i ABC
(crt. 188). Od sli~nosta na
triagolnicite OA1A’ i PA2A’
imame deka PA' = PA 2 = PA 2 ...(1).
OA' OA1 R
Analogno se dobivat ravenstvata
PB' PB 2 ...(2), PC' PC2 ...(3) i
= =
OB' R OC' R
PD' PD2 ...(4).
=
OD' R

Crte` 188

206
Odgovori, upatstva i re[enija
So sobirawe na ravenstvata (1), (2), (3) i (4) go dobivame ravenstvoto
PA ' PB' PC' PD' PA 2 + PB2 + PC2 + PD2
+ + + = ...(5).
OA ' OB' OC' OD' R
Od VBCDO+VCDAO+VABDO+VABCO=VBCDP+VCDAP+VABDP+VABCP dobivame
B ⋅ OA 1 B ⋅ OB1 B ⋅ OC1 B ⋅ OD1 B ⋅ PA 2 B ⋅ PB 2 B ⋅ PC 2 B ⋅ PD 2
+ + + = + + + . Mno`ej}i go
3 3 3 3 3 3 3 3
3
poslednoto ravenstvo so dobivame
B
PA 2 + PB 2 + PC 2 + PD 2 = OA 1 + OB1 + OC1 + OD1 = 4R …(6). Zamenuvaj}i go
ravenstvoto (6) vo ravenstvoto (5) se dobiva baranoto ravenstvo.
688. ]e go razgledame tetraedarot ABCD (crt. 189) vpi{an vo
paralelopipedot AKBLNDMC. Neka АС e normalna so BD и AD e normalna so
BC. Bidej}i KL e paralelna so CD, toga{ KL e normalna so AB od kade sleduva
deka AKBL e romb.
Analogno se doka`uva deka site {est yida na paralelogramot AKBLNDMC se
rombovi, {to zna~i deka site rabovi se ednakvi. Neka rabovite gi ozna~ime
2 2
 AB   
so х. Toga{   +  LK  = x 2 odnosno AB 2+ CD 2=4x2.
 2   2 
   
2 2 2 2
Аналогно се добива дека AC + BD = 4x и AD
2 2 2 2 2
+ BC = 4x од каде следува AB + CD = AC +
2 2 2
BD = AD + BC …(1).
Obratno, neka va`at ravenstvata (1). Za
paralelogramite AKBL i BMCL va`at
2 2 2 2
ravenstvata AB + CD = AB + LK
2 2 2 2 2 2
= 2 AL + 2 BL i BC + AD = BC + LM
2 2
= 2 CL + 2 BL (vidi zada~a 397). Crte` 189
2 2 2 2
Бидејќи AB + CD = BC + AD sleduva deka AL = CL . Analogno se doka`uva
deka site ostanati rabovi se ednakvi, {to zna~i deka site yidovi se rombovi,
od kade sleduva deka spritivnite rabovi na tetraedarot ABCD se normalni.
690. Ako stranite AB i AC ne se ednakvi toga{ gi
konstrurame simetri~nite to~ki B1 i C1 na B i C vo
odnos na simetralata na ®BAC (crt. 190).
Vo ~etiriagolnikot BC1CB1 (ramnokrak trapez)
va`i neravenstvoto CC 1 + BB 1 < BC + B 1C 1 , a od
toa {to CC1 = AC , BB1 = AB , B1C1 = BC imame deka
AC + AB < BC + BC = 2 ⋅ BC . Ako AB = AC odnosno ako
triagolnikot ABC e ramnostran, toga{
AC + AB = 2 ⋅ BC . Na toj na~in dobivame deka ako
agolot pri temeto A e 60± toga{ AC + AB ≤ 2 ⋅ BC . Crte` 190
Analogno, se dobiva deka AC + AD ≤ 2 ⋅ CD i AB + AD ≤ 2 ⋅ BD . Sobiraj}i gi
poslednite tri neravenstva go dobivame baranoto neravenstvo.

207
Odgovori, upatstva i re[enija

1
691. Neka a, b, c se bo~nite rabovi na dadeniot tetraedar. Toga{, P1= ab ,
2
1 1
P2= ac i P3= bc . Od neravenstvoto megu aritmeti~kata i geometriskata
2 2
ab + bc + ca 33 2 2 2
sredina dobivame P1+ P2+P3= ¥ a b c ...(1).
2 2
Od druga strana, pravata niz trirabniot agol pri vrvot na tetraedarot
(normalna na osnovata) obrazuva agli α, β, γ za koi va`i ravenstvoto
2 2 2
cos α + cos β + cos γ = 1. Vo na{iot slu~aj dobivame H + H + H = 1 ,
2 2 2
2 2 2
a b c
3
a2b2c 2 a2b2c 2 a2b2c 2
odnsono H2 = 2 2 2 2 2
≤ = .
a b+b c +a c 3
3 a4b 4c 4 3
3
Od ovde sleduva neravenstvoto a 2b2c 2 ≥ 3H2 . Od (1) i (2) dobivame
P1+P2+P3¥
33 2 2 2 9 2 .
a b c ≥ H
2 2
692. Primenuvaj}i go svojstvoto za simetrala na
agol kaj triagolnicite PBC, PAC i PAB gi
AC 1 PA
dobivame ravenstvata (crt. 191) = ,
BC 1 PB

BA 1 PB CB1 PC BA 1 CB 1
= i = . Sleduva deka ÿ ÿ
CA 1 PC AB 1 PA CA 1 AB 1

AC1 PB PC PA
= ÿ ÿ =1. Spored teoremata na ^eva
BC1 PC PA PB
pravite AA1, BB1 i CC1 se se~at vo edna to~ka. Crte` 191
2 2
4r R
693. 3 : 2 . 694. P = π . 695. V1:V2=h:h1=b:a.
(r + R) 2
696. Va : Vb : Vc = 1 : 1 : 1 . Neka h a , h b , h c se visini vo triagolnikot ABC.
a b c
2 2 2
Toga{ Va =
aha π,V chc
=
bhb π,V =
π. Od a·ha=b·hb=c·hc=2P sleduva
b c
3 3 3
1 1
2 ah a ⋅
Va ah a ah ⋅ h h a=a
= ,= a a = a =
Vb bh b 2 bh b ⋅ h b h b 1 1
bh b ⋅
b b.

208
Odgovori, upatstva i re[enija

1 1
Vb b i Vc c . 697. R= a 2 . 698. 8
Analogno se dobiva deka = = V = πR 3
Vc 1 Va 1 4 3
c a
(H = 2 ⋅ (2R)) . 699. VK:VT:Vc=1:2:3.

Tema 14: Eksponencijalni i logaritamski funkcii


700. 0. Upatstvo: Grupiraj po komplementarni agli.
17 1
702. 5, -1, , − . 703. 194. Izrazot go transformirame na sledniot na~in
4 4
x -x 2x -2x x -x 2
81 +81 =9 +2+9 -2=(9 +9 ) -2=194.
b logc a
704. logab= π . 705. a) ; b) Od vrskata logba = nao|ame deka
1- a logc b
2
14
log14
log14 28
=7 = 2log14 14 - log14 7 = 2 - a .
log35 28 =
log14 35 log14 7 ⋅ 5 log14 7 + log14 5 a+b
707. -10, 9. Logaritmiraj}i ja dadenata ravenka dobivame
2 k
(x +x-88)·lg21=lg441·lg10. Koristej}i go praviloto lgx =klgx i toa {to lg10=1,
2
imame (x +x-8)·lg21=2·lg21. Delej}i go poslednoto ravenstvo so lg21∫0, ja
2
dobivame kvadratnata ravenka x +x-90=0, ~ii re{enija se 9 i -10. Broevite 9
i -10 se re{enija i na dadenata eksponencijalna ravenka. 708. 248.
1
709. x=100. 710. . 711. x=100. 712. x=2, x=4. 713. x=3 i x=11. 714. x=6.
9
3
715. x= . 716. x=4. Upatstvo: Vnimavaj na definicionata oblast
3
1
log x 2 = log 4= .
x2 log 2 x
717. x=1. Dadenata ravenka
ekvivalentnae so ravenkata
3x 2x 4x x x 2x 2x
6 4 3 x 36 ⋅ 6 4 ⋅ 9
4x ⋅ 9x − 2 ⋅ + ⋅ = 0 , odnosno so 36 − + = 0 . Mno`ej}i
6 4 9 3 36
x x x 2x
go poslednoto ravenstvo so 36, imame 36·36 -12·36 ·6 +36 =0...(1). Delej}i ja
x 2x x
ravenkata (1) so 36 ∫0, dobivame ravenka 6 -12·6 +36=0, a voveduvaj}i smena
x 2
6 =y, ravenkata y -12y+36=0, so dvoen koren y=6. Vra}aj}i se vo smenata
1 1 1
dobivame deka x=1. 718. 2
2
i . 719. x1=100, x2= . 720. x=2. 721. x= .
10 4
2 2
722. -7. Upatstvo: Levata strana vo dadenata ravenka ima vrednost 1 vo
slu~aj koga osnovata ima vrednost 1 ili vrednost -1 pri paren eksponent,

209
Odgovori, upatstva i re[enija
kako i vo slu~aj koga eksponentot ima vrednost 0. 723. x=100; 724. x=0, x=2.
3
Zabele{ka x=1 ne e re{enie na ravenkata. 725. . 726. k=4.
2
727. x = 10 10 . Dadenoto ravenstvo e ekvivalentno so ravenstvoto
2 3 4 n n+1
+ 2 + 3 + ... + n −1 + n + ... = 8 …(1). Delej}i ja ravenkata (1) so
lgx lg x lg x lg x lg x
2 3 4 n 8
lgx>1 imame 2
+ 3
+ 4
+ ... + n
+ ... = …(2). Odzemaj}i ja (2) od (1) ja
lg x lg x lg x lg x lgx
1 1 1 1 10
dobivame ravenkata 2
+ 3
+ 4
+ ... + n
+ ... = 8 - , koja e
lg x lg x lg x lg x lgx
1
lg 2 x 10 3 3
ekvivalentna so ravenkata = 8- ~ii re{enija se lgx = i lgx = .
1 lgx 4 2
1−
lg x
a+b
728. x=20. 729. = 6 . Upatstvo: Dadenoto ravenstvo se transformira vo
c
a-2c 4c-b 10
vidot 3 =2 koe va`i samo vo slu~aj koga a-2c=4c-b=0. 730. x=10 .
732. log c + b a + log c −b a =
1 1 log a (c − b) + log a (c + b) log a (c − b) ⋅ (c + b)
= + = =
log a (c + b) log a (c − b) log a (c + b) ⋅ log a (c − b) log a (c + b) ⋅ log a (c − b)
log a (c 2 − b 2 ) log a a 2 2
= = =
loga (c + b) ⋅ loga (c − b) log a (c + b) ⋅ log a (c − b) log a (c + b) ⋅ log a (c − b)
= 2log c +b a ⋅ log c −b a, {to treba{e da se doka`e.

733. 3. Upatstvo: Vovedi gi smenite logab=x, logbc=y, logca=-(x+y).


1
734. 469. 735. . 736. 113. 737. 1<x<5.
6

Tema 15: Sistemi


Sistemi ravenki i neravenki
2 2 2 2
738. (12,13,57). 739. Ravenkata se zapi{uva vo vidot (xy) -7y =x +2xy+y ,
2 2
x+y x+y
odnosno y (x -7)=(x+y) . Ottuka x − 7 =   , odnosno x 2 − 
2
2 2 2
 = 7.
 y   y 
 x + y  x+y
Sleduva deka  x −  x +  = 7. Razgleduvame ~etiri sistemi:
 y  y 

210
Odgovori, upatstva i re[enija

 x+y  x+y  x+y  x+y


x − y = 1 x − y = 7 x − y = −1 x − y = −7
 

 ; ;  ;  , ~ii re{enija se
x + x + y = 7x + x + y = 1 x + x + y = −7 x + x + y = −1
 y  y  y  y
parovite (4,2), (4,-1), (-4,1) i (-4,-2). 740. (5,1,3). 741. (27,1), (1,27). 742. x=y=1,
1 1 (x + y )!
z=2. Upatstvo: prvata ravenka se zapi{uva vo vidot + = , vo koja
x! y! x! y!
desnata strana e priroden broj. 743. (2,3), (3,2). 744. a) x= ± 2 , y= ± 2 .
745. x1=14, y1=-11; x2=11, y2=-14. 746. x1=2, y1=1 i x2=1, y2=2 . 747. x=y=3.
1 
( ) ( )
748. (10,100), (100,10). 749. a) 4, 2 , 4,− 2 . 750. b)  ,64  , (8,2). 751. (3,3,3).
4 
Mno`ej}i ja vtorata ravenka so xyz i koristej}i ja tretata ravenka od
sistemot, dobivame deka xyz=27 ...(1).
b
Spored vietovite formuli − = x+ y+z = 9, od kade b=-9a;
a
c d
= xy + xz + yz = 27 , od kade c=27a i − = xyz = 27 , od kade d=-27a, pri {to
a a
3 2
x, y, z se koreni na ravenkata at +bt +ct+d=0 ...(2). Zamenuvaj}i gi dobienite
vrednosti za b, c i d vo (2), ravenkata se transformira vo oblik
3 2 3
t -9t +27t-27=0, odnosno vo ravenka (t-3) =0. Od tuka t=3. Bez gubewe na
op{tosta neka x=3. Zamenuvaj}i za x vo ravenkata (1) i vo prvata ravenka od
y ⋅ z = 9
sistemot, dobivame sistem  so re{enija y=z=3.
y + z = 6
12 12
752. x= , y=, z=-12. 753. (0,0,a);(0,a,0);(a,0,0).
7 5
2 2 2 2
Od (x+y+z) -(x +y +z )=2(xy+yz+zx) sleduva xy+yz+zx=0.
3 3 3 3
Od (x+y+z) -(x +y +z )=3(x+y)(y+z)(z+x) sleduva (x+y)(y+z)(z+x)=0.
Zna~i, xyz=(xy+yz+zx)(x+y+z)-(x+y)(y+z)(z+x)=0. Spored toa ako x=0 toga{ y=0,
 2 27 32 
z=a; ako y=0 toga{ z=0, x=a; ako z=0 toga{ x=0, y=a. 754.  , ,  .
3 8 3 
c −b a−c b−a
755. x = , y= ,
. Upatstvo: Doka`i deka x+y+z=0.
z=
a+b+c a+b+c a+b+c
756. Dadenite ravenki od sistemot se transformiraat vo vidot:
(x - y )(x + y ) = 2(x + y )(z + 1) (x + y )(x - y - 2z - 2) = 0

(y − z )(y + z ) = 2 (y + z )(x + 1) odnosno 
(y + z )(y - z - 2x - 2) = 0
.
(z − x )(z + x ) = 2(z + x )(y + 1) (z + x )(z − x − 2y − 2) = 0
 
Od ovde sleduvaat slednite ~etiri slu~ai:
1) Za x+y=y+z=z+x=0 dobivame x=y=z=0 i re{enie e trojkata (0,0,0).
2) Za x+y∫0, odnosno x-y-2z-2=0 i y+z=z+x=0 dobivame deka z=-1, x=y=1.
Zemaj}i gi i ostanatite dva sli~ni slu~aevi, dobivame deka vo ovoj
slu~aj re{enie na sistemot se trojkite (1,1,-1), (1,-1,1) i (-1,1,1).
3) Za x+y∫0, y+z∫0 i z+x=0 dobivame deka x=2,y=4 i z=-2. Zaedno so

211
Odgovori, upatstva i re[enija
ostanatite dva slu~ai, za re{enie na sistemot gi dobivame trojkite
(2,4,-2), (4,-2,2) i (-2,2,4).
x - y - 2z - 2 = 0
4) Za x+y∫0, y+z∫0 i z+x∫0 go dobivame sistemot y - z - 2x - 2 = 0 ~ie
z − x − 2y − 2 = 0

edinstveno re{enie e trojkata (-1,-1,-1).
757. x=y=z=2. Da pretpostavime deka x e najgolemata od trite nepoznati
4 4 4 4
veli~ini, odnosno x¥y i x¥z. Toga{ ≤ , a ottuka z = 3 − ≥3− = y.
zx zy zx zy
4 4 4 4
Sega, xz¥xy, odnosno ≤ , od kade = x . No, po
z =3− ≥3−
zx xy zx xy
pretpostavka x¥z, pa sleduva deka x=z. Zamenuvaj}i vo tretata ravenka za x=z,
3 2
ja dobivame ravenkata z -3z +4=0 ~ii re{enija se z=2 i z=-1. Toga{ z=x=2. Od
prvata ravenka dobivame deka y=2.
758. x=y=z=1. Mno`ej}i gi trite ravenki od sistemot, dobivame xyz=1. Jasno,
barem edna od promenlivite e pogolema ili ednakva na 1. ]e doka`eme deka
x=y=z=1. Bez gubewe na op{tosta mo`e da pretpostavime deka z e najgolemata
od nepoznatite, pa z¥1 i z¥y. Sega, z+1¥y+1, pa od prvata ravenka dobivame
1 1 1 1
deka i x¥1. Toga{ §1, §1 i §1, pa y= §1§x. Od neravenstvoto y§x,
z y x xz
dobivame deka 1+y§1+x i od tretata ravenka od sistemot sleduva z§1. Jasno,
1+ y 2
toga{ mora z=1. Od 1=z= , dobivame deka x=y. Taka, 1=xyz=x odnosno
1+ x
 15 1 
x=1=y. 759. 15 i 4. Re{enija na sistemot se: (12,3) i  ,  . 760. 2.
 4 4
761. xy + yz + zx = 2 2 . Neka AF = x, FB = y i FC = z (crt. 192), kade to~kata
F e to~ka na Ferma, odnosno aglite AFB, BFC i CFB se ednakvi i iznesuvaat
120±. Od kosinusnata teorema gi dobivame
2 2
ravenstvata x2 + y2 + xy = AB , y 2 + z2 + yz = BC
2 2 2
i z + x + zx= CA .
Zna~i, AB= 2 , BC = 3 i CA = 5 , od kade se
dobiva deka triagolnikot ABC e pravoagolen
so prav agol vo temeto B. Plo{tinata na
triagolnikot ABC e zbir od plo{tinite na
triagolnicite AFB, BFC i CFA. Crte` 192

o o o
6 xysin120 yzsin120 zxsin120 3
Spored toa, P∆ABC = = + + = ⋅ (xy + yz + zx ) ,
2 2 2 2 4
a ottuka xy+yz+zx= 2 2 .

212
Odgovori, upatstva i re[enija
3 2
762. Go razgleduvame polinomot P(t)=t +t z+ty+x ~ii nuli se a, b i c. Zna~i
P(t)=(t-a)(t-b)(t-c), od kade spored Vietovite formuli dobivame deka
x=-abc, y=ab+bc+ca, z=-(a+b+c).
763. (0,-1,3), (0,3,-1), (-1,0,3), (-1,3,0), (3,0,-1), (3,-1,0). Po sreduvaweto na
vtorata ravenka i koristewe na prvata ravenka se dobiva ravenstvoto
xy+yz+zx=-3. Na sli~en na~in, od tretata ravenka go dobivame ravenstvoto
x + y + z = 2

xyz=0. Zna~i dadeniot sistem e ekvivalenten so sistemot xy + yz + zx = −3 .
xyz = 0

Od poslednata ravenka dobivame deka x=0 ili y=0 ili z=0, pa sistemot se
zamenuva so vkupnost na tri ednostavni sistemi:
I. x=0 , yz=-3, y+z=2.
II. y=0 , xz=-3, x+z=2.
III. z=0 , xy=-3, x+y=2.
Nivni re{enija se trojkite (0,-1,3), (0,3,-1), (-1,0,3), (-1,3,0), (3,0,-1), (3,-1,0).
764. (1,1,1,1), (3,-1,-1,-1), (-1,3,-1,-1), (-1,-1,3,-1), (-1,-1,-1,3).
765. a=b=c=d=2. 766. (1,2,3,4,5).
767. x1+x2020=1010. Ja razgleduvame razlikata na vtorata ravenka od sekoj
ozna~en sistem so prvata ravenka od istiot sistem i ja dobivame slednata
,,{ema,,:

x + x = 1    
 1 2 
x3 − x1 = 1   
x2 + x3 = 2 x4 + x1 = 2  
  x5 − x1 = 2 
 3
x + x 4 = 3   x 6 + x1 = 3
   x7 − x1 = 3
x4 + x5 = 4   
x + x = 5  
 5 6  
... 


...

...   
x x2018 + x1 = 1009 
2017 + x 2018 = 2017  x 2019 − x1 = 1009
  x2020 + x1 = 1010
x2018 + x2019 = 2018  
 
x2019 + x2020 = 2019 




1
Zna~i x1+x2020=1010. 768. xi=2 za i=2k-1, kœ{1,2,…,50}; xi= za i=2k, kœ{1,2,…,50}.
2
1 3 20
769. x1=x2=…=xn= . 770. m = - . 771. mœ (-¶, − 5 )»( − 1 ,2). 772. .
n +1 4 2 2 3

213
Odgovori, upatstva i re[enija

A ⋅ ai 3x + y = 2z y = 2z − 3x
773. x i = , i=1, 2, …, n. 774.  ⇔
a1 + a 2 + ... + a n  x + 2y + 3z = 25 5x = 7z − 25
  2z   z
y = 2z − 3x y = 2z − 3 z + − 5  y = 15 − 2z −
   5 ⇔ 5 Za da bide yœN
⇔ 2z ⇔  .
 x = z + − 5  2z  2z
 x = z+ −5
5 x = z + 5 − 5  5
treba z=5. Toga{ x=2 i y=4. Zna~i 1 petel ~ini 2 denara;1 guska ~ini 4 denara
i 1 fazan ~ini 5 denara.
775. Jas imam 28 godini, a ti ima{ 21 godini. Ja sostavuvame slednata tabela:

Pred y godini Sega Posle x-2y god.


Jas x 2(x-y) 3x-4y
Ti x- y x 2(x-y)
2(x − y) − x = x − (x − y)
Od tabelata se dobiva sistemot  , ~ie re{enie e x=21,
3x − 4y + 2(x − y) = 63
y=7. Spored toa jas imam 28 godini, a ti ima{ 21 godini.
776. 65 km/h i 55 km/h.
777. 1350m. Neka V1 e brzinata na prviot brod a V2 brzinata na vtoriot brod
dodeka rastojanieto me|u dvete mesta e s. Spored uslovite na zada~ata gi
V ⋅ t = 500 V ⋅ t = x + 150 + 150
imame slednive sistemi ravenki  1 1 i  1 2 , kade
V2 ⋅ t1 = s − 500 V2 ⋅ t 2 = 500 + 500 + x
t1 e vremeto pominato do prvoto sre}avawe, t2 e vremeto pominato od prvoto
do vtoroto sre}avawe, a x e rastojanieto ozna~eno na crte` 193.

Crte` 193

Delej}i gi ravenstvata vo sekoj od sistemite gi dobivame ravenstvata


V1 500 V x + 300
= …(1) i 1 = …(2). Zamenuvaj}i go ravenstvoto x=s-650 vo
V2 s − 500 V2 x + 1000
500 s − 350
(2) i izedna~uvaj}i gi ravenstvata (1) i (2), dobivame =
s − 500 s + 350
2
odnosno kvadratna ravenka s -1350s=0. Re{enija na kvadratnata ravenka se
s=0 i s=1350. Zna~i, rastojanieto me|u dvete mesta bilo 1350 metri.
778. 3. Mno`ej}i ja prvata ravenka so z, vtorata ravenka so x i tretata
ravenka so y i sobiraj}i gi trite neravenstva, se dobiva neravenstvoto
xyz(27x+27y+27z)¥27x+27y+27z, odnosno (xyz-1)·27(x+y+z)¥0. Ottuka sleduva
deka xyz¥1. Od x+y+z¥3· 3 xyz ≥ 3 dobivame deka minimalnata vrednost na
zbirot x+y+z e 3.

214
Odgovori, upatstva i re[enija
779. Dadenite neravenstva se ekvivalentni so neravenstvata:
2(a3-a2) ¥(a2-a1)
2(a4-a3) ¥(a3-a2)
....
...
...
2(a1-a100) ¥(a100-a99)
2(a2-a1) ¥(a1-a100).
Koristej}i gi poslednite neravenstva dobivame
2a2=2(a2-a1) + 2(a1-a100) + 2(a100-a99)+....+2(a5-a4)+2(a4-a3)+2a3
¥a1-a100+a100-a99+a99-a98+...+a4-a3+a3-a2+2a3, odnosno a1-3a2+2a3§0.

Analogno se dobivaat i neravenstvata


a2-3a3+2a4§0
...

...
a99-3a100+2a1§0
a100-3a1+2a2§0.

Vrz osnova na dadenite i poslednite dobieni neravenstva, sleduva deka


a1-3a2+2a3=0 ñ 2(a3-a2)=(a2-a1)
a2-3a3+2a4=0 ñ 2(a4-a3)=(a3-a2)
...
...
...
a99-3a100+2a1=0 ñ 2(a1-a100)=(a100-a99)
a100-3a1+2a2=0 ñ 2(a2-a1)=(a1-a100).
Mno`ej}i gi poslednite 100 ravenstva, go dobivame ravenstvoto
100
(2 -1)ÿ(a2-a1)·(a3-a2)·(a4-a3)·...·(a100-a99)·(a1-a100)=0.
100
Bidej}i 2 -1∫0, sleduva deka barem edna od 100-te razliki e ednakva na
nula. Bez gubewe na op{tosta neka a2-a1=0 odnosno a1=a2. Zamenuva}i nazad vo
ravenstvata dobivame a1=a2=...=a100.

Tema 16: Elementi od trigonometrija

3 5 5− 5 . ± ±
780. cosα = . 781. . 782. 783. cos36 -sin18 =
4 7 8
1
cos36 °cos18 ° − sin18 °cos18 °
(cos18 ° + cos54 ° ) − 1 (sin0 ° + sin36 ° )
= 2 2 =
cos18 ° cos18 °
1 cos18 ° + cos54 ° − sin36 ° 1 cos18 ° 1
· = = .
2 cos18 ° 2 cos18 ° 2

215
Odgovori, upatstva i re[enija

89
784. S= . S=cos21±+cos22±+…+cos289± ...(1)
2
2 2 2
S=sin (90±-1±)+sin (90±-2±)+...+sin (90±-89±)
2 2 2
S=sin 89±+sin 88±+…+sin 1±
2 2 2
S=sin 1±+sin 2±+…+sin 89± ...(2)
Sobiraj}i gi ravenkite (1) i (2) dobivame
2 2 2 2 2 2
2S=( sin 1±+ cos 1±)+( sin 2±+ cos 2±)+…+( sin 89±+ cos 89±)
89
2S=1+1+…+1 sleduva 2S = 89, odnosno S= .
2
1 1
785. a) − ; b) . 789. sin2 α + sin2β + sin2 γ = 1.
8 2
791. Od dadeniot odnos i od toa {to α+β+γ= π dobivame deka
π 2π 4π
α= , β= i γ= . Od sinusnata teorema gi dobivame ravenstvata
7 7 7
1 1 1 1 1 1
= , = i = .
a 2R ⋅ sin α b 2R ⋅ sin β c 2R ⋅ sinγ
1 1 1
Zna~i treba da se doka`e deka = + . Od druga strana
π 2π 4π
sin sin sin
7 7 7
2π 4π 3π π 3π
sin + sin 2sin cos sin
1 1 1
+ = 7 7 = 7 7 = 7 = .
2π 4π 2π 4π π π  3π  π 3π π
sin sin sin ⋅ sin 2sin cos sin  π −  sin sin sin
7 7 7 7 7 7  7  7 7 7
792. a:b:c= 5 : 2 2 : 3 . 799. Prvo }e go doka`eme ravenstvoto
cos 2 α = 1 − cos 2 β − cos 2 γ − 2cosα ⋅ cosβ ⋅ cosγ …(1).
cos 2 α = cos 2 (β + γ ) = (cosβ ⋅ cosγ - sinβ ⋅ sinγ ) 2
= cos 2 β ⋅ cos 2 γ - 2sinβ ⋅ sin γ ⋅ cosβ ⋅ cosγ + (1 - cos 2 β) ⋅ (1 − cos 2 γ)
= 1 − cos 2 β − cos 2 γ + 2cos 2 β ⋅ cos 2
γ - 2sinβ ⋅ sin γcosβ ⋅ cosγ
= 1 − cos 2 β − cos 2 γ + 2cosβ ⋅ cos γ ⋅ (cosβ ⋅ cosγ - sinβ ⋅ sin γ)
= 1 − cos 2 β − cos 2 γ + 2cosβ ⋅ cos γ ⋅ cos(β + γ)
= 1 − cos 2 β − cos 2 γ − 2cosβ ⋅ cos γ ⋅ cosα .
Taka, (sinα + sinβ + sinγ ) 2 − (1 + cosα + cosβ + cosγ ) 2
= sin 2 α + sin 2 β + sin 2 γ + 2sinα ⋅ sinβ + 2sinα ⋅ sinγ + 2sinβ ⋅ sinγ − 1 − cos 2 α - cos 2 β

- cos 2 γ - 2cosα - 2cosβ - 2cosγ - 2cosα ⋅ cosβ - 2cosα ⋅ cosγ - 2cosβ ⋅ cosγ .

Koristej}i go ravenstvoto cosα = -cos(β + γ ) = sinβ ⋅ sin γ - cosβ ⋅ cosγ imame

(sinα + sinβ + sinγ ) 2 − (1 + cosα + cosβ + cosγ ) 2

216
Odgovori, upatstva i re[enija

= sin 2 α + sin 2 β + sin 2 γ + 2cosα + 2cosβ + 2cos γ - 2cosα - 2cosβ - 2cos γ


− 1 − cos 2 α - cos 2 β - cos 2 γ = sin 2 α + sin 2 β + sin 2 γ − 1 − cos 2 α - cos 2 β - cos 2 γ
= 2 - 2(cos 2 α + cos 2 β + cos 2 γ ) = 2(1 − cos 2 α - cos 2 β − cos 2 γ ) = 4cosαcosαcosγ .
22 22 sin(45 ° + α)
801. a)2 ; b)2 . Upatstvo: a) 1+tgα=tg45º+tgα= ;
cos45 ° ⋅ cos α
sin(α − 45°)
b) 1-ctgα=ctg45º-ctgα= .
sin45°sinα
sin1o
802. 1. Da go razgledame n-tiot ~len vo zbirot koj mo`e da
cos( n - 1)o ⋅ cosno
se transformira vo razlika na sledniot na~in:
sin1o sin[no - (n - 1)o ] sinno ⋅ cos(n- 1)o cosno ⋅ sin(n- 1)o o
= = − = tgno − tg(n − 1) .
cos(n- 1)o ⋅ cosno cos(n- 1)o ⋅ cosno cos(n- 1)o ⋅ cosno cos(n- 1)o ⋅ cosno
sin1o sin1o sin1o sin1o
Taka imame + + + ... +
cos0 o ⋅ cos1o cos1o ⋅ cos2 o cos2 o ⋅ cos3 o cos2024 o ⋅ cos2025 o
=tg1±-tg0±+tg2±-tg1±+tg3±-tg2±+…+tg2024±-tg2023±+tg2025±-tg2024±=tg2025±-tg0±
=tg2025±=tg(11·180±+45±)=tg45±=1.
803. Najgolema 9, najmala 1. Bidej}i a 2 + b 2 = 1 , sleduva deka postoi
2 2
c d
α ∈ (− π, π ) za koj sinα = a i cosα = b . Bidej}I   +   =1, sleduva deka
2 2
c d
postoi β ∈ (− π, π ) za koj sinβ = i cosβ = . Od (a - c )2 + (b - d)2 = 5 - 4cos(α - β)
2 2
− 1 ≤ cos (α − β ) ≤ 1 imame deka 1 = 5 - 4 • 1 § (a - c) + (b - d)
2 2
i od toa {to
tgα + tgβ
§ 5 - 4 • (- 1) = 9 . 808. Imame deka tg(α + β) = tg( − γ ) i ottuka = −tgγ ,
1 − tgα ⋅ tgβ
8
od kade sleduva tgα + tgβ + tgγ = tgα ⋅ tgβ ⋅ tgγ . 809. - (sin α + cos α ) . 810. .
9
π π π
811. a) x = kπ ili x = (2k + 1) kœZ; b) x = (2k + 1) , kœZ. 812. x =
+ kπ
10 10 2
π 2 π
ili x = ± + kπ , kœZ. 813. x = 2kπ ili x = π + 2k π , kœZ. 814. x = + kπ ,
6 3 4
kœZ.
815. Koristej}i ja formulata 2cos 2 α = 1 + cos2 α dobivame
2 2
cos2x + cos4x + 2cos 3x = 0 ⇔ 2cos3xcosx + 2cos 3x = 0
⇔ 2cos3x(cos x + cos3x) = 0 ⇔ 4cos3x ⋅ cos2x ⋅ cosx = 0
⇔ cos3x = 0 ∨ cos2x = 0 ∨ cosx = 0
π π π
~ii re{enija se: x = (2k + 1) ; x = (2k + 1) ; x = (2k + 1) ; kœZ.
6 4 2

217
Odgovori, upatstva i re[enija

2 π
816. x = arctg + kπ ili x = − + kπ , kœZ. 817. 9 π . 818. n=12.
5 4
α+β  γ γ γ  α+β α+β
820. sin = sin 90° −  = cos ; sin = sin 90° −  = cos .
2  2 2 2  2  2
α+β α−β γ α+β α−β γ γ
sinα + sinβ + sinγ = 2sin cos + sin2 = 2sin cos + 2sin cos
2 2 2 2 2 2 2
γ α−β γ γ α−β α+β α β γ
= 2cos  cos + sin  = 2cos  cos + cos  = 4cos cos cos .
2 2 2 2 2 2  2 2 2
2 2 2 2
821. P=PóABC+PóACD=(absinβ+cdsinδ)/2 …(1); a +b -2abcosβ=c +d -2cdcosδ…(2).
2 2 2 2 2 2 2
16P =4(a b sin β+2abcdsinβsinδ+c d sin δ)
2 2 2 2 2 2
=4a b (1-cos β)+8abcdsinβsinδ+4c d (1-cos δ) odnosno
2 2 2 2 2 2 2 2 2 2 2
16P =4a b -4a b cos β+8abcdsinβsinδ+4c d -4c d cos δ ...(3). Od (2) se dobiva
2 2 2 2 2 2 2 2 2 2 2 2
(a +b -c -d ) =(2abcosβ-2cdcosδ) =4a b cos β-8abcdcosβcosδ+4c d cos δ
2 2 2 2 2 2 2 2 2 2 2
odnosno 4a b cos β+4c d cos δ=(a +b -c -d ) +8abcdcosβcosδ .
Od poslednoto ravenstvo i ravenstvoto (3) dobivame
2 2 2 2 2 2 2 2 2 2
16P =4a b +4c d -(a +b -c -d ) -8abcdcosβcosδ+8abcdsinβsinδ,
2 2 2 2 2 2 2
16P =4(ab+cd) -(a +b -c -d ) -8abcd(1+cosβcosδ-sinβsinδ),
2 2 2 2 2 2 2 2 2
16P =(2ab+2cd-a -b +c +d )(2ab+2cd+a +b -c -d )-8abcd[1+cos(β+δ)],
2β+δ β+δ
2 2 2 2
16P =[(c+d) -(a-b) ][(a+b) -(c-d) ]-8abcd[1+cos
2
-sin2 ],
2 2
c + d−a+b c + d+a −b a+b −c + d a +b +c −d β+δ
16P2 = 16 ⋅ ⋅ ⋅ ⋅ − 8abcd ⋅ 2cos2 ,
2 2 2 2 2
2β+δ
2
P =(s-a)·(s-b)·(s-c)·(s-d)-abcd·cos .
2
α+β γ+δ
822.Upatstvo: Koristete go ravenstvoto cos = -cos .
2 2
823. a) Upatstvo: Koristete gi adicionite teoremi i kosinusna teorema;
α+β 180 o − γ γ
sin sin cos
b) I re{enie: 2 = 2 = 2 =1
cos
γ cos
γ cos
γ
2 2 2
α+β γ α+β γ α+β
sin 2sin sin 4Rsin sin
II re{enie: 2 = 2 2 = 2 2
γ γ γ c
cos 2sin cos
2 2 2
γ α β α β γ α β γ α β
4Rsin (sin cos + cos sin ) 4Rsin sin cos + 4Rsin cos sin
= 2 2 2 2 2 = 2 2 2 2 2 2
c c
γ α β β γ α α β
4Rsin sin sin cos 4Rsin sin cos sin
2 2 2 2 + 2 2 2 2
β α
sin sin
= 2 2
c

218
Odgovori, upatstva i re[enija

β α
cos cos
r 2 +r 2
β α β α
sin sin rctg + rctg
= 2 2 = 2 2 …(1). Od
c c
β α
crte` 194 imame rctg = m i rctg = n .
2 2
Zamenuvaj}i gi poslednite dve ravenstva
vo ravenstvoto (1) dobivame
α+β
sin
2 = m + n = c = 1. Crte` 194
γ c c
cos
2

α α α
sin 2sin ⋅ cos
v) Imame 2 2 2
β γ= α β γ . Koristej}i go ravenstvoto 820
cos ⋅ cos 2cos ⋅ cos ⋅ cos
2 2 2 2 2
α
sin
2 sinα 2Rsinα
dobivame
β γ = sinα + sinβ + sinγ = Rsinα + Rsinβ + Rsinγ , od kade so
cos ⋅ cos
2 2 2
koristewe na sinusna teorema se dobiva baranoto ravenstvo.
g) Koristej}i go ravenstvoto od zada~a 823 v) go dobivame ravenstvoto
α β γ
sin sin sin
2 2 2 abc
⋅ ⋅ = 3 . Taka,
β γ α γ cos α ⋅ cos β s
cos ⋅ cos cos ⋅ cos
2 2 2 2 2 2
α β γ abc α β γ
tg ⋅ tg ⋅ tg = 3 ⋅ cos ⋅ cos ⋅ cos . Koristej}i ja zada~a 820 dobivame
2 2 2 s 2 2 2
α β γ abc sinα + sinβ + sinγ abc R
tg ⋅ tg ⋅ tg = 3 ⋅ = ⋅ ⋅ (sinα + sinβ + sinγ ) …(1). Od
2 2 2 s 4 4R s 3
abc
druga strana, = P = r ⋅ s …(2). Koristej}i ja sinusna teorema imame
4R
a b c
( + + )
a b c s
sinα + sinβ + sinγ = + + = 2 2 2 = …(3). Zamenuvaj}i gi
2R 2R 2R R R
α β γ R s r
ravenstvata (2) i (3) vo (1) dobivame tg ⋅ tg ⋅ tg = r ⋅ s ⋅ 3 ⋅ = .
2 2 2 s R s
α β γ
d) Upatstvo: 4cos cos cos = sinα + sinβ + sinγ i sinusna teorema;
2 2 2
|) Upatstvo: a=2Rsinα, b=2Rsinβ, c=2Rsinγ;
e) Upatstvo: sinusna teorema i 823 |);

219
Odgovori, upatstva i re[enija

ab ⋅ sinγ α β γ s
`) Upatstvo: r·s=P= i 4cos cos cos = ;
2 2 2 2 R
β-γ α β−γ α β γ β γ
cos 2cos ⋅ cos 2cos (cos ⋅ cos + sin ⋅ sin )
z) 2 = 2 2 = 2 2 2 2 2 .
α α α sin α
sin 2sin ⋅ cos
2 2 2
β-γ α β γ β γ
cos 4Rcos (cos cos + sin sin )
Koristej}i ja sinusna teorema imame 2 = 2 2 2 2 2
α a
sin
2
α
4Rcos
β γ
cos cos + 4Rcos sin sin
α β γ
= 2 2 2 2 2 2 . Od zada~a 795 b) imame
a
β- γ β γ
cos s + s ⋅ tg tg
2 = 2 2 , a od zada~a 823 g) dobivame
α a
sin
2
r
s+ s⋅
β-γ α α
cos s ⋅ tg s + r ⋅ ctg
2 = 2 = 2 …(1). Taka, od crte` 191 dobivame
α a a
sin
2
β-γ
α cos
rctg = n …(2). Zamenuvaj}i (2) vo (1) imame 2 = s+n = b+c .
2 α a a
sin
2
826. Neka O e centar na opi{anata kru`nica, a , M, N i P se proekciite na O
vrz AB, BN i AC, soodvetno. Od triagolnikot AMO sleduva OM=Rcosγ=dc
(crt. 195). Od triagolnikot BNO sleduva ON =Rcosα=da.
Od triagolnikot CPO sleduva OP =Rcosβ=db. Zna~i,
da+db+dc=R(cosα+cosβ+cosγ).
Od zada~a 357 sleduva R+r=R(cosα+cosβ+cosγ),
r
odnosno cos α + cos β + cos γ = 1 + 4 . Od zada~a 823 `)
4R
α β γ
sleduva cos α + cos β + cos γ = 1 + 4sin sin sin .
2 2 2

Crte` 195
100
827. . Upatstvo: So koristewe na sinusna teorema va`i ravenstvoto
7
a+b +c
sinα + sinβ + sinγ = .
2R

220
Odgovori, upatstva i re[enija
Od ednakvost na tangentnite otse~ki i
definicija za kotangens imame deka
α β γ a+b+c
ctg + ctg + ctg = (crt. 196).
2 2 2 2r

α
828. 2m ⋅ cos α ⋅ ctg .
2
Crte` 196

829. 2Pcosαcosβcos γ . Okolu ~etiriagolnikot MBCP mo`e da se opi{e


kru`nica bidej}i ®BPC=®CMB=90± (crt. 197).
Zna~i, ®APM=β, spored toa triagolnikot APM e sli~en so triagolnikot ABC,
so koeficient na sli~nost AP : AB = cos α .
2
Od ovde sleduva ravenstvoto PóAMP:PóABC=cos α
2
odnosno PóAMP=P·cos α. Analogno, se dobivat
2 2
ravenstvata PóBNM=P·cos β i PóNCP=P·cos γ . Toga{
PóMNP=P-P·cos α-P·cos β-P·cos γ =2P·cosαcosβcos γ .
2 2 2

1 9
830. P =
4
(m − n ) ⋅ tgα . 831. . Upatstvo: Se koristat
2 2
2
absin γ
formulite P = , P=rs i sin(180º-γ)=sinγ.
2
Crte` 197

832. Jasno e deka P∆ABC : P∆MNP = 4 : 1 (crt. 198) od kade sleduva deka
1 abc
P∆MNP = P∆ABC = . Od druga strana
4 16R
xzsin(180 o - β) xysin(180 o - γ ) yzsin(180 o - α) xzsin β xysin γ yzsin α
P∆MNP = + + = + + .
2 2 2 2 2 2

a b
Od sinusna teorema imame: sinα = , sinβ = i
2R 2R
c
sinγ = . Zamenuvaj}i vo ravenkata (1) dobivame
2R
xzb xyc yza abc
+ = . Mno`ej}i go poslednoto
4R 4R 4R 16R
ravenstvo so 4R se dobiva baranoto ravenstvo.
Crte` 198

833. Neka triagolnikot ABC e pravoagolen so γ = 90± pri {to 0<α<β< γ <180±
P - Q = sinα + sinβ + sinγ - (cosα + cosβ + cos γ + 1)
α+β α-β  α+β α-β 
= 2sin cos + 1 -  2cos cos + 1
2 2  2 2 

221
Odgovori, upatstva i re[enija

α-β α+β α+β α-β γ γ


= 2cos  sin - cos  = 2cos  cos - sin  = 0 odnosno P=Q.
2  2 2  2  2 2
Neka P=Q, odnosno sinα + sinβ + sinγ = cosα + cosβ + cosγ + 1. Od tuka se dobiva
sin α - sin (90° - α ) + sin β - sin (90° - β ) + sin γ - sin (90° - γ ) = 1 .
2 sin (α - 45°) + 2 sin (β - 45°) + 2 sin (γ - 45°) = 1
2
sin(α - 45°) + sin(β - 45°) + sin(γ - 45°) =
2
sin(α - 45°) + sin(β - 45°) = sin45°- sin(γ - 45°)
α+β  α-β γ  γ
2sin - 45° ⋅ cos = 2cos ⋅ sin 45°- 
 2  2 2  2
 γ α-β γ  γ
sin 45°- . ⋅ cos = cos ⋅ sin 45°- 
 2  2 2  2 
 γ  α-γ γ
sin 45°-  ⋅  cos - cos  = 0
 2  2 2
 γ  β  γ
sin (45°- sin 45°- sin 45°-  = 0 .
 2  2  2
Sleduva deka eden od aglite α, β ili γe 90±, pa triagolnikot АВС e
α
pravoagolen. 834. a=b=2r·cos .
2
835. Neka AC1 = m = AB ; BC1 = n = BA i CA 1 = CB 1 = q (crt. 199).
1 1

o α+β
xy ⋅ sin(180 − )
c ⋅ r xy ⋅ sin∠AOB 2
= = =
2 2 2
o
o 180 − γ o o γ
xy ⋅ sin(180 − ) xysin(180 − 90 + )
= 2 = 2
2 2
γ xy ⋅ q
c ⋅ r = xy ⋅ cos odnosno c ⋅ r =
...(1). Crte` 199
2 z
yzm xzn
Analogno se dobivat ravenstvata a ⋅ r = ...(2), b ⋅ r = ...(3). Mno`ej}i gi
x y
3 abc xyz
(1), (2) i (3) imame abcr =xyzmnq odnosno = 3 ...(4). Od triagolnikot
mnq r
α r β r γ r
AOC1 imame sin = ; sli~no imame deka sin = ; sin = pa
2 x 2 y 2 z

222
Odgovori, upatstva i re[enija

α β γ r3 α m
sin sin sin = ...(5) . Od triagolnikot AOC1 imame cos = sli~no
2 2 2 xyz 2 x
β n γ q α β γ mnq
imame deka cos = ; cos = pa cos cos cos = ...(6).
2 y 2 z 2 2 2 xyz

Od (5) i (6) dobivame 8sin


α β γ α β γ 8r 3mnq odnosno
⋅ sin ⋅ sin ⋅ cos ⋅ cos ⋅ cos =
2 2 2 2 2 2 (xyz)2
8r 3 mnq
sinα ⋅ sinβ ⋅ sinγ = ...(7). Od sinusnata teorema imame deka
(xyz) 2
abc 8r 3 mnq abc
sinα ⋅ sinβ ⋅ sinγ = ...(8). Od (7) i (8) va`i =
8R3 ( xyz ) 2
8R 3
64r 3 R 3 abc xyz
2
= = 3 , od kade sleduva deka (xyz)3 = 64r 6R3 odnosno
( xyz ) mnq r
xyz = 4r 2R .
836. a+b=13. Neka S = sin2 x 1 + sin2 x 2 + sin2 x 3 + ... + sin2 x 2019 , od sinusna teorema
za 2019-agolnik vpi{an vo kru`nica so radius R=1/2 imame:
a1 a2 a3 a 2019
= = = ... = = 2R = 1 odnosno
sinx1 sinx 2 sinx 3 sinx 2019
a1 = sinx1; a 2 = sinx 2 ; a 3 = sinx 3 ;..., a 2019 = sinx 2019 . .
Zada~ata se transformira vo presmetuvawe na zbir od kvadrati na strani na
2019-agolnik vpi{an vo kru`nica so dijametar 1.
Lema: n-agolnik (n¥4) vpi{an vo kru`nica so dijametar 1 ima barem eden
vnatre{en agol najmalku 90±. Namaluvaj}i edno teme dobivame (n-1)-agolnik
~ija vrednost na S ne se namaluva.
Dokaz: Da go razgledame n-agolnikot A1A2A3…An-1An (crt. 200). Spored
2 2 2
kosinusna teorema z ¥x +y . Otfrlaj}i go temeto An (~ij vnatre{en agol e
2 2 2
najmalku 90±) dobivame n-1 agolnik vo koj x +y se zamenuva so z i pritoa se
dobiva zbir na kvadrati koj e pogolem od prethodniot zbir. Prodol`uvaj}i
taka da gi otfrlame temiwata, doa|ame do triagolnik ABC so strani a, b i c.
Na{ata zada~a se sveduva na odreduvawe
maksimalnata vrednost na
2 2 2
S = AB + BC + CA . Od teoremata na Lajbnic
imame
1 
⋅  AB + BC + CA 
2 2 2 2 2 2 2
PA + PB + PC = 3PT +
3  
kade P e proizvolna to~ka, a T te`i{te na
triagolnikot ABC. Zamenuvaj}i P ª O
imame
3 ⋅  OA + OB + OC  = 9OT + AB + BC + CA
2 2 2 2 2 2 2

 
2 2
odnosno S = 3 ⋅ 3R2 − 9OT = 9R2 − 9OT ≤ 9R2. Crte` 200
1 9
Maksimalnata vrednost na S e 9R 2 = 9 ⋅ = . Zna~i, а+b=13.
4 4

223
Odgovori, upatstva i re[enija

2 2 2
837. Upatstvo: Zameni TC 1 =m; ®C1TO=j i doka`i deka c =12m +8 OT ·m·cosj,
2
c 3
2
a potoa koristi go ravenstvoto 18m =2tc =a +b -
2 2
.
2
. 838. −
2 2
839. Od triagolnikot AMC imame deka ®AMC=180±-[j+(α-j)]=180±-α. Sleduva
deka sin(®AMC)=sinα.
Primenuvaj}i ja sinusna teorema za triagolnicite CAM, ABM i BCM dobivame
sinϕ , sinϕ i sinϕ . Imame
MA = b MB = c MC = a
sinα sinβ sinγ

PóABC=PóABM+ PóBCM+ PóCAM = MA ⋅ MB ⋅ sinβ + MB ⋅ MC ⋅ sinγ + MC ⋅ MA ⋅ sinα ,


2 2 2
1 sin2ϕ 1 sin2ϕ 1 sin2ϕ
PóABC= bc + ca + ab i ottuka sleduva
2 sinα 2 sinβ 2 sinγ

PóABC= P∆ABC
sin 2ϕ sin 2ϕ sin 2ϕ .
+ P∆ABC + P∆ABC
sin 2 α sin 2β sin 2 γ
Delej}i go poslednoto ravenstvo so sin2ϕ ·PóABC se dobiva barenoto ravenstvo.

841. I re{enie: Povlekuvame prava р || AB niz to~ka E (crt. 201).


Neka р I АD={M} i р I ВС={N}. Neka
AM = BN = x . Toga{ DM = CN = a − x .
x
Od triagolnikot АМE imame deka tg15° =
a
2
a
(а strana na kvadratot), odnosno x = ctg75° .
2
Od triagolnikot МED sleduva deka
a−x
tg α = = 2 − ctg75 ° = 2 − ctg(45 ° + 30 ° ) = 3 . Crte` 201
a
2
Zna~i α = 60° od kade dobivame deka 75±=®EAD=®AED
odnosno triagolnikot AED e ramnokrak od kade
sleduva deka DE = AD = a . Analogno se poka`uva
deka EC = BC = a .
II re{enie: Neka F e to~ka vo vnatre{nosta na
kvadratot ABCD taka {to ®FAD=®FDA=15± (crt. 199).
Toga{ DAFD@DBEA od kade sleduva ravenstvoto
AF = AE , a bidej}i ®FAE=60± sleduva deka
triagolnikot AEF e ramnostran. Crte` 202

224
Odgovori, upatstva i re[enija

Zna~i EF = FD i ®DFE=150±, pa DEFD@DAFD od kade dobivame deka


ED = AD , odnosno DE = DC . Od ®CDE=90±-2ÿ15±=60± i DE = DC dobivame
deka triagolnikot CDE e ramnostran.
2a a 3
842. L = . 843. R= .
2 α 3sin2 α
cos
2
845. k . So α i β }e gi ozna~ime aglite koi gi
zafa}aat bo~nite strani ASB i BSC so
ramninata na osnovata ABC, soodvetno. Bidej}i
CS¦SB i AS¦BS (crt. 203), triagolnicite ASB
i BSC se ortogonalni proekcii na triagolnikot
ABC vrz ramninite na triagolnicite ASB i BSC,
soodvetno. Zaradi vovedenite agli α i β
P∆ASB P∆BSC
imame = cosα , = cosβ odnosno
P∆ABC P∆ABC
P∆ASB cosα
= …(1).
P∆BSC cosβ
Crte` 203

Od druga strana triagolnicite AOB I BOC se proekcii na ASB i BSC vrz


ramninata na osnovata ABC.
P P
Zatoa ∆AOB = cosα , ∆BOC = cosβ .
P∆ASB P∆BSC
P∆BOC 1

P∆ASB P∆BSC P∆BOC cosβ ⋅ P∆AOB cosβ …(2). Od (1) i (2) dobivame
Toga{ = = =k⋅
P∆BSC P∆AOB 1 cosα ⋅ P∆BOC cosα

P∆ASC P∆AOB

cosβ cosα cosα P∆ASB


k⋅ = odnosno = k . Zna~i od (1) dobivame = k.
cosα cosβ cosβ P∆BSC

846. P= π H sin α .
2

2
847. a) Od triagolnicite OBE i AEO gi dobivame
OE ϕ OA
ravenstvata sinα = , tg = toga{
OB 2 OE
ϕ OA
sinα ⋅ tg = (crt. 204). No, OA = OB zna~i
2 OB
ϕ
sinα = ctg .
2
Crte` 204

225
Odgovori, upatstva i re[enija

ϕ OA AE
b) Od triagolnicite AEO i ABE imame deka sin = , sin(®ABE)= . No od
2 AE AB
o β β AE
triagolnikot ABV se dobiva deka ®ABE= 90 − toga{ cos = . Zna~i
2 2 AB
ϕ β OA AE OA 2
sin ⋅ cos = ⋅ = sin45o = .
2 2 AE AB AB 2
OP β AP
v) Od triagolnicite VPO, AVP i AOP imame deka cos γ = , tg = i
VP 2 VP
AP β ϕ β 2
= tg45 o = 1 toga{ tg = cosγ . Od ravenstvoto sin ⋅ cos = (vidi vo
PO 2 2 2 2
ϕ 2
istata zada~a pod b)) dobivame deka sin = od kade sleduva
2 β
2 ⋅ cos
2
2 ϕ 1 2 β 2
ravenstvoto 2sin = = 1 + tg = 1 + cos γ . Od formulata
2 β 2
cos 2
2
2 ϕ
dobivame deka cosϕ = −cos γ . 848. N=11. 849. α = 37o30' .
2
cosϕ = 1 − 2sin
2
3 2

2 3− 2
2 2
tgα = 3 ⋅ 2 + 3 − 2 − ( 2 ) = ( 3 − 2 )( 2 + 1) = = ,
2 −1 2 1

2 2
 75o 
105o 15o cos 90o − 75o
2cos sin  2  sin
sin60o − sin45o 2 2 =  = 2 = tg 75
o
i ottuka
tgα = =
sin45o − sin30o 75o 15o 75o 75o 2
2cos sin cos cos
2 2 2 2
75 o
α= .
2
851. Zamenuvaj}i za x+y=-z go dobivame neravenstvoto |cosx|+|cosy|+|cosz|¥1.
Imeno x+y+z=0 imame 1=|cos(x+y+z)|=|cosx·cos(y+z)-sinx·sin(y+z)|
§|cosx|·|cos(y+z)|+|sinx|·|sin(y+z)|§|cosx|+|sin(y+z)|§|cosx|+|siny|·|cosz|+|cosy|·|sinz|
§|cosx|+|cosy|+|cosz|.
852. Od a≥b i od toa {to sproti pogolem agol le`i pogolema strana imame
deka α≥β odnosno a-b i α−β se istovremeno negativni ili pozitivni pa
(a-b)·(α−β)≥0. Sli~no dobivame deka (b-c)·(β−γ)≥0 i (c-a)(γ−α)≥0. Spored toa,
(a-b)(α−β)+(b-c)(β−γ)+(c-a)(γ−α)≥0. Poslednoto neravenstvo se transformira
vo oblikot a(2α−β-γ)+b(-α+2β-γ)+c(-α-β+2γ)≥0. Od α+β+γ=π dobivame deka
π aα + bβ + cγ
3(aα+bβ+cγ)-π(a+b+c)≥0, odnosno ≤ . Bidej}i zbirot na dvete
3 a+b+c

226
Odgovori, upatstva i re[enija

strani e pogolem od tretata strana imame (b+c-a)α+(c+a-b)β+(a+b-c)γ>0,


odnosno a(-α+β+γ)+b(α-β+γ)+c(α+β-γ)>0. Zamenuvaj}i α+β+γ=π, dobivame
a α + b β + cγ π
-2(aα+bβ+cγ)+π(a+b+c)>0 odnosno < .
a+b+c 2
853. Neka vo pravoagolniot triagolnik ABC, AM e te`i{nata linija a AK
simetralata na ®BAC (crt. 205).
Zamenuvaj}i ®BAC=2α, ®ABC=2β, ®MAK= ϕ i
®MAC=α1 dobivame deka
1
®MAK=®MAC-®KAC=®MAC- ·®BAC odnosno
2
ϕ =α1-α…(1). Od (1) dobivame
tgα 1 - tgα
tgϕ = tg(α 1 - α ) = . Crte` 205
1 + tgα ⋅ tgα 1
BC 1 tgα 3
No, od tgα1 = = tg2α = sleduva tg ϕ =tg α …(2). Analogno, se nao|a
2AC 2 2
1 − tg α
deka tg ψ =tg β. Toga{ 1 +
3
( 3 )( )
tg ϕ ⋅ 1 + 3 tg ψ = 1 + tg α + tg β + tg α ⋅ tg β …(3). Od toa
tgα + tgβ
{to α+β=45±, dobivame tg(α + β ) = = 1 odnosno tgα+tgβ+tgα·tgβ=1, od
1 − tgα ⋅ tgβ
7
(
kade 1 + 3 )( )
tg ϕ ⋅ 1 + 3 tg ψ = 2 . 855. cos β =
25
.
856. To~kite A1, B1 i C1, se podno`ni to~ki na visinite spu{teni od temiwata
A, B i C, soodvetno. Neka k1(O1,R1), k2(O2,R2), k3(O3,R3), se opi{anite kru`nici
okolu triagolnicite AHB, BHC i AHC soodvetno. Niz to~kata N vo kru`incite
k1 i k3 povlekuvame dijametri HP1 i HP2 soodvetno. Od skladnosta na
triagolnicite BB1A i CC1A sleduva ®ABH=®ACH=α.
Isto taka ®AP3H=®ACH (kako periferni agli nad ist lak) i ®AP1H=®ABH.
Spored talesova teorema triagolnicite P3AH i P1AH se pravoagolni. Spored
AH AH
toa, = sinα i = sinα . Od poslednite ravenstva dobivame deka R1=R3.
2R 1 2R3
Analogno, R1=R2=R3.
1 1 1
857. = + . Koristej}i go ravenstvoto 823 z) imame
а b c
α β γ b+c α α−γ α−β
sin cos cos = ⋅ sin ⋅ sin ⋅ sin , od kade dobivame deka
2 2 2 a 2 2 2
α+γ α+β b+c α−γ α−β
sin ⋅ sin = ⋅ sin ⋅ sin .
2 2 a 2 2

227
Odgovori, upatstva i re[enija
Koristej}i gi adicionite teoremi vo poslednoto ravenstvo dobivame
α γ α γ α β α β
sin cos + cos sin sin cos + cos sin
2 2 2 2 ⋅ 2 2 2 2 = b + c . Delej}i ja prvata
α γ α γ sin α cos β − cos α sin β a
sin cos − cos sin
2 2 2 2 2 2 2 2
α γ α β
dropka od poslednoto ravenstvo so sin sin , a vtorata dropka so sin sin
2 2 2 2
γ α β α
ctg + ctg ctg + ctg
dobivame 2 2 ⋅ 2 2 = b+c . Od definicijata za
γ α β α a
ctg − ctg ctg − ctg
2 2 2 2
p n m n
+ +
trigonometriskite funkcii (crt. 191) imame r r ⋅ r r = b + c . Ottuka,
p n m n a
− −
r r r r
b c b+c b c b+c
⋅ = , odnosno ⋅ = …(1). Ravenstvoto (1)
p − (c − m) m − (b − p) a a−c a−b a
1 1 1 1 1 1
se zapi{uva kako = + , odnosno = + .
bc b c  1 1  bc b c
a−b−c + a − bc +  +
a b c a
 1 1    bc  1 1 
Poslednoto ravenstvo se razlo`uva kako  +  ⋅ a − 1 ⋅ 1 − ⋅  +  = 0 .
 b c    a  b c 
1 1 1
Taka dobivame deka = + ili a=b+c koe ravenstvo ne va`i dokolku a, b i
a b c
c se strani na triagolnik.

Tema 17: Analiti~ka geometrija


858. Ako to~kata A (x,y) gi zadovoluva uslovite od zada~ata, toga{ i to~kite
B(x,–y); C (–x,y) i D (–x,–y) isto taka gi zadovoluvaat uslovite. Zatoa }e gi
izbroime to~kite koi le`at vo prviot kvadrant, zaedno so to~kite od
2 2 2
pozitivniot del na x–oskata. Neka x + y § 10 .
2 2
Za x = 1, y § 99, pa y œ {1, 2, …, 9}. Za x = 2, va`i y § 96, od kade yœ{1, 2, …, 9}.
2
Za x = 3 i x=4, y œ {1, 2, …, 9}. Za x = 5, va`i y § 75, pa y œ {1, 2, …, 8}. Sli~no
za x = 6, y œ {1, 2, …, 8}. So istite presmetki, za x = 7, y œ {1, 2, …, 7}. Za x = 8,
2
y § 36 pa y œ {1, 2, …, 6} i za x=9, yœ{1, 2, 3, 4}.
Brojot na to~ki vo prviot kvadrant, zaedno so to~kite {to le`at na
pozitivniot del od x-oskata e 79. Baraniot broj e z = 4ÿ79 + 1 (kade edna
to~ka e koordinatniot po~etok). Od ovde dobivame deka baraniot broj e 317.
859. 296. 860. 48. 861. b) kœ(-¶,-3)»( 1 ,+¶). 862. To~kata P e presekot na
6
dijagonalite.

228
Odgovori, upatstva i re[enija
2
(a + b)
863. P = . Triagolnikot ABC go
4
postavuvame vo Dekartov koordinaten
sistem taka {to temeto C e identi~no so
koordinatniot po~etok O(0,0) (crt. 206).
Taka postaveniot pravoagolen
triagolnik gi ima slednite temiwa
A(b,0), B(0,a) i C(0,0). Koordinatite na
sprotivnite temiwata na kvadratot,
konstruiran nad hipotenuzata, se B(0,a) i
D(a+b,b) (triagolnikot AMD e skladen so
triagolnikot ABC).
C
Crte` 206
Pa taka, centarot na kvadratot konstruiran nad hipotenuzata ima koordinati
a+b a+b
T( , ). Koordinatite na centrite na kvadratite konstruirani nad
2 2
a a b b
katetite, se P( , − ) i Q( − , ). Jasno e deka pravata koja minuva niz
2 2 2 2
to~kite P i Q }e bide normalna so pravata TC. Taka, presmetuvaweto na
plo{tinata na baraniot triagolnik PTQ se sveduva na odreduvawe na
a+b
rastojanijata PQ i TC , koi se ednakvi na . Spored toa, plo{tinata na
2
2
(a + b)
baraniot triagolnik e P = .
4
864. 25 (crt. 207). Upatstvo: odredi gi koordinatite na to~kite A’ i B’ koi se
sliki na to~kite A i B pri osna simetrija so oski y i x soodvetno, a potoa
odredi ja dol`inata na otse~kata A ′B′ .
4 2
865.  ;  . Upatstvo: koga mravkata se
5 5
dvi`i desno ili nagore toga{ x odnosno y
koordinatata se zgolemuva, a koga se dvi`i
levo ili nadolu toga{ x odnosno y
koordinatata se namaluva. Posebno
razgleduvaj go dvi`eweto koe ja
opredeluva x-koordinatata, posebno za y-
koordinatata.
Crte` 207

866. (14,-22). Da gi razgledame to~kite koi le`at vo negativniot del na


y-oskata. Vo 7-ot ~ekor to~kata e (0,-1), vo 22-ot ~ekor to~kata (0,-2) i vo
45-ot ~ekor to~kata e (0,-3). Zna~i to~kata (0,-n) e vo ~ekorot
2 2 2
(2n+1) -(n+1)=4n +3n. Re{avaj}i ja ravenkata 4n +3n=2002 dobivame
(n-22)(4n+91)=0, odnosno n=22. Vo 2002-ot ~ekor to~kata e (0,-22). Dvi`ej}i
se na desno (horizontalno) za 14 ~ekori go dobivame 2016-tiot ~ekor, koj

229
Odgovori, upatstva i re[enija
2 2
odgovara na to~kata (14,-22). 867. A(-1,3). Upatstvo: x +2x+4=(x+1) +3¥3 i
2 2
y -6y+11=(y-3) +2¥2. 868. x=1; x=-1; y=1; y=2 6 x-5; y=-2 6 x-5.
n 1 n 1
869.Ortocentarot. 870. 90±. 871. Parot (2 - ,2 - ), za m=2n-1. 872. P=48.
2
873. P=3. Neka R={(x,y)œR : |x|+|y|+|x+y|§2} e oblasta ograni~ena so dadenata
neravenka. Da zabele`ime deka |a|=|-a| za sekoj a realen broj, od kade sleduva
deka oblasta R e simetri~na vo odnos na koordinatniot po~etok. Zatoa,
dovolno e da go razgledame samo slu~ajot koga y¥0. Vo prviot kvadrant koga
x¥0, y¥0, imame deka x+y¥0, odnosno x+y+x+y§2. Taka, delot od oblasta R vo
toj kvadrant e pravoagolniot triagolnik
2
ABO={(x,y)œR : x¥0, y¥0, x+y§1}. Vo vtoriot
kvadrant koga x§0 i y¥0 imame dva slu~ai
i toa: i) x+y¥0 ili ii) x+y§0.
Vo prviot slu~aj dadenoto neravenstvo se
transformira vo neravenstvoto
-x+y+x+y§2 od kade go dobivame
2
triagolnikot BCO={(x,y)œR : x§0, 0§y§1}.
Vo vtoriot slu~aj imame -x+y-(x+y)§2 od
kade go dobivame triagolnikot
2
CDO={(x,y)œR : -1§x§0 , y¥0} (crt. 208).
Zna~i, plo{tinata na oblasta ograni~ena
so dadenoto neravenstvo e 3.
Crte` 208
874. 1536 . Od vtorata neravenka e jasno deka y koordinatata na sekoja
to~ka od oblasta R e na rastojanie ne pogolemo od 16 od x—oskata. Od
ravenstvata x − y = − x − y = x − − y = − x − − y sleduva deka (x,y) e vo R
ako i samo ako (-x;y); (x,-y) i (-x,-y) se site vo R. Zna~i }e ja
presmetame plo{tinata na R vo I kvadrant (poradi simetrijata) i }e ja
pomno`ime so 4. Vo I kvadrant granicite se dvete oski i pravite x-y=16
i y=16. Dobivame deka oblasta R samo vo I kvadrant e pravoagolen
trapez so temiwa (0,0), (0,16),(16,0),(32,16). Zna~i, P = 4 16 + 32 ⋅ 16 = 1536.
2
 3
875. P=232. 876. P  3,  . 877. 36. 878. Za x=0, y=k i za y=0, x= ± k . Zna~i
 2
dobivame pravoagolnik so strani 2 k i k. Za plo{tinata va`i P1=2k k ,
2 2 4
dodeka plo{tinata na parabolata }e bide P= P1= 2k k = k k .
3 3 3
Plo{tinata }e bide priroden broj za k ∈ {32, 62, 92, ..., 422 } (k ≤ 2018) . Postojat
2
14 takvi broevi. 879. EF = 1 + 5 . 881. a = .
BC 2 3
882. Neka M e proizvolna to~ka za koj
MA : MB = m : n (m > n) (crt. 209). Na pravata AB
postojat dve to~ki P i Q koi ja delat otse~kata
AB vo ist odnos (vnatre{na i nadvore{na).
Pritoa PA : PB = QA : QB = m : n . Crte` 209

230
Odgovori, upatstva i re[enija
Zna~i P i Q le`at na baranoto geometrisko mesto na to~ki. Od uslovot
MA PA QA m
= , sleduva deka MP i MQ se simetrali (vnatre{na i
= =
MB PB QB n
nadvore{na) na agolot AMB. Toga{ agolot me|u MP i MQ e 90± i otse~kata PQ
se gleda od to~kata M pod prav agol. Ottuka to~kata M le`i na kru`nicata so
dijametar PQ . Obratno, neka M e
to~kata od kru`nicata so
dijametar PQ . Niz to~kata B
povlekuvame prava paralelna so
AM, koja ja se~e MP vo to~ka K i
MQ vo to~ka L. Od sli~nosta na
triagolnicite AMQ so BLQ i APM
MA QA m
so BPK, sleduva = = i
LB QB n
MA PA m
= . Toga{ LB = BK
=
BK PB n
odnosno MB e te`i{na linija vo
pravoagolniot triagolnik KML,
od kade MB = LB i
MA : MB = m : n . Crte` 210
2 2 1
883. x +y =25. Neka y=kx+p...(1) i y=- x+q...(2) se tangentite na elipsata
k
2 2
x y
+ = 1 me|usebno zaemno normalni. Od uslovot za tangenta na elipsa, gi
16 9
2 2 16 2
dobivame ravenkite 16x +9=p …(3) Kvadriraj}i gi
i + 9 = q ...(4).
2
k
ravenstvata (1) i (2) i zamenuvaj}i gi vo niv ravenstvata (3) i (4) dobivame
2 2 2 2 2 2 2 2
16x +9=y -2kxy+k x i 16+9k =k y +2kxy+x . Sobiraj}i gi poslednite dve
2 2 2 2
ravenstva dobivame 25+25k =(k +1)(x +y ), od kade sleduva deka re{enieto e
2 2
kru`nica so ravenka x +y =25.
2 2 2
884. 8x +9y =72r . So r1 i r2 gi ozna~uvame radiusite na kru`incite k1 i k2,
soodvetno (crt. 210).
Neka M e to~ka od baranoto geometrisko mesto na to~ki. Spored uslovot na
zada~ata imame deka PM = MQ odnosno MC 1 -r1=r2- MC 2 od kade MC 1 + MC 2
=r1+r2=6r (konstanten zbir). Od poslednoto ravenstvo, a spored definicijata
za elipsa, sleduva deka baranoto geometrisko mesto na to~ki e elipsa so
fokusi vo to~kite C1, C2 i zbir na rastojanija do fokusite 6r. Toga{ 2a=6r, od
2 2 2 2 2
kade a=3r. Od ravenkata a -b =c , nao|ame deka b =8r . Zna~i, ravenkata na
2 2 2
elipsata glasi 8x +9y =72r . 885. parabola.
886. Kru`nicite x +(y-1) =2 i x +(y+1) =2. 887. Kru`nicata x +y =n .
2 2 2 2 2 2 2

231
Odgovori, upatstva i re[enija
888. ^etiriagolnikot ACDF e tetiven i zatoa va`at ravenstvata
®DAF=®DCFñ®BAH=®BCM (crt. 211). ^etiriagolnikot BCEF e isto taka
tetiven ~etiriagolnik, pa ®EBF=®ECFñ®ABH=®ACM.
Od ednakvosta na perifernite agli nad ist lak i ve}e dobienite ravenstva
za aglite dobivame deka ®BAM=®BCM=®BAH i
®ABM=®ACM=®ABH. Od poslednite ravenstva sleduva
deka to~kata M od kru`nicata k e slika na
ortocentarot H pri osna simetrija so oska AB i
obratno, bilo koja to~ka M od kru`nicata k }e se
preslika vo ortocentarot H na triagolnikot ABC.
Zna~i baranoto geometrisko mesto na to~ki (lokus) e
isto taka kru`nica koja e slika na kru`nicata k pri
osna simetrija so oska AB.

Crte` 211

889. Kru`nica. 890. Prava p koja minuva niz to~kata C i e paralelna so


1 2
pravata AB. 891. Parabolata y=(x-5) .
2
892. Gi konstruirame kruænicite k 1 i k 2 so centri vo to~kite A i B
soodvetno i radius AB (crt. 212). Neka
to~kata C e sredina na otse~kata AB i
to~kite A1 ∈ k2 i B1 ∈ k1 se simetri~ni na A
i B vo odnos na centrite na kruænicite.
To~kata M mo`e da bide vrv na
ramnokrakiot triagolnik ili edno od
dvete temiwa na osnovata. ]e gi
razgledime slednite slu~ai:
1) To~kata M e vrv na ramnokrakiot
triagolnik ABM ako i samo ako M e to~ka
od simetralata s na otse~kata AB bez
to~kata C.
Crte` 212
2) To~kata M e teme na osnovata BM na triagolnikot ABM, ako i samo ako
Mœk1 bez to~kite B i B 1 .
3) To~kata M e teme na osnovata AM na triagolnikot ABM, ako i samo ako
Mœk2 bez to~kite A i A 1 .
Baranoto geometrisko mesto na to~ki e unijata na
kruænicite k1, k2 i pravata s bez to~kite A, B, C, A1 i
B1.
893. Neka M e proizvolna to~ka od vnatre{nosta na
triagolnikot ABC i MM1, MM2 , MM3 se rastojanija
do stranite na triagolnikot (crt. 213).
Poznato e deka MM1 + MM2 + MM3 = h (h-visina vo
triagolnikot ABC).
Crte` 213

232
Odgovori, upatstva i re[enija

h
Toga{ edna od stranite na triagolnikot M1M2M3 mora da e pomala od
2
(soglasno neravenstvata me|u stranite na triagolikot).
Gi povlekuvame srednite linii EF, FD, DE. Za sekoja to~ka od triagolnicite
AED, EBF, CDF, edno od rastojanijata do stranite na triagolnikot ABC e
h h
pogolemo ili ednakvo na . Za MϗDEF va`i MM 1 < , pa
2 2
h h
MM2 + MM3 = h − MM1 > h − = , odnosno MM2 + MM3 > MM1 . Baranoto
2 2
geometrisko mesto na to~ki e vnatre{nosta na triagolnikot EFD obrazuvan od
srednite linii na triagolnikot ABC.

894. Neka N e to~ka za koja MN = DA . Toga{


®NAM=®DMA i
®NBM=®BMC, pri {to
~etiriagolnikot AMBN e
tetiven. Bidej}i
dijagonalite vo toj
~etiriagolnik se ednakvi,
sleduva deka AM || BN ili
BM || AN. Vo prviot slu~aj
®AMD=®MAN=®AMB
(crt. 214), a vo vtoriot
slu~aj ®BMC=®MBN=®BMA
(crt. 215).
Crte` 214 Crte` 215
Ako ®AMB=®AMD toga{ ®AMB+®BMC=180± i M le`i na dijagonalata AC, a ako
®BMA=®BMC, toga{ to~kata M le`i na dijagonalata BD. Zna~i, to~kata M
le`i na edna od dijagonalite na rombot ABCD.
895. Neka to~kata O e presekot na
pravite AB i CD, a X to~kata koja
pripa|a na baranoto geometrisko
mesto na to~ki.
Na polupravite OA i OD gi
konstruirame otse~kite OM i ON
ednakvi so otse~kite AB i CD,
soodvetno (crt. 216). Toga{
PóABX+PóCDX=PóOMX+PóONX=PóOMN+PóMXN.
Crte` 216

Od toa {to plo{tinata na triagolnikot OMN e konstantna, sleduva deka


treba da bide konstantna i plo{tinata na triagolnikot MXN, a toa e
ispolneto vo slu~aj koga to~kata X le`i na pravata p, paralelna so pravata
MN. Toga{ baranoto geometrisko mesto na to~ki se to~kite od otse~kata PQ,
kade p…AD={P} i p…BC={Q}.

233
Odgovori, upatstva i re[enija
896. Neka to~kata B e so koordinati (-1,0), to~kata C so koordinati (1,0) i
neka to~kata A(x,y) e to~ka od
baranoto geometrisko mesto na
to~ki (crt. 217). Ako E(m,0) e
prese~nata to~ka na pravite BC i
AD toga{ -1<m<1. Bidej}i to~kata
E e sredina na otse~kata AD,
dobivame deka koordinatite na
to~kata D se (2m-x,-y).
Od toa {to ~etiriagolnikot ABDC
e tetiven, imame EB ⋅ EC = EA ⋅ ED .
Zamenuvaj}i gi nivnite koordinati
(rastojanie me|u dve to~ki) go
dobivame ravenstvoto
2 2 2 2 2
(1-m) ·(1+m) =[(m-x) +y ] , koea se
sveduva na ravenkata
2 2 2
2m -2xm+x +y -1=0.
Crte` 217
Bidej}i to~kata D e edinstvena takva to~ka koja gi zadovoluva uslovite od
zada~ata sleduva deka diskriminantata na poslednata kvadratna ravenka (po
2 2 2
nepoznata m) e 0. Taka, dobivame deka 4x -8(x +y -1)=0 od kade sleduva
ravenstvoto
x2
+ y 2 = 1...(1). Zna~i, geometriskoto mesto na to~ki A, e
2
mno`estvoto od to~ki (x,y) koi ja zadovoluvat ravenkata (1) i pritoa to~kite
A, B, C obrazuvat triagolnik.
Spored ravenstvoto (1), re{enie na zada~ata e elipsa so golema oska 2 ⋅BC i
mala oska BC , pri {to to~kite od elipsata koi le`at na pravata BC ne
pripa|at na baranoto geometrisko mesto na to~ki.
897.Neka e dadena to~kata K koja le`i na stranata AB (crt. 218). Neka CM e
te`i{na linija povle~ena od to~kata C. PóAMC=PóMBC Ako KªM, toga{ pravata
CM e odgovorot na zada~ata. Neka KTM.
Toga{ AK < KB ili AK > KB .
Ako AK < KB toga{ PóAKC<PóBKC .
Za da bide ispolnet uslovot od zada~ata, potrebno
e kon triagolnikot AKC da se dodade triagolnik
~ija plo{tina e ednakva so plo{tinata na
triagolnikot CKM . Neka KN e baranata prava
(NœBC).
Crte` 218

Za da va`i PóCNM =PóKNM potrebno e pravata CK da e paralelna so pravata MN.


Na toj na~in zada~ata se sveduva na odreduvawe na to~ka N koja e presek na
stranata BC so prava niz to~kata M, paralelna so pravata CK. Vo slu~aj koga
AK > KB odreduvaweto na takva prava sleduva po analogija, so toa {to
2
to~kata NœAC. 898. a) (-1/2,1/4); b) u=-x -x.

234
Odgovori, upatstva i re[enija

899. Neka otse~kite AB i CD le`at soodvetno na pravite p i q. Da


zabele`ime deka, pri fiksirani pravi p i q, baranoto geometrisko mesto na
to~ki se opredeluva samo od
dol`inite na otse~kite AB i CD i
konstantata a, bez da zavisi od
polo`bata na tie otse~ki na
pravite p i q (plo{tinata na
triagolnicite AMB i CMD ne se
menuva pri razli~na polo`ba na
otse~kite AB i CD). Zatoa dovolno e
da go razgledame slu~ajot koga
otse~kite AB i CD imaat zaedni~ko
teme vo prese~nata to~ka na pravite
p i q (crt. 219) (AªC).
Crte` 219

Neka M e to~ka od baranoto geometrisko mesto, koja le`i vo vnatre{nosta na


agolot BAD. Neka P∆AMB + P∆CMD = a 2 . ]e go razgledame triagolnikot BMD.

P∆BMD = P∆AMB + P∆CMD − P∆ABD = a2 − P∆ABD . Bidej}i P∆ABD e konstantna veli~ina

(ne zavisi od izborot na M), sleduva deka P∆BMD e konstanta, od kade pak M
le`i na pravata KL paralelna so pravata BD ( BD e fiksna dol`ina, P∆BMD e
fiksna, pa i visinata vo triagolnikot, spu{tena kon BD e nepromenliva).
2 2
2a 2a
To~kite K i L vo odnos na pravite q i p se na rastojanie hK = , hL =
CD AB
…(1) (za ve`ba poka`i deka P∆AKD = P∆ABL = a2 ).
Obratno, neka M e proizvolna to~ka od pravata KL, pri {to to~kite K i L se
na rastojanija opredeleni so ravenkite (1) vo odnos na pravite q i p. Toga{
2
2 AK P∆ADK a P∆ABL CL
P∆ADK = a = P∆ABL . Od odnosot = = = = , dobivame
AB P∆ADB P∆ADB P∆CBD CD
deka KL e paralelna so BD.
2
Toga{ P∆AMB + P∆CMD = P∆ABD + P∆BMD = P∆ABD + P∆BKD = P∆AKD = a . Jasno,
to~kata M le`i na baranoto geometrisko mesto. Od slu~aite koga BªC, BªD i
AªD gi dobivame ostanatite strani na paralelogramot ELKF. Zna~i, baranoto
geometrisko mesto se to~kite od stranite na paralelogramot ELKF.

235
Odgovori, upatstva i re[enija

Tema 18: Nizi i progresii


900. 2017, 901. a100=9901, S100=333400. 902. Koristime deka
3x 2 + 3x + 1 = (x + 1)3 − x 3 . Zamenuvaj}i za xœ{1,2,...,n} i sobiraj}i gi n-te
ravenstva dobivame dobivame 3(12 + 2 2 + .. + n 2 ) + 3(1 + 2 + ... + n) + n = (n + 1) 3 − 13
n(n + 1)(2n + 1)
od kade se dobiva S 2 = .
6
2
n 2 (n + 1) (  ) 1 
; b) n(n + 1)(2n + 1) 3n + 3n − 1 . 904. S = 21 - n +1  ⋅ Upatstvo:
2
903. a)
4 30  2 
 2 
1 2n + 1 − 1
Pomno`ete go ravenstvoto so 1 - . 905. S = . 906. 64. 907. 403.
2 2
908. a2010=2055. 909. 902. Upatstvo: a2=-96 i an=2+an-2=4+an-4=(n-2)+a2. 910. 13.
911. a) 1, 3. Upatstvo: Za n¥5, N=33+10m=3+10(m+3) odnosno N zavr{uva na
cifrata 3. 912. S=1. Upatstvo: n!=n!·(n+2)-(n+1)!
913. nœ{1,2,3}. Jasno e deka n=1 e re{enie na zada~ata. Neka n ¥2 toga{ zbirot
n- 2 n -1
2
Ѕ se zapi{uva kako S = n(n - 1)! + n ⋅ 3 ⋅ 4 ⋅ ... ⋅ (n − 1) + ... + n .(n - 1) + n n n -1
+
(n - 1)! (n - 1)! (n - 1)! (n - 1)! (n - 1)!
2 n−2 n-1
n(n - 1)!+n ⋅ 3 ⋅ 4 ⋅ ... ⋅ (n - 1) + ... + n ⋅ (n - 1) + 2 ⋅ n . Broitelot vo posledniot izraz
S=
(n - 1)!
n 1
e deliv so n-1 ako (n-1)|2ÿn - , odnosno (n-1)|2. Zna~i n=2 ili n=3.
2+n 1
914. Sn=2- n . 915. a500=1. 916. a2009= .
2 2
2
917. a2013=2013 . Ako zamenime za m=n vo dadenoto ravenstvo dobivame
1 1
a2m+a0= ·(a2m+a2m) od kade a0=0. Za n=0 va`i ravenstvoto am+am= (a2m+a0)
2 2
1
odnosno a2m=4·am …(1). Ako m=n+2 toga{ a2n+2+a2= ·(a2n+4+a2n) …(2). Od
2
ravenstvoto (1) sleduva a2n+2=a2·(n+1)=4·an+1 i a2=4a1=4, od kade dobivame
a2n+2+ a2=4·an+1+4·a1=4·(an+1+1) …(3). Od druga strana od (2) i (1) imame deka
1
a2n+2+ a2= ·(4·an+2+4·an)=2·an+2+2·an ...(4). Od ravenstvata (3) i (4) se dobiva
2
rekurzivnata relacija an+2=2·an+1-an+2, a0=0,a1=1. Od ova relacija se dobiva
2
deka a2=4, a3=9, a4=16, … od kade mo`eme da pretpostavime deka an=n koja
pretpostavka }e ja doka`eme so principot na matemati~ka indukcija.
1±) Za n=0 i n=1 tvrdeweto e to~no;
2 2
2±) Da pretpostavime deka an=n i an+1=(n+1) za nekoe n ¥0.
2 2 2 2
Toga{ an+2=2·(n+1) -n +2=n +4n+4=(n+2) so {to dokazot e zavr{en. Zna~i,
2
a2013=2013 .
918. Upatstvo: Doka`i deka 100...0 3
1231 = 100...0
123 300...0 1231 za n¥0.
123 300...0
n n n n

236
Odgovori, upatstva i re[enija
n +1
919. b) Neka N = 4 ⋅ 11...1 { ⋅ 10 + 9 . No,
{ ⋅ 10 + 8 ⋅ 11...1
n +1 n
k
10 − 1
11...1
{ = 10
k −1
+ 10
k −2
+ ... + 10 + 1 = . Sleduva deka
k 10 − 1
2
10n + 1 − 1 10n − 1 4 ⋅ 102(n + 1) + 4 ⋅ 10n + 1 + 1  2 ⋅ 10n + 1 + 1 
N= 4⋅ ⋅ 10n +1 + 8 ⋅ ⋅ 10 + 9 = = .
9 9 9  3 
 
n−1
2
920. (n + 1)! ÿn. 921. S = () ()
. Upatstvo: Koristi deka n0 + 1n + ... + nn = 2n i ako n ()
n!
e neparen broj toga{ n () ()
n n n ( ) () ()
n
0 + 2 + ... + n-1 = 1 + 3 + ... + n .
n ()
11 x 4 + x 3 + x 2 - x + 1 1  - x 2 + 2x x 2 + 2 
922. . Izrazot = + 3
12 x6 - 1 2  x 3 + 1 x - 1 
1 1 -1 -1 1 
=  + 2 + + 2  .Od ovde imame:
2  x - 1 x + x + 1 x + 1 x - x + 1

∞ ∞ ∞
x4 + x3 + x2 − x + 1 1  1 1  1  1 1 

x=2
6
x −1
= ∑ − +  2 ∑ − 2 
2 x=2 x − 1 x + 1 2 x=2  x − x + 1 x + x + 1
1  1 1  1 1 11
= ⋅ +  + ⋅ = . 923. 59334.
2  1 2  2 3 12
n +1
924. 2012. Jasno e deka n +1 >1 od kade sleduva
n
3 44 2013
2+3 + + ...2013 > 1 + 1 + ... + 1 = 2012 .
2 3 2012 142 2012
43

 1
1 +  + 11+42
1 + ...
4 43 +1
4
n + 1  1  n  n n 1 1 1
Od n+1 = n+1 1 +  ⋅ 1 ≤ = 1+ = 1+ −
n  n n+1 n(n + 1) n n+1
3 44 2013
sleduva deka 2 +3 + + ... + 2013
2 3 2012
1 1 1 1 1 1 1 1
§2012+1- + − + − + ... + − = 2013 − <2013.
2 2 3 3 4 2012 2013 2013
3 4 2013
Zna~i, 2012< 2 + 3 + 4 + ... + 2013 <2013 od kade
2 3 2012
 3 4 2013 
 2 + 3 + 4 + ... + 2013  =2012.
 2 3 2012 
925. Neka x=m+a, kade [x]=m i {x}=a.Toga{, {x²}={m²+2am+a²} i {x}²=a². Od
uslovot na zada~ata imame deka {x²}={x}². No, bidej}i drobnite delovi na dva
broja se ednakvi ako nivnata razlika e cel broj, toga{ imame deka
m²+2am+a²-a²=k, kœZ odnosno m²+2am=k. Bidej}i, m² e cel broj sleduva deka

237
Odgovori, upatstva i re[enija

j j
2am e cel broj. Neka 2am=j, toga{ a= odnosno x=m+ . Za m=1,
2m 2m
1 3 9 5 11
0 ≤ j<2·1; x1=1+0=1 i x2=1+ = . Za m=2, 0 ≤ j<2·2; x1=2, x2= , x3= , x4= .
1⋅ 2 2 4 2 4
Op{to, za m<x<m+1 postojat 2m re{enija za 0 ≤ j<2m. Neka T(m) e zbirot na
 98

ovie 2m re{enija. Toga{ S= 

∑ T(m)  +99 (kade +99 e od faktot deka x=99 e
m =1

re{enie).
1 2 2m − 1
T(m)=m+(m+ )+(m+ )+....+(m+ ),
2m 2m 2m
2m  1
· m + ( m + 2m − 1 ) =m (2m + 2m − 1 ) =2m +m- .
2
T(m)=
2  2m  2m 2
98  1 98 98 98 1
Zna~i , S=99+ ∑  2m 2 + m −  =99+2 ∑ m2 + ∑ m - ∑
m =1  2 m=1 m =1 m =1 2

98 ⋅ 99 ⋅ 197 98 ⋅ 99 1
=99+2· + -98· =99+637098+4851-49=641999
6 2 2
[S]=[641999]=641999 .Baraniot broj e 641999.
 m n +1 + 1  mn+1 + 1 2 m2 − 1
926. Ja razgleduvame funkcijata   kade = m − .
m
n −1
+ 1 mn−1 + 1 mn−1 + 1
Razgleduvame dva slu~ai:
m2 − 1
I) koga n ≥ 3 toga{ m2 − 1 ≤ mn−1 + 1 , pa ≤ 1 . Vo toj slu~aj imame deka:
mn−1 + 1
m2 − 1  n+1 
≤ m 2 , odnosno  m + 1 =m -1;
2
m 2 − 1 ≤ m 2 − n −1
n −1
m +1  m + 1
mn+1 + 1 m3 + 1
II) koga n = 2 , imame = = m2 − m + 1 .
mn−1 + 1 m +1
k
Vo prviot slu~aj S( k, n )= ∑ (m
m=2
2
)
− 1 (ne zavisi od n) zna~i S(10,112)=S(10,57)

odnosno S(10,112)-S(10,57)+S(10,2)=S(10,2).
S(10,2) }e go presmetame od vtoriot slu~aj
10 10 10 10 10
S(10,2 ) = ∑  m2 − m + 1 = ∑  m2 − m + 1 − 1 = ∑ m2 − ∑ m + ∑ 1 − 1
m=2   m=1   m=1 m=1 m=1
10 ⋅ 11⋅ 21 10 ⋅ 11
= − + 10 − 1 . Baranata vrednost e 399.
6 2
n a−n +a
927. n=a. Upatstvo. Re{i ja ravenkata = .
n +1 a −n +a +1
928. Ako levata strana vo ravenstvoto ja pomno`ime i podelime so n!
n!⋅(n + 1) ⋅ (n + 2 ) ⋅ ... ⋅ (2n − 1) ⋅ 2n (2n )! (2 ⋅ 4 ⋅ ... ⋅ 2n ) ⋅ (1 ⋅ 3 ⋅ ... ⋅ (2n − 1))
dobivame = =
n! n! n!

238
Odgovori, upatstva i re[enija

2 ⋅n!⋅(1⋅3 ⋅...⋅(2n −1))


n
= 2 ⋅ (1⋅3 ⋅...⋅ (2n −1)) . 929. (n+1)!-1. Dadeniot zbir }e go
n
=
n!
zapi{eme vo oblikot 2·1!+3·2!+4·3!+…+n·(n-1)!+(n+1)·n!-(1!+2!+3!+…+(n-1)!+n!)
=2!+3!+4!+…+n!+(n+1)!-(1!+2!+3!+…+(n-1)!+n!)=(n+1)!-1.
k −1 1 1
930. Upatstvo: Koristi go ravenstvoto = − .
k! (k − 1)! k!
931. Od xn = xn+1 + 2xn-1 dobivame deka xn+1 - xn = 2(xn - xn-1) . Od ovde dobivame
3
( ) (
deka xn - xn-1 = 2 xn - 1 - xn - 2 = 2 ÿ 2(xn- 2 - xn-3 ) = ... = 2n- 2 x 2 - x . Spored toa va`at
1
)
ravenstvata:
x 2 - x1 = x 2 - x1 ...(1)
x3 - x 2 = 2(x 2 - x1) ...(2)
x 4 - x3 = 22 (x 2 - x1) ...(3)
...
...
...
x 2014 - x 2013 = 22012 (x 2 - x1) ...(2013).
Sobiraj}i gi poslednite 2013 ravenstva dobivame
( ) ( )
x 2014 - x 1 = (x 2 - x 1 ) 1 + 21 + .... + 2 2012 = (x 2 - x 1 ). 2 2013 - 1 . Bidej}i 22013 − 1 ≠ 0 i
x 2014 = x 1 sleduva deka x 2 = x1. Od (2) sleduva deka x 3 - x 2 = 0 odnosno
x 3 = x 2 = x1 i.t.n. Od 2013-te ravenstva sleduva deka
x 2014 = x 2013 = ... = x 2 = x1 .
2 ⋅ (2 − 1)
932. Prviot ~len od vtoroto mno`estvo se zapi{uva kako 2 ⋅ + 1.
2
3 ⋅ (3 − 1)
Prviot ~len od vtoroto mno`estvo se zapi{uva kako 2 ⋅ + 1 . Prviot
2
~len od n–toto mno`estvo se zapi{uva kako 2 n(n − 1) + 1 = n 2 − n + 1 . Zna~i
2
sekoe mno`estvo pretstavuva aritmeti~ka progresija so prv ~len
a1 = n 2 − n + 1 , razlika d=2 i op{t ~len a n = n 2 + n − 1 . Sleduva deka
a1 + a n 2n 2
Sn = ⋅n = ⋅ n = n3 .
2 2
933. Upatstvo: Mno`estvoto prirodni broevi da se razbie na podmno`estva
od parovi dvojki na sledniot na~in (31-j,j), jœ{1,2,…,15}.
936. ]e gi razgledame ostatocite na site broevi zapi{ani vo taplicata pri
delewe so 2020. Vo prvata redica pri simetrija vo odnos na sredniot ~len
2020
, ostatokot “k” pominuva vo ostatok “–k” na simetri~niot ~len, koj e na
2
isto rastojanie vo odnos na sredniot ~len. ]e ja razgledame simetrijata vo
odnos na pravata koja minuva niz srednite ~lenovi. Ako vo nekoja redica se
nao|at broevite а1, а2, ..., аn so zbir Ѕ toga{ vo slednata redica se nao|at

239
Odgovori, upatstva i re[enija
broevite a1+a2, a2+a3, …, an-1+an ~ii zbir e 2Ѕ-а1-аn. No poradi simetrijata
dobivame deka а1+аn =0, odnosno zbirot na sledniot red se udvojuva. Vo
2020
prviot red zbirot na ~lenovite (ostatocite) e vo vtoriot 2020 i t.n.
2
Zna~i, vo posledniot red se dobiva broj deliv so 2020.
937. Da pretpostavime deka megu broevite ak, kœ{1,2,...,n-1} ima i pozitivni
broevi. Neka ar e prviot od tie broevi odnosno ar-1<0, ar>0. Toga{ ar-ar-1>0. Od
uslovot na zada~ata va`i ak -1-2ak+ak+1¥0, odnosno ak+1-ak¥ak-ak-1 ...(1). Od (1)
imame deka ar+1-ar>0, ar+2-ar+1>0, ..., an–an-1>0 odnosno 0<ar<ar+1<ar+2<...<ak-1<an .
No toa protivre~i na uslovot an=0, bidej}i dobivme deka an>0. Zna~i, site ak,
kœ{1,2,...,n-1} se negativni.
938. Упатство: Primenete go Principot na matemati~ka indukcija.
939.Od uslovot na zada~ata imame
a1–a0>0, a2–a1=2(a1-a0), a3–a2=2(a2–a1), …, a100–a99=2(a99–a98), od kade sleduva
a 2 − a1 a 3 − a 2 a − a 99
deka = = ... = 100 = 2 . Mno`ej}i gi site 99 odnosi
a1 − a 0 a 2 − a1 a 99 − a 98
a 100 − a 99 99
dobivame deka = 2 99 odnosno а100=а99+(а1-а0)·2 . Od toa {to а99>0 i
a1 − a 0
99
а1–а0≥0 dobivame а100>2 . 942. 1.
943. Upatstvo: a) So koristewe na principot na matemati~ka indukcija;
b) So principot na matemati~ka indukcija }e doka`eme deka
Fm+n=Fm-1·Fn+Fm·Fn+1. Za n=1 imame Fm+1=Fm-1+Fm=Fm-1·1+Fm·1=Fm-1·F1+Fm·F1+1.
Za n=2 imame Fm+2=Fm+Fm+1=2Fm+Fm-1=Fm-1·1+Fm·2=Fm-1·F2+Fm·F2+1.
Neka tvrdeweto e to~no za n=k-1 i n=k, odnosno neka va`at
Fm+k-1=Fm-1·Fk-1+Fm·Fk i Fm+k=Fm-1·Fk+Fm·Fk+1. Toga{ za n=k+1 dobivame
Fm+k-1=Fm+k+Fm+k-1=Fm-1·Fk+Fm·Fk+1+Fm-1·Fk-1+Fm·Fk=Fm-1(Fk+Fk-1)+Fm(Fk+1+Fk)
=Fm-1·Fk+1+Fm·Fk+2. Toga{, F2n=Fn+n=Fn-1·Fn+Fn·Fn+1=Fn(Fn-1+Fn+1), odnosno Fn|F2n, pa
F2n
zatoa e cel broj. 944. Upatstvo: Dokazot sleduva so indukcija
Fn
1
123 08 - 120 33...3
k+1
120 33...3 123 08 =1083 ÿ 10 . 945. 335. 947. 6m-n6 = . 2
k - trojki k -1 trojki
2 2 2 2
948. Od uslovite vo zada~ata imame b =a +d i c =a +2d, ottuka sleduva
c 2 − a2 2 2
d = b2 − a2 = . U{te, od razlikata na dvete ravenstva dobivame d=c -b .
2
1 c −b c −b 1 c −a c −a 1 b−a b−a
Toga{, = 2 2
= , = 2 2
= i = 2 2
= .
b+c c −b d c+a c −a 2d a+b b −a d
1 c − a c − b 2b − c − a 1 b − a c − b 2b − c − a
Bidej}i = = + i = = + , va`i
c+a 2d d 2d a+b d d d
tvrdeweto. 949. Od Sn=Sm sleduva n(a1+an)=m(a1+am) odnosno
2 2
(m-n)a1+ma1+m(m-1)d-na1-n(n-1)d=0. Imame 2(m-n)a1+(m -m-n +n)d=0,
odnosno 2(m-n)a1+(m-n)(m+n-1)d=0. Bidej}i m∫n, dobivame 2a1+(m+n-1)d=0,
x
odnosno a1+am+n=0 a ottuka sleduva deka Sm+n=0. 950. Od Sn=2 sleduva
n x+1 x+1
(a1 + an ) = 2x , odnosno n(a1+a2)=2 . Toga{ n e delitel na 2 pa zatoa n e
2
stepen so osnova 2. 951. Ako m=n toga{ jasno e deka tvrdeweto e to~no. Neka

240
Odgovori, upatstva i re[enija

Sm m2 2a1 + (m − 1)d m
m∫n. Od = sleduva = , odnosno 2a1(n-m)=(n-m)d. Od m∫n
Sn n2 2a1 + (n − 1)d n
a a + (m − 1)d a1(1 + (m − 1) ⋅ 2) 2m − 1
dobivame d=2a1. Toga{ m = 1 = = .
an a1 + (n − 1)d a1(1 + (n − 1) ⋅ 2) 2n − 1
1 1 1
952. Upatstvo: Iskoristi deka = − .
ak ⋅ ak +1 d ⋅ ak d ⋅ ak +1
1 ak − ak +1
953. Upatstvo: Iskoristi deka = . 955. x=log25.
ak + ak +1 d
956. Od uslovite vo zada~ata sleduva tgβ-tgα=tgγ-tgα, odnosno
sin(β − α) sin(γ − β)
= i ottuka dobivame cosγ·sin(β-α)=cosα·sin(γ-β). Od
cosα ⋅ cosβ cosβ ⋅ cosγ
poslednoto ravenstvo sleduva sin(γ+β-α)-sin(γ-β+α)=sin(α+γ-β)-sin(α-γ+β).
Koristej}i deka α, β i γ se agli vo triagolnik dobivame
sin(180±-2α)-sin(180±-2β)=sin(180±-2β)-sin(180±-2γ), odnosno sin2α, sin2β i
sin2γ se posledovatelni ~lenovi na aritmeti~ka progresija.
BO 2
958. Treba da doka`eme deka (spored talesova teorema za
=
OP 1
proporcionalni otse~ki). Od toa {to BO e simetrala na agolot β imame
CP CB a BO a
= = . Od toa {to CO e simetrala na agolot γ imame = .
AP BA c OP CP
BO c
Analogno, od toa {to AO e simetrala na agolot α imame = . Od
OP AP
a c a+c a+c
poslednite dve ravenstva se dobiva = = = . No, od
CP AP CP + AP b
BP a 2b
uslovot na zada~ata imame a+c=2b pri {to sleduva = 2. = =
OP CP b
959. a8=348 .Od uslovot na zada~ata imame deka (A-D)(a-d)=1440, Aÿa=1716,
(A+D)(a+d)=1848. 2Dd=(A-D)(a-d)+(A+D)(a-d)-2Aÿa=1440+1848-2ÿ1716=-144.
Taka dobivame deka Aÿd+D·a=204. Zna~i, osmiot ~len e
(A+6D)(a+6d)=Aÿa+36Dÿd+6(Aÿd+Dÿa)=1716+36(-72)+6ÿ204=348.
2
960. Od svojstvata za geometriska progresija imame deka lgmÿlgp=(lgn) . No, od
2
log a x lgx lgx  lgx 
toa {to = log a b dobivame deka ⋅ =   , od kade
log b x logm x logp x  logn x 
2
sleduva logmxÿlogpx=(lognx) , {to treba{e da se doka`e.
1008
962. a1·a2·…·a2017=2 · 2 . Od prvoto ravenstvo dobivame deka S2017=2
2017
q −1
odnosno a1 =2 ...(1). Delej}i ja vtorata ravenka so q dobivame deka
q −1

241
Odgovori, upatstva i re[enija

1 1 1 1 1 1 1 1
+ + ... + = 1 odnosno + + ... + + = od kade
a1 ⋅ q a2 ⋅ q a 2017 ⋅ q a2 a a 2017 q⋅a q
3 2017
2017 2017
se dobiva ravenstvoto a1·q ·(q-1)=q(q -1) …(2). Mno`ej}i gi ravenstvata
2 2 1008 2
(1) i (2) go dobivame ravenstvoto a1 = odnosno (a1·q ) =2 od kade se
q2016
dobiva deka a1009= 2 . Od svojstvoto na geometriska progresija da
2
a1·an=a2·an-1=…=ak·an-(k-1) dobivame deka a1·a2017=a2·a2016=…=a1008·a1010=a1009 =2.
1008
Zna~i, a1·a2·…·a2017=(a1·a2017)·(a2·a2016)·…·(a1008·a1010)·a1009=2 · 2 .
963. Od |a1-2a2|=|a2-2a3|=...=|a2018-2a2019|=|a2019-2a1|=k sleduva
a1-2a2=≤k
a2-2a3=≤k

a2018-2a2019=≤k
a2019-2a1=≤k.
Sobiraj}i gi poslednite ravenstva dobivame
(a1+a2+...+a2019)-2(a2+...+a2019+ a1)= ±
1k4 k2
±4 ...4
±4 ±3k . Sleduva 0=k·(≤1≤1≤…≤1)
2019
2018
odnosno k=0. Ottuka imame a1=2a2=4a3=…=2 a2019. Od a1+a2+...+a2019=0
2018 2017
sleduva a2019·(2 +2 +…+1)=0 odnosno a2019=0. Zna~i, a1=a2=...=a2019=0.
6 2
964. a+b+c=111. log6a+log6b+log6c =log6abc=6, odnosno abc=6 . Bidej}i aÿc=b
3 6 2
imame deka b =6 , b=6 , b=36. Od toa {to b-a e poln kvadrat sleduva deka
2 4
aœ{11,20,27,32,35}. Bidej}i a|b odnosno a|6 sleduva deka a=27. Od ovde
imame deka c=48, pa a+b+c=27+36+48=111.
3(9 + 65) 3(9 − 65) 3(9 − 65) 3(9 + 65)
966. a) 3,6,12; 12,6,3; ,-6, i , -6, .
2 2 2 2
b)15,45,135 i 125,-175,245.
968. Bez gubewe na op{tosta, da pretpostavime deka triagolnicite ABO i
ABC se sli~ni. Toga{ nivnite agli se ednakvi. Aglite na triagolnikot ABO se
α β γ (α, β i γ se aglite na triagolnikot ABC). Jasno, α
, i 90o + ≠ α . Neka
2 2 2 2
α β β α
= β . Toga{ ≠ α, β pa = γ . Zna~i β = 2 γ, α = 2β =22 γ . Sli~no, ako = γ.
2 2 2 2
969. Neka a1, a2, a3, ..., an, an+1 se ~lenovi na aritmeti~ka progresija (a1≠a2), a
b1, b2, b3, ..., bn, bn+1 ~lenovi na geometriskata progresija pri {to a1=b1 i
a2=b2. Bez razlika dali geometriskata progresija e opa|a~ka ili raste~ka
najgolemiot i najmaliot ~len se b1 i bn+1. Od neravenstvoto na Evklid imame:
bn+1+b1>b2+bn, od kade se dobiva b3>b2+(b2-b1), b4>b3+(b2-b1), ..., bn+1>bn+(b2-b1).
Sobiraj}i gi neravenstvata dobivame bn+1>b2+(n-1)(b2-b1). Osven toa
b2+(n-1)(b2-b1)=a2+(n-1)(a2-a1)=a1+n·d=an+1. Zna~i, bn+1>an+1 {to treba{e da se
doka`e.
2 b c b c
970. Od b =ac sleduva = . Toga{ log x
= log x odnosno
a b a a
1 1 1
logxb − logxa = logxc − logxb . Zna~i, logxa = , logxb = i logxc =
loga x logb x logc x
obrazuvaat aritmeti~ka progresija.

242
Odgovori, upatstva i re[enija

4 52 676
971. 4, 20, 100 ili , , . 973. 1. 974. 1.
9 9 9
975. Da zabele`ime deka ako nizata x, y, z obrazuva harmoniska progresija
1 1 1
toga{ nizata , , obrazuva aritmeti~ka progresija. Od toa {to nizata
x y z
tgα, tgβ, tgγ e harmoniska progresija sleduva deka postoi dœR za koj
1 1 1 1
= − d ...(1) i = + d ...(2). Neka to~kite A1, B1 i C1 se podno`nite
tgα tgβ tgγ tgβ
to~ki na visinite spu{teni od to~kite A, B i C, soodvetno. Od definicija za
1 AB1 AC1
trigonometriskite funkcii gi dobivame ravenstvata = = ...(3),
tgα BB CC1
1

1 BC1 BA 1 1 CA 1 CB1
= = ...(4) i = = ...(5). Od ravenstvata (3) i (4); (4) i
tgβ CC AA tgγ AA BB
1 1 1 1

1 1 AC1 + BC1 c
(5); (3) i (5) posledovatelno dobivame + = = ...(6) ,
tgα tgβ CC1 CC1

1 1 BA 1 + CA 1 a 1 1 CB1 + AB1 b
+ = = ...(7) , + = = ...(8) .
tgβ tgγ AA 1 AA 1 tgα tgγ BB1 BB1
Od ravenstvata (1) i (6); (2) i (7); (1), (2) i (8) posledovatelno imame
2 c 2 a 2 b 2
= + d ...(9) , = − d ...(10) , = ...(11) . Neka k = .
tgβ CC tgβ AA tgβ BB tgβ
1 1 1
2 2 2
Od ravenstvata (9), (10), (11) sleduvat ravenstvata c =(k-d) · CC1 ...(12),
2 2 2 2 2 2
a =(k+d) · AA 1 ...(13), b =k · BB 1 ...(14). Od formulite za plo{tina na

2 4P 2 2 4P 2
triagolnik gi imame ravenstvata AA 1 = ...(15), BB 1 = ...(16),
a2 b2
2 4P 2
CC1 = ...(17). Kombiniraj}i gi ravenstvata (12) so (17), (13) so (15) i (14)
c2
2 2
so (16), gi dobivame ravenstvata c =2P·(k-d)=2k·P-2d·P; b =2k·P;
2 2 2 2
a =2P·(k+d)=2k·P+2d·P od kade se gleda deka nizata a , b , c obrazuva
416π 9π 2
aritmeti~ka progresija. 976. V = . 977. . 978. S = .
189 13 3

243
Odgovori, upatstva i re[enija

Tema 19: Elementi od kombinatorika i verojatnost


979. 24 na~ini. Bidej}i vlezot e mo`en preku 6 uli~ki a izlezot preku 4,
imame deka pominuvaweto preku plo{tadot e mo`no na 6x4=24 na~ini.
980. 7920 na~ini. 981. 2 =64 na~ini. 982. m+m
6 n+p
( )( )
ÿ n+np . 983. 26.
984. a)5050; b)171700. Neka A e brojot na kocki vo poslednoto nivo, a B e
vkupniot broj kocki vo piramidata. Gi koristime poznatite sumi:
n ⋅ (n + 1) 2 n ⋅ (n + 1) ⋅ (2n + 1)
1 + 2 + ... + n = ; 1 + 22 + ... + n2 = .
2 6
100 ⋅ 101
Toga{ A = 1 + 2 + ... + 100 = = 5050 i B = 1 + (1 + 2) + (1 + 2 + 3) + ... + (1 + 2 + ... + 100)
2
1⋅ 2 + 2 ⋅ 3 + ... + 100 ⋅ 101 1⋅ (1 + 1) + 2 ⋅ (2 + 1) + ... + 100 ⋅ (100 + 1)
= =
2 2
12 + 2 2 + ... + 100 2 1 + 2 + ... + 100 100 ⋅ 101 ⋅ 201 5050
= + = + = 171700.
2 2 12 2
986. Krugot, vo koj k-tata ramnina ja se~e sferata, se presekuva so sekoja od
ostanatite ramnini vo dve to~ki i sledstveno se deli na 2(k-1) delovi. Zna~i
F(k) (brojot na delovi na sferata pri delewe so k ramnini) e za 2(k-1) pove}e
od F(k-1), odnosno F(k)=F(k-1)+2(k-1). Bidej}i F(1)=2, sleduva deka
F(n)=2·(1+1+2+3+...+(n-1))=2+n(n-1).
987. Sekoe dvi`ewe od A do C sodr`i 4
dvi`ewa so dol`ina 1 nagore (g) i 7 dvi`ewa
so dol`ina 1 nadesno (d). Zna~i, sekoj izbran
pat od A do C pretstavuva permutacija so
povtoruvawe od 4+7=11 elementi ((g) se
povtoruva 4 pati, (d) se povtoruva 7 pati). Na
crte`ot vo tekstot na zada~ata (vidi crt. 59) e
pretstavena permutacijata ddgdddggddg. Zna~i
baranit broj e P4,7(11)=330. Crte` 220
2
988. Da. Proizvolen triagolnik mo`e da se podeli na n sli~ni triagolnici,
2
1+3+5+…+(2n-1)=n (crt. 220). So spojuvawe na 4 ili 9 takvi triagolnici se
dobiva triagolnik sli~en so dadeniot, a vkupniot broj triagolnici se
2
namaluva za 3 ili 8. Od 45 -1ÿ8-1·3=2025-11=2014, zaklu~uvame deka e mo`no
proizvolen triagolnik ABC da se podeli na 2014 sli~ni triagolnici.
989. 216. Neka Na e vkupniot broj triagolnici kaj koi {to edna od stranite e
paralelna so stranata a. Analogno gi definirame Nb, Nc, Nab, Nac, Nbc i Nabc.
Toga{ N=7 (N-vkupniot broj triagolnici), Na=Nb=Nc=7 , Nab=Nac=Nbc=7 i
3 2

3 2 3
Nabc=1. Baraniot broj e N-(Na+Nb+Nc)+(Nab+Nac+Nbc)-Nabc=7 -3≅7 +3≅7-1=6 .
990. 48. Mravkata mo`e da dojde do temeto B pravej}i 3 ili 5 ~ekori. Patot so
dol`ina 3 }e go ozna~ime so XYZ. Postojat 3!=6 takvi pati{ta. Patot so
dol`ina 5 }e go ozna~ime so XYZXX, XYZYY ili XYZZZ. Postojat 3ÿ
5!
= 60 takvi pati{ta. No, vo ovoj slu~aj imame povtoruvawe na
3!⋅1!⋅1!
mo`nostite da se stigne so tri “~ekori”. Brojot na povtoruvawa e 3ÿ3!=18 i
kone~no, vkupniot broj e 60+6-18=48. 991. 13 lu|e. 992. 6. 993. 330. 994. 41.

244
Odgovori, upatstva i re[enija

re{enie: Da zabele`ime deka za kœ{0,1,…,n} va`i k ⋅ n


995. I re{enie:  n-1 
k = n ⋅  k -1  . ( )
Toga{ 1 + 2 2 + 3 3 + ... + n = n ⋅  n0- 1  +  n1- 1  +  n 2- 1  + ... +  n
( ) ( ) ( ) ( ) - 1  =n·2n-1.
n n n n

       n - 1 
II re{enie:
re{enie: Neka
n
() ( ) ( )
n n
S= 1 + 2 2 + 3 3 + ... + n .
n
() Jasno e deka
( ) () ( )
n n n n
0 + 2 1 + 3 2 + ... + (n + 1) n
() n
=S+2 , odnosno n
( ) ( ) ( )
n n
n + 2 n-1 + 3 n-2 + ... + (n + 1) 0 ( )
n
n
=S+2 …(1). So sobirawe na poslednite dve ravenstva dobivame
n n
(n+2)ÿ2 =2(S+2 ) od kade {to sleduva baraniot identitet.
re{enie: Diferenciraj}i go izrazot (1 + x)n = n
III re{enie: () ()
n n ()2 n n
0 + 1 ⋅ x + 2 ⋅ x + ... + n ⋅ x , ()
dobivame 1 () ()
2 3 () n ()
n + 2 n ⋅ x + 3 n ⋅ x 2 + ... + n ⋅ n ⋅ xn − 1 = n ⋅ (1 + x)n - 1 . Za x=1 se dobiva

baranoto ravenstvo. 998. a) -2 b) 2 999. C2n .


50 49 n

1000. 281250. Brojot na takvi {estcifreni broevi {to po~nuvaat so parna


cifra e 4·  5  ·5 5 =40·5 (brojot po~nuva na 2, 4, 6 ili 8, drugite parni cifri
2· 3 5
2
mo`e da se nao|aat na  5  pozicii), dodeka tie {to po~nuvaat so neparna
2
cifra gi ima vkupno 5·  5  ·5 =50·5 . Zna~i vkupno ima 90·5 =281250 takvi
5 5 5
2
broevi.
1001. Ako poslednata cifra e 1 toga{ ima C 38 takvi broevi. Ako poslednata
cifra e 3 toga{ ima C 36 takvi broevi, i ako poslednata cifra e 5 toga{ ima
C 34 takvi broevi. Vkupniot broj takvi broevi e C 38 + C 36 + C 34 =80.
1002. 357. 1003. 66660.
1004. 50 delovi. So povlekuvawe na nova dijagonala, brojot na delovi
formirani od prethodno povle~enite dijagonali se zgolemuva za m+1, kade
{to m e brojot na to~ki vo koi novata dijagonala gi se~e prethodno
povle~enite. Op{to, brojot na delovi na koi se deli n-agolnik so svoite
dijagonali e D+P+1 , kade {to D e brojot na site dijagonali, a P e brojot na
nivnite prese~ni to~ki. Edna to~ka se dobiva kako presek na dve dijagonali,
n ⋅ (n − 3 )
odnosno od 4 temiwa na mnoguagolnikot. Zna~i, D= ,
2
n ⋅ (n − 1) ⋅ (n − 2) ⋅ (n − 3 )
P = Cn4 = i vkupniot broj na delovi e
24
n ⋅ (n − 1) ⋅ (n − 2 ) ⋅ (n − 3 ) n ⋅ (n − 3 )
+ + 1 . 1005. 102. Eden od broevite mora da
24 2
bide 5. Toga{ drugite dva broja mora da imaat proizvod deliv so 4. Postojat
dve mo`nosti, dvata da se parni broevi ili edniot da e deliv so 4 a drugiot
e neparen broj. Vo prviot slu~aj postojat 48 =3x4x4 mo`nosti i toa 3 pozicii
za brojot 5 i dve pozicii za sekoj od ~etirite parni broja.Vo vtoriot slu~aj
ima 54=3x2x9 mo`nosti i toa 3 pozicii i dva izbora za broj deliv so 4 (4 i 8)

245
Odgovori, upatstva i re[enija
i 9 na~ini za popolnuvawe na drugite dva broja so 5 i eden neparen broj.
n
7 ( )
1006. n=11. Upatstvo: kn = . 1007. x=7, y=3.
k -1
5( )
1009.Upatstvo:Kvadratot da se razdeli na 25 kvadrati so strana 0,2 .
1010. Neka A e edna od to~kite i k1 e kru`nica so centar vo A i radius 1. Ako
site ostanati to~ki se vo vnatre{nosta na k1 toga{ tvrdeweto va`i. Neka
to~kata B e nadvor od k1, odnosno AB > 1 . Neka k2 e kru`nica so centar vo B i
radius 1. Neka C e proizvolna to~ka, razli~na od A i B. Od AB > 1 sleduva
deka AC < 1 ili BC < 1 , odnosno C e vo vnatre{nosta na k1 ili C e vo
vnatre{nosta na k2. Zna~i sekoja od 23-te to~ki, razli~na od A i B, se nao|a
ili vo vnatre{nosta na k1 ili vo vnatre{nosta na k2. Spored principot na
Dirihle, vo k1 ili vo k2 ima najmalku 12 to~ki. Ako tie se vo k1, toga{ zaedno
so A pravat najmalku 13 to~ki. Analogno za k2 i B. So toa tvrdeweto e
poka`ano.
1013. Pome|u posledovatelni 20 broevi ima dva koi zavr{uvaat na 0. Pri toa
barem pred eden od niv se nao|a cifra razli~na od 9. Neka toj broj e N, a n e
zbirot na negovite cifri. Toga{ broevite N, N+1, N+2, …, N+9 i N+19 imaat
zbir na cifri n, n+1, n+2, …, n+9, n+10, odnosno edinaeset posledovatelni
broja. Jasno e deka barem eden od niv e deliv so 11.
1014. Neka broevite se a1, a2, ..., a8. Bez gubewe na op{tosta mo`eme da
pretpostavime deka 1§a1<a2<...<a8<16. Vkupniot broj pozitivni razliki {to gi
davaat ovie broevi e 8
()
2 = 28 , dodeka od 1§ai<aj<16 sleduva deka
1§ai-aj§14, odnosno ai-aj mo`e da primi najmnogu 14 razli~ni vrednosti.
Kone~no, bidej}i 28 razliki primaat vrednosti od mno`estvo so 14 elementi
i bidej}i nemo`e da se slu~i dve razliki da se 14 (ai-aj=14 ako i samo ako
ai=15, aj=1), od principot na Dirihle sleduva deka najmalku 3 razliki se
ednakvi me|usebe.
1015. Me|u 50 broja postoi broj koj zavr{uva na 0, a cifrata pred nulata ne e
6, 7, 8, 9. Neka e toa brojot N, a neka n e sumata na negovite cifri. Toga{
broevite N, N+1, N+2, …, N+49 gi imaat kako zbir na svoite cifri broevite
n, n+1, n+2, …, n+13 koi se 14 posledovatelni broevi. Jasno e deka barem
eden od tie broevi e deliv so 14.
1016. Neka se toa broevite a1, a2, ..., a11. Pome|u 11 prirodni broevi postojat
pet para ~ij zbir pri delewe so 6 dava paren ostatok, odnosno toa se
ostatocite 0, 2 ili 4. Bez gubewe na op{tosta neka se toa parovite (a1,a2),
(a3,a4), (a5,a6), (a7,a8), (a9,a10). Ako barem tri para imaat ednakvi ostatoci pri
delewe so 6 toga{ zbirot na tie parovi e baranoto re{enie. Vo sprotivno
postojat tri para koi imaat ostatoci 0, 2 i 4, pa zbirot na tie 3 para e isto
2
taka deliv so 6 , {to treba{e da se doka`e. 1017. 2n (5n-3)
1018. Da gi ozna~ime figurite so A1, A2, ..., A100, a nivnite plo{tini so
P1, P2, ..., P100. Od uslovot na zada~ata imame deka P1+P2+...+P100>99. So Bi }e
ja ozna~ime figurata koja ja dopolnuva Ai do kvadrat. Toga{ plo{tinata na Bi
}e bide 1-Pi (i = 1,2,...,100). Zna~i, zbirot od plo{tinite na figurite
B1, B2, ..., B100 }e bide (1 - Ρ 1 ) + (1 - Ρ 2 ) + ... + (1 - Ρ 100 ) = 100 - (Ρ 1 + Ρ 2 + ... + Ρ 100 ) ,
{to e pomalo od 1, odnosno od plo{tinata na kvadratot. Toa zna~i deka vo

246
Odgovori, upatstva i re[enija
kvadratot postoi to~ka koja {to ne pripa|a na nitu edna od figurite
B1, B2, ..., B100 , {to zna~i deka taa to~ka pripa|a na site figuri A1, A2, ..., A100.
n ⋅ (n + 1)
1019. + 1 . Upatstvo: Iskoristi ja rekurentnata vrska f(n)=f(n-1)+n.
2
1020. Neka A1, A2, ..., An se levite, a B1, B2, ..., Bn se desnite kraevi na dadenite
otse~ki. Toga{ proizvolna to~ka Ai (i=1,2,...,n) }e leæi na levo vo odnos na
proizvolna to~ka B j (j=1,2,...,n) ili }e se sovpadne so nea. Navistina, ako
pretpostavime deka to~kata Ai se nao|a desno od to~kata Bj,toga{ otse~kite
A i B i i A jB j nemaat zaedni~ka to~ka. Me|u to~kite Ai ja izbirame najdesnata
to~ka, neka e toa Ak, a pome|u to~kite B j ja izbirame najlevata to~ka, neka e
toa Bm. Toga{ otse~kata A k B m se sodræi vo site dadeni otse~ki A i B i
(i=1,2,...,n). Jasno e deka, dokolku to~kite A k i Bm se sovpa|aat, toga{
otse~kata A k B m pominuva vo to~ka.
1021. Neka S-e plo{tina na masata, Si-plo{tina na i-tiot prekriva~
(i=1,2,3),Si,j-plo{tina na sovpa|awe na i-tiot i j-tiot prekriva~ (1≤i<j≤3) i
S1,2,3-plo{tina na sovpa|awe na site prekriva~i. Od formulata za
vklu~uvawe i isklu~uvawe imame deka S-(S1+S2+S3)+(S1,2+S1,3+S2,3)-S1,2,3≥0 od
kade se dobiva deka S1,2+S1,3+S2,3≥S1+S2+S3-S=3. Od poslednoto neravenstvo
2
sleduva deka barem edna od plo{tinite S1,2, S1,3 i S2,3 ne e pomala od 1m .
1023. Neka nizata e a1, a2, ..., a29, a30. Da pretpostavime deka zbirot na sekoi
tri posledovatelni ~lenovi na nizata e pomal od 2350. Toa povlekuva deka
zbirot na sekoi tri posledovatelni ~lenovi e pomal ili ednakov na 2300
(Zo{to?) odnosno
a 1 + a 2 + a 3 ≤ 2300
a 2 + a3 + a 4 ≤ 2300
...
.
...
...
a 30 + a 1 + a 2 ≤ 2300
So sobirawe na site neravenstva se dobiva a1+a2+...+a30§23000. No od druga
strana a1+a2+...+a30=50+100+150+...+1500=23250. Dobivame kontradikcija,
odnosno po~etnata pretpostavka ne e dobra. Zna~i postojat 3 posledovatelni
~lenovi vo nizata ~ij zbir e pogolem ili ednakov na 2350.
1024. Da. So matemati~ka indukcija }e poka`eme deka za sekoj n=1+3k, k¥0,
kvadratot mo`e da se podeli na n pomali kvadrati (crt. 221).
Za k=0 tvrdeweto e to~no.
Da pretpostavime deka za nekoe n=3k+1, k¥0, tvrdeweto va`i.
Ako eden od kvadratite se podeli na 4 ednakvi kvadrati toga{
vkupniot broj kvadrati se zgolemuva za 3 , pa imame deka
n=1+3k+3=1+3(k+1), {to zna~i deka tvrdeweto e to~no i za k+1.
Od 2014=1+3ÿ671 sleduva pozitivniot odgovor. Crte` 221

1025. Da. Ako kockata mo`e da se podeli na x pomali kocki, toga{ mo`e da se
podeli i na x+7 pomali kocki (edna od tie x-kocki mo`e da se podeli na 8

247
Odgovori, upatstva i re[enija
ednakvi kocki). Povtoruvaj}i ja postapkata kone~en broj pati, mo`eme da ja
podelime i na x+kÿ7 kocki. Od 2012=38+282·7 i od faktot {to kocka so rab a
3a a
mo`e da se podeli na edna kocka so rab i 37 kocki so rab , sleduva
4 4
pozitivniot odgovor. 1026. 12. Od k ⋅ n ( )  n-1 
k = n ⋅  k -1  , za k=1,2,…,n, sleduva deka
  +  2016  + ... +  2016  = 2017 ⋅ 2 2016 . Od 26ª14(mod 25),
S = 2017 2016
0   1   2016 
     
10 2000 2016
2 ª-1(mod 25) i 2 ª1(mod 25), sleduva deka 2 ª-14ª11(mod 25). Od
2016
2017ª17(mod25) i 2 ª11(mod 25) sleduva deka Sª187ª12(mod 25). Zna~i
ostatokot pri delewe na S so 25 e 12.
1027. Nau~nicite }e gi gledame kako temiwa na graf. Sekoi dve temiwa se
povrzani so rebro koe e oboeno so edna od boite crvena, `olta i sina, vo
zavisnost od temata na koja eden par nau~nici me|usebno se dopi{uva.
Treba da go doka`eme slednovo: Postojat tri temiwa koi me|usebno se
povrzani so rebra so ista boja. Izbirame proizvolno teme A. Od nego
izleguvaat 16 rebra, sekoe od niv e oboeno so edna od trite boi. Spored
principot na Dirihle, postojat 6 rebra so ista boja, da re~eme crvena. Da gi
razgledame krajnite to~ki na ovie 6 rebra (A ja smetame za po~etna). Postojat
2 mo`nosti:
1) Postojat dve to~ki (od 6-te krajni) koi se povrzani so rebro so crvena
boja. Toga{ tie, zaedno so A, formiraat crven triagolnik(site strani mu se
crveni).
2)Ne postojat dve to~ki povrzani so crveno rebro. Toga{ sekoi dve to~ki se
povrzani so rebro so `olta ili sina boja. Neka B e edno od krajnite temiwa.
Od nego izleguvaat 5 rebra kon drugite krajni to~ki koi se oboeni so `olta
ili sina boja. Spored principot na Dirihle me|u tie 5 rebra postojat 3 rebra
so ista boja, da re~eme sina. Da gi razgledame sega krajnite to~ki na ovie 3
rebra. Ako me|u tie 3 to~ki postojat 2 povrzani so sino rebro, toga{ tie,
zaedno so temeto B, formiraat sin triagolnik, vo sprotivno formiraat `olt
triagolnik. Zna~i vo sekoj slu~aj postoi triagolnik ~ii strani se oboeni so
ista boja.
1030. Induktivno }e poka`eme deka postojat n+1 lu|e koi me|usebno se
poznavaat. O~igledno postojat dvajca vo kompanijata {to se poznavaat. Neka
k§n. Ako postojat k lu|e koi me|usebno se poznavaat, toga{ od uslovot na
zada~ata postoi ~ovek koj se poznava so tie k lu|e. Ottuka sleduva deka
postojat n+1 lu|e koi me|usebno se poznavaat, da gi ozna~ime so A1, A2, ..., An+1.
Za preostanatite n lu|e, od uslovot na zada~ata, postoi nekoj od
A1, A2, ..., An+1 koj gi poznava site niv, neka toa e Ak. Toga{ sleduva deka Ak se
poznava so site lu|e.
1033. Da pretpostavime deka koi bilo dve to~ki koi se na rastojanie 1 se
oboeni so razli~ni boi. Da go razgledame ramnostraniot triagolnik ABC so
strana 1, za koj site temiwa se so razli~ni boi. Neka A1 e simetri~na to~ka na
A vo odnos na pravata BC. Toga{ A1 e oboena so razli~na boja vo odnos na
to~kite B i C, zna~i so ista boja kako i to~kata A. Jasno AA 1 = 3 .

Konstruirame kru`nica so centar vo A i radius 3 . Site to~ki od taa


kru`nica }e bidat oboeni so ista boja. Jasno e deka na taa kru`nica postojat
dve to~ki koi se na rastojanie, {to protivre~i na pretpostavkata.

248
Odgovori, upatstva i re[enija
2
1034. Formirame 4x82 pravoagolnici vo ramninata, taka {to (x,y)œZ i 0§x§3,
0§y§81. Vo sekoja redica so 4 to~ki i 3 mo`ni boi (za sekoja to~ka), postojat
4
3 =81 razli~ni boewa. Spored principot na Dirihle od 82 redici postojat
dve redici so ist raspored na boite na ~etirite
to~ki. Isto taka po kolona postojat ~etiri to~ki
(vo eden red) i tri boi, {to zna~i ista boja se
javuva dvapati. Vo sekoja od dvete redici postoi
korespondencija po dve to~ki, so boja koja se
javuva dvapati. Tie formiraat pravoagolnik ~ii
temiwa se so ista boja .
1035. ]e gi ozna~ime boite so X i Y. Da
pretpostavime deka ne postoi ramnostran
triagolnik so istobojni temiwa. Neka ABC e
proizvolen ramnostran triagolnik. Crte` 222
Poradi pretpostavkata, bez gubewe od op{tosta, mo`eme da pretpostavime
deka A i B se oboeni so X, a C e so Y (ozna~uvame A=X, B=Y, C=Y). Sekoja
strana na triagolnikot ABC so dve to~ki ja razdeluvame na 3 ednakvi delovi
(crt. 222).

I. Ako B2=Y toga{ (poradi pretpostavkata) A2=X, C1=Y, A1=X, C2=Y.


Bidej}i triagolnikot OC1C2 e ramnostran, sleduva deka O=X, no
toga{ OA1A2 e ramnostran triagolnik ~ii temiwa se oboeni so
ista boja (X). Zna~i B2∫Y.
II. Ako B2=X toga{ C2=Y. Imame dve mo`nosti:
1) O=X. Toga{ B1=Y i A2=Y. No toga{ triagolnikot A2B1C2 e
ramnostran so istobojni temiwa (Y).
2) O=Y. Toga{ C1=X i A1=X, no vo toj slu~aj triagolnikot A1B2C1 e
ramnostran so istobojni temiwa. Zna~i B2∫X.
Od prethodnoto sleduva deka po~etnata pretpostavka ne e to~na, odnosno
postoi ramnostran triagolnik so istobojni temiwa.
1036. To~kite so celobrojni kordinati gi imenuvame so jazli, a boite gi
ozna~uvame so 1, 2 i 3. Gi razgleduvame slednite slu~ai:
Ako na sekoja vertikalna prava jazlite se oboeni so ista boja,
izbirame proizvolen jazol so taa boja (da pretpostavime so boja 1). Niz
jazolot povlekuvame dve normalni pravi pod agol 45° vo odnos na
vertikalata i izbirame na tie pravi to~ki oboeni so boja 2 i boja 3.
Dobieniot triagolnik gi ispolnuva uslovite od zada~ata. Analogno se
postapuva ako site jazli od edna horizontala se oboeni so ista boja.
Neka postoi vertikala V na koja postojat jazli oboeni so to~no dve boi
(neka se toa boite 1 i 2). Toga{ izbirame proizvolen jazol C so boja 3, koj e na
ista horizontala so jazol A od V so boja 1 i zaedno so jazolot B so boja 2 od V
go formiraat baraniot triagolnik. Ako postoi vertikala V na koja site 3 boi
se zastapeni, toga{ izbirame horizontala h na koja ne se site to~ki so ista
boja. Neka to~kata na presek e A so boja 1. Toga{ izbirame na h to~ka B so boja
2 (ili 3) a na V to~ka C so boja 3 (ili 2). 1037. 4(3n − 4 ) .
1038. 186. Od uslovot na zada~ata sleduva deka neparen broj i paren broj ne
smeat da bidat oboeni so ista boja. Nie mo`eme da izbereme edna od trite
boi za neparnite broevi, a drugite dve boi da gi koristime za ostanatite 5
5
parni broevi. Toa mo`e da se napravi na 3ÿ2 =96 na~ini. Isto taka mo`eme da

249
Odgovori, upatstva i re[enija
izbereme edna boja za parnite broevi, a drugite dve boi da gi koristeme za
ostanatite 5 neparni broja, ~ie boewe e mo`no povtorno na 96 na~ina. No, vo
dvata slu~ai imame isto obojuvawe (duplo broewe) vo 3ÿ2=6 slu~ai, koga site
broevi so ista parnost se so ista boja. Zna~i vkupniot broj na na~ini na
obojuvawe e 2ÿ96-6=186.

1039. 600 (temiwa na triagolnikot mo`at da bidat i


prese~nite to~ki na tetivite). Da zememe m to~ki na
edniot lak i n to~ki na drugiot lak.
0
1 .) Ako trite to~ki (temiwa na triagolnikot) le`at na
kru`nicata, toga{ dve to~ki od ist lak se povrzani, {to ne
mo`e poradi uslovot na zada~ata.
0
2 .) Ako dve to~ki le`at na kru`nicata toga{ tretoto teme
e vo nejzinata vnatre{nost (crt. 223). Crte` 223
Prodol`uvaj}i gi stranite dobivame 4 temiwa, po dve na sekoj lak.Tie ~etiri
to~ki formiraat dva triagolnika. Zna~i sekoj par to~ki od edniot lak so
sekoj par to~ki od drugiot lak formiraat po dva triagolnika. Vo ovoj slu~aj
imame vkupno 2 m ( )( )
n
2 ⋅ 2
triagolnici.
0
3 .) Neka edno teme na triagolnikot le`i na eden od dvata laka i dve temiwa
se vo vnatre{nosta na kru`nicata k (crt. 224).
Prodol`uvaj}i gi nivnite strani dobivame u{te edna to~ka na
istiot lak i tri to~ki na drugiot lak. Sekoj par od edniot lak
so sekoja trojka od drugiot lak, i obratno, formiraat po dva
triagolnika.
Ottuka, brojot na triagolnici so edno teme na kru`nicata se
( )( ) ( )( )
vkupno 2 ⋅ m ⋅ n + 2 ⋅ m ⋅ n . Vkupniot broj na triagolnici e
2 3 3 2
[( ) ( ) ( ) ( ) ( ) ( )]
m n m n m n
T(m, n ) = 2 ⋅ 2 ⋅ 2 + 2 ⋅ 3 + 3 ⋅ 2 . Konkretno, za m=n=5,

[( ) ( ) ( ) ( ) ( ) ( )]
5 5 5 5 5 5
se dobiva T (5,5 ) = 2 ⋅ 2 ⋅ 2 + 2 ⋅ 3 + 3 ⋅ 2 = 600. Crte` 224
1040. 204. Sekoj kvadrat so dimenzii 1x1 e opredelen na 8ÿ8 na~ini.Sekoj
kvadrat so dimenzii 2x2 e opredelen na 7ÿ7 na~ini. Sekoj kvadrat so dimenzii
2 2 2 2 2
8x8 e opredelen na 1ÿ1 na~in. Zna~i baraniot broj e x = 1 + 2 + 3 ...7 + 8 .
Od ravenstvoto 12 + 22 + 32 + ... + n2 = n(n + 1) ⋅ (2n + 1) sleduva deka x=204. 1041. 768.
6
1042. 576. Bidej}i figurite na crni poliwa vo sosedni koloni nemo`at da se
nao|aat vo ist red, dovolno e da gi razgledame oddelno smestuvawata na
figurite na crnite poliwa vo parnite i vo neparnite
koloni. I vo dvata slu~aja brojot na mo`nosti e 4!, od kade
{to sleduva deka vkupniot broj mo`nosti e 4!4!=576.
1043. Da pretpostavime deka tablata mo`e da se pokrie so
15 figuri od oblik i edna od oblik . Kolonite na
tablata gi boime naizmeni~no so crna i bela boja, kako na
crte` 225. Figurata od prviot oblik pokriva neparen broj
poliwa so ista boja (3 so 1). Crte` 225
Petnaeset takvi figuri }e pokrijat neparen broj crni i neparen broj beli
poliwa. Zaedno so figurata koja pokriva 2 beli i 2 crni poliwa,
povtorno }e bidat pokrieni neparen broj crni i neparen broj beli poliwa.

250
Odgovori, upatstva i re[enija
Me|utoa, so vakvoto boewe na tablata, brojot na crni i beli poliwa e paren.
Zna~i tablata ne mo`e da se pokrie so dadenite figuri.
1044. Jasno e deka na krajot ne postojat dve raznobojni figuri nitu vo edna
redica nitu vo edna kolona. Neka na krajot crni figuri ima vo a redici i b
koloni. Beli figuri mo`e da ima vo najmnogu 2n-a redici i 2n-b koloni.
Toga{ crni figuri ne se pove}e od ab, a beli figuri ne se pove}e od
2 2 4
(2n-a)(2n-b). Dobivame deka ab(2n-a)(2n-b)=a(2n-a)b(2n-b)≤n ÿn =n , od kade
{to sleduva tvrdeweto.
1045. Broevite vo tabelata gi zapi{uvame na sledniot na~in:

1 2 3 ... (n-1) n
...
1+n 2+n 3+ n ... (n-1) + n n+n
1 + 2n 2 + 2n 3 + 2n (n-1) + 2n n +2n
...
... ...
...
1 + (n- 2 + (n-2)n 3 + (n-2)n ... (n-1) + (n-2)n n + (n-2)n
2)n
1 + (n- 2 + (n-1)n 3 + (n-1)n (n-1) + (n-1)n n + (n-1)n
1)n

Neka spored uslovot na zada~ata to~no n/2 broevi go promenat svojot znak vo
sekoja redica, no taka {to so toa se promenat znacite i na to~no n/2 broja vo
sekoja kolona. Sobiraj}i gi prvite broevi od zbirovite po vertikala, se
dobiva deka zbirot na tie broevi vo sekoja kolona e nula. Ostanuva da se
soberat broevite koi pretstavuvaat vtori sobiroci vo sekoja od }eliite.
Sobiraj}i gi tie broevi po horizontal, se dobiva deka zbirot vo sekoja
redica e nula, od kade {to sleduva deka zbirot na site broevi e nula.
1046. 12869. Ako k e brojot na edine~ni kvadrati (1 ≤ k ≤ 8) toga{ postojat
(k8 ) na~ini na izbor na k koloni i (k8 ) na~ini na izbor na k redici.
8
k =1 k( )( )
k
8
k =0 k
( )( )
TVkupniot broj na~ini e ∑ 8 ⋅ 8 = − 1 + ∑ 8 ⋅ 8 = −1 + 16 = 12869 .
k 8 ( )
1047. Da zabele`ime deka dvi`ej}i se od edno do drugo pole po vertikala
ili horizontala mo`eme da napravime najmnogu 14 dvi`ewa (maksimalno
rastojanie me|u dve poliwa postaveni vo sprotivni temiwa). Jasno e deka
dvi`ej}i se od poleto na koe e zapi{an brojot 1 do poleto na koe e zapi{an
brojot 64 po najkratkiot pat }e se napravi najmnogu 14 dvi`ewa. Da go
pretpostavime sprotivnoto, odnosno deka razlikata na koi bilo dva sosedni
broja e pomala od 5. Toga{ brojot zapi{an vo edno pole posle edno dvi`ewe
se zgolemuva za ne pove}e od 4. Posle 14 dvi`ewa brojot se zgolemuva za ne
pove}e od 4·14=56. Kako 1+56=57 ne e pogolem od 64 doa|ame, do
protivre~nost na pretpostavkata. 1048. 3 u~enici; 15 poeni.
1049. Da pretpostavime deka prviot den {ahistot odigral a1 partija, za dva
dena odigral a2 partii, za tri dena a3 partii i t.n. Posle 77 dena, {ahistot
odigral a77 partii.
Da ja razgledame slednata niza a1, a2, ...,a77, a1+20, a2+20, ..., a77+20. Vo nizata
ima 154 ~lenovi, site pomali ili ednaki na 132+20=152 (bidej}i 77 dena se 11

251
Odgovori, upatstva i re[enija
nedeli, a77§11·12=132). Sleduva deka barem dva ~lena vo nizata se ednakvi.
Pome|u ~lenovite a1, a2, ..., a77 nema ednakvi bidej}i {ahistot sekoj den igral
barem edna partija. Od tie pri~ini sleduva deka nema ednakvi ~lenovi
pome|u a1+20, a2+20, ..., a77+20. Zna~i postojat k i m takvi {to ak=am+20,
odnosno ak-am=20. Poslednovo poka`uva deka za k-m posledovatelni denovi
od (m+1)-viot do k-tiot, {ahistot odigral to~no 20 partii.
1050. 21 600. Vo prvata redica mo`e da se obele`i eden od 6-te kvadrati koi
ne se vo }o{e. Vo poslednata redica mo`e da se obele`i eden od 5-te
kvadrati (gi izzemame ko{iwata i poleto koe se nao|a vo ista kolona so
obele`aniot kvadrat vo prvata redica). Vo vtorata redica mo`e da se izbira
pome|u 6 edine~ni kvadrati, vo tretata pome|u 5, ..., vo {estata pome|u 2 i vo
sedmata ima samo eden izbor. Vkupniot broj na~ini za obele`uvawe na
tablata e 6.5.6!=21600.
1 6 . 1055. Upatstvo: P(6 ) ⋅ V74 1
1051. 216. 1052. 4 . 1053. . 1054. P= = .
31 3 55 P(10 ) 6
20
1056. . Ako prviot tim e pobednik toga{ mora da pobedi vo pet od sedumte
81
odigrani igri. Edna od 5-te igri mora da bide i poslednata 7-ma igra. Taka,
vo prvite 6 igri, prviot tim mora da pobedi vo 4, a izgubi vo 2 igri.
6 2 4 1 2 2
Verojatnosta toa da se slu~i e P1 = [( 2 ) ⋅ ( 3 ) ⋅ ( 3 ) ] ⋅ ( 3 ) . Od druga strana,
verojatnosta vtoriot tim da pobedi posle 7 odigrani igri e
P2 = [( 62 ) ⋅ ( 13 ) 4 ⋅ ( 32 ) 2 ] ⋅ ( 13 ) . Zna~i, baranata verojatnost e
160 20 20
P = P1 + P2 = + = .
729 729 81
9
1057 . Neka rezultatot e nere{en posle odreden broj poedine~ni igri i
13
neka toa e posledniot nere{en rezultat. Vo slednite dve igri ili pobeduva
2 2
3
ili pobeduva Ivan so verojatnost   =
9 2 4
Daniel so verojatnost   = .
5 25  
5 25
9
25 9
Verojatnosta Daniel da bide kone~en pobednik e = .
9 4 13
+
25 25
2017 4034 2017
1058. 79. Brojot na pozitivni deliteli na 12 =2 ⋅3 e
4035ÿ2018.Brojot na pozitivnite deliteli na 12 =2 ⋅3 e 2010 4020 2010

4021ÿ2011.Verojatnosta deka pozitiven delitel na 12


2017
}e bide delitel i na
4021 ⋅ 2011
12
2010
e .
4035 ⋅ 2018
Od NZD(4035,4021)=NZD(4035,14)=NZD(3,14)=1;
NZD(4035,2011)=NZD(4035,13)=NZD(5,13)=1;
NZD(2018,4021)=NZD(2018,-15)=NZD(8,-15)=1;
NZD(2018,2011)=NZD(7,2011)=NZD(7,2)=1,sleduva deka dropkata e
neskratliva. Taka a=4021ÿ2011 i b=4035ÿ2018.

252
Odgovori, upatstva i re[enija
a+b=4021ÿ2011+4035ÿ2018ª{(-13)ÿ(-6)+1ÿ1}(mod 2017)ª79(mod 2017).
4 2
1059. Od 2025=3 ·5 sleduva deka 2025 ima 15 deliteli
od koi 10 se delivi so 5. Zna~i verojatnosta slu~ajno
10 2
izbran delitel na 2025 da e deliv so 5 e P = = , od
15 3
kade {to sleduva a=2, b=3 i a+b=5.
1060. 417 . Postojat 4 crveni kvadrati so dimenzii 2x2
512
(crt. 226).Neka Pijkl e verojatnosta da go dobieme i, j, k ili
l-ti kvadrat.
Crte`. 226
P1=P2=P3=P4=1/16fl ∑ Р i =1/4
P12=P13=P24=P34=1/64, P14=P23=1/128 fl ∑ Рij =5/64,
P123=P124=P134=P234=1/256, fl ∑ Р =1/64 i Pijkl=P1234=1/512.
ijk
37 .
P=1-( ∑ Р i − ∑ Р ij + ∑ Р ijk − ∑ Р ijkl )=1-1/4+5/64-1/64+1/512=417/512. 1061.
72
1062. 23 . Postojat 4 edine~ni kocki so 3 oboeni strani, 20 edine~ni kocki
96
so 2 oboeni strani, 28 edine~ni kocki so eden oboen yid i 12 edine~ni kocki
bez oboen yid. Za kocka so 3 oboeni strani, verojatnosta deka to~no 2 oboeni
1
strani }e bidat vidlivi e , dodeka za kocka so 2 oboeni strani,
2
4 2
verojatnosta deka dvete }e bidat vidlivi e , odnosno . Kone~no,
6 3
4 1 20 2 23 1 1
baranata verojatnost e P = ⋅ + ⋅ = . 1063. P= . 1064. .
64 2 64 3 96 8 1009
1065. 6 crveni i 3 sini top~iwa. Neka h e brojot na crvenite, a u brojot na
sinite top~iwa. Neka A, B i C se slednite nastani:
A: ,,dvete izvle~eni top~iwa se so crvena boja””
B: ,,dvete izvle~eni top~iwa se so razli~na boja””
C: ,,dvete izvle~eni top~iwa se so sina boja””.
Neka P(A), P(B) i P(C) se verojatnostite na soodvetnite nastani A, B i C. Od
uslovot na zada~ata imame deka P(A)=5P(C) …(1) i P(B) =6P(C) …(2).
2 2
Cx = x ⋅ (x − 1) ; P(C ) = Cy = y ⋅ (y − 1) ;
P (A ) = 2
Cx + y ( x + y ) ⋅ (x + y − 1) 2
Cx + y ( x + y ) ⋅ (x + y − 1)
1 1

P(B ) =
Cx ⋅ Cy = 2xy
.
2
Cx+ y ( x + y ) ⋅ (x + y − 1)

253
Odgovori, upatstva i re[enija
Zamenuvaj}i gi poslednite tri ravenstva vo ravenstvata (1) i (2) go dobivame
 x ⋅ (x - 1) y ⋅ (y - 1)
 (x + y ) ⋅ (x + y − 1) = 5 (x + y ) ⋅ (x + y − 1) x + y ≠ 0
 x ⋅ (x - 1) = 5y ⋅ (y - 1) 
sistemot  ñ  , x + y ≠ 1
 2xy
=6
y ⋅ (y - 1) 2xy = 6y ⋅ (y - 1) y ≠ 0
 (x + y ) ⋅ (x + y − 1) (x + y ) ⋅ (x + y − 1) 
~ii re{enija se (0,1) i (6,3). Zna~i, odgovorot e 6 crveni i 3 sini top~iwa.

Tema 20:
20: Funkcii.Funkcionalni ravenki
− 13
1066. f(2019)=-11. 1067. f(2) = . 1068. f(3)=26. 1069. r=0 ¤ r=-4.
16
3 2 2
f(x)=x –6x +9x+r=(x–a) ÿ(x–b)
3 2 3 2 2 2
f(x)=x – 6x + 9x + r = x – ( 2a + b )x + (a + 2ab )x – a b
2a + b = 6
i r=–a b. Re{enija na sistemot se parovite (3,0) i
2
Imame deka 
2
a + 2ab = 9
(1,4).
1070. f(2011)=2011. Od f(f(1)+x+y)=f(f(1)+f(f(x)+f(y)))=1+f(x)+f(y) sleduva deka
f(x)+f(y) e funkcija od x+y, odnosno f(x)+f(y)=g(x+y). Od poslednoto ravenstvo
imame deka f(x+1)+f(1)=g(x+2)=f(x)+f(2), odnosno f(x+1)-f(x)=f(2)-f(1). Sleduva
deka f(x)=ax+b. Zamenuvaj}i vo po~etnoto ravenstvo x=y=1, dobivame
f(f(1)+f(1))=2=f(2a+2b)=a(2a+2b)+b…(1), a za x=1, y=2 se dobiva
f(f(1)+f(2))=3=f(3a+2b)=a(3a+2b)+b…(2). Re{avaj}i go dobieniot sistem ravenki
(1) i (2) dobivame b=0, a=1. Kone~no, f(x)=x.
2 2
1071. Neka n>1. Od f(1)+f(2)+...+f(n)=n f(n) i f(1)+f(2)+...+f(n-1)=(n-1) f(n-1)
2 2 n −1
sleduva (n-1) f(n-1)+f(n)=n ·f(n), odnosno f(n) = ⋅ f(n − 1) ...(1). Od (1)
n +1
1 2 3
sleduvaat slednite ravenstva f(2) = ⋅ f(1) , f(3) = ⋅ f(2) , f(4) = ⋅ f(3) , …
3 4 5
n−2
f(n − 1) = ⋅ f(n − 2) , f(n) = n − 1 ⋅ f(n − 1) . So nivno mno`ewe se dobiva deka
n n +1
f(n) = 2 ⋅
(n − 1)!
⋅ f(1) , odnosno f(n) =
2020
…(2). Od (2) sleduva
(n + 1)! n ⋅ (n + 1)
1 1 x
f(2020) = . 1072. − . 1073. f(x) =
.
2021 x x
1+
2018
1074. f(0)=2;f(1)=-2. Ako vo po~etnata ravenka x se zameni so 1-x, se dobiva
3
f(1-x)+( -x)ÿf(x)=1 ...(1). Po~etnata ravenka i (1) go obrazuvaat sistemot
2

254
Odgovori, upatstva i re[enija

  1
f(x) +  x + f(1 - x) = 1 1 2 1
  2
 . Od sistemot, za x∫ se dobiva f(x)= . Za x= , od
 3 − x f(x) + f(1 − x) = 1 2 1 − 2x 2
 2 
1 1
 ,x =
 1 1 2 2
po~etnata ravenka se dobiva f   = . Zna~i, f(x)=  . Ottuka
2 2  2 ,x ≠ 1
1 − 2x 2
1 1  1
f(0)=2 i f(1)=-2. 1075. Ako vovedeme smena =t , odnosno x= , dobivame f  
x t t
1 1 1
+8f(t)=-63· . Zna~i f   +8f(x)=-63· . Ako prvata ravenka od sistemot
t x x
  1
 f(x) + 8f   = - 63x 1
 x ja pomno`ime so - i ja sobereme so vtorata ravenka

 f  1  + 8f(x) = - 63 8
  x  x
1 63 1 8
dobivame - f(x)+8f(x)= x-63 , odnosno f(x)=x- .
8 8 x x
6 4 3 2
1076. 1809. Neka f(x)= x − 4x − 6x − 4x + 1 . Da zabele`ime deka korenite na
1
ravenkata f(x)=0 se isti so korenite na ravenkata f   =0. Delej}i ja
x
3 4 1 1
ravenkata so x dobivame x 3 - 4x - 6 -− 3 = 0 …(1). So smenata y = x +
x x x
3
ravenkata (1) se transformira vo y - 7y - 6 = 0 odnosno (y+1)(y+2)(y-3)=0.
1 1 1
Ottuka y=-1, y=-2 ili y=3, odnosno x + = 1 , x + = -2 ili x + = 3 .
x x x
1 1
Maksimalna mo`na vrednost izrazot x 16 + 16 dostiga pri x + = 3 . Toga{
x x
1 1 1
x 2 + 2 = 3 2 - 2 = 7 ; x 4 + 4 = 7 2 - 2 = 49 - 2 = 47 ; x 8 + 8 = 47 2 - 2 = 2207 ;
x x x
1
x 16 + 16 = 2207 2 - 2 .
x
( )
Kone~no, m = 22072 − 2 ≡ 190 2 − 2 (mod 2017) ≡ 36098(mod 2017) ≡ 1809(mod 2017) .
1077. f(x)=x+1 xœQ.
i) Za y=0 ravenkata se transformira vo f(0)=f(0)f(x)-f(x)+1, odnosno vo
(f(0)-1)(f(x)-1)=0. Ottuka dobivame deka f(0)=1 ili f(x)=1 za sekoj realen
broj x. Re{enieto f(x)=1, xœR otpa|a poradi uslovot f(1)=2. Zna~i
ostanuva f(0)=1;

255
Odgovori, upatstva i re[enija

ii) Za xœQ i y=1 se dobiva f(x)=f(1)f(x)-f(x+1)+1=2f(x)-f(x+1)+1, od kade {to


se dobiva f(x+1)=f(x)+1, xœQ;
iii) Od ii) i od f(0)=1 se dobiva deka f(k)=k+1, kœZ;
iv) Od ii) isto taka se dobiva f(x+m)=f(x)+m (xœQ, mœZ).
1 1 1 1
v) Za x=kœZ i y= , od (iv) imame 2=f(1)=f(k· )=f(k)f( )-f(k+ )+1
k k k k
1 1 1 1
=(k+1)f( )-(k+f( ))+1, od kade {to se dobiva f( )=1+ .
k k k k
1 m 1 1
vi) Kone~no za x=m, y= (m,nœZ) dobivame f( )=f(m)f( )-f(m+ )+1
n n n n
1 1 m
=(m+1)( +1)-(m+ +1)+1= +1. Dobivame deka za sekoj racionalen
n n n
broj x va`i f(x)=x+1. Zna~i, edinstveno re{enie na funkcionalnata
ravenka e f(x)=x+1. 1078. x=1, x=-3.
x3 - x +1
. Zаменувајќи го x во равенката f(x)+f 
1  1
1079. f(x)=  =x…(1) sо
2x(x - 1)  1 - x  1 − x
 1   x - 1 1 x -1
добиваме f   +f  = …(2). Ako x го замениме со добиваме f
 1- x   x  1- x x
 x - 1 x -1
  +f(x)= …(3). Собирајќи gi ravenkite (1) и (3) и одземај}и go
 x  x

x3 - x +1 x3 - x +1
ravenstvoto (2) добиваме 2f(x)= odnosno f(x)= .
x(x - 1) 2x(x - 1)
1 1 1
1080. f(x) = x + − . Ako x≠0 ili x≠1, dropkata e razli~na od 0 i 1, pa vo
x 2 1− x
1
dadenoto ravenstvo mo`eme x da go zamenime so , dobivaj}i go pritoa
1− x
ravenstvoto  1   x − 1
f  + f =−
(1 − x )2 …(1). Ponatamu, ako x≠0 ili x≠1,
 1− x   x  x
x −1
dropkata e isto taka razli~na od 0 i 1, pa vo dadenata ravenka x go
x
x -1  x − 1 x2
zamenuvame so , dobivaj}i go ravenstvoto f   + f(x) = …(2).
x  x  x -1
Sobiraj}i gi dadenata ravenka i ravenstvoto (2), a potoa odzemaj}i ja
2x 2 − x + 2
ravenkata (1), se dobiva 2f(x) = , od kade dobivame deka
x
1 1 2
f(x) = x +− . 1081. f(x)=x. 1082. f(x)=x . Ako vo vtorata ravenka zememe y=0,
x 2
dobivame f(x)=f(x)+f(0)+0 odnosno f(0)=0. Ottuka 0=f(0)=f(1+(-1))=f(1)+f(-1)-2
odnosno f(1)+f(-1)=2. Od druga strana, ako vo vtorata ravenka zamenime y=-x i

256
Odgovori, upatstva i re[enija
2
ja iskoristime prvata ravenka, dobivame 0=f(x+(-x))=f(x)+f(-x)-2x
. . 2 . . 2 2 2
=f(x 1)+f(x (-1))-2x =f(x) f(1)+f(x) f(-1)-2x =2f(x)-2x odnosno f(x)=x .
1083. f(x)=x; f(x)=x+1; f(x)=x-1. 1084. f(x)=≤x.
2
1085.f(x)=≤x, xœR. Jasno e deka f(0)=0. Za y=x dobivame f(xf(x)=x ...(1). Od (1)
2 2
sleduva deka f(f(1))=1 i f(f(1)f(f(1)))=(f(1)) , odnosno (f(1)) =1. Zna~i f(1)=≤1. ]e
doka`eme deka f e injekcija.
i) f(1)=1. Za y=1 po~etnata ravenka se transformira vo f(x)+f(f(x))=2x ...(2). Od
(2) sleduva injektivnost na f.
ii) f(1)=-1. Od (1) dobivame f(-f(-1))=1. Od (1), za x=1 se dobiva f(-1)=1. Za y=-1,
po~etnata ravenka se transformira vo f(x)+f(-f(x))=-2x, od kade sleduva
1 1
injektivnosta na f. ]e doka`eme deka f( )= ...(3). Ako vo po~etnata ravenka
x f(x)
1 1 1 1 1
stavime y=f( )· i go iskoristime (1), dobivame f( f( )f(x))=f( ). Od
x x x x x
1
injektivnosta na f se dobiva (3). Za f(1)=1, ako vo (2) x go zamenime so i go
x
1 1 2 1 1 2
iskoristime (3), dobivame + = , odnosno + = , {to e
f(x) f (f(x)) x f(x) 2x - f(x) x
ekvivalentno so f(x)=x, xœR. Analogno, za f(1)=-1 se dobiva f(x)=-x, xœR.
2
1086. f(x) = 1-x . Upatstvo: Zamenete go х so (1-х) vo dadenata ravenka.
x
1087. f(x)=0 i f(x)= . O~igledno e deka f(x)=0 e re{enie. Neka f(x) e drugo,
2
nenulto re{enie na funkcionalnata ravenka. Postoi aœR taka {to f(a)∫0.
Zamenuvaj}i gi ulogite na x i y vo funkcionalnata ravenka se dobiva
f(a)
yf(x)=f(xf(y)+yf(x))=xf(y) (x,yœR). Za y=a se dobiva f(x) = x = cx , kade {to
a
f(a)
c= e konstanta. Zamenuvaj}i f(x)=cx vo po~etnata ravenka, se dobiva
a
2 1 0
2c xy=cxy, od kade {to se dobiva c=0 ili c= . Ottuka, osven f(x)= go
2 2
x
dobivame i re{enieto f(x)= .
2
1088. f(x)=x i f(x)=-x. Ako vo funkcijata stavime x=0 dobivame
f(f(y))=f(0)+y…(1). Dokolku x i y si gi zamenat mestata i stavime x=0, dobivame
f(y)=f(y+f(0))…(2). Od (1) i (2) se dobiva f(y)=f(y+f(0))=f(f(f(y))=f(0)+f(y) odnosno
f(0)=0. Ottuka (1) preminuva vo f(f(y))=y. Ponatamu, f(x+y)=f(x+f(f(y))=f(x)+f(y), od
kade sleduva deka f(x)=Aÿx. Zamenuvaj}i vo dadenata funkcija dobivame deka
A(x+Ay)=Ax+y odnosno A=≤1.
1089. f(y)=y+1, "yœR. Od a) sleduva f(x+f(y))=f(x+y)+1 =f(y+x)+1=f(y+f(x))…(1).
Bidej}i f e strogo monotono raste~ka, f e injektivna, pa od (1) sleduva deka
x+f(y)=y+f(x). Ako vo poslednoto ravenstvo zamenime y=0 dobivame deka "xœR
va`i f(x)=x+f(0) …(2). Od a) dobivame x+f(y)+f(0)=x+y+f(0)+1 odnosno f(y)=y+1,
"yœR. 1090. F(x)=kx, kœR. 1091. f(x)=3x-2x.

257
Odgovori, upatstva i re[enija
1092. f(x)=x+1. Zamenujvaj}i x=y=z=1 vo dadenoto ravenstvo i ozna~uvaj}i
3 2
f(1)=a dobivame a -a=6, odnosno (a-2)(a +2a+3)=0. Od poslednoto ravenstvo se
dobiva a=2 odnosno f(1)=2. Za proizvolno x i y=z=1 dobivame deka 3f(x)=3x+3,
odnosno f(x)=x+1. So neposredna proverka se poka`uva deka funkcijata
f(x)=x+1 ja zadovoluva dadenata ravenka.
x +1
1093. f(x)=2x+1 .Upatstvo: Vovedete smena = t.
x −1
1094. ]e doka`eme prvo deka f e biekcija.
I. Neka f(x1)=f(x2). Toga{ imame deka f(x1)+y+1=f(x2)+y+1, "yœZ. Ottuka
f(f(x1)+y+1)=f(f(x2)+y+1) odnosno x1+f(y)+1=x2+f(y)+1. Od poslednoto
sleduva deka x1=x2, odnosno f e injekcija.
II. Neka kœZ. Toga{ k=(k-1-f(y))+f(y)+1=f(f(k-1-f(y)+y+1), "yœZ. Ottuka
sleduva deka f e i surjekcija, sleduva deka f e biekcija;
III. Bidej}i f e surjekcija, postoi AœZ taka {to f(A)=-1. Toga{ za sekoj
yœZ va`i f(y)=f(f(A)+y+1)=A+f(y)+1, od kade sleduva deka A=-1. Isto
taka, postoi BœZ taka {to f(B)=0 i toga{ za sekoj yœZ va`i
f(y+1)=f(f(B)+y+1)=B+f(y)+1.
Zna~i dobivme deka za sekoj yœZ va`i f(y+1)=f(f(B)+y+1)=B+f(y)+1, odnosno
f(y+1)-f(y)=B+1=const. Od poslednoto lesno se poka`uva deka za sekoj xœZ,
f(x)=(B+1)x+f(0) ili ako stavime C=B+1 i D=f(0) se dobiva f(x)=Cx+d, "xœZ.
Zamenuvaj}i go izrazot za f vo po~etnata funkcionalna ravenka, se dobiva
slednoto:
f(f(x)+y+1)=x+f(y)+1, "x,yœZ
ñ f(Cx+D+y+1)=x+Cy+D+1, "x,yœZ
ñ C(Cx+D+y+1)+D=x+Cy+D+1, "x,yœZ
2
ñ C x+CD+C=x+1, "x,yœZ
2
ñ (C -1)x+CD+C-1=0, "x,yœZ

C 2 − 1 = 0 C = ±1
 C = 1 C = −1
ñ  ñ  1− C ñ  ¤  .
CD + C − 1 = 0 D = D = 0 D = −2
 C
Zna~i dobivame f(x)=x ili f(x)=-x-2. Lesno se proveruva deka i dvete funkcii
go zadovoluvaat uslovot na zada~ata.
2021
1095. f (x ) = x . Za y=0 se dobiva f(0)=0. Zamenuvaj}i vo ravenkata za x=0,
2020
se dobiva deka f(-y)=-f(y), {to zna~i deka funkcijata e neparna. Od
f(4ab)=f(4ba) sleduva ravenstvoto (a+b)f(a-b)=(a-b)f(a+b). Vo poslednoto
ravenstvo za b=a-2020 se dobiva (2a-2020)f(2020)=2020f(2a-2020). Ako vo
poslednoto ravenstvo zememe x=2a-2020 i iskoristime f(2020)=2021, ja
dobivame baranata funkcija.

258
Prilog
 Princip na vklu~uvawe
Ako A, B i C se tri mno`estva toga{
A ∪B ∪C = A + B + C − A ∩B − B ∩C − C ∩ A + A ∩B ∩C .
Brojot A go ozna~uva vkupniot broj elementi {to gi sodr`i mno`estovoto A.
 Princip na isklu~uvawe
Neka A,B, C ⊆ M se dadeni mno`estva. Ozna~uvame X = M \ X , komplement na
dadeno mno`estvo. Toga{
A ∩B ∩C = M − A − B − C + A ∩B + B ∩C + C ∩ A − A ∩B ∩C .
 Logi~ki zakon kontrapozicija
Ako p i q se logi~ki iskazi toga{ (p ⇒ q) ⇔ (¬q ⇒ ¬p ) .
 Nekoi formuli za razlo`uvawe na mno`iteli
(
x n − y n = (x − y ) ⋅ x n−1 + x n−2 y + x n−3 y 2 + ... + x 2 y n−3 + xyn−2 + y n−1 ; )
n
x +y n
= (x + y ) ⋅ (x n−1
−x n −2
y+x n−3 2 2 n−3
y − ... + x y − xy n−2
+y n−1
), za n-neparen broj;
(x + y + z) ⋅ (xy + yz + zx ) − xyz = (x + y ) ⋅ (y + z ) ⋅ (z + x ) .
 Relacii me|u stepenska, aritmeti~ka, geometriska i harmoniska sredina
r r r
a1 + a 2 + ... + a n
Neka a 1, a 2 ,..., a n se n pozitivni realni broevi. Ako K n = r ,
n
a 1 + a 2 + ... + a n n
r>1, An = , G n = n a 1 ⋅ a 2 ⋅ ... ⋅ a n i Hn =
n 1 1 1
+ + ... +
a1 a 2 an
toga{ K n ≥ A n ≥ G n ≥ Hn .
 Neravenstvo na Nesbit
a b c
Za pozitivnite realni broevi a, b i c va`i + + ≥ 3.
b+c c +a a+b
 Neravenstvo na ^ebi{ev
Ako broevite a1, a2, …, an i b1, b2, …, bn gi ispolnuvaat uslovite:
a) a1≤a2≤…≤an i b1≤b2≤…≤bn ili a1≥a2≥…≥an i b1≥b2≥…≥bn toga{
1
a1b1+a2b2+…+anbn≥ (a1+a2…+an)(b1+b2+…bn);
n
b) a1≤a2≤…≤an i b1≥b2≥…≥bn ili a1≥a2≥…≥an i b1≤b2≤…≤bn toga{
1
a1b1+a2b2+…+anbn≤ ( a1+a2+…+an)(b1+b2+…+bn).
n
 Neravenstvo na Jensen Neka f(x) e konveksna funkcija na intervalot I, pri
{to x1, x2, …, xnœI, kade IÕR. Ako f(x) e neprekinata funkcija na intervalot I
 x + x 2 + ... + x n
 f(x 1 ) + f(x 2 ) + ... + f(x n )
toga{ f  1  ≤ .
 n  n
 x + x 2 + ... + x n  f(x 1 ) + f(x 2 ) + ... + f(x n )
Ako f(x) e konkavna toga{ va`i f  1  ≥ .
 n  n

259
 Neravenstvo na Bernuli
n
Ako e n priroden broj, a x realen broj pogolem od -1 toga{ va`i (1+x) ¥1+nx.
Ravenstvo va`i ako i samo ako n=1 ili x=0.
 Evklidovo neravenstvo vo triagolnik
Zbirot od dol`inite na dve strani vo triagolnik e pogolem od dol`inata na
tretata strana
 Pravilo na simetrala (bisektrisa)
Ako AD e bisektrisa vo triagolnikot ABC toga{ va`i BA : BD = CA : CD .
 Talesova teorema za proporcionalni otse~ki
Neka dve pravi gi se~at kracite na daden agol. Tie pravi se paralelni ako i
samo ako soodvetnite (otse~eni) otse~ki od kracite se proporcionalni.
 Talesova teorema
teorema za periferen agol
Periferniot agol nad dijametar vo kru`nica e prav.
 Pitagorova teorema
Zbirot od kvadratite na katetite vo pravoagolen triagolnik e ednakov so
kvadratot od hipotenuzata.
 Apolonieva teorema
Neka a, b i c se strani vo triagolnikot ABC. Ako ta e te`i{nata linija
 a  2 
povle~ena od temeto A toga{ va`i b 2 + c 2 = 2   + t a 2  .
 2  
 Stepen na to~ka vo odnos na kru`nica
2
Stepen na to~ka P vo odnos na kru`nica k(O,r) e brojot OP − r 2 .
2
Vo zavisnost od znakot na OP − r 2 odreduvame dali to~kata P e vnatre{na
to~ka za kru`nicata (ako znakot e minus), nadvore{na to~ka (ako znakot e
plus), a ako stepenot e 0 toga{ P le`i na kru`nicata.
Ako AD i BC se dve tetivi na kru`nica k koi se se~at vo to~ka P
(nadvore{na ili vnatre{na za kru`nicata), toga{ stepenot na to~kata P vo
odnos na k e ednakov na PA ⋅ PD = PB ⋅ PC .
 Radikalna oska
Mno`estvoto to~ki so ednakov stepen vo odnos na dve nekoncentri~ni
kru`nici se vika radikalna oska.
 Priznak za delivost so 7
Brojot a 1a 2 ...a n e deliv so 7 ako i samo ako razlikata a 1a 2 ...a n-1 − 2a n e
deliva so 7.
 Priznak za delivost so 11
Ako razlikata od zbirovite na cifrite od neparniot i parniot razred e
deliv so 11 toga{ i dadeniot broj e deliv so 11.
 Formula za odreduvawe vkupen broj deliteli na daden priroden broj
Neka prirodniot broj m = p1n1 ⋅ p 2 n2 ⋅ ... ⋅ p k nk , kade p1, p 2 , ..., p n se razli~ni
prosti broevi. Vkupniot broj deliteli na brojot m e
n = (n1 + 1) ⋅ (n 2 + 1) ⋅ ... ⋅ (nk + 1) .
 Moavrova formula
Za sekoj realen broj x i cel broj n va`i (cosx + i ⋅ sinx )n = cos(nx ) + i ⋅ sin(nx ) .
 Nekoi formuli za presmetuvawe plo{tina na triagolnik

260
ab ⋅ sinγ bc ⋅ sinα ca ⋅ sinβ abc
i) P = = = ; ii) P = ; iii) P = rs ; iv) P = s(s − a )(s − b )(s − c ) .
2 2 2 4R
 Vieotovi formuli
Ako x1, x 2 , ..., x n se koreni na ravenkata a n x n + a n -1x n -1 + ... + a1x + a 0 = 0 toga{
a n −1
x1 + x 2 + ... + x n = (− 1) ⋅ ;
an
an − 2
x1x 2 + x 1x 3 + ... + x n -1x n = (− 1) ⋅
2
;
an

a0
x1 ⋅ x 2 ⋅ ... ⋅ x n = (− 1) ⋅
n
.
an
 Sinusna teorema
Vo sekoj triagolnik so strani a, b, c i agli α, β, γ va`i
a b c
= = = 2R , kade {to R e radiusot na opi{anata kru`nica na
sinα sinβ sinγ
triagolnikot.
 Kosinusna teorema
Vo sekoj triagolnik so strani a, b, c i agli α, β, γ va`i:
a 2 = b 2 + c 2 − 2bc ⋅ cosα ; b 2 = c 2 + a 2 − 2ca ⋅ cosβ ; c 2 = a 2 + b 2 − 2ab ⋅ cosγ .
 Tangensna teorema
Vo sekoj triagolnik so strani a, b, c i agli α, β, γ va`i:
α+β β+γ γ+α
tg tg tg
a+b 2 b+c 2 c+a 2 .
i) = ; ii) = ; iii) =
a-b α-β b-c β-γ c-a γ -α
tg tg tg
2 2 2
 Princip na matemati~ka indukcija
Neka P(n) e tvrdewe koe va`i za prirodnite broevi. Ako P(1) e to~no
tvrdewe i ako od to~nosta na P(k) sleduva to~nost na P(k+1) toga{ P(n) e
to~no tvrdewe za sekoj priroden broj n.
 Cel del od x (oznaka [x])
Cel del od realniot broj x e najgolemiot cel broj pomal od x, odnosno,
[x] = k ako и samo ako k ≤ x < k + 1 .
 Droben del od x (oznaka {x})
Droben del od realniot broj x se presmetuva so formulata {x}=x-[x].
 Fibona~ievi broevi
Broevite od nizata (Fn) definirana so rekurentnata ravenka Fn=Fn-1+Fn-2, so
F1=F2=1, se vikat fibona~ievi broevi
 Mala teorema na Ferma
p
Ako p e prost broj i a e cel broj, toga{ a ªa(modp). Ako a i p se zaemno prosti
p-1
toga{ a ª1(modp).
 Princip na Dirihle
Ako m top~iwa rasporedime vo n kutii, pri m>n toga{ postoi barem edna
kutija koja sodr`i najmalku dve top~iwa.
 Ko{ieva funkcionalna ravenka f(x+y)=f(x)+f(y).

261
Blagodarnica

Posebna blagodarnost im izrazuvame na Valentina Miovska,


Aneta Gacovska i Del~o Le{kovski za nivniot nesebi~en pridones
za oformuvawe i podobruvawe na kvalitetot na ovaa kniga, za {to
najiskreno i od se srce im se zablagodaruvame.

262
Koristena literatura

[1] Voved vo teorija na broevi - Don~o Dimovski, Risto Mal~eski,


Kostadin Tren~evski;
[2] Izbrani sodr`ini od elementarna matematika (Biblioteka
„SIGMA“) - Risto Mal~eski, Aleksa Mal~eski 1994 (Skopje);
[3] Metodi za doka`uvawe na neravenstva – Zoran [terjov 2008
(Probi{tip);
[4] Identi~ni transformacii – Pavle M. Mili~i} 2002 (Belgrad);
[5] Nejednakosti. Funkcionalne jedna~ine - Vladimir BaltiÑ 2004
(Belgrad);
[6] Sbornik ot zada~ za matemati~eski olimpiadi - St. Budurov,
V.Florov 1966 (Sofija);
[7] Sbornik ot podrobno re{eni zada~i po matematika - Cvetanka
Stoilkova 1995 (Sofija);
[8] SBORNIK KONKURSNÃIH ZADA^ PO MATEMATIKE S
RE[ENIœMI - VASILI• KU…ENKO 1965 (Leningrad);
[9] WISCONSIN MATHEMATICS SCIENCE & ENGINEERING TALENT
SEARCH 1999-2009 (Wisconsin);
[10] [osta SorosivsÏka olimpiada z matematiki 1999-2000 (Kiev);
[11] Harvard – MIT Mathematics Tournament, 2003-2008 (Cambridge)
Boston;
[12] USA – Mathematical Talent Search – Art of Problem Solving
Foundation 1999-2000 ; (Washington)
[13] The i-TEST American internet mathematical competition, 2007
(Franclin);
[14] Zada~i po planimetrii - V.V.Prasolov 2006 (Moskva);
[15] Sbornik zada~ kievskih matemati~eskih olimpiad 1984 (Kiev);
[16] Algebra i teoriя ~isel dlя matemati~eskih {kol - N. B.
Alfutova, A. V. Ustinov 2002 (Moskva);
[17] VSEROSSIœSKIE OLIMPIADÃI [KOLÃNIKOV PO
MATEMATIKE - N.G.Agahanov, I.I.Bogdanov, P.A.Ko`evnikov,
O.K.Podlipski£, D.A.Tere{in 1993-2006 (Moskva);
[18] „Mathematical Excalibur“ 2000-2010 (Hong Kong);
[19] @urnal „Kvant“ 2003-2009 (Moskva);
[20] Crux Mathematicorum with Mathematical Mayhem – Canadian
Mathematical Society 2001-2010 (Canada);
[21] ,,SIGMA”” - Sojuz na matemati~ari na R. Makedonija 1992-2010
(Skopje);
[22] Sigmina riznica - (Biblioteka ,,SIGMA””) 2008 (Skopje);

263
[23] Math@Mac Online Mathematics Competition, McMaster University,
2006-2009 (Canada);
[24] Mathematics Competition, Florida Atlantic University, 2006-2007
(Boca Raton);
[25] Geometri~eskie olimpiadÎ im. I.F. [arÏigina – A.
A.Zaslavski£, V.Œ. Protasov, D.I. [arÏigin 2007 (Moskva);
[26] Massachusetts Mathematics Olympiad 1998-2007 (USA);
[27] Moskovskie matemati~eskie regatÎ - A. D. Blinkov, E. C.
GorskaÔ, V. M. Gurovic 2007 (Moskva);
[28] Moskovskie Matemati~eskie olimpiadÎ 1993-2005 - A. œ.
KanelÏ - Belov, A. K. KovalÏd`i, R. M. FÒdorov, I. V. œÈenko
2006 (Moskva);
[29] 101 Problems in Algebra - Titu Andreescu, Zuming Feng 2001 (USA);
[30] San Jose Math Circle (mathematical competition) – 2009 (San Jose);
[31] Državno natjecanje učenika srednjih škola Republike Hrvatske –
Hrvatsko Matematičko Društvo (2002-2009) Zagreb;
[32] Complex Numbers from A to … Z – Titu Andreescu, Dorin Andrica
2001 (Romania);

264
265

Das könnte Ihnen auch gefallen